Prep Id 2016

You might also like

Download as pdf or txt
Download as pdf or txt
You are on page 1of 309

12/12/2016 January

Welcome  mohammed alsaiary [ Logout ]

Home PREP Pearls FAQs My Bookmarks CME Information

Overview Home  > January

Editorial Board
January   Enter Keyword Search
January
Question View:   All (8) Jump to Question
February
Print this Page Add to my Bookmarks Page 8 of 115
March

April Assessment History Mode: Learner  Exam 

May ASSESSMENT PROGRESS:  Total Questions:  8  Questions Answered:  0  Correct Answers:  0


June

July You currently have 8 questions unanswered in this assessment

August

September
Question: 1
October
A 3­year­old, unimmunized child presents to the pediatrician’s office with a 3­day history of fever and a 1­day history of pruritic rash.
November Physical examination reveals a fever of 38.9°C and a papulovesicular rash on the trunk and extremities, which appears in crops at
different stages of evolution and excoriation.
December
Of the following, the compound MOST likely to inhibit the virus causing this child’s illness is a

A. messenger RNA inhibitor
B. neuraminidase inhibitor
C. nucleoside analogue
Claim Credit
D. pH channel blocker
Evaluation
E. reverse transcriptase inhibitor
My Learning Plan

Submit   Reset  

Page 8 of 115

Contact Us
Links may open in separate window
Copyright © 2016   American Academy of Pediatrics. All rights reserved. Pop­up Blocker may need to be disabled

http://2016.prepid.courses.aap.org/script/january?req=201612121342157346# 1/1
12/12/2016 January

Welcome  mohammed alsaiary [ Logout ]

Home PREP Pearls FAQs My Bookmarks CME Information

Overview Home  > January

Editorial Board
January   Enter Keyword Search
January
Question View:   All (8) Jump to Question
February
Print this Page Add to my Bookmarks Page 8 of 115
March

April Assessment History Mode: Learner  Exam 

May ASSESSMENT PROGRESS:  Total Questions:  8  Questions Answered:  1  Correct Answers:  0


June

July You currently have 7 questions unanswered in this assessment

August

September
Question: 1
October
A 3­year­old, unimmunized child presents to the pediatrician’s office with a 3­day history of fever and a 1­day history of pruritic rash.
November Physical examination reveals a fever of 38.9°C and a papulovesicular rash on the trunk and extremities, which appears in crops at
different stages of evolution and excoriation.
December
Of the following, the compound MOST likely to inhibit the virus causing this child’s illness is a

A. messenger RNA inhibitor
B. neuraminidase inhibitor
C. nucleoside analogue
Claim Credit
D. pH channel blocker
Evaluation
E. reverse transcriptase inhibitor
My Learning Plan

Incorrect View Peer Results
Correct Answer: C Average Percent Correct:  80.58%

This child has varicella, a disease caused by varicella­zoster virus (VZV), a member of the DNA virus
family of Herpesviridae. The compound most likely to inhibit the virus causing this child’s illness is a
nucleoside analogue. The nucleoside analogues include 2 nucleoside analogues of deoxyguanosine,
acyclovir and its valine ester valacyclovir, and ganciclovir and its valine ester valganciclovir. These
compounds incorporate into viral DNA during the viral replicative cycle and exert antiviral activity by
inhibiting viral DNA polymerase. These compounds are active against members of the Herpesviridae
family of viruses. Acyclovir has specific activity against herpes simplex virus (HSV) types 1 and 2 and
VZV, whereas ganciclovir, in addition to inhibiting HSV and VZV, has extended activity against cytomegalovirus as well.

An example of a messenger RNA inhibitor is ribavirin. Ribavirin is also a nucleoside analogue and a guanosine analogue with
broad spectrum antiviral activity. It inhibits a wide variety of RNA viruses primarily by interfering with viral mRNA formation,
and is not likely to exert adequate antiviral activity against VZV.

The neuraminidase (NA) inhibitors, oseltamivir and zanamivir, inhibit the ability of both influenza virus type A and B from
penetrating through the mucoproteins present in respiratory secretions and reduces the ability of influenza virus to attach to
the surface of cells. The NA inhibitors also prevent release of influenza virus particles from infected cells and reduce cell to
cell spread of virus within the respiratory tract, lessening the intensity of the viral infection.

Examples of a pH channel blocker are amantadine and rimantadine, which are members of the class of antivirals called
adamantanes (also known as tricyclic amines). They inhibit replication of susceptible strains of influenza A virus by blocking
the ion channel pump function of the viral M2 protein by decreasing the amplitude and frequency of the M2 ion channel
opening. By acting as a pH channel blocker, the resultant change in intracellular pH primarily inhibits viral uncoating and to a
lesser extent, viral assembly.

The reverse transcriptase inhibitors include at least 6 nucleoside reverse transcriptase inhibitors (NRTIs), zidovudine,
didanosine, stavudine, lamivudine, abacavir, and emtricitabine; nucleotide reverse transcriptase inhibitors (NtRTIs) which
include tenofovir; and the non­nucleoside reverse transcriptase inhibitors (NNRTIs), which include nevirapine, efavirenz, and
etravirine. These compounds inhibit HIV by inhibiting reverse transcriptase through a variety of different mechanisms.

Antiviral agents inhibit viral replication by interfering with 1 or more of the basic steps in the viral replicative cycle. These steps
include:

Attachment ­ specific binding between viral capsid proteins and cell receptors on the cell surface; neuraminidase
inhibitors inhibit this step of viral replication in influenza A and B viruses

http://2016.prepid.courses.aap.org/script/january?req=201612121350174543&status=submit# 1/2
12/12/2016 January
Entry or penetration ­ the virus fuses with the cell membrane and enters the cell
Uncoating ­ which removes the viral capsid and releases the viral nucleic acid; amantadine and rimantadine inhibit this
step of viral replication in influenza A virus
Nucleic acid replication­ viral gene expression and viral nucleic acid synthesis that takes place in nucleus for DNA
viruses and in cytoplasm for RNA viruses, inhibitors of viral DNA polymerase, such as acyclovir, ganciclovir, cidofovir,
and foscarnet inhibit this step of viral replication for DNA viruses; inhibitors of mRNA expression, such as ribavirin, also
inhibit this step; the antiretroviral agents that are integrase inhibitors and reverse transcriptase inhibitors inhibit HIV
replication at this ste
Protein synthesis ­ viral proteins are synthesized, and may include both structural and enzymatic proteins
Assembly ­ viral proteins and viral nucleic acid assemble into mature virions; amantadine and rimantadine also inhibit
this step of viral replication for influenza A virus; protease inhibitors inhibit this step of HIV replication
Release – mature virion is released from the cell through lysis, which bursts or destroys the cell, or through budding,
which allows enveloped viruses to acquire a modified piece of the cell’s membrane as it exits the cell; neuraminidase
inhibitors also inhibit this step of viral replication in influenza A and B viruses

PREP Pearls
The main steps in viral replication for most viruses associated with human disease include attachment, entry, uncoating,
nucleic acid and protein synthesis, assembly and release.

Antiviral agents target a specific step in viral replication.

American Board of Pediatrics Content Specification(s)
Know the major steps in the viral replication cycle and which ones are targets for antiviral and interferon therapy

Suggested Readings
Harrison G. Antiviral Agents. In: Cherry J, Harrison G, Kaplan S, Steinbach W, Hotez P, eds. Feigin and Cherry’s Textbook of
Pediatric Infectious Diseases, 7th ed. Philadelphia, PA: Saunders Elsevier; 2014: 3265­3290.

Seeborg FO, Paul ME, Shearer WT. Human immunodeficiency virus and acquired immunodeficiency syndrome In: Cherry J,
Harrison G, Kaplan S, Steinbach W, Hotez P, eds. Feigin and Cherry’s Textbook of Pediatric Infectious Diseases, 7th ed.
Philadelphia, PA: Saunders Elsevier; 2014: 2609­2619.

Comment On This Question

Page 8 of 115

Contact Us
Links may open in separate window
Copyright © 2016   American Academy of Pediatrics. All rights reserved. Pop­up Blocker may need to be disabled

http://2016.prepid.courses.aap.org/script/january?req=201612121350174543&status=submit# 2/2
12/13/2016 January

Welcome  mohammed alsaiary [ Logout ]

Home PREP Pearls FAQs My Bookmarks CME Information

Overview Home  > January

Editorial Board
January   Enter Keyword Search
January
Question View:   All (8) Jump to Question
February
Print this Page Add to my Bookmarks Page 9 of 115
March

April Assessment History Mode: Learner  Exam 

May ASSESSMENT PROGRESS:  Total Questions:  8  Questions Answered:  1


June

July You currently have 7 questions unanswered in this assessment

August

September
Question: 2
October
You are asked to evaluate a 2­year­old girl with recurrent middle ear infections. She is otherwise previously well, growing and
November developing normally, and has received all recommended childhood vaccines to date. Since 8 months of age, she has had 5 separate
episodes of acute otitis media. The first 4 infections were treated for 10 days, each with amoxicillin, and each time resolved
December
completely. The most recent infection began 2 weeks ago, and despite initial treatment with amoxicillin (40 mg/kg per day), fevers
and bilateral ear pain persisted. The antibiotic treatment was switched to cefdinir 1 week ago, but when she was re­evaluated by her
primary care physician 2 days ago, the child was still having fevers and her left tympanic membrane was ruptured. A significant
amount of purulent drainage was noted in the left external auditory canal. The right tympanic membrane remained erythematous,
with a layer of pus visible behind it. The primary care physician obtained a blood culture, a complete blood cell count, and a culture
of the left ear drainage, then administered a dose of intramuscular ceftriaxone. On presentation to your office the next day, the child
Claim Credit continued to have fever and the drainage from the left ear continued. Laboratory results from the prior day revealed a normal
complete blood cell count, negative blood culture for 1 day, and an ear culture growing 3+ Streptococcus pneumoniae. You consider
Evaluation
performing additional diagnostic studies.
My Learning Plan
Of the following test results, an explanation for the persistence of the child’s current infection would MOST likely be revealed by

A. antibiotic susceptibility of the S pneumoniae
B. computed tomography of the mastoids bilaterally
C. serum quantitative immune globulin assay
D. ultrasonography of the spleen

Submit   Reset  

Page 9 of 115

Contact Us
Links may open in separate window
Copyright © 2016   American Academy of Pediatrics. All rights reserved. Pop­up Blocker may need to be disabled

http://2016.prepid.courses.aap.org/script/january?req=201612121928585699 1/1
12/13/2016 January

Welcome  mohammed alsaiary [ Logout ]

Home PREP Pearls FAQs My Bookmarks CME Information

Overview Home  > January

Editorial Board
January   Enter Keyword Search
January
Question View:   All (8) Jump to Question
February
Print this Page Add to my Bookmarks Page 9 of 115
March

April Assessment History Mode: Learner  Exam 

May ASSESSMENT PROGRESS:  Total Questions:  8  Questions Answered:  2  Correct Answers:  1


June

July You currently have 6 questions unanswered in this assessment

August

September
Question: 2
October
You are asked to evaluate a 2­year­old girl with recurrent middle ear infections. She is otherwise previously well, growing and
November developing normally, and has received all recommended childhood vaccines to date. Since 8 months of age, she has had 5 separate
episodes of acute otitis media. The first 4 infections were treated for 10 days, each with amoxicillin, and each time resolved
December
completely. The most recent infection began 2 weeks ago, and despite initial treatment with amoxicillin (40 mg/kg per day), fevers
and bilateral ear pain persisted. The antibiotic treatment was switched to cefdinir 1 week ago, but when she was re­evaluated by her
primary care physician 2 days ago, the child was still having fevers and her left tympanic membrane was ruptured. A significant
amount of purulent drainage was noted in the left external auditory canal. The right tympanic membrane remained erythematous,
with a layer of pus visible behind it. The primary care physician obtained a blood culture, a complete blood cell count, and a culture
of the left ear drainage, then administered a dose of intramuscular ceftriaxone. On presentation to your office the next day, the child
Claim Credit continued to have fever and the drainage from the left ear continued. Laboratory results from the prior day revealed a normal
complete blood cell count, negative blood culture for 1 day, and an ear culture growing 3+ Streptococcus pneumoniae. You consider
Evaluation
performing additional diagnostic studies.
My Learning Plan
Of the following test results, an explanation for the persistence of the child’s current infection would MOST likely be revealed by

A. antibiotic susceptibility of the S pneumoniae
B. computed tomography of the mastoids bilaterally
C. serum quantitative immune globulin assay
D. ultrasonography of the spleen

Correct View Peer Results

Average Percent Correct:  72.82%

Antibiotic resistance is the most likely explanation for this child’s persistent infection. Each of the earlier
bouts of otitis media was successfully treated with amoxicillin, but the current infection persisted
despite changing from amoxicillin to an oral cephalosporin (cefdinir), and then to a parenteral
cephalosporin (ceftriaxone). The culture obtained from the external ear canal revealed 3+
Streptococcus pneumoniae, despite empiric ß­lactam antibiotic treatment. Now that a culture result is
available, antibiotic susceptibility results would be expected to reveal that the isolate is penicillin­ and
cephalosporin­resistant. Susceptibility results for other antibiotic classes will be most helpful in guiding
further treatment with an appropriate antibiotic.

Results from computed tomography of the mastoids would not reveal the reason for the persistent infection, but may be
necessary in some patients who show clinical evidence of mastoiditis as a complication of the middle ear infection. The
patient described in the vignette has a history consistent with eustachian tube dysfunction, however, mastoid imaging is not
useful in confirming that possibility. Patients with recurrent or persistent sinopulmonary infections may require diagnostic
testing for humoral immune deficiency, including measurement of serum immunoglobulin concentrations. Abnormal results
would typically be low yield in the patient described, as she is otherwise well, has not had recurrent or severe lower
respiratory tract infections, and has previously responded quite well to typical courses of oral antibiotics for her middle ear
infections. Ultrasonography of the spleen, to determine whether asplenia explains recurrent pneumococcal disease, is best
reserved as part of the evaluation of patients who develop recurrent pneumococcal bacteremia, not persistent pneumococcal
otitis media.

Persistent or recurrent pneumococcal infection in a child warrants special attention, especially when the infections progress to
complications, or when bacteremia recurs. A complete medical history and physical examination may point to the possibility of
a primary immune deficiency, an anatomic abnormality, or an underlying chronic illness as the primary risk factor. Even when
a specific risk factor for the persistence or recurrence of the infection is identified, exploring the possibility that antibiotic failure
has contributed to the persistence or recurrence of the infection is important. Lack of patient or parent adherence to

http://2016.prepid.courses.aap.org/script/january?req=201612121929378833 1/2
12/13/2016 January
previously prescribed treatments should also be considered while other causes of treatment failure are explored. A careful
review of microbiologic data (when available) may indicate that the pneumococcus responsible for the infection is resistant to
the antibiotic selected. In a child who is otherwise healthy, daycare attendance, young age, or recent antibiotic treatment may
be important clues that the infection has recurred or persisted because of antimicrobial resistance.

Any known condition that predisposes to invasive infection with pneumococcus may be immediately evident by history, such
as asplenia, HIV infection, diabetes mellitus, or humoral immune deficiency. When underlying risk factors for recurrent or
persistent pneumococcal infections are not immediately obvious, further diagnostic evaluation may be warranted. For
example, a child who has developed a persistent pneumococcal otitis media that also has a history of frequent recurrent and
severe sinopulmonary infections requiring lengthy courses of antibiotic treatment should be evaluated for a humoral immune
deficiency. A history of frequent exposure to antibiotics in the past identifies any current pneumococcal infection as higher risk
for being caused by an antibiotic resistant strain. Recurrent or persistent pneumococcal infections may also be explained by
anatomic defects. The child described has a history typical for eustachian tube dysfunction, as each infection was localized to
the middle ear, but a child with a history of recurrent otitis media, otorrhea, and known sensorineural hearing loss who
presents with recurrent pneumococcal meningitis may have an underlying anatomic defect such as Mondini dysplasia.

Many children with recurrent or persistent pneumococcal infections have healthy immune systems and normal anatomy, but
are unlucky to have been infected with a pneumococcal strain that is resistant to the medications most commonly used.
Otherwise healthy children are at increased risk for antibiotic resistant pneumococcal infection if they have been treated
recently with antibiotics. As mentioned, other risk factors for antibiotic resistant pneumococcal infection in this group of
patients include daycare attendance and young age.

When evaluating a child with recurrent or persistent pneumococcal infection, it is important to review available antimicrobial
susceptibility testing on laboratory isolates. Careful review of those susceptibility data, in the context of prior treatment
prescribed, will usually allow a quick assessment of whether antibiotic failure is responsible for the ongoing illness and
provide options to which the isolate is susceptible. Particular care in antibiotic choice is important when switching from empiric
to definitive treatment of pneumococcal meningitis, as the interpretation of some susceptibility data rely on different
breakpoint minimal inhibitory concentrations (MICs) for meningeal infections and nonmeningeal infections.

In most instances, antimicrobial resistance for a drug class is absolute, however, resistance for pneumococcus to penicillin
occurs in a graded fashion so that, depending on the MIC of penicillin, therapy may still prove successful by using higher
doses, by switching from oral to parenteral therapy, or by using an alternative ß­lactam drug. This can be explained by the
mechanism of ß­lactam resistance seen in pneumococci. ß­lactam antibiotics interfere with bacterial replication by binding to
and interfering with the activity of penicillin­binding proteins (PBPs). These transpeptidases are important for cell wall
biosynthesis, and when their function is impaired in the presence of a ß­lactam antibiotic, treatment is successful. Alterations
can occur in PBPs expressed by pneumococci through a series of mutations that lower their affinity for penicillin, and when
sufficient changes have occurred, the affinity to bind other ß­lactams is also reduced. When the affinity is altered sufficiently
that the antibiotic can no longer bind to and interfere with the PBP activity, the antibiotic fails. The extent of the changes in the
PBPs will dictate how susceptible (or resistant) the isolate is to each of the ß­lactam drugs. If there are sufficient alterations in
the PBPs, pneumococcus will become fully resistant to all members of this antibiotic class. Fortunately, to date, neither
vancomycin resistance nor acquisition of ß­lactam resistance via expression of a ß­lactamase have been described in
pneumococci.

PREP Pearls
Children with persistent or recurrent pneumococcal infection warrant consideration for further diagnostic testing, and careful
review of available antibiotic susceptibility data.

Young age, childcare center attendance, and recent treatment with antibiotics are known risk factors for the development of
antibiotic resistant pneumococcal infections.

Many strains of pneumococci remain exquisitely susceptible to penicillin, while others have emerged that are resistant to all
ß­lactam antibiotics. Pneumococci that are ß­lactam­resistant are more likely to be resistant to other antibiotic classes.

American Board of Pediatrics Content Specification(s)
Know the importance of investigating antimicrobial resistance by pneumococcus as the cause of persistent or recurrent
infection (eg, patients with upper or lower respiratory tract infections or those with meningitis or acute otitis media)

Understand the factors that predispose a patient to infection with an antibiotic resistant Streptococcus pneumoniae
infection (child‑care center attendance, young age, recent antibiotic treatment)

Suggested Readings
Alsina L, Basteiro MG, de Paz HD, et al. Recurrent invasive pneumococcal disease in children: underlying clinical conditions,
and immunological and microbiological characteristics. PLoS One. 2015;10(3):e0118848. doi:
http://dx.doi.org/10.1371/journal.pone.0118848

Cornick JE, Bentley SD. Streptococcus pneumoniae: the evolution of antimicrobial resistance to beta­lactams,
fluoroquinolones and macrolides. Microbes Infect. 2012;14(7­8):573­583. doi: http://dx.doi.org/10.1016/j.micinf.2012.01.012

Comment On This Question

Page 9 of 115

Contact Us
Links may open in separate window
Copyright © 2016   American Academy of Pediatrics. All rights reserved. Pop­up Blocker may need to be disabled

http://2016.prepid.courses.aap.org/script/january?req=201612121929378833 2/2
12/13/2016 January

Welcome  mohammed alsaiary [ Logout ]

Home PREP Pearls FAQs My Bookmarks CME Information

Overview Home  > January

Editorial Board
January   Enter Keyword Search
January
Question View:   All (8) Jump to Question
February
Print this Page Add to my Bookmarks Page 10 of 115
March

April Assessment History Mode: Learner  Exam 

May ASSESSMENT PROGRESS:  Total Questions:  8  Questions Answered:  2


June

July You currently have 6 questions unanswered in this assessment

August

September
Question: 3
October
The parents of one of your patients with HIV infection have just adopted another child from an orphanage in a resource­limited
November country. They call you because they were just informed that the child received immunizations prior to leaving for the United States
approximately 2 weeks ago and would like to know if they need to take any specific precautions.
December
Of the following, the vaccine that would necessitate that minimal contact be kept between the 2 children for at least another month is

A. mumps, measles, rubella
B. oral poliovirus
C. oral typhoid
Claim Credit
D. varicella zoster
Evaluation

My Learning Plan
Submit   Reset  

Page 10 of 115

Contact Us
Links may open in separate window
Copyright © 2016   American Academy of Pediatrics. All rights reserved. Pop­up Blocker may need to be disabled

http://2016.prepid.courses.aap.org/script/january?req=201612121930596507&q­page=3&status=submit 1/1
12/13/2016 January

Welcome  mohammed alsaiary [ Logout ]

Home PREP Pearls FAQs My Bookmarks CME Information

Overview Home  > January

Editorial Board
January   Enter Keyword Search
January
Question View:   All (8) Jump to Question
February
Print this Page Add to my Bookmarks Page 10 of 115
March

April Assessment History Mode: Learner  Exam 

May ASSESSMENT PROGRESS:  Total Questions:  8  Questions Answered:  3  Correct Answers:  2


June

July You currently have 5 questions unanswered in this assessment

August

September
Question: 3
October
The parents of one of your patients with HIV infection have just adopted another child from an orphanage in a resource­limited
November country. They call you because they were just informed that the child received immunizations prior to leaving for the United States
approximately 2 weeks ago and would like to know if they need to take any specific precautions.
December
Of the following, the vaccine that would necessitate that minimal contact be kept between the 2 children for at least another month is

A. mumps, measles, rubella
B. oral poliovirus
C. oral typhoid
Claim Credit
D. varicella zoster
Evaluation

My Learning Plan
Correct View Peer Results
Average Percent Correct:  83.50%

Household contacts of immunocompromised persons, including those with HIV infection, should not
receive oral poliovirus vaccine (OPV). If vaccination is necessary for any household member or close
contact, inactivated poliovirus vaccine should then be given. If OPV is given inadvertently, close
contact between the vaccine recipient and HIV­infected patient should be minimized for approximately
4 to 6 weeks after immunization. Household members should be counseled on practices that would
minimize exposure of the HIV­infected child to excreted OPV in the stool and educated on practices
such as appropriate hand hygiene.

Aside from OPV, family members living in the same household as an HIV­infected individual should receive all immunizations
indicated for their age, including annual influenza vaccine. An exception is if there is a specific contraindication for the
individual household member that prevents them from receiving the vaccine such as a previous history of anaphylaxis.

In general, it is safe to immunize household contacts with all inactivated and live attenuated vaccines such as mumps­
measles­rubella (MMR) and varicella­zoster virus (VZV). In fact, household contacts of immunocompromised individuals,
including those with HIV infection, should be immunized with both vaccines if they have no evidence of immunity to decrease
the likelihood that wild­type VZV or MMR viruses would be introduced into the household. Transmission of vaccine­strain VZV
from healthy individuals has been documented, however, there is no contraindication for vaccination of household contacts.
Similarly, healthy household contacts of an HIV­infected individual should receive the zoster vaccine (Zostavax®) if indicated.
If an immunized individual develops a rash, in the case of both vaccines, direct contact with the HIV­infected person without
evidence of immunity should be avoided for the duration of the rash.

Measles vaccine virus is not shed nor do individuals transmit the virus after immunization. Therefore, HIV­infected individuals
are not at risk of acquiring measles infection from the vaccine virus if household members receive the vaccine. Immunized
individuals also do not transmit mumps vaccine virus. On the other hand, small amounts of rubella virus are shed after
immunization, although no evidence of transmission of vaccine virus from immunized children has been found. Oral typhoid
vaccine (Ty21a) should not be given to immunocompromised individuals, including those with HIV infection. However, there is
no contraindication for vaccination with Ty21a for those living in the household with an immunocompromised patient. While
vaccine virus can be shed transiently in the stool of vaccine recipients, secondary transmission of vaccine virus has not been
documented.

Live­attenuated influenza vaccine (LAIV) may also be given to household contacts of individuals with HIV infection, as long as
the HIV­infected individual is not severely immunocompromised and the individuals receiving the vaccine have no
contraindications for its use. If a person caring for a severely immunocompromised individual does get the LAIV, then contact
with the immunocompromised individual should be avoided for 7 days after getting the vaccine. Live­attenuated yellow fever
virus vaccine may also be given to household contacts of immunocompromised patients if indicated for travel.

http://2016.prepid.courses.aap.org/script/january?req=201612121931234637 1/2
12/13/2016 January
No specific infection control measures are recommended for individuals living with an HIV­infected patient. Standard
precautions such as good hand washing should be followed to prevent transmission of common illnesses such as respiratory
tract or diarrheal diseases from family members to the immunocompromised individual. Universal precautions should also be
followed in the case of any injury, trauma, or nosebleeds resulting in an open wound or active blood loss. Items such as
razors and toothbrushes should not be shared. The risk of HIV transmission to a household member from a child who bites is
negligible because saliva usually does not contain any blood. HIV testing would be warranted only in the event of a human
bite from an HIV­infected individual if there was a break in the skin or evidence of blood in the saliva of the HIV­infected
individual.

PREP Pearls
Family members of patients with HIV infection should receive all inactivated vaccines for age.

Live­attenuated vaccines may also be given to household contacts of HIV­infected patients with the exception of oral
poliovirus vaccine.

When poliovirus vaccine is indicated, the inactivated poliovirus vaccine should be used.

Apart from good hand washing and standard precautions, there are generally no specific infection control measures for
family members of HIV­infected patients.

American Board of Pediatrics Content Specification(s)
Recommend infection control measures and vaccinations for family members of a patient with HIV infection

Suggested Readings
American Academy of Pediatrics. Measles. In: Kimberlin DW, Brady MT, Jackson MA, Long SS, eds. Red Book: 2015 Report
of the Committee on Infectious Diseases. 30th ed. Elk Grove Village, IL: American Academy of Pediatrics; 2015: 535­547.

American Academy of Pediatrics. Poliovirus infections. In: Kimberlin DW, Brady MT, Jackson MA, Long SS, eds. Red Book:
2015 Report of the Committee on Infectious Diseases. 30th ed. Elk Grove Village, IL: American Academy of Pediatrics; 2015:
644­650.

American Academy of Pediatrics. Varicella­Zoster infections. In: Kimberlin DW, Brady MT, Jackson MA, Long SS, eds. Red
Book: 2015 Report of the Committee on Infectious Diseases. 30th ed. Elk Grove Village, IL: American Academy of Pediatrics;
2015: 846­860.

Deshpandie AK, Jadhav SK, Bandivdekar AH. Possible transmission of HIV infection due to human bite. AIDS Res Ther.
2011;8:16.  DOI: http://dx.doi.org/10.1186/1742­6405­8­16

Richman KM, Rickman LS. The potential for transmission of human immunodeficiency virus through human bites. J Acquir
Immune Defic Syndr. 1993;6(4):402­406.

US Centers for Disease Control and Prevention. Live attenuated influenza vaccine (LAIV). US Centers for Disease Control
and Prevention website. http://www.cdc.gov/flu/about/qa/nasalspray.htm. Updated August 21, 2015

US Food and Drug Administration. Vitotif® (Typhoid Vaccine Live Oral TY21a) package insert. US Food and Drug
Administration website. http://www.fda.gov/downloads/BiologicsBloodVaccines/Vaccines/ApprovedProducts/UCM142807.pdf

Comment On This Question

Page 10 of 115

Contact Us
Links may open in separate window
Copyright © 2016   American Academy of Pediatrics. All rights reserved. Pop­up Blocker may need to be disabled

http://2016.prepid.courses.aap.org/script/january?req=201612121931234637 2/2
12/13/2016 January

Welcome  mohammed alsaiary [ Logout ]

Home PREP Pearls FAQs My Bookmarks CME Information

Overview Home  > January

Editorial Board
January   Enter Keyword Search
January
Question View:   All (8) Jump to Question
February
Print this Page Add to my Bookmarks Page 11 of 115
March

April Assessment History Mode: Learner  Exam 

May ASSESSMENT PROGRESS:  Total Questions:  8  Questions Answered:  3  Correct Answers:  2


June

July You currently have 5 questions unanswered in this assessment

August

September
Question: 4
October
The pediatric infectious diseases service is called by a pediatrician to discuss the immunization needs for an 11­year­old girl. She
November has multiple allergies that include immediate hypersensitivity to yeast and tree nuts. The girl is otherwise healthy. Her medications
include loratadine and epinephrine injection for use as needed.
December
Of the following, the vaccine that is MOST concerning to use in this girl is

A. human papilloma virus (HPV4 or HPV9)
B. live­attenuated influenza vaccine (LAIV)
C. meningococcal conjugate vaccine, quadrivalent (MCV4)
Claim Credit
D. tetanus, diphtheria, acellular pertussis (Tdap)
Evaluation

My Learning Plan
Submit   Reset  

Page 11 of 115

Contact Us
Links may open in separate window
Copyright © 2016   American Academy of Pediatrics. All rights reserved. Pop­up Blocker may need to be disabled

http://2016.prepid.courses.aap.org/script/january?q­page=4&req=201612121931239637 1/1
12/13/2016 January

Welcome  mohammed alsaiary [ Logout ]

Home PREP Pearls FAQs My Bookmarks CME Information

Overview Home  > January

Editorial Board
January   Enter Keyword Search
January
Question View:   All (8) Jump to Question
February
Print this Page Add to my Bookmarks Page 11 of 115
March

April Assessment History Mode: Learner  Exam 

May ASSESSMENT PROGRESS:  Total Questions:  8  Questions Answered:  4  Correct Answers:  3


June

July You currently have 4 questions unanswered in this assessment

August

September
Question: 4
October
The pediatric infectious diseases service is called by a pediatrician to discuss the immunization needs for an 11­year­old girl. She
November has multiple allergies that include immediate hypersensitivity to yeast and tree nuts. The girl is otherwise healthy. Her medications
include loratadine and epinephrine injection for use as needed.
December
Of the following, the vaccine that is MOST concerning to use in this girl is

A. human papilloma virus (HPV4 or HPV9)
B. live­attenuated influenza vaccine (LAIV)
C. meningococcal conjugate vaccine, quadrivalent (MCV4)
Claim Credit
D. tetanus, diphtheria, acellular pertussis (Tdap)
Evaluation

My Learning Plan
Correct View Peer Results
Average Percent Correct:  60.19%

The human papillomavirus vaccines (HPV2, HPV4, HPV9) are contraindicated in people with a history
of hypersensitivity to any vaccine component, yeast (HPV4 and HPV9), or latex (HPV2). The rubber
stopper in the prefilled HPV2 syringe contains latex and should not be administered to latex­sensitive
persons. The HPV vaccines are not recommended for use during pregnancy. The HPV4 and HPV9
vaccines may be administered to lactating women; the HPV2 vaccine was not studied in lactating
women. Vaccine providers, particularly when vaccinating adolescents, should be encouraged to
observe patients (with patients seated or lying down) for 15 minutes after vaccination to decrease the
risk for injury should they faint. If syncope develops, patients should be observed until symptoms resolve. The meningococcal
conjugate vaccine, quadrivalent (MCV4), live­attenuated influenza vaccine (LAIV), and tetanus, diphtheria, acellular pertussis
(Tdap) vaccines can safely be administered to the girl in the vignette.

Three HPV vaccines are licensed by the US Food and Drug Administration for use in the United States (Table). All 3 vaccines
contain noninfectious recombinant virus­like particles (VLPs) assembled from the major capsid L1 proteins of the HPV types
with an adjuvant. Adjuvants are commonly used in vaccines to accelerate, prolong, or enhance antigen­specific immune
responses. The HPV2 vaccine contains AS04, a proprietary adjuvant. Each of the L1 proteins in the HPV4 and HPV9
vaccines are produced by separate fermentations in recombinant Saccharomyces cerevisiae and self­assembled into VLPs.
The fermentation process involves growth of S cerevisiae on chemically­defined fermentation media that include vitamins,
amino acids, mineral salts, and carbohydrates. The VLPs are released from the yeast cells by cell disruption and purified by a
series of chemical and physical methods. The purified VLPs are adsorbed on preformed aluminum­containing adjuvant,
amorphous aluminum hydroxyphosphate sulfate.

* Required *
Take Survey  

PREP Pearls
Three human papillomavirus (HPV) vaccines are licensed by the US Food and Drug Administration and contain
noninfectious recombinant virus­like particles assembled from the major capsid L1 proteins of the HPV types with an
adjuvant.

The HPV2, HPV4, and HPV9 vaccines are contraindicated in people with a history of hypersensitivity to any vaccine
component, yeast (HPV4 and HPV9), or latex (HPV2).

http://2016.prepid.courses.aap.org/script/january?req=201612121932473405&status=submit 1/2
12/13/2016 January

American Board of Pediatrics Content Specification(s)
Understand the contraindications of the human papillomavirus (HPV) vaccine

Understand the components of the two available vaccines for the prevention of human papillomavirus (HPV)

Understand the components of the two available vaccines for the prevention of human papillomavirus (HPV)

Suggested Readings
American Academy of Pediatrics. Human papillomaviruses. In: Kimberlin DW, Brady MT, Jackson MA, Long SS, eds. Red
Book: 2015 Report of the Committee on Infectious Diseases. 30th ed. Elk Grove Village, IL: American Academy of Pediatrics;
2015: 576­583.

GlaxoSmithKline. Highlights of prescribing information: Cervarix. GlaxoSmithKline website.
https://www.gsksource.com/pharma/content/dam/GlaxoSmithKline/US/en/Prescribing_Information/Cervarix/pdf/CERVARIX­
PI­PIL.PDF

Merck. Highlights of prescribing information: Gardasil 9. Merck website.
https://www.merck.com/product/usa/pi_circulars/g/gardasil_9/gardasil_9_pi.pdf

Merck. Highlights of prescribing information: Gardasil. Merck website.
https://www.merck.com/product/usa/pi_circulars/g/gardasil/gardasil_pi.pdf

US Centers for Disease Control and Prevention. Human papillomavirus (HPV). US Centers for Disease Control and
Prevention website. http://www.cdc.gov/hpv/

Comment On This Question

Page 11 of 115

Contact Us
Links may open in separate window
Copyright © 2016   American Academy of Pediatrics. All rights reserved. Pop­up Blocker may need to be disabled

http://2016.prepid.courses.aap.org/script/january?req=201612121932473405&status=submit 2/2
12/13/2016 January

Welcome  mohammed alsaiary [ Logout ]

Home PREP Pearls FAQs My Bookmarks CME Information

Overview Home  > January

Editorial Board
January   Enter Keyword Search
January
Question View:   All (8) Jump to Question
February
Print this Page Add to my Bookmarks Page 12 of 115
March

April Assessment History Mode: Learner  Exam 

May ASSESSMENT PROGRESS:  Total Questions:  8  Questions Answered:  4  Correct Answers:  3


June

July You currently have 4 questions unanswered in this assessment

August

September
Question: 5
October
A previously healthy 4­month­old female infant presents with 2 days of fever of 38.8°C, increased fussiness, and decreased oral
November intake. She is ill­appearing and a complete sepsis evaluation is undertaken in the emergency department. Fluid resuscitation is
administered and parenteral ceftriaxone is initiated empirically prior to admission to the inpatient unit. Urinalysis is consistent with
December
urinary tract infection (50 white blood cells/high power field, 10 red blood cells/high power field, positive leukocyte esterase, positive
nitrite) and her initial blood culture reveals a Gram­negative rod within 12 hours of hospital admission.

Of the following, the imaging study that is MOST likely to be beneficial in the management of this patient is

A. computed tomography of the abdomen

Claim Credit B. renal and bladder ultrasonography

Evaluation C. technetium nuclear scan
D. voiding cystourethrogram
My Learning Plan

Submit   Reset  

Page 12 of 115

Contact Us
Links may open in separate window
Copyright © 2016   American Academy of Pediatrics. All rights reserved. Pop­up Blocker may need to be disabled

http://2016.prepid.courses.aap.org/script/january?q­page=5&req=201612121932479030 1/1
12/13/2016 January

Welcome  mohammed alsaiary [ Logout ]

Home PREP Pearls FAQs My Bookmarks CME Information

Overview Home  > January

Editorial Board
January   Enter Keyword Search
January
Question View:   All (8) Jump to Question
February
Print this Page Add to my Bookmarks Page 12 of 115
March

April Assessment History Mode: Learner  Exam 

May ASSESSMENT PROGRESS:  Total Questions:  8  Questions Answered:  5  Correct Answers:  4


June

July You currently have 3 questions unanswered in this assessment

August

September
Question: 5
October
A previously healthy 4­month­old female infant presents with 2 days of fever of 38.8°C, increased fussiness, and decreased oral
November intake. She is ill­appearing and a complete sepsis evaluation is undertaken in the emergency department. Fluid resuscitation is
administered and parenteral ceftriaxone is initiated empirically prior to admission to the inpatient unit. Urinalysis is consistent with
December
urinary tract infection (50 white blood cells/high power field, 10 red blood cells/high power field, positive leukocyte esterase, positive
nitrite) and her initial blood culture reveals a Gram­negative rod within 12 hours of hospital admission.

Of the following, the imaging study that is MOST likely to be beneficial in the management of this patient is

A. computed tomography of the abdomen

Claim Credit B. renal and bladder ultrasonography

Evaluation C. technetium nuclear scan
D. voiding cystourethrogram
My Learning Plan

Correct View Peer Results
Average Percent Correct:  79.61%

The infant in the vignette has urosepsis with a Gram­negative organism. She is previously healthy
without history of urinary tract infection. Given that this is a first time urinary tract infection, the patient
should undergo renal and bladder ultrasonography to evaluate for anatomic abnormalities that may
require further evaluation. The timing of the renal ultrasound is dependent upon the clinical severity of
the child’s illness. In the setting of suspected renal abscess, obstructive uropathy, or failure to improve
within 48 hours of initiation of therapy, it is prudent to obtain an ultrasound early in the course.
However, in infants who exhibit a quick clinical response, renal ultrasonography can be safely delayed
and, if obtained too early, may be misleading, owing to edema of the renal parenchyma or dilation of the renal collecting
system caused by bacterial endotoxin release. Computed tomography (CT) is rarely indicated in the setting of urinary tract
infection. Inappropriate use of CT results in unnecessary radiation exposure with little to no patient benefit. However, CT can
be useful in the setting of suspected abscess that cannot be confirmed by ultrasonography. Nuclear scan with technetium­
labeled dimercaptosuccinic acid (DMSA) has a higher sensitivity for detecting pyleonephritis than renal ultrasonography. The
DMSA scan also has the advantage of identifying renal scarring and is useful in research studies to ensure that all subjects
meet inclusion criteria. However, findings on DMSA scans rarely affect acute clinical management and are not indicated in
children with a first­time urinary tract infection (UTI). Voiding cystourethrogram (VCUG) should be obtained after a first febrile
UTI only if abnormalities (eg, hydronephrosis or congenital abnormality) are found on the ultrasound. This recommendation is
based on a pooled analysis of 1,100 infants between 2 and 24 months of age with known vesicoureteral reflux (VUR). The
data showed that the majority of infants with VUR have low­grade disease and that the infants are unlikely to benefit from
antibiotic prophylaxis, which does not prevent renal scarring.

Following the publication of the American Academy of Pediatrics’ febrile infant UTI clinical practice guideline, a randomized
controlled trial was conducted to determine if antibiotic prophylaxis is beneficial for children with VUR after a first or second
febrile UTI. While the data showed that antibiotic prophylaxis does decrease the risk of subsequent UTI, it does so without the
benefit of prevention of renal scarring. Further, when infection does occur, the risk of infection with a resistant organism is
increased. Some controversy remains among physicians who care for infants with VUR regarding whether antibiotic
prophylaxis is indicated for those with high­grade VUR; open communication and discussion among infectious diseases
physicians, nephrologists, and/or urologists is essential in the optimal management of infants and children with high­grade
VUR.

Choosing empiric treatment for UTI is largely dependent upon patient­related factors. In infants, parenteral therapy is often
initiated in the setting of febrile UTI and the inability to maintain adequate oral intake. Clinical appearance and objective
measures of sepsis are also major determining factors. For older, well­appearing infants and young children without high fever

http://2016.prepid.courses.aap.org/script/january?req=201612121934040296&status=submit 1/2
12/13/2016 January
and in whom adequate oral intake is achievable, it is reasonable to initiate enteral therapy and arrange follow­up to assess
clinical improvement and culture results. Commonly used empiric antibiotics for UTI are listed in the Table. Other factors that
should come into consideration include contact with a person who is known to be colonized with or has had infection caused
by a multidrug­resistant Gram­negative organism (eg, an extended spectrum β­lactamase producing Escherichia coli), recent
international travel, or frequent exposure to antibiotics in the past. Along with risk factor assessment, empiric therapy should
be based on the local antibiogram. Definitive therapy is based on culture results, and transition from parenteral to enteral
formulations can be considered when clear clinical improvement has been made, bacteremia (when present) has cleared, and
the patient has adequate oral intake. The clinical practice guideline for diagnosis and management of febrile UTI in infants
and children 2 to 24 months of age suggests that a minimum of 7 days should be provided, based on evidence of inferiority
with shorter therapy courses. However, there are no comparative trials that have examined the optimal duration of therapy.
Thus, it is recommended that therapy be continued between 7 and 14 days’ duration.

PREP Pearls
Infants and children with a first­time febrile urinary tract infection (UTI) do not need to undergo voiding cystourethrogram
unless the renal and bladder ultrasonography reveals an abnormality.

Renal and bladder ultrasonography should be obtained in all infants and children with first­time febrile UTI.

Antibiotic choice, route of administration, and therapy length should be dependent upon clinical status of the patient, risk
factors for multidrug­resistant infection, and clinical response to therapy.

American Board of Pediatrics Content Specification(s)
Know the appropriate imaging studies in patients with suspected or proven urinary tract infection

Plan the appropriate treatment of urinary tract infection (drugs, duration of treatment, prophylaxis), including initial empiric
therapy

Suggested Readings
American Academy of Pediatrics. Urinary tract infection: clinical practice guideline for the diagnosis and management of the
initial UTI in febrile infants and children 2 to 24 months. Pediatrics. 2011;128(3):595­610. doi:
http://dx.doi.org/10.1542/peds.2011­1330

Hoberman A, Greenfield SP, Matoo TK, et al. Antimicrobial prophylaxis for children with vesicoureteral reflux. N Engl J Med.
2014;370(25):2367­2376. doi: http://dx.doi.org/10.1056/NEJMoa1401811

Comment On This Question

Page 12 of 115

Contact Us
Links may open in separate window
Copyright © 2016   American Academy of Pediatrics. All rights reserved. Pop­up Blocker may need to be disabled

http://2016.prepid.courses.aap.org/script/january?req=201612121934040296&status=submit 2/2
12/13/2016 January

Welcome  mohammed alsaiary [ Logout ]

Home PREP Pearls FAQs My Bookmarks CME Information

Overview Home  > January

Editorial Board
January   Enter Keyword Search
January
Question View:   All (8) Jump to Question
February
Print this Page Add to my Bookmarks Page 13 of 115
March

April Assessment History Mode: Learner  Exam 

May ASSESSMENT PROGRESS:  Total Questions:  8  Questions Answered:  5  Correct Answers:  4


June

July You currently have 3 questions unanswered in this assessment

August

September
Question: 6
October
You are seeing 2 refugee boys in your clinic, one from a country in sub­Saharan Africa, the other from southeast Asia. The boys
November appear well and are said to have not experienced any recent illnesses prior to their migration.

December Of the following, the screening test you would ONLY order for the boy from sub­Saharan Africa compared to the other boy is

A. blood smear for malaria
B. interferon­γ release assay for tuberculosis
C. nontreponemal test for syphilis

Claim Credit D. stool examination for ova and parasites

Evaluation

My Learning Plan Submit   Reset  

Page 13 of 115

Contact Us
Links may open in separate window
Copyright © 2016   American Academy of Pediatrics. All rights reserved. Pop­up Blocker may need to be disabled

http://2016.prepid.courses.aap.org/script/january?q­page=6&req=201612121934044984 1/1
12/13/2016 January

Welcome  mohammed alsaiary [ Logout ]

Home PREP Pearls FAQs My Bookmarks CME Information

Overview Home  > January

Editorial Board
January   Enter Keyword Search
January
Question View:   All (8) Jump to Question
February
Print this Page Add to my Bookmarks Page 13 of 115
March

April Assessment History Mode: Learner  Exam 

May ASSESSMENT PROGRESS:  Total Questions:  8  Questions Answered:  6  Correct Answers:  5


June

July You currently have 2 questions unanswered in this assessment

August

September
Question: 6
October
You are seeing 2 refugee boys in your clinic, one from a country in sub­Saharan Africa, the other from southeast Asia. The boys
November appear well and are said to have not experienced any recent illnesses prior to their migration.

December Of the following, the screening test you would ONLY order for the boy from sub­Saharan Africa compared to the other boy is

A. blood smear for malaria
B. interferon­γ release assay for tuberculosis
C. nontreponemal test for syphilis

Claim Credit D. stool examination for ova and parasites

Evaluation

My Learning Plan Correct View Peer Results


Average Percent Correct:  56.31%

Malaria caused by Plasmodium falciparum is very common in sub­Saharan Africa (SSA). Current
practice for a refugee from a SSA country that is endemic for P falciparum is to treat presumptively with
an artesunate­combination therapy (ACT) for asymptomatic malaria prior to departure. If this was not
done for some reason, screening for malaria should be conducted upon arrival to the United States.
This does not apply to refugees outside SSA (eg, southeast Asia), who should not receive empirical
therapy. Therefore, for the child in the vignette from SSA, the unique screening consideration is a test
for malaria. All the other screening tests listed (for ova and parasites, syphilis, tuberculosis) should be
done routinely for all refugees.

Thousands of internationally adopted, refugee, and immigrant children arrive in the United States each year. There is wide
variability in the availability of healthcare in their countries of origin. In addition, children may have lived for extended periods
in refugee camps or under unsanitary conditions en route to the United States. For these reasons, medical evaluation of these
children is urgent, particularly to screen for infectious diseases and to catch­up on immunizations. Adopted children need
similar comprehensive evaluation, as the pre­immigration visa examination is limited to screening for certain infectious
diseases. Moreover, information provided by the agency may be limited or unreliable. HIV and hepatitis infections are more
likely in children adopted from orphanages or foster care. The long­term consequences of undiagnosed infectious diseases
include chronic infection, growth stunting, developmental delay, and even death.

All internationally adopted, refugee, and immigrant children should be screened for immunizations. The child should be
assumed to be unvaccinated for any vaccine not documented in the medical paperwork. The written documentation must be
clear as to dates of administration, intervals between doses, and the child's age at the time of vaccination. If the vaccine
sequences did not conform to accepted US Advisory Committee on Immunization Practices (ACIP) or state schedules, the
child should be revaccinated according to the catch­up schedule. While laboratory evidence of immunity to individual
infectious agents is an acceptable alternative, it is usually more expedient to administer the age­appropriate vaccines.

The Table contains a list of screening tests for infectious diseases for internationally adopted, refugee, and immigrant
children. Refugee children warrant evaluation and possible treatment that exceeds that recommended for adopted and
immigrant children.

PREP Pearls
There are differences in the screening needs of international adoptee, immigrant, and refugee children.

Sub­Saharan Africa refugees should have received presumptive treatment for falciparum malaria prior to departure.

Risk factors dictate need for screening for sexually transmitted infections and bloodborne viruses.

http://2016.prepid.courses.aap.org/script/january?req=201612121935400005&status=submit 1/2
12/13/2016 January

American Board of Pediatrics Content Specification(s)
Know what medical evaluation (including screening) internationally adopted and immigrant children should receive

Recognize the long‑term consequences of infectious diseases that infect internationally adopted and immigrant children

Suggested Readings
American Academy of Pediatrics. Medical evaluation for infectious diseases for internationally adopted, refugee, and
immigrant children. In: Kimberlin DW, Brady MT, Jackson MA, Long SS, eds. Red Book: 2015 Report of the Committee on
Infectious Diseases. 30th ed. Elk Grove Village, IL: American Academy of Pediatrics; 2015: 194­201.

US Centers for Disease Control and Prevention. Refugee health guidelines: guidelines for pre­departure and post­arrival
medical screening and treatment of U.S.­bound refugees. US Centers for Disease Control and Prevention website.
http://www.cdc.gov/immigrantrefugeehealth/guidelines/refugee­guidelines.html. Updated November 12, 2013

Comment On This Question

Page 13 of 115

Contact Us
Links may open in separate window
Copyright © 2016   American Academy of Pediatrics. All rights reserved. Pop­up Blocker may need to be disabled

http://2016.prepid.courses.aap.org/script/january?req=201612121935400005&status=submit 2/2
12/13/2016 January

Welcome  mohammed alsaiary [ Logout ]

Home PREP Pearls FAQs My Bookmarks CME Information

Overview Home  > January

Editorial Board
January   Enter Keyword Search
January
Question View:   All (8) Jump to Question
February
Print this Page Add to my Bookmarks Page 14 of 115
March

April Assessment History Mode: Learner  Exam 

May ASSESSMENT PROGRESS:  Total Questions:  8  Questions Answered:  6  Correct Answers:  5


June

July You currently have 2 questions unanswered in this assessment

August

September
Question: 7
October
A community pediatrician contacts you stating that a sales representative from a vaccine manufacturer is urging her to provide
November meningococcus serogroup B vaccine for her patients. She asks you if this is a good idea. In researching the topic, you find an
analysis performed by the US Centers for Disease Control and Prevention showing that the cost per quality­adjusted life­year
December
(QALY) is $8,700,000 if the immunization series is started at 11 years of age and $4,100,000 if started at 16 years of age.

Of the following, the MOST accurate interpretation of these figures is that

A. meningococcal disease mortality is lower for adolescents than for pre­teenagers
B. net costs for length and quality of life are lower for the 16­year­old immunization group
Claim Credit C. QALYs are cheaper per year for adolescents than for pre­teenagers
Evaluation D. the cost of 1 year of life lost is ($8,700,000 – $4,100,000)/(16­11) = $920,000

My Learning Plan

Submit   Reset  

Page 14 of 115

Contact Us
Links may open in separate window
Copyright © 2016   American Academy of Pediatrics. All rights reserved. Pop­up Blocker may need to be disabled

http://2016.prepid.courses.aap.org/script/january?q­page=7&req=201612121935404848 1/1
12/13/2016 January

Welcome  mohammed alsaiary [ Logout ]

Home PREP Pearls FAQs My Bookmarks CME Information

Overview Home  > January

Editorial Board
January   Enter Keyword Search
January
Question View:   All (8) Jump to Question
February
Print this Page Add to my Bookmarks Page 14 of 115
March

April Assessment History Mode: Learner  Exam 

May ASSESSMENT PROGRESS:  Total Questions:  8  Questions Answered:  7  Correct Answers:  5


June

July You currently have 1 questions unanswered in this assessment

August

September
Question: 7
October
A community pediatrician contacts you stating that a sales representative from a vaccine manufacturer is urging her to provide
November meningococcus serogroup B vaccine for her patients. She asks you if this is a good idea. In researching the topic, you find an
analysis performed by the US Centers for Disease Control and Prevention showing that the cost per quality­adjusted life­year
December
(QALY) is $8,700,000 if the immunization series is started at 11 years of age and $4,100,000 if started at 16 years of age.

Of the following, the MOST accurate interpretation of these figures is that

A. meningococcal disease mortality is lower for adolescents than for pre­teenagers
B. net costs for length and quality of life are lower for the 16­year­old immunization group
Claim Credit C. QALYs are cheaper per year for adolescents than for pre­teenagers
Evaluation D. the cost of 1 year of life lost is ($8,700,000 – $4,100,000)/(16­11) = $920,000

My Learning Plan

Incorrect View Peer Results
Correct Answer: B Average Percent Correct:  58.25%

Quality­adjusted life­year (QALY) is an estimation that assigns a value, usually characterized as a
number between 0 (death) and 1 (perfect health), combining both quality and length of life into a single
outcome. In some health economic studies, assumptions are made to provide actual dollar values for a
QALY, which can then be utilized to compare costs and benefits of an intervention such as vaccination.
Thus, a QALY is not just a measure of mortality, and therefore response choices A and D are incorrect
because they focus only on mortality. Also, because of the many assumptions and variables used in
determining cost per QALY, response choice C is much too simplified and is incorrect. Response
choice B essentially restates the meaning of QALY, with the understanding that many variables are included in the complete
assessment of cost per QALY.

Cost­effectiveness analyses (CEAs) differ in significant ways from cost­benefit analyses (CBAs), though sometimes these
terms are used interchangeably (and incorrectly) in some studies. So it is understandable that many clinicians are confused
by these terms.

Most simply stated, a CEA is a unidirectional study looking at either quantity or quality of life changed with a particular
intervention. Stated differently, a CEA identifies and assigns dollar amounts to the costs of a program and relates this to
program effectiveness or outcomes. The CBA is a more complex analysis, looking at both quantity and quality of life, as well
as more subjective issues such as society’s willingness to pay for specific outcomes. It usually will assign monetary figures to
outcomes, such as determining cost per QALY. The CBA is the only type of study that ultimately addresses whether the total
benefits of an intervention outweigh the total costs. Some researchers also define an intermediate type of study, the cost­
utility analysis (CUA), or term CUA as a subset of CEA. The CUAs incorporate measures of both quantity and quality of life
benefits, including use of QALY methodology, but does not address overall societal benefits as thoroughly as a CBA. The
discussion of meningococcal vaccine in the vignette is a component of a CBA.

Although CEAs and CBAs are very useful for healthcare economic decisions, they are limited by the complexity of the
assumptions made. Changing the assumptions may change economic forecasts dramatically. Given that much is unknown
about performance of meningococcal B vaccines in large scale use, major assumptions used in the example included
applying historical incidence and case­fatality rates to the current analysis, assigning initial vaccine effectiveness of 85% to
95% with waning of protection over 10 years, and cost of vaccine series (including vaccine itself, administration, adverse
event costs, and wastage) at $402.

Another important issue for CEAs and CBAs is the difference in perspectives of groups involved, which include not only the
individual vaccine recipient, but also society, third­party payors, and caregivers. For example, some researchers have

http://2016.prepid.courses.aap.org/script/january?req=201612121937108618&status=submit 1/2
12/13/2016 January
suggested that QALYs need modification when utilized in small children, where health benefits and costs of caregivers may
take on greater meaning.

For assessing quality of CEAs and CBAs, reports should contain clear statements regarding:

1. Perspective of the analysis (eg, individual, society)
2. Benefits of the interventions
3. Cost components
4. Timing of costs and benefits (eg, vaccine costs are high early, benefits increase later)
5. Sensitivity analyses for all assumptions
6. Summary measurement (eg, cost per QALY)

Making a checklist of these items can help clinicians decide how to advise their patients on new interventions such as
meningococcal B vaccine. Currently, the meningococcal B vaccine can be considered for adolescents and young adults 16 –
23 years of age (preferred age 16 – 18 years) to provide short term protection against most strains of serogroup B disease.
Note that meningococcal B vaccine is recommended for individuals > 10 years of age in high risk groups, including persistent
complement deficiencies, use of eculizumab, and functional or anatomic asplenia, and in meningococcal B outbreaks.

* Required *
Take Survey  

PREP Pearls
A cost­effectiveness analysis compares costs of an intervention to a predefined health outcome. A cost­benefit analysis
extends this analysis to assign a monetary value to the outcomes, such that total costs can be weighed in economic terms.

Quality­adjusted life­year is a measure that combines both quality and length of life into a single outcome summarized by a
number rating.

American Board of Pediatrics Content Specification(s)
Differentiate cost­benefit from cost­effectiveness analysis

Understand how quality‑adjusted life years are used in cost analyses

Understand the multiple perspectives (eg, of an individual, payor, society)that influence interpretation of cost­benefit and
cost­effectiveness analyses

Suggested Readings
Griebsch I, Coast J, Brown J. Quality­adjusted life­years lack quality in pediatric care: a critical review of published cost­utility
studies in child health. Pediatrics. 2005;115(5):e600­e614..

MacNeil J. Considerations for use of serogroup B meningococcal (MenB) vaccines in adolescents. US Centers for Disease
Control and Prevention website. http://www.cdc.gov/vaccines/acip/meetings/downloads/slides­2015­06/mening­03­
macneil.pdf. Updated July 14, 2015

MacNeil JR, et al. Use of serogroup B meningococcal vaccines in adolescents and young adults: recommendations of the
Advisory Committee on Immunization Practices. 2015. Morbid Mortal Weekly Rep. 2015;64(41):1171­1176..

Marshall JK, Cawdron R, Yamamura DLR, Ganguli S, Lad R, O'Brien BJ. Use and misuse of cost­effectiveness terminology in
the gastroenterology literature: a systematic review. Am J Gastroenterol. 2002;97(1):172­179. doi:
http://dx.doi.org/10.1111/j.1572­0241.2002.05433.x

Udvarhelyi IS, Colditz GA, Rai A, Epstein AM. Cost­effectiveness and cost­benefit analyses in the medical literature: are the
methods being used correctly? Ann Intern Med. 1992;116(3):238­244. doi: http://dx.doi.org/10.7326/0003­4819­116­3­238

Weinstein MC, Siegel JE, Gold MR, Kamlet MS, Russell LB. Recommendations of the panel on cost­effectiveness in health
and medicine. JAMA. 1996;276(15):1253­1258. doi: http://dx.doi.org/10.1001/jama.276.15.1253

Comment On This Question

Page 14 of 115

Contact Us
Links may open in separate window
Copyright © 2016   American Academy of Pediatrics. All rights reserved. Pop­up Blocker may need to be disabled

http://2016.prepid.courses.aap.org/script/january?req=201612121937108618&status=submit 2/2
12/13/2016 January

Welcome  mohammed alsaiary [ Logout ]

Home PREP Pearls FAQs My Bookmarks CME Information

Overview Home  > January

Editorial Board
January   Enter Keyword Search
January
Question View:   All (8) Jump to Question
February
Print this Page Add to my Bookmarks Page 15 of 115
March

April Assessment History Mode: Learner  Exam 

May ASSESSMENT PROGRESS:  Total Questions:  8  Questions Answered:  7  Correct Answers:  5


June

July You currently have 1 questions unanswered in this assessment

August

September
Question: 8
October
You are seeing an unimmunized 15­month­old child in clinic after a prolonged hospitalization for tetanus. The child received tetanus
November immune globulin and metronidazole followed by tetanus vaccine prior to hospital discharge. The parents of the child are now
interested in having this child and the rest of their children immunized with the routine childhood vaccine series.
December
Of the following, the MOST accurate response regarding the timing of rubella vaccine following receipt of tetanus immune globulin is
that

A. no delay is necessary
B. rubella vaccine should be delayed for 3 months
Claim Credit C. rubella vaccine should be delayed for 6 months
Evaluation D. rubella vaccine should be delayed for 11 months

My Learning Plan

Submit   Reset  

Page 15 of 115

Contact Us
Links may open in separate window
Copyright © 2016   American Academy of Pediatrics. All rights reserved. Pop­up Blocker may need to be disabled

http://2016.prepid.courses.aap.org/script/january?q­page=8&req=201612121937116118 1/1
12/13/2016 January

Welcome  mohammed alsaiary [ Logout ]

Home PREP Pearls FAQs My Bookmarks CME Information

Overview Home  > January

Editorial Board
January   Enter Keyword Search
January
Question View:   All (8) Jump to Question
February
Print this Page Add to my Bookmarks Page 15 of 115
March

April Assessment History Mode: Learner  Exam 

May ASSESSMENT PROGRESS:  Total Questions:  8  Questions Answered:  8  Correct Answers:  6


June

July You currently have 2 required survey(s) that are incomplete. Please complete the survey(s) for the following
question(s) on this assessment:  Question 4, Question 7
August

September

October Question: 8
November You are seeing an unimmunized 15­month­old child in clinic after a prolonged hospitalization for tetanus. The child received tetanus
immune globulin and metronidazole followed by tetanus vaccine prior to hospital discharge. The parents of the child are now
December
interested in having this child and the rest of their children immunized with the routine childhood vaccine series.

Of the following, the MOST accurate response regarding the timing of rubella vaccine following receipt of tetanus immune globulin is
that

A. no delay is necessary

Claim Credit B. rubella vaccine should be delayed for 3 months

Evaluation C. rubella vaccine should be delayed for 6 months
D. rubella vaccine should be delayed for 11 months
My Learning Plan

Correct View Peer Results

Average Percent Correct:  57.28%

The suggested interval between receipt of immune globulin and vaccination are the same for measles,
mumps, and rubella (MMR), and vary in length based on the dose and type of immune globulin
received (Table 1). In the setting of tetanus prophylaxis or treatment with tetanus immune globulin
(TIG), the interval between immune globulin receipt and rubella­containing vaccine in the form of MMR
should be 3 months. No vaccine delay is needed in the setting respiratory syncytial virus prophylaxis
with palivizumab, as this is a monoclonal antibody and should not interfere with development of
immunity to measles, mumps, or rubella. Similarly, no delay is needed in the setting of receipt of
washed red blood cells. Rubella­containing vaccine should be delayed for 4 months following receipt of rabies immune
globulin as postexposure prophylaxis. Receipt of MMR vaccine should be delayed for 6 months in those who received
botulinum immune globulin, whole blood transfusion, or high­dose measles immune globulin. Additionally, MMR vaccine
should be delayed for 11 months in children who receive between 1.6 to 2 g/kg of intravenous immune globulin intravenous
for idiopathic thrombocytopenic purpura or Kawasaki disease. The rationale for delaying MMR vaccine in these circumstances
is related to the decreased likelihood of developing an adequate immune response if given too early.

Other contraindications to rubella immunization include history of anaphylaxis to neomycin, history of severe allergic reaction
to any component of the vaccine, immune suppression due to malignancy or immune suppressive therapy related to another
disease process (eg, dermatomyositis), and pregnancy or individuals attempting to become pregnant (Table 2). Although
theoretically possible, congenital defects have never been reported in infants born to women who received vaccine early in
pregnancy. When possible, vaccine should be given to children 4 weeks prior to receipt of biologic modifiers for autoimmune
diseases (eg, Crohn disease).

The presence of a personal history or family history of seizures from any etiology is a precaution for vaccine containing
measles, mumps, rubella, and varicella (MMRV). Data suggests that, in the setting of personal or family history of febrile
seizures or epilepsy, there is an increased risk for febrile seizures after vaccination compared to children without such
histories. There is no contraindication for immunization in the setting of breastfeeding, asymptomatic or mildly symptomatic
HIV infection, household contacts of immune­compromised people, the child of a pregnant mother, or minor febrile illnesses.
However, the tuberculin skin test, when indicated, should be performed on the same day as MMR is given or should be
delayed 4 to 6 weeks, as the vaccine may interfere with skin test reactivity.

PREP Pearls

http://2016.prepid.courses.aap.org/script/january?req=201612121942469003&status=submit 1/2
12/13/2016 January
The suggested interval between receipt of immune globulin and vaccination are the same for measles, mumps, and rubella,
and vary in length based on the dose and type of immune globulin received.

The measles, mumps, rubella, and varicella (MMRV) vaccine carries an increased risk of febrile seizures in children with a
personal or family history of seizures.

There is no contraindication for vaccine for a child in the setting of maternal pregnancy, maternal breastfeeding, or those
with household contacts who are immune suppressed.

American Board of Pediatrics Content Specification(s)
Manage a patient who has a contraindication (or precaution) for rubella immunization

Suggested Readings
American Academy of Pediatrics. Rubella. In: Kimberlin DW, Brady MT, Jackson MA, Long SS, eds. Red Book: 2015 Report
of the Committee on Infectious Diseases. 30th ed. Elk Grove Village, IL: American Academy of Pediatrics; 2015: 688­695.

US Centers for Disease Control and Prevention. Rubella. In: Hamborsky J, Kroger A, Wolfe C, eds. Epidemiology and
Prevention of Vaccine­Preventable Diseases. 13th ed. Atlanta, GA: US Centers for Diseases Control and Prevention; 2015:
325­340.

Comment On This Question

Page 15 of 115

Contact Us
Links may open in separate window
Copyright © 2016   American Academy of Pediatrics. All rights reserved. Pop­up Blocker may need to be disabled

http://2016.prepid.courses.aap.org/script/january?req=201612121942469003&status=submit 2/2
12/13/2016 February

Welcome  mohammed alsaiary [ Logout ]

Home PREP Pearls FAQs My Bookmarks CME Information

Overview Home  > February

Editorial Board
February   Enter Keyword Search
January
Question View:   All (8) Jump to Question
February
Print this Page Add to my Bookmarks Page 17 of 115
March

April Assessment History

May ASSESSMENT PROGRESS:  Total Questions:  8  Questions Answered:  0  Correct Answers:  0

June

July You currently have 8 questions unanswered in this assessment

August

September Question: 1
October In preparation for discharge from a neonatal intensive care unit, the parents of twin males born at 27 weeks’ gestational age were
November provided anticipatory guidance about respiratory syncytial virus (RSV) infection. You inform them that the twins will receive their first
dose of palivizumab at the start of the local RSV season, with subsequent doses every 4 weeks until the season ends. The father
December informs you that the family will be moving to Florida and asks if there are differences in the RSV season there that he should know
about.

Of the following, the MOST accurate response to the father’s question is that the start of the RSV season in Florida tends to be

A. earlier in the calendar year, and lasts longer than activity in the rest of the United States

Claim Credit B. earlier in the calendar year, but lasts shorter than activity in the rest of the United States
C. later in the calendar year, and lasts longer than activity in the rest of the United States
Evaluation
D. later in the calendar year, but lasts shorter than activity in the rest of the United States
My Learning Plan

Submit   Reset  

Page 17 of 115

Contact Us
Links may open in separate window
Copyright © 2016   American Academy of Pediatrics. All rights reserved. Pop­up Blocker may need to be disabled

http://2016.prepid.courses.aap.org/script/february?q­page=1&req=201612121952572027 1/1
12/13/2016 February

Welcome  mohammed alsaiary [ Logout ]

Home PREP Pearls FAQs My Bookmarks CME Information

Overview Home  > February

Editorial Board
February   Enter Keyword Search
January
Question View:   All (8) Jump to Question
February
Print this Page Add to my Bookmarks Page 17 of 115
March

April Assessment History

May ASSESSMENT PROGRESS:  Total Questions:  8  Questions Answered:  1  Correct Answers:  0

June

July You currently have 7 questions unanswered in this assessment

August

September Question: 1
October In preparation for discharge from a neonatal intensive care unit, the parents of twin males born at 27 weeks’ gestational age were
November provided anticipatory guidance about respiratory syncytial virus (RSV) infection. You inform them that the twins will receive their first
dose of palivizumab at the start of the local RSV season, with subsequent doses every 4 weeks until the season ends. The father
December informs you that the family will be moving to Florida and asks if there are differences in the RSV season there that he should know
about.

Of the following, the MOST accurate response to the father’s question is that the start of the RSV season in Florida tends to be

A. earlier in the calendar year, and lasts longer than activity in the rest of the United States

Claim Credit B. earlier in the calendar year, but lasts shorter than activity in the rest of the United States
C. later in the calendar year, and lasts longer than activity in the rest of the United States
Evaluation
D. later in the calendar year, but lasts shorter than activity in the rest of the United States
My Learning Plan

Incorrect View Peer Results

Correct Answer: A Average Percent Correct:  49.50%

Surveillance to document trends in seasonality of respiratory syncytial virus (RSV) infection nationwide
is performed by the US Centers for Disease Control and Prevention’s National Respiratory and Enteric
Virus Surveillance System. A typical seasonal pattern of RSV disease activity for the United States is
shown in Figure 1. When regional data are reported, Florida is shown separately from the other states
in the southeastern United States because disease onset tends to start earlier in the calendar year, and
lasts much longer than activity in both neighboring states and in the rest of the continental United
States (Figure 2). Similarly, in warm and tropical climates outside of the United States, RSV activity
has less distinctive winter month peaks, with illness occurring year­round in some areas.

Respiratory syncytial virus infection is known to cause a full spectrum of respiratory complaints and findings in all regions of
the United States and all over the world. Once a child is infected, the manifestations of illness depend on a variety of host
factors, most notably age and immune competence. In the first month of life, the primary manifestation may be periodic
breathing or apnea with few (if any) findings on auscultation of the lungs. Such patients often require hospitalization to monitor
for apnea. Infants who are infected beyond the newborn period will develop nasal congestion, impressive coryza, and cough.
Those who progress to lower respiratory tract involvement (up to 40% of them) develop bronchiolitis with tachypnea and
wheezing. Hospitalization should be considered for infants with hypoxemia, prolonged feeding difficulties leading to
dehydration, or those with moderate­to­severe respiratory distress that could quickly lead to fatigue. A physical examination
finding of crackles, with radiographic evidence of pulmonary infiltrates may suggest the development of RSV pneumonia,
herald the development of a bacterial super­infection, or be indicative of aspiration caused by feeding difficulty. While it is not
possible to predict which patients with RSV infection will develop respiratory failure and require mechanical ventilation, infants
with known risk factors, especially hemodynamically significant congenital heart disease, prematurity, and chronic lung
disease of prematurity, require the highest level of vigilance.

Respiratory syncytial virus infection, even when severe, does not confer immunity to reinfection. Beyond infancy, reinfection is
commonplace, but most of these repeat infections are limited to the upper respiratory tract. Older children with RSV infection
typically develop cough and congestion. Reinfection of the lower airways, when it does occur, manifests as tracheobronchitis.
Even among children (and adults) without asthma or underlying lung disease, RSV infection can trigger wheezing and induce
airway reactivity, sometimes for as long as 8 weeks.

The connection between lower respiratory tract RSV infection during the first months of life and the development of
subsequent recurrent wheezing is also well known, although the mechanism for this association is not understood. One
possibility is that the virus alters early lung development to promote an asthma­like phenotype. Another possibility is that the

http://2016.prepid.courses.aap.org/script/february?req=201612121954475310&status=submit 1/2
12/13/2016 February
infant who is already genetically predisposed to asthma is also at risk for severe symptoms during his or her initial RSV
infection.

Host immune competence plays an important role in disease manifestations, morbidity, and mortality of RSV infection. In
immune­compromised children, symptoms of RSV infection include congestion and cough, sometimes with fever. Wheezing
may not be present, even when the infection progresses to the lower respiratory tract. Progression from minor upper
respiratory symptoms to fulminant lower respiratory disease with signs and symptoms of pneumonia can occur rapidly. The
degree of immune dysfunction correlates with illness severity. For example, RSV infection has been reported to occur in as
many as 50% of stem cell transplant patients, often acquired nosocomially. In this population, lower respiratory tract RSV
infection has a mortality rate approaching 100%. Severe and often fatal RSV infection also occurs in infants with combined T­
and B­cell deficiencies.

Members of each of the 2 major strain groups of RSV, RSV A and RSV B, co­circulate during the peaks of RSV activity, with
group A strains predominating during most seasons. While early studies suggested that outbreaks and individual infections
were more severe when caused by group A RSV strains, more recent studies have questioned that association.

PREP Pearls
In most of the United States, respiratory syncytial virus (RSV) disease activity is seasonal and fairly predictable during the
winter months.

In the state of Florida, RSV disease activity tends to start earlier and last longer than that seen in the rest of the country.

The association between RSV infection early in life and the risk for recurrent wheezing episodes suggests a possible link
between this infection and the development of asthma.

American Board of Pediatrics Content Specification(s)
Appreciate the epidemiology of respiratory syncytial virus, including the geographic, seasonal, and strain patterns

Appreciate the relative importance of respiratory syncytial virus in causing various types of respiratory illness according to
age in ambulatory and hospitalized patients

Recognize the possible association of respiratory syncytial virus infection in the development of asthma

Suggested Readings
Escobar GJ, Masaquel AS, Li SX, Walsh EM, Kipnis P. Persistent recurring wheezing in the fifth year of life after laboratory­
confirmed, medically attended respiratory syncytial virus infection in infancy. BMC Pediatr. 2013;13:97. DOI:
http://dx.doi.org/10.1186/1471­2431­13­97

Hall CB, Weinberg GA, Blumkin AK, et al. Respiratory syncytial virus­associated hospitalizations among children less than 24
months of age. Pediatrics. 2013;132(2):e341­e348. DOI: http://dx.doi.org/10.1542/peds.2013­0303

Haynes AK, Prill MM, Iwane MK, Gerber SI. Respiratory syncytial virus­­United States, July 2012­June 2014. MMWR Morb
Mortal Wkly Rep. 2014;63(48):1133­1136. http://www.cdc.gov/mmwr/preview/mmwrhtml/mm6348a3.htm

Comment On This Question

Page 17 of 115

Contact Us
Links may open in separate window
Copyright © 2016   American Academy of Pediatrics. All rights reserved. Pop­up Blocker may need to be disabled

http://2016.prepid.courses.aap.org/script/february?req=201612121954475310&status=submit 2/2
12/13/2016 February

Welcome  mohammed alsaiary [ Logout ]

Home PREP Pearls FAQs My Bookmarks CME Information

Overview Home  > February

Editorial Board
February   Enter Keyword Search
January
Question View:   All (8) Jump to Question
February
Print this Page Add to my Bookmarks Page 18 of 115
March

April Assessment History

May ASSESSMENT PROGRESS:  Total Questions:  8  Questions Answered:  1  Correct Answers:  0

June

July You currently have 7 questions unanswered in this assessment

August

September Question: 2
October You are asked to evaluate a previously healthy, 9­year­old boy who has had nasal drainage and cough for the past 21 days. His
November illness began with rhinorrhea, cough, and low­grade fever. Over the past 14 days, he developed a profuse thick nasal drainage and
worsening cough. He was evaluated by his primary care physician 5 days ago and diagnosed with a viral illness. Over the past 4
December days, he has developed more profuse nasal drainage, worsening cough, and high fever. He denies chest pain or increased work of
breathing. His mother reports that he has a history of a hive­like rash that developed 1 week into treatment with amoxicillin for an ear
infection several years ago. On physical examination, his temperature is 39°C, his pulse rate is 90 beats/min, his respiratory rate is
20 breaths/min, and his blood pressure 100/60 mm Hg. He has purulent nasal drainage bilaterally and erythematous, swollen nasal
turbinates. His lungs are clear to auscultation. He has no facial tenderness or periorbital edema.

Of the following, the MOST appropriate initial antimicrobial agent for this child is
Claim Credit

Evaluation A. azithromycin
B. cefuroxime
My Learning Plan
C. clindamycin
D. trimethoprim­sulfamethoxazole

Submit   Reset  

Page 18 of 115

Contact Us
Links may open in separate window
Copyright © 2016   American Academy of Pediatrics. All rights reserved. Pop­up Blocker may need to be disabled

http://2016.prepid.courses.aap.org/script/february?req=201612121955100936 1/1
12/13/2016 February

Welcome  mohammed alsaiary [ Logout ]

Home PREP Pearls FAQs My Bookmarks CME Information

Overview Home  > February

Editorial Board
February   Enter Keyword Search
January
Question View:   All (8) Jump to Question
February
Print this Page Add to my Bookmarks Page 18 of 115
March

April Assessment History

May ASSESSMENT PROGRESS:  Total Questions:  8  Questions Answered:  2  Correct Answers:  0

June

July You currently have 6 questions unanswered in this assessment

August

September Question: 2
October You are asked to evaluate a previously healthy, 9­year­old boy who has had nasal drainage and cough for the past 21 days. His
November illness began with rhinorrhea, cough, and low­grade fever. Over the past 14 days, he developed a profuse thick nasal drainage and
worsening cough. He was evaluated by his primary care physician 5 days ago and diagnosed with a viral illness. Over the past 4
December days, he has developed more profuse nasal drainage, worsening cough, and high fever. He denies chest pain or increased work of
breathing. His mother reports that he has a history of a hive­like rash that developed 1 week into treatment with amoxicillin for an ear
infection several years ago. On physical examination, his temperature is 39°C, his pulse rate is 90 beats/min, his respiratory rate is
20 breaths/min, and his blood pressure 100/60 mm Hg. He has purulent nasal drainage bilaterally and erythematous, swollen nasal
turbinates. His lungs are clear to auscultation. He has no facial tenderness or periorbital edema.

Of the following, the MOST appropriate initial antimicrobial agent for this child is
Claim Credit

Evaluation A. azithromycin
B. cefuroxime
My Learning Plan
C. clindamycin
D. trimethoprim­sulfamethoxazole

Incorrect View Peer Results
Correct Answer: B Average Percent Correct:  61.39%

The child in the vignette likely has acute bacterial sinusitis (ABS) based on the persistence of upper
respiratory symptoms (lasting longer than 10 days and not improving) and the presence of high fever
and purulent nasal drainage present for 3 to 4 consecutive days in a generally ill appearing child. The
diagnosis of ABS can also be made in children who have worsening of respiratory symptoms after
initial improvement. Recent American Academy of Pediatrics (AAP) guidelines have stressed the
importance of making a clinical diagnosis of ABS and initiating appropriate antimicrobial treatment
according to the these clinical criteria. Thus, the boy in the vignette should be treated with an
antimicrobial agent. The choice of antimicrobial agent should take into consideration the microbiology of ABS, the likelihood of
antimicrobial resistance, cost, safety, tolerability, and potential allergic reactions. The bacteriology of ABS include
Streptococcus pneumoniae (30%), nontypeable Haemophilus influenzae (30%), and Moraxella catarrhalis (10%). About half
of infections caused by H influenzae and 50% to 75% of infections caused by M catarrhalis will resolve spontaneously, while
only about 15% of pneumococcal ABS will resolve spontaneously. About 15% to 40% of pneumococcal strains are
nonsusceptible to penicillin, with half of these strains being highly resistant. Except for the highly resistant strains of
pneumococci, most strains will respond to conventional dosages of amoxicillin. Given the possible nontype I allergy to
penicillin, the boy in the vignette may be safely treated with a second­ or third­generation cephalosporin, such as cefuroxime,
cefdinir, or cefpodoxime. Azithromycin and trimethoprim­sulfamethoxazole do not have reliable activity against S pneumoniae
or H influenzae, and are not recommended for the treatment of ABS. Clindamycin does not have activity against H influenzae
or M catarrhalis and thus is not an appropriate initial choice for ABS. However, clindamycin used in combination with a third­
generation oral cephalosporin, such as cefixime, is an appropriate option for children who have a history of type I immediate
reaction to penicillin. Linezolid and levofloxacin are alternative agents in the treatment of ABS in children who have type I
allergic reactions to penicillin, although the AAP guidelines stress that cephalosporins are likely to be safe in such patients.
Nevertheless, use of a non­β­lactam agent or referral to an allergist for skin testing before initiation of therapy may be
preferred by some clinicians.

Amoxicillin remains the agent of choice for children with uncomplicated ABS in situations where antimicrobial resistance is not
suspected and there is no history of allergy to penicillin. This is based on the safety, tolerability, low cost, and narrow
antimicrobial spectrum of this agent. Standard dose amoxicillin (45 mg/kg per day) is recommended for children older than 2
years of age who have mild or moderate symptoms, do not attend childcare centers, and who have not received antibiotic
therapy during the previous 4 weeks. Higher doses of amoxicillin (80­90 mg/kg per day) may be used in communities with a

http://2016.prepid.courses.aap.org/script/february?req=201612121957170626&status=submit 1/2
12/13/2016 February
high prevalence of nonsusceptible S pneumoniae. High­dose amoxicillin­clavulanate (80­90 mg/kg per day) may be used for
children with ABS who have moderate­to­severe illness, those younger than 2 years of age, those attending childcare
centers, or those who have recently been treated with an antimicrobial agent. Although the ideal duration of treatment for ABS
has not been established, a common approach is to continue treatment for 7 days after signs and symptoms have resolved.
Adjuvant therapies such as antihistamines, decongestants, mucolytics, and nasal irrigations have not been studied
systematically and are not recommended for ABS in children. Limited data on the use of intranasal steroids as an adjunct to
oral antimicrobial therapy in children with ABS suggest a modest beneficial effect on symptoms. However, given the limited
data and methodological flaws of available studies done exclusively in children, no definite recommendation regarding their
use can be made.

The diagnosis of ABS can be made on stringent clinical grounds, therefore radiographic studies are not indicated for
uncomplicated ABS. Furthermore, plain radiographs, contrast­enhanced computed tomography, or magnetic resonance
imaging in children will not differentiate between bacterial sinusitis and uncomplicated upper respiratory tract infection. Thus,
abnormal images of the sinuses cannot confirm the diagnosis of ABS. Transillumination of the sinuses to assess for the
presence of fluid for the diagnosis of ABS is unreliable in children and not recommended. Imaging studies, specifically
contrast­enhanced computed tomography and magnetic resonance imaging with contrast, should be reserved for children
who are suspected of having a complication of ABS, such as orbital cellulitis or central nervous system complications.

* Required *
Take Survey  

PREP Pearls
The diagnosis of acute bacterial sinusitis should be made based on stringent clinical criteria.

Amoxicillin, with or without clavulanate, is the first­line treatment for uncomplicated acute bacterial sinusitis.

Radiographic studies cannot confirm the presence of acute bacterial sinusitis and are not recommended for uncomplicated
acute bacterial sinusitis.

American Board of Pediatrics Content Specification(s)
Appreciate the relative value and accuracy of diagnostic tests for sinusitis

Plan antimicrobial and other aspects of therapy for acute sinusitis

Suggested Readings
Kristo A, Uhari M, Luotonen J, et al. Paranasal sinus findings in children during respiratory infection evaluated with magnetic
resonance imaging. Pediatrics. 2003;111(5 Pt 1):e586–e589.
http://pediatrics.aappublications.org/content/111/5/e586.full.pdf+html

Pichichero ME. A review of evidence supporting the American Academy of Pediatrics recommendation for prescribing
cephalosporin antibiotics for penicillin allergic patients. Pediatrics. 2005;115(4):1048–1057. DOI:
http://dx.doi.org/10.1542/peds.2004­1276

Revai K, Dobbs LA, Nair S, Patel JA, Grady JJ, Chonmaitree T. Incidence of acute otitis media and sinusitis complicating
upper respiratory tract infection: the effect of age. Pediatrics. 2007;119(6):31408­e1412. DOI:
http://dx.doi.org/10.1542/peds.2006­2881

Wald ER, Applegate KE, Bordley C, et al. Clinical practice guideline for the diagnosis and management of acute bacterial
sinusitis in children aged 1 to 18 years. Pediatrics. 2013;132(1):e262­e280. DOI: http://dx.doi.org/10.1542/peds.2013­1071

Comment On This Question

Page 18 of 115

Contact Us
Links may open in separate window
Copyright © 2016   American Academy of Pediatrics. All rights reserved. Pop­up Blocker may need to be disabled

http://2016.prepid.courses.aap.org/script/february?req=201612121957170626&status=submit 2/2
12/13/2016 February

Welcome  mohammed alsaiary [ Logout ]

Home PREP Pearls FAQs My Bookmarks CME Information

Overview Home  > February

Editorial Board
February   Enter Keyword Search
January
Question View:   All (8) Jump to Question
February
Print this Page Add to my Bookmarks Page 19 of 115
March

April Assessment History

May ASSESSMENT PROGRESS:  Total Questions:  8  Questions Answered:  2  Correct Answers:  0

June

July You currently have 6 questions unanswered in this assessment

August

September Question: 3
October A 5­year­old girl is referred to you for evaluation of recurrent pneumonia. The parents relate that their child is generally healthy,
November however, she has been treated on multiple occasions for pneumonia. The parents deny other recurrent infections. Her immunizations
are up­to­date, she takes no routine medications, and has had good growth. Previous episodes were characterized by coughing,
December more pronounced at night and in the early morning hours, and wheezing. Upon discussion with the pediatrician, the previous
radiographs have shown dense multifocal infiltrates. The child is currently asymptomatic and her physical examination is
unremarkable.

Of the following, the MOST likely underlying cause of this child’s recurrent episodes of pneumonia is

A. agammaglobulinemia
Claim Credit
B. asthma
Evaluation
C. bronchopulmonary dysplasia
My Learning Plan D. lymphocytic interstitial pneumonitis
E. segmental bronchiectasis

Submit   Reset  

Page 19 of 115

Contact Us
Links may open in separate window
Copyright © 2016   American Academy of Pediatrics. All rights reserved. Pop­up Blocker may need to be disabled

http://2016.prepid.courses.aap.org/script/february?q­page=3&req=201612121957176095 1/1
12/13/2016 February

Welcome  mohammed alsaiary [ Logout ]

Home PREP Pearls FAQs My Bookmarks CME Information

Overview Home  > February

Editorial Board
February   Enter Keyword Search
January
Question View:   All (8) Jump to Question
February
Print this Page Add to my Bookmarks Page 19 of 115
March

April Assessment History

May ASSESSMENT PROGRESS:  Total Questions:  8  Questions Answered:  3  Correct Answers:  1

June

July You currently have 5 questions unanswered in this assessment

August

September Question: 3
October A 5­year­old girl is referred to you for evaluation of recurrent pneumonia. The parents relate that their child is generally healthy,
November however, she has been treated on multiple occasions for pneumonia. The parents deny other recurrent infections. Her immunizations
are up­to­date, she takes no routine medications, and has had good growth. Previous episodes were characterized by coughing,
December more pronounced at night and in the early morning hours, and wheezing. Upon discussion with the pediatrician, the previous
radiographs have shown dense multifocal infiltrates. The child is currently asymptomatic and her physical examination is
unremarkable.

Of the following, the MOST likely underlying cause of this child’s recurrent episodes of pneumonia is

A. agammaglobulinemia
Claim Credit
B. asthma
Evaluation
C. bronchopulmonary dysplasia
My Learning Plan D. lymphocytic interstitial pneumonitis
E. segmental bronchiectasis

Correct View Peer Results
Average Percent Correct:  69.31%

In this generally healthy child with recurrent wheezing and cough, multiple episodes of pneumonia, and
a radiograph depicting dense multifocal infiltrates, the most likely explanation is asthma. Asthma,
through diffuse inflammation or mucus plugging of airways, is the most likely cause of recurrent
pulmonary atelectasis. Children with immunologic abnormalities such as humoral immune deficiencies
(eg, agammaglobulinemia) may have recurrent pneumonia with dense focal or multifocal infiltrates, but
would not be expected to remain otherwise healthy and asymptomatic. Bronchopulmonary dysplasia
and lymphocytic interstitial pneumonitis are generally associated with interstitial pneumonia. Segmental
bronchiectasis can result in recurrent pneumonia, but at a single site.

Children with recurrent pneumonia have 2 or more episodes of pneumonia in a year period, or more than 3 episodes in any
period, and radiographic resolution between the episodes. Of all children hospitalized for pneumonia, an estimated 8% have
recurrent pneumonia. The underlying causes of recurrent pneumonia are shown in Table 1. Important distinctions to make in
order to develop a differential diagnosis and a plan for evaluation of a patient with recurrent pneumonia are whether the area
of the lung involvement for each episode is the same or different, and the characteristics of the lung infiltrate (eg,
consolidated/focal or interstitial/diffuse/atelectatic). Recurrent pneumonia at a single site is generally the result of atelectasis,
obstructing lesions (either intraluminal or extrinsic), congenital abnormalities, or segmental bronchiectasis (Table 2). For
recurrent pneumonia not confined to a single site, it is important to distinguish between dense focal or multifocal infiltrates
versus diffuse interstitial infiltrates (Table 3). Causes of dense focal or multifocal infiltrates include aspiration and respiratory
tract disorders (including asthma, cystic fibrosis, and congenital heart disease), immunologic abnormalities (eg,
agammaglobulinemia, complement deficiency, HIV infection, and common variable immunodeficiency), and other diseases
such as sickle cell disease. Diffuse interstitial infiltrates have multiple etiologies, including bronchopulmonary dysplasia or
lymphocytic interstitial pneumonitis.

The medical history and physical examination guide the work­up of any patient with recurrent pneumonia. For example, for
recurrent pneumonia at a single site, evaluations may include bronchoscopy (to detect intraluminal or extrinsic obstructing
lesions, biopsy of a mass, obtain specimens for microscopy and culture) or computed tomography (to evaluate more distal
airways (eg, bronchiectasis), congenital cystic anomalies, and bronchopleural fistulas. For recurrent pneumonia not confined
to a single site, the work­up would differ according to characteristics of the recurrent episodes (dense focal/multifocal
infiltrates versus diffuse interstitial infiltrates). Some etiologies of recurrent dense infiltrates may be more clearly identifiable
due to the past medical history (eg, cystic fibrosis, recurrent aspiration due to seizures, immunologic abnormalities). Other

http://2016.prepid.courses.aap.org/script/february?req=201612121958392790&status=submit 1/2
12/13/2016 February
etiologies may require further investigation (eg, fluoroscopic examination during a barium meal to evaluate upper
pharyngopalatal coordination in children with suspected neurologic dysfunction). Lung biopsy may be required to establish the
etiology of recurrent pneumonia, especially with a noninfectious cause of recurrent, interstitial pneumonia.

PREP Pearls
Important distinctions to make in order to develop a differential diagnosis and a plan for evaluation of a patient with
recurrent pneumonia are whether the area of the lung involvement for each episode is the same or different, and the
characteristics of the lung infiltrate (eg, consolidated/focal or interstitial/diffuse/atelectatic).

Recurrent pneumonia at a single site is generally the result of atelectasis, obstructing lesions (either intraluminal or
extrinsic), congenital abnormalities, or segmental bronchiectasis.

For recurrent pneumonia not confined to a single site, it is important to distinguish between dense focal or multifocal
infiltrates versus diffuse interstitial infiltrates.

Etiologies of dense focal or multifocal infiltrates include respiratory tract disorders and aspiration (including asthma, cystic
fibrosis, and congenital heart disease), immunologic abnormalities, and other diseases such as sickle cell disease.

Diffuse interstitial infiltrates have multiple etiologies, including bronchopulmonary dysplasia or lymphocytic interstitial
pneumonitis.

American Board of Pediatrics Content Specification(s)
Plan the evaluation of a patient with chronic or recurrent pneumonia based on clinical and laboratory manifestations

Formulate a differential diagnosis and know the predisposing factors for a patient with suspected chronic or recurrent
pneumonia, including anatomic and immunologic abnormalities, and underlying diseases including cystic fibrosis and
foreign body

Suggested Readings
Murray DL, Mani CS. Persistent and recurrent pneumonia. In: Long SS, Pickering LK, Prober CG, eds. Principles and
Practice of Pediatric Infectious Diseases. 4th ed. New York, NY: Churchill Livingstone Elsevier; 2012:245­252.

Comment On This Question

Page 19 of 115

Contact Us
Links may open in separate window
Copyright © 2016   American Academy of Pediatrics. All rights reserved. Pop­up Blocker may need to be disabled

http://2016.prepid.courses.aap.org/script/february?req=201612121958392790&status=submit 2/2
12/13/2016 February

Welcome  mohammed alsaiary [ Logout ]

Home PREP Pearls FAQs My Bookmarks CME Information

Overview Home  > February

Editorial Board
February   Enter Keyword Search
January
Question View:   All (8) Jump to Question
February
Print this Page Add to my Bookmarks Page 20 of 115
March

April Assessment History

May ASSESSMENT PROGRESS:  Total Questions:  8  Questions Answered:  3  Correct Answers:  1

June

July You currently have 5 questions unanswered in this assessment

August

September Question: 4
October A 14­year­old adolescent girl with systemic lupus erythematosus on high­dose steroid therapy developed fever and cough 1 week
November ago. She was admitted to the hospital 4 days ago when a chest radiograph demonstrated a right­sided cavitary lesion (Figure 1).
Antimicrobial therapy with piperacillin­tazobactam, vancomycin, and azithromycin was begun.
December
She did not improve after 4 days on this regimen, and a sputum Gram stain is shown in Figure 2.

Of the following additional treatments, the one MOST likely to improve her pulmonary infection is

A. amikacin
B. cefepime
Claim Credit
C. isoniazid, rifampin, pyrazinamide, and ethambutol
Evaluation
D. trimethoprim­sulfamethoxazole
My Learning Plan

Submit   Reset  

Page 20 of 115

Contact Us
Links may open in separate window
Copyright © 2016   American Academy of Pediatrics. All rights reserved. Pop­up Blocker may need to be disabled

http://2016.prepid.courses.aap.org/script/february?q­page=4&req=201612121958397319 1/1
Figure 1. Plain chest radiograph.

Reprinted with permission from the American Academy of Pediatrics. Red


Book Online Visual Library.
Figure 2. Sputum Gram stain.

Reprinted with permission from the American Academy of Pediatrics. Red Book
Online Visual Library. Accessed September 7, 2015.
http://redbook.solutions.aap.org/data/Books/1484/086_07.jpeg
12/13/2016 February

Welcome  mohammed alsaiary [ Logout ]

Home PREP Pearls FAQs My Bookmarks CME Information

Overview Home  > February

Editorial Board
February   Enter Keyword Search
January
Question View:   All (8) Jump to Question
February
Print this Page Add to my Bookmarks Page 20 of 115
March

April Assessment History

May ASSESSMENT PROGRESS:  Total Questions:  8  Questions Answered:  4  Correct Answers:  2

June

July You currently have 4 questions unanswered in this assessment

August

September Question: 4
October A 14­year­old adolescent girl with systemic lupus erythematosus on high­dose steroid therapy developed fever and cough 1 week
November ago. She was admitted to the hospital 4 days ago when a chest radiograph demonstrated a right­sided cavitary lesion (Figure 1).
Antimicrobial therapy with piperacillin­tazobactam, vancomycin, and azithromycin was begun.
December
She did not improve after 4 days on this regimen, and a sputum Gram stain is shown in Figure 2.

Of the following additional treatments, the one MOST likely to improve her pulmonary infection is

A. amikacin
B. cefepime
Claim Credit
C. isoniazid, rifampin, pyrazinamide, and ethambutol
Evaluation
D. trimethoprim­sulfamethoxazole
My Learning Plan

Correct View Peer Results
Average Percent Correct:  76.24%

The Gram stain is most suggestive of Nocardia species; cavitary pulmonary nocardiosis is seen in
immunocompromised patients, including in patients with systemic lupus erythematosus. A modified
acid­fast stain would lend further support to the diagnosis and the organism will likely grow in culture if
plates are held at least 5 days. Amikacin and cefepime have some activity against Nocardia species,
but are not the preferred regimen. Isoniazid, rifampin, pyrazinamide, and ethambutol are the preferred
treatments for pulmonary tuberculosis, also a cause of cavitary lung lesions in adolescents, but the
Gram stain morphology is not typical for Mycobacterium tuberculosis. Trimethoprim­sulfamethoxazole
(TMP­SMX) is considered the treatment of choice for nocardiosis in most circumstances.

Sulfonamides were one of the first classes of antibiotic used in humans, so it is not particularly surprising that many bacteria
now are commonly resistant to this antibiotic class. Sulfonamides are seldom used alone, but as part of combination therapy
(eg, trimethoprim) may have utility in both prophylaxis and treatment of selected infections.

Table. Prophylactic and Therapeutic Uses for Sulfonamides.

Prophylactic Uses (primarily trimethoprim­sulfamethoxazole)

Chronic granulomatous disease
Otitis media
Pneumocystis jirovecii pneumonia
Urinary tract infection

Therapeutic Uses

Trimethoprim­Sulfamethoxazole

Brucellosis
Pertussis (alternative to macrolide)
Pneumocystis jirovecii pneumonia
Bacterial infections with known trimethoprim­sulfamethoxazole susceptibility (eg, susceptible Escherichia coli
urinary tract infection, Shigella or Yersinia species enteritis)

Pyrimethamine­sulfadiazine

http://2016.prepid.courses.aap.org/script/february?req=201612122002004049&status=submit 1/2
12/13/2016 February
Toxoplasmosis

Pyrimethamine­sulfadoxine

Uncomplicated falciparum malaria (in combination with artesunate)

Sulfamethoxazole may also have a role in treatment of rifampin­resistant Mycobacterium kansasii infections, if susceptible to
sulfonamide.

PREP Pearls
Sulfonamides are rarely utilized as monotherapy for any pediatric infection

Sulfamethoxazole in combination with trimethoprim has both prophylactic and therapeutic uses.

Trimethoprim­sulfamethoxazole susceptibility patterns vary widely; proper use should involve both awareness of local
patterns and susceptibility testing of specific isolates.

American Board of Pediatrics Content Specification(s)
Know clinical uses of sulfonamides for treatment (Nocardia, rifampin‑resistant Mycobacterium kansasii, Escherichia coli)

Know the clinical uses of sulfonamides for prophylaxis (urinary tract infection, otitis media, chronic granulomatous disease,
Pneumocystis jiroveci)

Know clinical uses of sulfonamide combination therapy for toxoplasma, P. falciparum

Suggested Readings
Bradley JS, Sauberan JB. Antimicrobial agents. In: Long SS, Pickering LK, Prober CG, eds. Principles and Practice of
Pediatric Infectious Diseases. 4th ed. Philadelphia, PA: Churchill Livingstone Elsevier; 2012:1453­1484.

Garcia C, McCracken GH Jr. Antibacterial therapeutic agents. In: Cherry J, Demmler­Harrison GJ, Kaplan SL, Steinbach WJ,
Hotez P, eds. Feigin and Cherry’s Textbook of Pediatric Infectious Diseases. 7th ed. Philadelphia, PA: Saunders Elsevier;
2013:3182­3241.

World Health Organization. Guidelines for the Treatment of Malaria. 3rd ed. Geneva, Switzerland: World Health Organization;
2015: 316 pp.

Comment On This Question

Page 20 of 115

Contact Us
Links may open in separate window
Copyright © 2016   American Academy of Pediatrics. All rights reserved. Pop­up Blocker may need to be disabled

http://2016.prepid.courses.aap.org/script/february?req=201612122002004049&status=submit 2/2
12/13/2016 February

Welcome  mohammed alsaiary [ Logout ]

Home PREP Pearls FAQs My Bookmarks CME Information

Overview Home  > February

Editorial Board
February   Enter Keyword Search
January
Question View:   All (8) Jump to Question
February
Print this Page Add to my Bookmarks Page 21 of 115
March

April Assessment History

May ASSESSMENT PROGRESS:  Total Questions:  8  Questions Answered:  4  Correct Answers:  2

June

July You currently have 4 questions unanswered in this assessment

August

September Question: 5
October You are discussing the different vaccines available for prevention of hepatitis A infection with your trainees. Apart from the coverage,
November they want to know the difference between the combination hepatitis A/B vaccine and the 2 hepatitis A vaccines.

December Of the following, the MOST accurate statement about the combination hepatitis A/B vaccine is

A. the composition is 720 ELU of hepatitis A and 20 µg of hepatitis B antigens
B. it contains small amounts of aluminium salts, neomycin, yeast, and gelatin
C. it has been studied for pre­ and postexposure prophylaxis
D. it should not be used for immunocompromised individuals
Claim Credit

Evaluation
Submit   Reset  
My Learning Plan

Page 21 of 115

Contact Us
Links may open in separate window
Copyright © 2016   American Academy of Pediatrics. All rights reserved. Pop­up Blocker may need to be disabled

http://2016.prepid.courses.aap.org/script/february?q­page=5&req=201612122002008735 1/1
12/13/2016 February

Welcome  mohammed alsaiary [ Logout ]

Home PREP Pearls FAQs My Bookmarks CME Information

Overview Home  > February

Editorial Board
February   Enter Keyword Search
January
Question View:   All (8) Jump to Question
February
Print this Page Add to my Bookmarks Page 21 of 115
March

April Assessment History

May ASSESSMENT PROGRESS:  Total Questions:  8  Questions Answered:  5  Correct Answers:  3

June

July You currently have 3 questions unanswered in this assessment

August

September Question: 5
October You are discussing the different vaccines available for prevention of hepatitis A infection with your trainees. Apart from the coverage,
November they want to know the difference between the combination hepatitis A/B vaccine and the 2 hepatitis A vaccines.

December Of the following, the MOST accurate statement about the combination hepatitis A/B vaccine is

A. the composition is 720 ELU of hepatitis A and 20 µg of hepatitis B antigens
B. it contains small amounts of aluminium salts, neomycin, yeast, and gelatin
C. it has been studied for pre­ and postexposure prophylaxis
D. it should not be used for immunocompromised individuals
Claim Credit

Evaluation
Correct View Peer Results
My Learning Plan
Average Percent Correct:  61.39%

There are 3 vaccines currently available for use against hepatitis A virus (HAV) infection in the United
States. The Table details the characteristics of each vaccine. The combined hepatitis A and hepatitis B
(recombinant) vaccine differs from the other 2 inactivated hepatitis A vaccines in that it contains the full
adult dose of hepatitis B antigen (Engerix­B® adult dose), as well as half the adult dose of hepatitis A
antigen (Havrix®­ 720 ELU). It may be given in 2 separate schedules, with the 4­dose schedule
recommended for individuals at high risk who may need protective antibody sooner.

The combination A/B vaccine has not been studied for postexposure prophylaxis unlike the 2 HAV­only vaccines. It may be
used in immunocompromised individuals, although the response may be suboptimal depending on the degree of
immunosuppression of the individual. None of the HAV vaccines contain any preservatives. The combination A/B vaccine
does not contain any gelatin, although it does contain aluminium salts, neomycin, and yeast (Table).

All the available HAV vaccines are highly immunogenic, and at least 95% and 99% of healthy children, adolescents, and
adults have protective antibody concentrations when measured 1 month after immunization with the first and second doses of
the vaccines, respectively.

The 2 HAV­only vaccines are prepared from cell culture­adapted HAV that has been propagated in human fibroblasts, purified
from cell lysates, formalin inactivated, and then adsorbed onto an aluminium hydroxide adjuvant.

Both the single antigen HAV vaccines may be used interchangeably. They may also be given simultaneously with other
vaccines. Pre­immunization serologic testing for anti­HAV is not recommended for young children because of their expected
low prevalence of infection. Postimmunization serologic testing is also not recommended because of the high seroconversion
rates in adults and children. An exception to this is immunization in HIV­infected adults and adolescents who have CD4
counts less than 200 cells/µL where serologic testing is recommended 1 month after vaccination is complete.

PREP Pearls
Both of the currently available HAV vaccines, as well as the combined hepatitis A and B (recombinant) vaccines, are highly
immunogenic in healthy children, adolescents, and adults; the efficacy in clinical studies has been reported to be between
94% to 100%.

The combined hepatitis A and B (recombinant) vaccine contains the full adult dose of hepatitis B antigen (Engerix­B®), but
half the adult dose of hepatitis A vaccine (Havrix®), while the HAV­only vaccines contain inactivated HAV, propagated in
human diploid fibroblasts.

Neither of the HAV vaccines contain any preservatives.

http://2016.prepid.courses.aap.org/script/february?req=201612122003075614&status=submit 1/2
12/13/2016 February

American Board of Pediatrics Content Specification(s)
Know the composition of hepatitis A vaccine, including nature of antigens, source, adjuvants, chemicals, and different
products

Know the efficacy of hepatitis A vaccine

Suggested Readings
American Academy of Pediatrics. Hepatitis A. In: Kimberlin DW, Brady MT, Jackson MA, Long SS, eds. Red Book: 2015
Report of the Committee on Infectious Diseases. 30th ed. Elk Grove Village, IL: American Academy of Pediatrics; 2015: 391­
399.

GlaxoSmithKline. Havrix®­full prescribing information. GlazoSmithKline website.
https://www.gsksource.com/pharma/content/dam/GlaxoSmithKline/US/en/Prescribing_Information/Havrix/pdf/HAVRIX.PDF

GlaxoSmithKline. Twinrix®­full prescribing information. GlazoSmithKline website.
https://www.gsksource.com/pharma/content/dam/GlaxoSmithKline/US/en/Prescribing_Information/Twinrix/pdf/TWINRIX.PDF

Merck & Co., Inc. Vaqta®­full prescribing information. Merck & Co., Inc website.
http://www.merck.com/product/usa/pi_circulars/v/vaqta/vaqta_pi.pdf

Comment On This Question

Page 21 of 115

Contact Us
Links may open in separate window
Copyright © 2016   American Academy of Pediatrics. All rights reserved. Pop­up Blocker may need to be disabled

http://2016.prepid.courses.aap.org/script/february?req=201612122003075614&status=submit 2/2
12/13/2016 February

Welcome  mohammed alsaiary [ Logout ]

Home PREP Pearls FAQs My Bookmarks CME Information

Overview Home  > February

Editorial Board
February   Enter Keyword Search
January
Question View:   All (8) Jump to Question
February
Print this Page Add to my Bookmarks Page 22 of 115
March

April Assessment History

May ASSESSMENT PROGRESS:  Total Questions:  8  Questions Answered:  5  Correct Answers:  3

June

July You currently have 3 questions unanswered in this assessment

August

September Question: 6
October An interdisciplinary conference between infectious diseases and immunology was held at your hospital where fellows from both
November divisions reviewed key components of the human immune system. The next day on rounds, your infectious diseases team is
consulted on an infant with influenza virus infection. The fellow refers to her talk the previous day as she begins to quiz a medical
December student about the infant’s immune response to influenza infection. She discusses the substance that increases host defenses
through upregulation of viral antigen presentation by increasing the expression of major histocompatibility complex antigens.

Of the following, this substance is MOST likely

A. complement

Claim Credit B. interferon
C. interleukin
Evaluation
D. leukotriene
My Learning Plan

Submit   Reset  

Page 22 of 115

Contact Us
Links may open in separate window
Copyright © 2016   American Academy of Pediatrics. All rights reserved. Pop­up Blocker may need to be disabled

http://2016.prepid.courses.aap.org/script/february?q­page=6&req=201612122003079988 1/1
12/13/2016 February

Welcome  mohammed alsaiary [ Logout ]

Home PREP Pearls FAQs My Bookmarks CME Information

Overview Home  > February

Editorial Board
February   Enter Keyword Search
January
Question View:   All (8) Jump to Question
February
Print this Page Add to my Bookmarks Page 22 of 115
March

April Assessment History

May ASSESSMENT PROGRESS:  Total Questions:  8  Questions Answered:  6  Correct Answers:  3

June

July You currently have 2 questions unanswered in this assessment

August

September Question: 6
October An interdisciplinary conference between infectious diseases and immunology was held at your hospital where fellows from both
November divisions reviewed key components of the human immune system. The next day on rounds, your infectious diseases team is
consulted on an infant with influenza virus infection. The fellow refers to her talk the previous day as she begins to quiz a medical
December student about the infant’s immune response to influenza infection. She discusses the substance that increases host defenses
through upregulation of viral antigen presentation by increasing the expression of major histocompatibility complex antigens.

Of the following, this substance is MOST likely

A. complement

Claim Credit B. interferon
C. interleukin
Evaluation
D. leukotriene
My Learning Plan

Incorrect View Peer Results

Correct Answer: B Average Percent Correct:  53.47%

The substance most likely to increase the expression of major histocompatibility complex (MHC)
antigens is interferon (IFN). Interferons are a group of naturally occurring signaling proteins produced
and secreted by immune cells such as white blood cells, natural killer cells, fibroblasts, macrophages,
epithelial cells, dendritic cells, etc. The mechanism of action of IFN is complex, involving modulation of
the response of the immune system to viruses, bacteria, cancer, and other foreign substances.
Interferons do not directly kill viral or cancerous cells, but boost the immune system response and
reduce the growth of cancer cells by regulating the action of several genes that control the secretion of
numerous cellular proteins that affect growth. Interferons increase host defenses by upregulating antigen presentation
through increased expression of MHC antigens, MHC I and MHC II, and increased immunoproteasome activity (Table 1).

The proteins and glycoproteins that constitute the classical complement pathway activate binding of an antibody to an
antigen, facilitating phagocytosis of an organism (specific immune response). The alternative complement system and
mannose­binding lectin pathways can be activated by C3 hydrolysis or antigens without the presence of antibodies
(nonspecific immune response). Leukotrienes consist of a family of eicosanoid inflammatory mediators produced in
leukocytes; they use lipid signaling to convey information to either the cell producing them (autocrine signaling) or neighboring
cells (paracrine signaling) in order to regulate immune responses. Interleukins are cytokines responsible for communication
between white blood cells; they also recruit immune cells to the site of infection and may promote healing of damaged tissue.

There are 3 types of IFNs (Table 2). When used in the systemic therapy, IFNs are administered by intramuscular or
subcutaneous injection and are generally well tolerated. The most frequent adverse effects are flu­like symptoms: fever,
malaise, fatigue, headache, myalgia, dizziness, hair thinning, and depression. Erythema, pain, and induration at the site of
injection are also frequently observed. Interferon therapy causes neutropenia, which may lead to immunosuppression.
Available human IFNs are manufactured using recombinant DNA technology. There are multiple indicated uses (Table 3).

PREP Pearls
Interferons (IFN) are a group of naturally occurring signaling proteins, produced and secreted by immune cells.

The mechanism of action of IFN is complex, involving modulation of the response of the immune system to viruses,
bacteria, cancer, and other foreign substances.

There are 3 types of IFNs. Interferons do not directly kill viral or cancerous cells, but boost the immune system response
and reduce the growth of cancer cells by regulating the action of several genes that control the secretion of numerous

http://2016.prepid.courses.aap.org/script/february?req=201612122004489879&status=submit 1/2
12/13/2016 February
cellular proteins that affect growth.

American Board of Pediatrics Content Specification(s)
Know the three types of interferon, their major mechanisms of actions, and their role in host response to infection and as
therapy for infectious diseases

Know the indications for interferons for therapy of infectious diseases in humans

Suggested Readings
Dobbs K, Domínguez Conde C, Zhang SY, et al. Inherited DOCK2 deficiency in patients with early­onset invasive infections.
N Engl J Med. 2015;372(25):2409­2422. DOI: http://dx.doi.org/10.1056/NEJMoa1413462

Durbin RK, Kotenko SV, Durbin JE. Interferon induction and function at the mucosal surface. Immunol Rev. 2013;255(1):25­
39. DOI: http://dx.doi.org/10.1111/imr.12101

Lin FC, Young HA. Interferons: success in anti­viral immunotherapy. Cytokine Growth Factor Rev. 2014;25(4):369­376. DOI:
http://dx.doi.org/10.1016/j.cytogfr.2014.07.015

Lopušná K, Režuchová I, Betáková T, et al. Interferons lambda, new cytokines with antiviral activity. Acta Virol.
2013;57(2):171­179. DOI: http://dx.doi.org/10.4149/av_2013_02_171

Comment On This Question

Page 22 of 115

Contact Us
Links may open in separate window
Copyright © 2016   American Academy of Pediatrics. All rights reserved. Pop­up Blocker may need to be disabled

http://2016.prepid.courses.aap.org/script/february?req=201612122004489879&status=submit 2/2
12/13/2016 February

Welcome  mohammed alsaiary [ Logout ]

Home PREP Pearls FAQs My Bookmarks CME Information

Overview Home  > February

Editorial Board
February   Enter Keyword Search
January
Question View:   All (8) Jump to Question
February
Print this Page Add to my Bookmarks Page 23 of 115
March

April Assessment History

May ASSESSMENT PROGRESS:  Total Questions:  8  Questions Answered:  6  Correct Answers:  3

June

July You currently have 2 questions unanswered in this assessment

August

September Question: 7
October A 4­year­old previously healthy boy presents with watery, nonbloody mucoid diarrhea with mild dehydration. An enzyme
November immunoassay for rotavirus antigen is reported as positive. The mother is concerned that other household members will also develop
diarrhea.
December
The household member that is MOST at risk for developing severe rotavirus disease is the

A. 2­week­old sibling
B. 21­month­old sibling
C. 27­year­old nanny
Claim Credit
D. 67­year­old grandfather
Evaluation

My Learning Plan
Submit   Reset  

Page 23 of 115

Contact Us
Links may open in separate window
Copyright © 2016   American Academy of Pediatrics. All rights reserved. Pop­up Blocker may need to be disabled

http://2016.prepid.courses.aap.org/script/february?q­page=7&req=201612122004494408 1/1
12/13/2016 February

Welcome  mohammed alsaiary [ Logout ]

Home PREP Pearls FAQs My Bookmarks CME Information

Overview Home  > February

Editorial Board
February   Enter Keyword Search
January
Question View:   All (8) Jump to Question
February
Print this Page Add to my Bookmarks Page 23 of 115
March

April Assessment History

May ASSESSMENT PROGRESS:  Total Questions:  8  Questions Answered:  7  Correct Answers:  3

June

July You currently have 1 questions unanswered in this assessment

August

September Question: 7
October A 4­year­old previously healthy boy presents with watery, nonbloody mucoid diarrhea with mild dehydration. An enzyme
November immunoassay for rotavirus antigen is reported as positive. The mother is concerned that other household members will also develop
diarrhea.
December
The household member that is MOST at risk for developing severe rotavirus disease is the

A. 2­week­old sibling
B. 21­month­old sibling
C. 27­year­old nanny
Claim Credit
D. 67­year­old grandfather
Evaluation

My Learning Plan
Incorrect View Peer Results

Correct Answer: B Average Percent Correct:  35.64%

Rotavirus infection is a common cause of diarrhea in the first 5 years of life worldwide, causing an
estimated 453,000 deaths annually in the developing world. The highest incidence occurs in children 4
to 23 months of age. It is this age group that develops the most severe clinical manifestations with
profuse diarrhea and dehydration. Term neonates and infants younger than 3 months of age are
typically asymptomatic or develop mild disease. This is thought to be because of the protective effects
of maternal antibodies. Adults also develop mostly asymptomatic infections, as recurrent exposures
and infections lead to some immunity. Based on age, the 21­month­old sibling is at greatest risk for
developing severe disease, especially if unvaccinated.

Following an incubation period of 1 to 3 days, the symptoms of rotavirus include vomiting with or without fever for 1 to 3 days,
followed by frequent, profuse diarrhea for 4 to 8 days. Severe rotavirus disease includes complications such as dehydration,
metabolic acidosis, and organ dysfunction. Groups at risk for developing severe rotavirus disease include
immunocompromised hosts such as those with AIDS, X­linked agammaglobulinemia, DiGeorge syndrome, severe combined
immunodeficiency (SCID), and bone marrow and renal transplant patients. Viral shedding is prolonged (months to years) and
hepatorenal replication has been documented in the immunocompromised host. Normal hosts usually shed virus for the
duration of illness.

The majority of children have been exposed to rotavirus infection by 5 years of age. Immunity after natural infection does
develop, offering some protection from subsequent episodes. Such naturally derived immunity also leads to less severe
symptoms in subsequent infections. Recurrent infections are necessary to sustaining immunity. Studies have shown that
protection from severe infection is seen in 82% after 1 natural infection and reaches up to 100% after 2 natural infections. In
addition, 2 live oral vaccines are available: the pentavalent rotavirus vaccine given as a 3­dose series and the monovalent
rotavirus vaccine given as a 2­dose series. The vaccines are contraindicated in patients with SCID and those with
intussusception. Precautions include other immunocompromising conditions, moderate­to­severe illness including
gastroenteritis and chronic intestinal tract disease, and spina bifida or bladder exstrophy.

PREP Pearls
Rotavirus disease can be severe in unvaccinated infants and immunocompromised patients.

Maternal antibodies, natural infection, and vaccination are protective against rotavirus disease.

American Board of Pediatrics Content Specification(s)

http://2016.prepid.courses.aap.org/script/february?req=201612122005519827&status=submit 1/2
12/13/2016 February
Understand differences in the clinical manifestations of rotavirus infection according to age (eg, primary versus recurrent
infection)

Understand differences in clinical manifestations and complications of rotavirus infection between normal hosts and those
with underlying diseases who are at risk for complicated or severe disease

Know the usual course of rotavirus infection, both clinical and virologic (shedding) in normal and compromised hosts

Suggested Readings
American Academy of Pediatrics. Rotavirus infections. In: Kimberlin DW, Brady MT, Jackson MA, Long SS. eds. Red Book:
2015 Report Of The Committee On Infectious Diseases. 30th ed. Elk Grove Village, IL: American Academy of Pediatrics;
2015:684­688.

Dormitzer PR. Rotaviruses. In: Bennett JE, Dolin R, Blaser MJ, eds. Mandell, Douglas, and Bennett's Principles and Practice
of Infectious Diseases. 8th ed. Philadelphia, PA: Saunders Elsevier; 2015:1854­1864.

Yen C, Cortese MM. Rotaviruses. In: Long SS, Pickering LK, Prober CG, eds. Principles and Practice of Pediatric Infectious
Diseases: Expert Consult. 4th ed. Philadelphia, PA: Saunders Elsevier; 2012:1094­1096.

Comment On This Question

Page 23 of 115

Contact Us
Links may open in separate window
Copyright © 2016   American Academy of Pediatrics. All rights reserved. Pop­up Blocker may need to be disabled

http://2016.prepid.courses.aap.org/script/february?req=201612122005519827&status=submit 2/2
12/13/2016 February

Welcome  mohammed alsaiary [ Logout ]

Home PREP Pearls FAQs My Bookmarks CME Information

Overview Home  > February

Editorial Board
February   Enter Keyword Search
January
Question View:   All (8) Jump to Question
February
Print this Page Add to my Bookmarks Page 24 of 115
March

April Assessment History

May ASSESSMENT PROGRESS:  Total Questions:  8  Questions Answered:  7  Correct Answers:  3

June

July You currently have 1 questions unanswered in this assessment

August

September Question: 8
October You are called by a pediatrician for advice because he has seen 4 healthy, adolescent boys this week who presented with “spider
November bites” on the mid chest. The lesions were pustular and draining, and cultures from all 4 adolescents grew methicillin­resistant
Staphylococcus aureus (MRSA). He prescribed oral trimethoprim­sulfamethoxazole to which all isolates were susceptible and told
December each patient to keep the lesions covered until healed. He reports that they attend the same school, but do not play football or
wrestle.

Of the following, the MOST appropriate advice to give this pediatrician is to

A. inform the school nurse that the adolescents are excused from school due to MRSA infection

Claim Credit B. notify the health department of the outbreak in order to find the source
C. recommend decolonization measures (eg, chlorhexidine baths and mupirocin to the nares) for the adolescents and their
Evaluation
families
My Learning Plan D. send the MRSA isolates to your infectious diseases or hospital laboratory for molecular typing

Submit   Reset  

Page 24 of 115

Contact Us
Links may open in separate window
Copyright © 2016   American Academy of Pediatrics. All rights reserved. Pop­up Blocker may need to be disabled

http://2016.prepid.courses.aap.org/script/february?q­page=8&req=201612122005525451 1/1
12/13/2016 February

Welcome  mohammed alsaiary [ Logout ]

Home PREP Pearls FAQs My Bookmarks CME Information

Overview Home  > February

Editorial Board
February   Enter Keyword Search
January
Question View:   All (8) Jump to Question
February
Print this Page Add to my Bookmarks Page 24 of 115
March

April Assessment History

May ASSESSMENT PROGRESS:  Total Questions:  8  Questions Answered:  8  Correct Answers:  4

June

July You currently have 2 required survey(s) that are incomplete. Please complete the survey(s) for the following
question(s) on this assessment:  Question 2, Question 8
August

September

October Question: 8
November You are called by a pediatrician for advice because he has seen 4 healthy, adolescent boys this week who presented with “spider
bites” on the mid chest. The lesions were pustular and draining, and cultures from all 4 adolescents grew methicillin­resistant
December Staphylococcus aureus (MRSA). He prescribed oral trimethoprim­sulfamethoxazole to which all isolates were susceptible and told
each patient to keep the lesions covered until healed. He reports that they attend the same school, but do not play football or
wrestle.

Of the following, the MOST appropriate advice to give this pediatrician is to

A. inform the school nurse that the adolescents are excused from school due to MRSA infection
Claim Credit
B. notify the health department of the outbreak in order to find the source
Evaluation
C. recommend decolonization measures (eg, chlorhexidine baths and mupirocin to the nares) for the adolescents and their
My Learning Plan families
D. send the MRSA isolates to your infectious diseases or hospital laboratory for molecular typing

Correct View Peer Results

Average Percent Correct:  39.60%

Skin and soft tissue (SSTI) infection is the most frequent manifestation of methicillin­resistant
Staphylococcus aureus (MRSA) infection in the community. However, an outbreak or infection cluster is
suggested by the presentation of 4 patients of the same age with infection in the same anatomic
location within a 1­week period. Therefore, the health department should be notified of an increase in
disease above what is normally expected so that an investigation can begin to identify the cause.

Establishing the existence of an outbreak requires determining if the observed cases exceed the
expected number of cases. The ultimate goal of the investigation is to determine if there is: 1) a true outbreak, 2) sporadic and
unrelated cases of the same disease, or 3) different diseases presenting as clinically similar illnesses. The investigation
begins with establishing a case definition. Confirmed cases are those with positive culture or appropriate laboratory test
results. Probable cases lack laboratory confirmation but have typical clinical features, while possible or suspect cases have
some consistent clinical features. Data collection proceeds to characterize the outbreak by person, place, and time.
Hypotheses are developed regarding the etiologic agent, mode of transmission, and likely mechanism of exposure. Analytic
epidemiology is used to evaluate the associations between exposure and disease, taking into account the role of chance.

Community outbreaks of MRSA infection have been associated with crowded settings (eg, daycare, correctional facilities),
poor hygiene, sharing of personal items (eg, towel, razors, sports equipment), body piercings, and frequent skin contact (eg,
wrestling). In the real­life outbreak described in the vignette, the adolescents were members of a club fencing team and
shared equipment, including sensor wires that fencers wear under their clothes to record when an opponent’s weapon makes
contact. These wires had no routine schedule for cleaning and were determined to be the cause of MRSA transmission.

Strategies to limit the transmission of S aureus and prevent spread of infections in the community include:

Optimizing hand hygiene
Minimizing skin trauma
Practicing appropriate wound care, including keeping them covered
Optimizing personal hygiene, including bathing after activities involving skin­to­skin contact
Avoiding sharing personal items (eg, towels, razors, clothing, sports equipment)
Cleaning of shared equipment between uses
Regular cleaning of clothing and environmental surfaces

http://2016.prepid.courses.aap.org/script/february?req=201612122007488638&status=submit 1/2
12/13/2016 February
Children with MRSA wounds that can be covered do not need to be excused from school. In addition, there are limited data to
support the use of nasal mupirocin and chlorhexidine baths or washes for patients with MRSA infection or their close
contacts. Decolonization regimens may be considered on an individual basis for patients with multiple documented MRSA
infection recurrences, or when MRSA transmission continues to occur in a household or other cohort with close contact.
Patients at risk for recurrent staphylococcal infections include those with risk factors previously described, as well as those
who are obese, have disorders of neutrophil function, or have chronic skin conditions. In an outbreak situation, MRSA isolates
with similar susceptibility profiles may undergo molecular typing by the health department, but clinicians caring for affected
patients do not need to order molecular testing.

* Required *
Take Survey  

PREP Pearls
Community outbreaks of methicillin­resistant Staphylococcus aureus infection have been associated with crowded settings,
poor hygiene, sharing of personal items, body piercings, and frequent skin contact.

Patients are also at risk for recurrent staphylococcal infections if they are obese, have disorders of neutrophil function, or
chronic skin conditions.

Establishing the existence of an outbreak requires determining if the observed cases exceed the expected number of
cases.

The first step in an outbreak investigation is establishing a case definition.

American Board of Pediatrics Content Specification(s)
Know the approach to investigating and managing community outbreaks of CA­MRSA infections

Suggested Readings
American Academy of Pediatrics. Staphylococcal infections. In: Kimberlin DW, Brady MT, Jackson MA, Long SS, eds. Red
Book: 2015 Report of the Committee on Infectious Diseases. 30th ed. Elk Grove Village, IL: American Academy of Pediatrics;
2015:715­732.

Gorwitz RJ, Jernigan DB, Powers JH, et al. Strategies for clinical management of MRSA in the community: summary of an
experts’ meeting convened by the Centers for Disease Control and Prevention March 2006. US Centers for Disease Control
and Prevention website. http://www.cdc.gov/mrsa/community/clinicians/index.html

US Centers for Disease Control and Prevention. Healthcare­associated infections. US Centers for Disease Control and
Prevention website. http://www.cdc.gov/hai/outbreaks/outbreaktoolkit.html. Updated January 24, 2013

Comment On This Question

Page 24 of 115

Contact Us
Links may open in separate window
Copyright © 2016   American Academy of Pediatrics. All rights reserved. Pop­up Blocker may need to be disabled

http://2016.prepid.courses.aap.org/script/february?req=201612122007488638&status=submit 2/2
12/13/2016 March

Welcome  mohammed alsaiary [ Logout ]

Home PREP Pearls FAQs My Bookmarks CME Information

Overview Home  > March

Editorial Board
March   Enter Keyword Search
January
Question View:   All (8) Jump to Question
February
Print this Page Add to my Bookmarks Page 26 of 115
March

April Assessment History Mode: Learner  Exam 

May ASSESSMENT PROGRESS:  Total Questions:  8  Questions Answered:  0  Correct Answers:  0


June

July You currently have 8 questions unanswered in this assessment

August

September
Question: 1
October
An 18­month­old child awoke from his nap and is found to be febrile to 39.7°C. He is brought to the emergency department where he
November was found to have a proptotic right auricle, a bulging right tympanic membrane, postauricular swelling, and postauricular tenderness
on the same side. He is fussy but consolable; his neurologic examination is normal. A complete blood cell count and a blood culture
December
are obtained.

A clinical diagnosis of otitis media with mastoiditis is made and is supported by computed tomography. The pediatric hospitalist team
calls you for guidance about further management for this child.

In addition to parenteral antibiotics, the MOST appropriate next step is

Claim Credit A. lumbar puncture
Evaluation B. magnetic resonance image

My Learning Plan C. mastoidectomy
D. myringotomy
E. observation

Submit   Reset  

Page 26 of 115

Contact Us
Links may open in separate window
Copyright © 2016   American Academy of Pediatrics. All rights reserved. Pop­up Blocker may need to be disabled

http://2016.prepid.courses.aap.org/script/march?q­page=1&req=201612122011068560 1/1
12/13/2016 March

Welcome  mohammed alsaiary [ Logout ]

Home PREP Pearls FAQs My Bookmarks CME Information

Overview Home  > March

Editorial Board
March   Enter Keyword Search
January
Question View:   All (8) Jump to Question
February
Print this Page Add to my Bookmarks Page 26 of 115
March

April Assessment History Mode: Learner  Exam 

May ASSESSMENT PROGRESS:  Total Questions:  8  Questions Answered:  1  Correct Answers:  0


June

July You currently have 7 questions unanswered in this assessment

August

September
Question: 1
October
An 18­month­old child awoke from his nap and is found to be febrile to 39.7°C. He is brought to the emergency department where he
November was found to have a proptotic right auricle, a bulging right tympanic membrane, postauricular swelling, and postauricular tenderness
on the same side. He is fussy but consolable; his neurologic examination is normal. A complete blood cell count and a blood culture
December
are obtained.

A clinical diagnosis of otitis media with mastoiditis is made and is supported by computed tomography. The pediatric hospitalist team
calls you for guidance about further management for this child.

In addition to parenteral antibiotics, the MOST appropriate next step is

Claim Credit A. lumbar puncture
Evaluation B. magnetic resonance image

My Learning Plan C. mastoidectomy
D. myringotomy
E. observation

Incorrect View Peer Results

Correct Answer: D Average Percent Correct:  51.19%

Mastoiditis is a suppurative complication of the mastoid air cells; it is a potential complication of otitis
media caused by the continuity of the mucoperiosteal lining of the mastoid with that of the middle ear.

Most experts recommend drainage of the middle ear by myringotomy immediately in the early course
for therapeutic and diagnostic purposes. A specific etiologic diagnosis is important because of the
increasing incidence of infections caused by penicillin­resistant pneumococci and methicillin­resistant
Staphylococcus aureus. With adequate parenteral antibiotic coverage and myringotomy, many children
with an acute onset of posterior auricular swelling and tenderness, minimal or no posterior fluctuance, and no signs of
intracranial complications, are likely to respond to antibiotic therapy and can be managed with immediate myringotomy
(Figure). Simple mastoidectomy should also be performed in every case of unsuccessful subperiosteal abscess drainage or
presence of intracranial complications. Complications and the indications for surgical intervention are listed in the Table.
Temporal bone imaging is recommended to evaluate for bony destruction and intracranial complications; magnetic resonance
imaging is not indicated because computed tomography has been performed. Meningitis is a serious but uncommon
complication of mastoiditis; lumbar puncture is recommended to be performed only in a child with meningeal signs.

PREP Pearls
Drainage of the middle ear by myringotomy should be provided immediately in the early course for therapeutic and
diagnostic purposes.

Imaging with temporal bone computerized tomography is recommended to confirm the clinical diagnosis.

With adequate parenteral antibiotic coverage, many children with an acute onset of posterior auricular swelling and
tenderness, minimal or no posterior fluctuance, and no signs of intracranial complications are likely to respond to antibiotic
therapy and can be managed with immediate myringotomy without mastoidectomy.

Surgical intervention is indicated if there is nonresponse, progression of swelling or fluctuance, persistence of fever, ear
pain, or purulent drainage despite parenteral antibiotics for 3 to 5 days.

http://2016.prepid.courses.aap.org/script/march?req=201612122013055869&status=submit 1/2
12/13/2016 March

American Board of Pediatrics Content Specification(s)
Plan the evaluation of a patient with suspected mastoiditis

Anticipate the complications of mastoiditis

Suggested Readings
Chesney J, Black A, Choo D. What is the best practice for acute mastoiditis in children? Laryngoscope. 2014;124(5):1057­
1058. DOI: http://dx.doi.org/10.1002/lary.24306

Lewis, K, Shapiro NL, Cherry JD. Mastoiditis. In: Feigin RD, Cherry J, Demmler­Harrison GJ, Kaplan SL, eds. Feigin and
Cherry’s Textbook of Pediatric Infectious Diseases. 6th ed. Philadelphia, PA: Saunders Elsevier; 2009:238­244.

Psarommatis IM, Voudouris C, Douros K, Giannakopoulos P, Bairamis T, Carabinos C. Algorithmic management of pediatric
acute mastoiditis. Int J Pediatr Otorhinolaryngol. 2012;76(6):791­796. DOI: http://dx.doi.org/10.1016/j.ijporl.2012.02.042

Comment On This Question

Page 26 of 115

Contact Us
Links may open in separate window
Copyright © 2016   American Academy of Pediatrics. All rights reserved. Pop­up Blocker may need to be disabled

http://2016.prepid.courses.aap.org/script/march?req=201612122013055869&status=submit 2/2
12/13/2016 March

Welcome  mohammed alsaiary [ Logout ]

Home PREP Pearls FAQs My Bookmarks CME Information

Overview Home  > March

Editorial Board
March   Enter Keyword Search
January
Question View:   All (8) Jump to Question
February
Print this Page Add to my Bookmarks Page 27 of 115
March

April Assessment History Mode: Learner  Exam 

May ASSESSMENT PROGRESS:  Total Questions:  8  Questions Answered:  1  Correct Answers:  0


June

July You currently have 7 questions unanswered in this assessment

August

September
Question: 2
October
You are on a medical center utilization committee that is charged with deciding what diagnostic kits and reagents to purchase for the
November diagnosis of influenza. The committee asks you to review all types of commercially available tests.

December Of the influenza diagnostic tests available commercially, the MOST accurate type of test for the detection of influenza viruses is

A. antigen detection by monoclonal antibody
B. antigen detection by direct fluorescent antibody (DFA)
C. RNA detection by molecular assay

Claim Credit D. virus isolation in cell culture

Evaluation

My Learning Plan Submit   Reset  

Page 27 of 115

Contact Us
Links may open in separate window
Copyright © 2016   American Academy of Pediatrics. All rights reserved. Pop­up Blocker may need to be disabled

http://2016.prepid.courses.aap.org/script/march?q­page=2&req=201612122013060400 1/1
12/13/2016 March

Welcome  mohammed alsaiary [ Logout ]

Home PREP Pearls FAQs My Bookmarks CME Information

Overview Home  > March

Editorial Board
March   Enter Keyword Search
January
Question View:   All (8) Jump to Question
February
Print this Page Add to my Bookmarks Page 27 of 115
March

April Assessment History Mode: Learner  Exam 

May ASSESSMENT PROGRESS:  Total Questions:  8  Questions Answered:  2  Correct Answers:  1


June

July You currently have 6 questions unanswered in this assessment

August

September
Question: 2
October
You are on a medical center utilization committee that is charged with deciding what diagnostic kits and reagents to purchase for the
November diagnosis of influenza. The committee asks you to review all types of commercially available tests.

December Of the influenza diagnostic tests available commercially, the MOST accurate type of test for the detection of influenza viruses is

A. antigen detection by monoclonal antibody
B. antigen detection by direct fluorescent antibody (DFA)
C. RNA detection by molecular assay

Claim Credit D. virus isolation in cell culture

Evaluation

My Learning Plan Correct View Peer Results


Average Percent Correct:  73.81%

Diagnostic tests available for influenza are antigen detection assays of different types, molecular (RNA
detection) assays, viral culture, and serology. There are no validated commercially available serologic
assays, and influenza serologic testing should not be ordered. Molecular RNA assays such as reverse
transcription polymerase chain reaction (RT­PCR) have the highest sensitivity of all the tests, have
excellent specificity, and several of the assays differentiate between influenza A and B and between
influenza A virus subtypes. They are in use as part of respiratory viral panels in hospitals, reference,
and public health laboratories. They are more expensive to perform than the other tests, so they are
more often used as a “reflex” test after a rapid influenza virus detection test. Rapid influenza diagnostic tests (RIDTs) are
currently the most commonly used tests because they detect influenza viral antigens in respiratory specimens within minutes,
making them point­of­care tests. Most RIDTs are enzyme immunoassays that use monoclonal antibodies to detect influenza A
and B conserved nucleoproteins, but do not differentiate between influenza A virus subtypes. The RIDTs have high specificity
and one advantage is that the antigen is easily detected in nasopharyngeal specimens: aspirates in infants and young
children, and nasopharyngeal swabs in older children. The sensitivity of RIDTs is low compared to molecular assays or viral
culture, and the utility of RIDTs is being evaluated on an ongoing basis because of variable sensitivity of test kits. Despite their
limitations, RIDTs are widely used, and their use has been shown to influence physician behavior in the management of
influenza with reduced antibiotic prescribing and increased antiviral prescribing. The development of RIDTs with better
sensitivity could have an even greater impact on physician decision­making and resource utilization. Direct fluorescent
antibody (DFA) and indirect fluorescent antibody staining tests for antigen detection are sensitive and specific. They are less
commonly used now, as they require nasal wash specimens that are rich in epithelial cells and are labor­intensive. Viral
culture in tissue cells, though the most specific test, has lower sensitivity because it can detect only viable viruses. It requires
3 to 7 days to complete and is now used primarily in research and vaccine development settings. As would be expected, the
positive predictive value of any influenza test is highest during peak influenza activity.

Children with influenza experience a sudden onset of symptoms in about two­thirds of cases. Fever is moderate­to­high
grade, but in about 10% of children, there is only subjective or low­grade fever. Nonspecific constitutional symptoms,
especially headache, anorexia, and malaise, occur in most children, but only one­third of children report chills or myalgia in
contrast to two­thirds of adults. Upper respiratory symptoms (cough, nasal discharge and congestion, and sore throat) are
present in most children, especially those younger than 5 years of age. Gastrointestinal symptoms (vomiting, diarrhea,
abdominal pain) occur in less than one­third of children, and are more common in younger than in older children. Influenza A
and B infections are usually clinically indistinguishable. Common physical signs are conjunctival and pharyngeal injection in
about 60%, and cervical adenopathy in about one­third of children. Pulmonary rales or wheezes are uncommon in the
absence of complicating pneumonia. Asymptomatic infection is documented in 10% to 30% of individuals during seasonal
influenza epidemics. The symptoms of influenza in neonates and young infants may not include fever, and the infection often
presents as apnea, respiratory failure, or a sepsis­like picture. The primary presentation in infants may be pneumonia,
bronchiolitis, or croup. Premature infants are at risk for severe illness.

http://2016.prepid.courses.aap.org/script/march?req=201612122014518666&status=submit 1/2
12/13/2016 March
Complications of influenza occur both in previously healthy children and in children with underlying comorbidities such as
chronic cardiac, pulmonary, neurologic, and neuromuscular conditions. Rapid progression of the illness that can result in
death is not uncommon; in most years, more than 100 children die of influenza annually in the United States. Complications of
influenza are listed in the Table.

The chief mode of transmission of influenza is person­to­person by large­particle respiratory droplets that are generated by
coughing or sneezing. The distance that large­particle droplets can travel from a patient who coughs or sneezes is about 6 ft
or less. Indirect contact transmission, such as transfer from virus­contaminated hands or fomites to the nose or mouth, plays
a smaller role in transmission. All respiratory secretions and bodily fluids of infected patients are potentially infectious. The
role of airborne (small particle aerosol) transmission is probably negligible. The transmissibility of influenza viruses can vary
by strain. The source of infection in seasonal epidemics is usually school­aged children, with secondary spread to adults. In
healthcare settings, standard precautions including hand hygiene and droplet precautions should be put in place for the
duration of illness while caring for all individuals suspected of having influenza.

PREP Pearls
Molecular RNA assays for influenza have the best combination of sensitivity and specificity. Point­of­care rapid diagnostic
tests are also in common use, though their accuracy varies.

Complications of influenza occur in previously healthy children and in children with underlying conditions; influenza can be
fatal.

The main mode of transmission of influenza is person to person by large­particle respiratory droplet spread, but contact
transmission occurs as well.

American Board of Pediatrics Content Specification(s)
Differentiate the clinical manifestations of influenza in patients according to age, severity, and underlying disease

Understand the sources and modes of spread of the influenza viruses, including appropriate infection control procedures
for influenza viruses (droplet, contact, airborne) and when they are indicated

Judge the relative value and accuracy of the currently available tests for the diagnosis of influenza virus infection

Suggested Readings
Blaschke AJ, Shapiro DJ, Pavia AT, et al. A national study of the impact of rapid influenza testing on clinical care in the
emergency department. J Ped Infect Dis. 2014;3(2):112­118. DOI: http://dx.doi.org/10.1093/jpids/pit071

US Centers for Disease Control and Prevention. Prevention strategies for seasonal influenza in healthcare settings. US
Centers for Disease Control and Prevention website.
http://www.cdc.gov/flu/professionals/infectioncontrol/healthcaresettings.htm. Updated January 9, 2013

Uyeki TM. Influenza viruses. In: Cherry J, Demmler­Harrison GJ, Kaplan SL, Steinbach WJ, Hotez P, eds. Feigin and Cherry's
Textbook of Pediatric Infectious Diseases. 7th ed. Philadelphia, PA: Saunders Elsevier; 2013: 2326­2359.

Comment On This Question

Page 27 of 115

Contact Us
Links may open in separate window
Copyright © 2016   American Academy of Pediatrics. All rights reserved. Pop­up Blocker may need to be disabled

http://2016.prepid.courses.aap.org/script/march?req=201612122014518666&status=submit 2/2
12/13/2016 March

Welcome  mohammed alsaiary [ Logout ]

Home PREP Pearls FAQs My Bookmarks CME Information

Overview Home  > March

Editorial Board
March   Enter Keyword Search
January
Question View:   All (8) Jump to Question
February
Print this Page Add to my Bookmarks Page 28 of 115
March

April Assessment History Mode: Learner  Exam 

May ASSESSMENT PROGRESS:  Total Questions:  8  Questions Answered:  2


June

July You currently have 6 questions unanswered in this assessment

August

September
Question: 3
October
A 4­year­old boy who recently emigrated from West Africa is referred to you because of an abnormal complete blood cell count
November showing a hemoglobin of 6.1 g/dL (61 g/L), mean corpuscular volume of 62 fL, platelet count of 347 x 103/μL (347 x 109/L), and
December white blood cell count of 6,900/μL (6.9 x 109/L) with 38% neutrophils, 35% lymphocytes, 8% monocytes, and 19% eosinophils. He is
asymptomatic and his physical examination is unremarkable except for pallor. Among other studies, a stool examination for ova and
parasites is shown in the Figure.

Of the following, the treatment agent MOST likely to result in cure is

A. albendazole
Claim Credit
B. metronidazole
Evaluation C. praziquantel
My Learning Plan D. pyrantel pamoate

Submit   Reset  

Page 28 of 115

Contact Us
Links may open in separate window
Copyright © 2016   American Academy of Pediatrics. All rights reserved. Pop­up Blocker may need to be disabled

http://2016.prepid.courses.aap.org/script/march?req=201612122111013064&q­page=3&status=submit 1/1
Figure.

Courtesy of the Centers for Disease Control (CDC)


12/13/2016 March

Welcome  mohammed alsaiary [ Logout ]

Home PREP Pearls FAQs My Bookmarks CME Information

Overview Home  > March

Editorial Board
March   Enter Keyword Search
January
Question View:   All (8) Jump to Question
February
Print this Page Add to my Bookmarks Page 28 of 115
March

April Assessment History Mode: Learner  Exam 

May ASSESSMENT PROGRESS:  Total Questions:  8  Questions Answered:  3  Correct Answers:  1


June

July You currently have 5 questions unanswered in this assessment

August

September
Question: 3
October
A 4­year­old boy who recently emigrated from West Africa is referred to you because of an abnormal complete blood cell count
November showing a hemoglobin of 6.1 g/dL (61 g/L), mean corpuscular volume of 62 fL, platelet count of 347 x 103/μL (347 x 109/L), and
December white blood cell count of 6,900/μL (6.9 x 109/L) with 38% neutrophils, 35% lymphocytes, 8% monocytes, and 19% eosinophils. He is
asymptomatic and his physical examination is unremarkable except for pallor. Among other studies, a stool examination for ova and
parasites is shown in the Figure.

Of the following, the treatment agent MOST likely to result in cure is

A. albendazole
Claim Credit
B. metronidazole
Evaluation C. praziquantel
My Learning Plan D. pyrantel pamoate

Incorrect View Peer Results

Correct Answer: A Average Percent Correct:  69.05%

The clinical scenario and stool microscopy is most consistent with a diagnosis of hookworm disease,
caused by either Ancylostoma duodenale or Necator americanus. Eggs of these organisms are
indistinguishable, but N americanus predominates in most of Africa. A meta­analysis of treatments for
hookworm disease showed cure rates of 72% (95% confidence interval, 59%–81%) for albendazole,
15% (1%–27%) for mebendazole, and 31% (19%– 42%) for pyrantel pamoate. Praziquantel has some
hookworm activity, but relatively low cure rates (25%–35%) in noncomparative treatment studies.
Metronidazole has no clinical utility in hookworm disease.

Hookworm larvae usually gain access to the human host by penetration of skin, with subsequent bloodstream migration to the
right heart and lungs. Skin penetration occasionally causes a pruritic reaction called “dew itch” (related to the fact that larvae
are prevalent in moist soil) or “ground itch.” After days to weeks, a mild pneumonitis may develop. Rarely, ingestion of A
duodenale results in gastrointestinal colonization, bypassing the cutaneous and respiratory phases. Once the larvae reach the
small intestine by either direct ingestion or via the portal circulation, they develop into adult worms that both feed on and lyse
human red blood cells.

The most common clinical presentation of hookworm disease is that of iron­deficiency anemia, though rarely patients may
present earlier with cutaneous or respiratory symptoms. Hypoproteinemia with intestinal blood loss can rarely present as
edema or anasarca. Findings are very nonspecific, so travel and exposure history is key to accurate diagnosis. Differential
diagnosis in returning travelers or immigrants includes malaria, schistosomiasis, malnutrition, and hemoglobinopathies. Direct
examination of stool for eggs is diagnostic. Prognosis is good with treatment, though long­standing anemia may result in
permanent growth and developmental sequelae.

Also worth noting are 2 unusual features of hookworm disease. Ancylostoma braziliense, found only in the western
hemisphere, is a zoonotic disease causing “creeping eruption” or cutaneous larva migrans. These individuals do not have
intestinal egg excretion. Treatment with albendazole or ivermectin is effective. Secondly, it has been hypothesized that
arrested A duodenale eggs can enter mammary glands and breast milk, resulting in infantile hookworm disease.

School­based “de­worming” programs hold some promise for control in endemic countries. Hookworm vaccine development
and testing is ongoing, as are clinical trials of combination antiparasitic therapies.

PREP Pearls

http://2016.prepid.courses.aap.org/script/march?req=201612122016318030&status=submit 1/2
12/13/2016 March
Hookworm disease primarily presents with the nonspecific symptoms of iron­deficiency anemia.

Albendazole is the treatment of choice for hookworm disease, though cure rates are relatively low.

American Board of Pediatrics Content Specification(s)
Understand the importance and geographic distribution of hookworm infestation (Ascaris duodenale, Necator americanus)

Recognize the clinical manifestations of hookworm infestation involving major organ systems (skin, pulmonary,
gastrointestinal)

Understand the method of diagnosis and treatment of hookworm infestation

Suggested Readings
Bethony J, Brooker S, Albonico M, et al. Soil­transmitted helminth infections: ascariasis, trichuriasis, and hookworm. Lancet.
2006;367(9521):1521­1532. DOI: http://dx.doi.org/10.1016/S0140­6736(06)68653­4

Hotez PJ, Brooker S, Bethony JM, Bottazzi ME, Loukas A, Xiao S. Hookworm infection. N Engl J Med. 2004;351(8):799­807.
DOI: http://dx.doi.org/10.1056/NEJMra032492

Hotez PJ. Parasitic nematode infections. In: Cherry JD, Harrison GJ, Kaplan SL, Steinbach WJ, eds. Feigin and Cherry’s
Textbook of Pediatric Infectious Diseases. 7th ed. Philadelphia, PA: Saunders Elsevier; 2014: 3012­3029.

Keiser J, Utzinger J. Efficacy of current drugs against soil­transmitted helminth infections: systematic review and meta­
analysis. JAMA. 2008;299(16):1937­1948. DOI: http://dx.doi.org/10.1001/jama.299.16.1937

Speich B, Ali SM, Ame SM, et al. Efficacy and safety of albendazole plus ivermectin, albendazole plus mebendazole,
albendazole plus oxantel pamoate, and mebendazole alone against Trichuris trichiura and concomitant soil­transmitted
helminth infections: a four­arm, randomised controlled trial. Lancet Infect Dis. 2015;15(3):277­284. DOI:
http://dx.doi.org/10.1016/S1473­3099(14)71050­3

Comment On This Question

Page 28 of 115

Contact Us
Links may open in separate window
Copyright © 2016   American Academy of Pediatrics. All rights reserved. Pop­up Blocker may need to be disabled

http://2016.prepid.courses.aap.org/script/march?req=201612122016318030&status=submit 2/2
12/13/2016 March

Welcome  mohammed alsaiary [ Logout ]

Home PREP Pearls FAQs My Bookmarks CME Information

Overview Home  > March

Editorial Board
March   Enter Keyword Search
January
Question View:   All (8) Jump to Question
February
Print this Page Add to my Bookmarks Page 29 of 115
March

April Assessment History Mode: Learner  Exam 

May ASSESSMENT PROGRESS:  Total Questions:  8  Questions Answered:  3  Correct Answers:  1


June

July You currently have 5 questions unanswered in this assessment

August

September
Question: 4
October
You have just presented a lecture in which you review antibiotics for the pediatric residents. Most of your discussion was centered on
November the use of ß­lactam and macrolide antibiotics for common infections, and you mentioned quinolones only in passing. A resident asks
you about his observation of the increasing use of this class of agents in outpatient treatment regimens. You spend a few more
December
minutes reviewing new data on safety of fluoroquinolones in children and list the infections for which fluoroquinolones are considered
effective therapy in children, despite the lack of US Food and Drug Administration­approved indications.

Of the following clinical uses, the one for which we have the BEST quality of clinical study data in children is

A. ciprofloxacin for salmonellosis and enteric fever

Claim Credit B. ciprofloxacin for shigellosis

Evaluation C. ciprofloxacin for urinary tract infections caused by Pseudomonas aeruginosa
D. levofloxacin for acute otitis media
My Learning Plan
E. levofloxacin for community­acquired pneumonia

Submit   Reset  

Page 29 of 115

Contact Us
Links may open in separate window
Copyright © 2016   American Academy of Pediatrics. All rights reserved. Pop­up Blocker may need to be disabled

http://2016.prepid.courses.aap.org/script/march?q­page=4&req=201612122016325374 1/1
12/13/2016 March

Welcome  mohammed alsaiary [ Logout ]

Home PREP Pearls FAQs My Bookmarks CME Information

Overview Home  > March

Editorial Board
March   Enter Keyword Search
January
Question View:   All (8) Jump to Question
February
Print this Page Add to my Bookmarks Page 29 of 115
March

April Assessment History Mode: Learner  Exam 

May ASSESSMENT PROGRESS:  Total Questions:  8  Questions Answered:  4  Correct Answers:  1


June

July You currently have 4 questions unanswered in this assessment

August

September
Question: 4
October
You have just presented a lecture in which you review antibiotics for the pediatric residents. Most of your discussion was centered on
November the use of ß­lactam and macrolide antibiotics for common infections, and you mentioned quinolones only in passing. A resident asks
you about his observation of the increasing use of this class of agents in outpatient treatment regimens. You spend a few more
December
minutes reviewing new data on safety of fluoroquinolones in children and list the infections for which fluoroquinolones are considered
effective therapy in children, despite the lack of US Food and Drug Administration­approved indications.

Of the following clinical uses, the one for which we have the BEST quality of clinical study data in children is

A. ciprofloxacin for salmonellosis and enteric fever

Claim Credit B. ciprofloxacin for shigellosis

Evaluation C. ciprofloxacin for urinary tract infections caused by Pseudomonas aeruginosa
D. levofloxacin for acute otitis media
My Learning Plan
E. levofloxacin for community­acquired pneumonia

Incorrect View Peer Results

Correct Answer: E Average Percent Correct:  13.10%

The fluoroquinolones (FQ) are broad spectrum antimicrobials active against Gram­negative pathogens
and the “atypical” bacteria Mycoplasma pneumoniae, Chlamydophila pneumoniae, Legionella
pneumophila, and, to a lesser extent, Gram­positive bacteria. They have in vitro activity against
staphylococcal species (including methicillin­resistant Staphylococcus aureus) and enterococcal
species, however, they are not preferred drugs to treat infections caused by these pathogens due to
concerns over clinical efficacy. Third­generation FQs (eg, moxifloxacin) also have enhanced activity
against anaerobic bacilli. In addition, they have activity against mycobacterial species, including
Mycobacterium tuberculosis and some nontuberculous mycobacteria.

The currently approved indications for ciprofloxacin or levofloxacin, for infections caused by susceptible organisms, in patients
18 years of age and older include: urinary tract infections (uncomplicated and complicated), chronic bacterial prostatitis,
community­acquired and nosocomial lower respiratory tract infections, acute sinusitis, skin and skin structure infections, bone
and joint infections, infectious diarrhea, typhoid fever, complicated intra­abdominal infections (used in combination with
metronidazole), empirical therapy for febrile neutropenic patients (in combination with piperacillin), and prophylaxis and
treatment of plague. They are also approved for adult and pediatric patients for the prevention of inhalational anthrax following
exposure to aerosolized Bacillus anthracis. The only other approved indication for pediatric patients is ciprofloxacin for
complicated urinary tract infections and pyelonephritis caused by Escherichia coli in children 1 to 17 years of age.
Nevertheless, the use of FQs for other infections is increasingly being accepted in pediatric practice (Table). Among the
nonapproved indications in the vignette, the best clinical study data is available for levofloxacin from prospective randomized
controlled trials in children 6 months of age and older with community­acquired pneumonia. In all of these trials, clinical cure
rates were noninferior to oral or intravenously administered comparator agents. Although the manufacturer has not submitted
an application for this pediatric indication, levofloxacin is a choice for second line therapy, especially for multidrug­resistant
strains of Streptococcus pneumoniae. Levofloxacin (and gatifloxacin) have been studied in children with recurrent or
persistent otitis media, but not in simple acute otitis media. With rising rates of ampicillin and trimethoprim­sulfamethoxazole
resistance in Shigella infections, ciprofloxacin is a recommend treatment choice, despite lack of randomized controlled data.
Ciprofloxacin was included in studies of Salmonella infections that included enteric fever, but not in studies that included
comparator drugs, and caution is to be exercised in children returning from South Asia because of a high prevalence of
decreased susceptibility to FQs. The American Academy of Pediatrics endorses ciprofloxacin as oral therapy for urinary tract
infections caused by Pseudomonas aeruginosa or other multidrug­resistant, Gram­negative bacilli in children 1 to 17 years of
age because of a lack of alternatives.

http://2016.prepid.courses.aap.org/script/march?req=201612122018278174&status=submit 1/2
12/13/2016 March
There is cumulative experience with safety of FQs in children younger than 18 years of age. Articular and epiphyseal plate
cartilage injury is documented in all animal models of FQ toxicity. This is a result of formation of magnesium chelate
complexes that impair cartilage matrix integrity in weight­bearing joints and has led to concern about impairment of growth
potential in children. Retrospective cohort and case control studies and case reports showed conflicting results, so data from
large prospective safety studies were requested by US Food and Drug Administration (FDA) under its program of pediatric
drug development. For ciprofloxacin, data from several prospective studies of the treatment of complicated urinary tract
infections and pyelonephritis performed from 2004 onward were analyzed. The studies were designed to capture any sign of
cartilage or tendon toxicity by eliciting a history of a comprehensive list of bone and joint complaints. In US children, a higher
rate of potential musculoskeletal toxicity was observed (21% ciprofloxacin, 11% comparator drug). Safety data on levofloxacin
were recorded in about 2,500 children enrolled in 3 large multicenter efficacy trials. At 2 month and 12 month time points,
spontaneously reported musculoskeletal adverse events (arthritis, arthralgias, tendinopathy, or gait abnormality) were twice
as likely (at 2%­4%) in levofloxacin­treated children compared to those treated with comparator antibiotics. Conversely,
cumulative 5­year musculoskeletal adverse events that included ongoing arthropathy, peripheral neuropathy, abnormal bone
development, scoliosis, walking difficulty, myalgia, tendon disorder, hypermobility syndrome, and spine, hip, or shoulder pain,
were more common in the control group (4% versus 2%). In a systematic review of ciprofloxacin safety in about 16,000
children enrolled in 105 studies, the estimated risk of a musculoskeletal adverse event was approximately 1 in 62.5 patients,
and all were reversible. The most commonly reported musculoskeletal event was arthralgia in a knee joint. From 23 controlled
clinical trials, the increased risk of an arthropathy was 57% from ciprofloxacin compared to the comparator antibiotic (odds
ratio, 1.57).

There is a FDA black box warning for FQs for the rare complication of tendon rupture, which occurs most commonly in one or
both Achilles tendons. The rate is 15 to 20 per 100,000 treated patients in adults and is associated with risk factors that
include advanced age, steroid therapy, renal disease, gout, and rheumatoid arthritis. To date, there have been no reports of
this rare complication in a child treated with a FQ, although this is not proof that the risk does not exist. A second black box
warning is for possible exacerbation of muscle weakness in persons with myasthenia gravis. Other adverse effects of FQs are
gastrointestinal symptoms, skin rashes, and allergic reactions. Infrequent adverse effects in children include neutropenia,
eosinophilia, elevated liver enzymes, prolongation of QT interval, and photosensitivity. Reported rates of neurologic events in
the FDA safety database, which includes data requested from prospective studies, are statistically similar between children
treated with a FQ or a comparator drug. It is prudent to avoid the use of a FQ in a patient who is receiving another drug that
may prolong the QT interval.

Fluoroquinolones are frequently clinically indicated in the treatment of children with complicated infections, therefore parents
and patients should be verbally counseled on the potential for adverse events. To date, there is no published evidence that
shows sustained injury to developing bones or joints, but FDA analysis of ciprofloxacin and levofloxacin safety data does
suggest the possibility of increased musculoskeletal adverse effects in children. In addition, the patient is to call their
healthcare provider at the first sign of any tendon pain, swelling, or inflammation.

* Required *
Take Survey  

PREP Pearls
Fluoroquinolones (FQs) are effective therapy for several pediatric infections other than urinary tract infections, despite lack
of US Food and Drug Administration indications.

Musculoskeletal symptoms occurred frequently in US trials of FQs in children, but permanent damage has not been
observed to date.

Parents and patients should be verbally counseled on the potential for adverse events, especially arthralgias and tendon
rupture.

American Board of Pediatrics Content Specification(s)
Know the spectrum of antibacterial activity of quinolones, and their approved indications for use

Know the indications for use of quinolones in children younger than 18 years of age, including UTI

Know the potential toxicity of quinolones including conduction disorders and tendon rupture

Suggested Readings
Adefurin A, Sammons H, Jacqz­Aigrain E, Choonara I. Ciprofloxacin safety in paediatrics: a systematic review. Arch Dis Child.
2011;96(9):874­880. DOI: http://dx.doi.org/10.1136/adc.2010.208843

Bradley JS, Jackson MA. The use of systemic and topical fluoroquinolones. Pediatrics. 2011;128(4):e1034­e1045. DOI:
http://dx.doi.org/10.1542/peds.2011­1496

Choi SH, Kim EY, Kim YJ. Systemic use of fluoroquinolone in children. Korean J Pediatr. 2013;56(5):196­201. DOI:
http://dx.doi.org/10.3345/kjp.2013.56.5.196

Comment On This Question

Page 29 of 115

Contact Us
Links may open in separate window
Copyright © 2016   American Academy of Pediatrics. All rights reserved. Pop­up Blocker may need to be disabled

http://2016.prepid.courses.aap.org/script/march?req=201612122018278174&status=submit 2/2
12/13/2016 March

Welcome  mohammed alsaiary [ Logout ]

Home PREP Pearls FAQs My Bookmarks CME Information

Overview Home  > March

Editorial Board
March   Enter Keyword Search
January
Question View:   All (8) Jump to Question
February
Print this Page Add to my Bookmarks Page 30 of 115
March

April Assessment History Mode: Learner  Exam 

May ASSESSMENT PROGRESS:  Total Questions:  8  Questions Answered:  4  Correct Answers:  1


June

July You currently have 4 questions unanswered in this assessment

August

September
Question: 5
October
A 16­year­old adolescent boy with AIDS due to HIV infection presents with odynophagia, retrosternal chest pain, and endoscopy­
November confirmed Candida esophagitis that has been unresponsive to oral fluconazole for 5 days, followed by posaconazole for 1 week. In
addition to his antifungal therapy, he is taking abacavir­lamivudine­dolutegravir for HIV infection. His temperature is 38°C, heart rate
December
is 90 beats/min, respiratory rate is 16 breaths/min, and blood pressure is 90/60 mm Hg. Physical examination is significant for a thin
male adolescent with pallor and poor dentition but no oral lesions. His CD4 cell count is 95/µL.

Of the following, the MOST likely cause of his ongoing symptoms is

A. drug­resistant infection

Claim Credit B. eosinophilic esophagitis

Evaluation C. medication esophagitis
D. viral coinfection
My Learning Plan

Submit   Reset  

Page 30 of 115

Contact Us
Links may open in separate window
Copyright © 2016   American Academy of Pediatrics. All rights reserved. Pop­up Blocker may need to be disabled

http://2016.prepid.courses.aap.org/script/march?q­page=5&req=201612122018282861 1/1
12/13/2016 March

Welcome  mohammed alsaiary [ Logout ]

Home PREP Pearls FAQs My Bookmarks CME Information

Overview Home  > March

Editorial Board
March   Enter Keyword Search
January
Question View:   All (8) Jump to Question
February
Print this Page Add to my Bookmarks Page 30 of 115
March

April Assessment History Mode: Learner  Exam 

May ASSESSMENT PROGRESS:  Total Questions:  8  Questions Answered:  5  Correct Answers:  1


June

July You currently have 3 questions unanswered in this assessment

August

September
Question: 5
October
A 16­year­old adolescent boy with AIDS due to HIV infection presents with odynophagia, retrosternal chest pain, and endoscopy­
November confirmed Candida esophagitis that has been unresponsive to oral fluconazole for 5 days, followed by posaconazole for 1 week. In
addition to his antifungal therapy, he is taking abacavir­lamivudine­dolutegravir for HIV infection. His temperature is 38°C, heart rate
December
is 90 beats/min, respiratory rate is 16 breaths/min, and blood pressure is 90/60 mm Hg. Physical examination is significant for a thin
male adolescent with pallor and poor dentition but no oral lesions. His CD4 cell count is 95/µL.

Of the following, the MOST likely cause of his ongoing symptoms is

A. drug­resistant infection

Claim Credit B. eosinophilic esophagitis

Evaluation C. medication esophagitis
D. viral coinfection
My Learning Plan

Incorrect View Peer Results
Correct Answer: D Average Percent Correct:  48.81%

The patient described in the vignette has AIDS due to HIV infection and Candida esophagitis that is
unresponsive to antifungal therapy. Most cases of esophageal candidiasis are caused by Candida
albicans and respond to therapy with fluconazole. Esophagitis caused by non­albicans Candida
species is less common and usually responds to posaconazole (age older than 12 years), itraconazole
solution, or voriconazole. Patients who do not demonstrate an adequate response to antifungal therapy
for Candida esophagitis, such as the patient described in the vignette, should be evaluated for
concomitant viral infections (eg, cytomegalovirus, herpes simplex virus), which occur commonly in HIV­
infected patients with CD4 cell counts less than 200/μL.

Candida esophagitis occurs most commonly in AIDS patients, but can also be observed in patients with hematologic
malignancies and idiopathic CD4+ lymphocytopenia. It occasionally occurs in HIV­negative patients taking inhaled
corticosteroids. The diagnosis of Candida esophagitis is presumed in at­risk patients with odynophagia, retrosternal pain, and
oral thrush; however, oral thrush may be absent. Treatment with antifungal therapy on the basis of the patient’s history and
examination may be employed, but further evaluation is warranted if symptoms do not improve within 3 to 4 days. The
diagnosis of Candida esophagitis is confirmed by endoscopic visualization of white mucosal plaques and biopsy confirming
the presence of invading yeast by histopathology and culture.

Oral fluconazole is the preferred treatment of Candida esophagitis. Intravenous fluconazole, an echinocandin (eg, micafungin,
caspofungin, and anidulafungin), or amphotericin B can be used in patients who cannot tolerate oral fluconazole. In children
with refractory Candida esophagitis, posaconazole (age older than 12 years), itraconazole solution or voriconazole are
recommended. Treatment is for 14 to 21 days, but depends on the patient’s immune status and clinical response. Fluconazole
prophylaxis is recommended for patients with recurrent Candida esophagitis.

Eosinophilic esophagitis typically presents with dysphagia, food impaction, and retrosternal chest pain in immunocompetent
teenagers and adults. Younger children may have nausea, feeding problems, and abdominal pain. There is a strong
association with other allergic disorders such as allergic rhinitis, atopic dermatitis, and asthma. The diagnosis is made by
esophageal biopsy that reveals an eosinophilic­predominant inflammation. Eosinophilic esophagitis in HIV­infected patients is
uncommon.

Patients with medication esophagitis usually present with retrosternal chest pain, dysphagia, and a history of ingestion of
medications known to cause esophageal injury (Table). The diagnosis is confirmed by endoscopic visualization of an
ulcerative lesion(s) with relatively normal surrounding mucosa. The patient described in the vignette does not have the
characteristic features of medication esophagitis.

http://2016.prepid.courses.aap.org/script/march?req=201612122020237232&status=submit 1/2
12/13/2016 March

PREP Pearls
Candida esophagitis most commonly occurs in immunocompromised patients and presents with odynophagia and
retrosternal chest pain.

The diagnosis of Candida esophagitis is confirmed by endoscopy with biopsy of white mucosal lesions.

Fluconazole, an echinocandin, or amphotericin B are the primary antifungal therapies recommended for patients with
esophageal candidiasis.

Viral coinfection should be considered in patients with Candida esophagitis who do not respond to appropriate antifungal
therapy within several days.

American Board of Pediatrics Content Specification(s)
Recognize typical predisposing factors and clinical characteristics of Candida esophagitis

Plan the treatment of Candida esophagitis (effective drugs, routes, duration)

Suggested Readings
American Academy of Pediatrics. Candidiasis. In: Kimberlin DW, Brady MT, Jackson MA, Long SS, eds. Red Book: 2015
Report of the Committee on Infectious Diseases. 30th ed. Elk Grove Village, IL: American Academy of Pediatrics; 2015: 275­
280.

Pappas PG, Kauffman CA, Andes D, et al. Clinical practice guidelines for the management of candidiasis: 2009 Update by the
Infectious Diseases Society of America. Clin Infect Dis. 2009;48(5):503­535. DOI: http://dx.doi.org/10.1086/596757

Comment On This Question

Page 30 of 115

Contact Us
Links may open in separate window
Copyright © 2016   American Academy of Pediatrics. All rights reserved. Pop­up Blocker may need to be disabled

http://2016.prepid.courses.aap.org/script/march?req=201612122020237232&status=submit 2/2
12/13/2016 March

Welcome  mohammed alsaiary [ Logout ]

Home PREP Pearls FAQs My Bookmarks CME Information

Overview Home  > March

Editorial Board
March   Enter Keyword Search
January
Question View:   All (8) Jump to Question
February
Print this Page Add to my Bookmarks Page 31 of 115
March

April Assessment History Mode: Learner  Exam 

May ASSESSMENT PROGRESS:  Total Questions:  8  Questions Answered:  5  Correct Answers:  1


June

July You currently have 3 questions unanswered in this assessment

August

September
Question: 6
October
You are speaking to a group of residents about the different branches and functions of the immune system.
November
Of the following, the MOST accurate statement regarding cell­mediated immunity is
December
A. activated cytotoxic T­lymphocytes induce apoptosis of cells
B. activation of the cytotoxic T­lymphocytes results in the production of plasma cells
C. antibody production is the major mechanism of protection
D. cytotoxic T­lymphocyte killing is antigen­nonspecific
Claim Credit

Evaluation
Submit   Reset  
My Learning Plan
Page 31 of 115

Contact Us
Links may open in separate window
Copyright © 2016   American Academy of Pediatrics. All rights reserved. Pop­up Blocker may need to be disabled

http://2016.prepid.courses.aap.org/script/march?req=201612122020551918 1/1
12/13/2016 March

Welcome  mohammed alsaiary [ Logout ]

Home PREP Pearls FAQs My Bookmarks CME Information

Overview Home  > March

Editorial Board
March   Enter Keyword Search
January
Question View:   All (8) Jump to Question
February
Print this Page Add to my Bookmarks Page 31 of 115
March

April Assessment History Mode: Learner  Exam 

May ASSESSMENT PROGRESS:  Total Questions:  8  Questions Answered:  6  Correct Answers:  2


June

July You currently have 2 questions unanswered in this assessment

August

September
Question: 6
October
You are speaking to a group of residents about the different branches and functions of the immune system.
November
Of the following, the MOST accurate statement regarding cell­mediated immunity is
December
A. activated cytotoxic T­lymphocytes induce apoptosis of cells
B. activation of the cytotoxic T­lymphocytes results in the production of plasma cells
C. antibody production is the major mechanism of protection
D. cytotoxic T­lymphocyte killing is antigen­nonspecific
Claim Credit

Evaluation Correct View Peer Results


My Learning Plan Average Percent Correct:  67.86%

Historically, the immune system has been separated into 2 branches:

1. cell­mediated immunity, an immune response that does not involve antibodies, but rather
involves the activation of phagocytes, antigen­specific cytotoxic T­lymphocytes (CTLs), and the
release of various cytokines in response to an antigen
2. humoral immunity, for which the protective function of immunization (development of antibodies)
is found in the cell­free bodily fluid or serum

With cell­mediated immunity, CD4 cells or helper T cells provide protection against different pathogens. Cell­mediated
immunity protects the body by activating antigen­specific CTLs that are able to induce apoptosis in body cells displaying
epitopes of foreign antigen on their surface, such as virus­infected cells, cells with intracellular bacteria, and cancer cells
displaying tumor antigens. Macrophages and natural killer cells are also activated, enabling them to destroy pathogens, and
various cells are stimulated to secrete a variety of cytokines that influence the function of other cells involved in adaptive and
innate immune responses. Cytotoxic T lymphocytes are not fully mature when they exit the thymus. They have a functional T­
cell receptor (TCR) that recognizes antigen, but they cannot lyse a target cell. They must differentiate into fully functional
effector CTL. Cytotoxic cells differentiate from a pre­CTL in response to 2 signals: specific antigen associated with class I
major histocompatibility complex (MHC), on a stimulator cell, and by cytokines produced by T helper 1 cells, especially
interleukin­2 and interferon­γ. Cytotoxic T lymphocyte killing is antigen­specific; to be killed by a CTL, the target cell must bear
the same class I MHC­associated antigen that triggered pre­CTL differentiation. Cytotoxic T lymphocytes are triggered to kill
when they recognize the target antigen associated with a cell surface MHC molecule. Cytotoxic T lymphocytes utilize several
mechanisms to kill target cells, some of which require direct cell to cell contact and others that result from the production of
certain cytokines. In all cases, death of the target cells is a result of apoptosis. Macrophages are also activated by the
cytokines produced by T helper 1 cells, are involved in presenting antigen to T helper cells, and have various effector
functions (eg, cytokine production, bactericidal and tumoricidal activities).

Humoral immunity (or antibody‐mediated immunity) involves B cells that recognize antigens or pathogens that are circulating
in the lymph or blood. The antigens bind to B cells, and interleukins or helper T cells costimulate the B cells to activate them
and initiate B­cell proliferation and the production of plasma cells. The plasma cells bear antibodies with the identical antigen
specificity as the antigen receptors of the activated B cells. The antibodies are released and circulate through the body,
binding to antigens.

Dendritic cells (DCs) are part of the DC system of antigen presenting cells (APC). They are specialized accessory cells whose
primary function is the initiation and modulation of adaptive immune responses through the stimulation of quiescent, naïve,
and memory B and T lymphocytes. Dendritic cells are found in all different tissues of the body. Initiation and modulation of B­
and T­cell functions occurs through the DCs. Dendritic cells have major effects on B­cell growth, differentiation, and
immunoglobulin secretion. Dendritic cells activate and expand T helper cells, which in turn induce B­cell growth and antibody
production. Naïve B cells respond uniquely to the secretion of soluble factors, especially interleukin­12 produced by interstitial

http://2016.prepid.courses.aap.org/script/march?req=201612122021502384&status=submit 1/2
12/13/2016 March
DCs. They stimulate the production of antibodies directly and the proliferation of B cells that have been stimulated by CD40L
on activated T cells. T lymphocytes, however, need the antigen to be processed and presented to them by an APC. Dendritic
cells capture and process antigens, and display large amounts of MHC­peptide complexes at their surface. They upregulate
their costimulatory molecules and migrate to lymphoid organs where they activate antigen­specific T cells. The T­cell antigen
receptors recognize fragments of antigens bound to molecules of the MHC on the surface of the APC. The peptide­binding
proteins are of 2 types (MHC class I and MHC class II), which stimulate cytotoxic T cells and helper T cells, respectively.
Dendritic cell activities are induced by infectious agents and inflammatory products, so that DCs are mobile sentinels found in
multiple tissue types with multiple specialized subtypes that bring antigens to T cells and express costimulators, such as
interleukin­12, for the stimulation of naïve B cells and the induction of adaptive immunity.

PREP Pearls
Protection provided by cell­mediated immunity is through activating antigen­specific cytotoxic T­lymphocytes that are able
to induce apoptosis in body cells displaying epitopes of foreign antigen on their surface.

Cytotoxic T­lymphocytes are activated in response to 2 signals: specific antigen associated with class I major
histocompatibility complex on a stimulator cell and by cytokines produced by T helper 1 cells, especially interleukin­2 and
interferon­γ.

Cytotoxic T­lymphocytes utilize several mechanisms to kill target cells, some of which require direct cell to cell contact and
others that result from the production of certain cytokines.

American Board of Pediatrics Content Specification(s)
Know that cell‑mediated immunity is dependent upon the interaction of T cells with macrophages and dendritic cells

Suggested Readings
Banchereau J, Steinman RM. Dendritic cells and the control of immunity. Nature. 1998;392(6673):245­252. DOI:
http://dx.doi.org/10.1038/32588

Bishop GA, Hostager BS. B lymphocyte activation by contact­mediated interactions and T lymphocytes. Curr Opin Immunol.
2001;13(3):278­285. DOI: http://dx.doi.org/10.1016/S0952­7915(00)00216­8

Broere F, Apasov SG, Sitkovsky MV, van Eden W. T cell subsets and T­cell mediated immunity. In: Nijkamp FP, Parnham MJ,
eds. Principles of Immunopharmacology. 3rd ed. Basel, Switzerland: Springer Basel AG; 2011: 15­27.

Dayer J­M. How T­lymphocytes are activated and become activators by cell­cell interaction. Eur Respir J Suppl.
2003;22(44):10s­15s. DOI: http://dx.doi.org/10.1183/09031936.03.00000403b

Comment On This Question

Page 31 of 115

Contact Us
Links may open in separate window
Copyright © 2016   American Academy of Pediatrics. All rights reserved. Pop­up Blocker may need to be disabled

http://2016.prepid.courses.aap.org/script/march?req=201612122021502384&status=submit 2/2
12/13/2016 March

Welcome  mohammed alsaiary [ Logout ]

Home PREP Pearls FAQs My Bookmarks CME Information

Overview Home  > March

Editorial Board
March   Enter Keyword Search
January
Question View:   All (8) Jump to Question
February
Print this Page Add to my Bookmarks Page 32 of 115
March

April Assessment History Mode: Learner  Exam 

May ASSESSMENT PROGRESS:  Total Questions:  8  Questions Answered:  6  Correct Answers:  2


June

July You currently have 2 questions unanswered in this assessment

August

September
Question: 7
October
A 15­year­old patient with cystic fibrosis (genotype G551D and ΔF508) is admitted for a pulmonary exacerbation and the pediatric
November infectious diseases service is consulted for assistance in determining antibiotic treatment. While on rounds, the pharmacist reviews
all the medications for the patient (Table 1).
December
The pharmacist reviews the drug ivacaftor and its recent breakthrough in understanding and treating the basic defect in cystic
fibrosis; he highlights the future impact this drug may have on lung function, chronic infection, and outcome for this patient.

The target of ivacaftor in this patient is MOST likely to potentiate the

A. calcium­activated chloride channel
Claim Credit
B. chloride ion transporter
Evaluation
C. Δ F508 allele
My Learning Plan D. transmembrane regulatory protein

Submit   Reset  

Page 32 of 115

Contact Us
Links may open in separate window
Copyright © 2016   American Academy of Pediatrics. All rights reserved. Pop­up Blocker may need to be disabled

http://2016.prepid.courses.aap.org/script/march?q­page=7&req=201612122021507228 1/1
Table 1. Patient Medications
Chronic Home Medications & Treatments
Albuterol solution nebulized twice daily

Continuous alternating therapy: tobramycin inhalation solution


and aztreonam lysine solution

DNAse inhalation solution nebulized daily

Fat soluble vitamins orally with enzymes twice daily

High fat/protein/calorie diet

Hypertonic saline 7% nebulized bid

Oral ivacaftor with enzymes twice daily

Pancrelipase enzymes orally with all meals/snacks

Vest therapy twice daily

Courtesy of K Moffett, MD
12/13/2016 March

Welcome  mohammed alsaiary [ Logout ]

Home PREP Pearls FAQs My Bookmarks CME Information

Overview Home  > March

Editorial Board
March   Enter Keyword Search
January
Question View:   All (8) Jump to Question
February
Print this Page Add to my Bookmarks Page 32 of 115
March

April Assessment History Mode: Learner  Exam 

May ASSESSMENT PROGRESS:  Total Questions:  8  Questions Answered:  7  Correct Answers:  2


June

July You currently have 1 questions unanswered in this assessment

August

September
Question: 7
October
A 15­year­old patient with cystic fibrosis (genotype G551D and ΔF508) is admitted for a pulmonary exacerbation and the pediatric
November infectious diseases service is consulted for assistance in determining antibiotic treatment. While on rounds, the pharmacist reviews
all the medications for the patient (Table 1).
December
The pharmacist reviews the drug ivacaftor and its recent breakthrough in understanding and treating the basic defect in cystic
fibrosis; he highlights the future impact this drug may have on lung function, chronic infection, and outcome for this patient.

The target of ivacaftor in this patient is MOST likely to potentiate the

A. calcium­activated chloride channel
Claim Credit
B. chloride ion transporter
Evaluation
C. Δ F508 allele
My Learning Plan D. transmembrane regulatory protein

Incorrect View Peer Results

Correct Answer: D Average Percent Correct:  30.95%

The field of cystic fibrosis (CF) is undergoing dramatic and rapid change with the recent approvals of
cystic fibrosis transmembrane conductance regulatory (CFTR) modulators, small molecular weight
drugs that target the basic function of the defective cystic fibrosis protein for a specific mutation.
Ivacaftor was specifically designed to treat patients who have at least one copy of the G551D mutation
in at least one of their CFTR genes; however, it does not affect calcium­activated chloride channel or
the chloride ion transporter. The ΔF508 allele is the genetic defect at chromosome 7. The G551D
mutation, which occurs in approximately 4.4% of CF patients, is called a "gating mutation" because it
impairs the regulated opening of the ion channel that is formed by the CFTR protein. Ivacaftor was developed using high­
throughput screening of large chemical libraries. It is US Food and Drug Administration (FDA)­approved for patients with
G551D mutations and other gating mutations, namely G178R, S549N, S549R, G551S, G1244E, S1251N, S1255P, G1349D,
and R117H. Therefore, all patients with CF should undergo genotyping to determine their mutation. Table 2 summarizes
results and benefits from clinical trials of these first­in­class drugs.

Individuals who are homozygous for the Δ F508 mutation are eligible to be treated with the combination of ivacaftor and
lumacaftor. Clinical trials revealed modest improvements in pulmonary function and reduction in the risk of pulmonary
exacerbations. As of July 2015, this drug combination is approved by the FDA for CF patients with this homozygous mutation.
The F508del mutation interferes with CFTR protein folding and channel gating activity. Lumacaftor partially corrects the CFTR
misfolding, while ivacaftor improves the gating abnormality. Unfortunately, neither drug is effective when used alone for
F508del homozygotes. The field of CF is rapidly changing as a more detailed understanding of the basic mechanisms of the
disease and first­in­class drugs aimed at the basic defect are targeted. Most importantly, the understanding of the disease
pathogenesis is changing as novel tools to dissect the basic molecular mechanisms of protein dysfunction and genomic
medicine become available.

The amount of expression of the CFTR protein is important in the pathogenesis of CF. The CFTR protein resides in the
plasma membrane of epithelial cells and has several functions. In the sweat duct, it acts to absorb chloride; in the lung, it
secretes chloride; and in the pancreas, it secretes chloride in exchange for bicarbonate. Organs that are affected by the
disease have abnormal chloride and fluid secretion, which impairs fluid movement and leads to ductular obstruction and
organ damage. Five classes of CF mutations result in different amounts of CFTR expression, and the range of severity of
disease is related to the lack of production of functioning CFTR. Classes of mutations produce CFTR that cannot be
activated, are misfolded and degraded before reaching the cell membrane, are decreased in abundance, or have altered
conductance. Variation in the severity of disease in individuals with the same genotype indicates other factors, including the
environment, genetic modifiers, and medical therapy, are also important in long­term outcomes. The diagnosis of CF depends

http://2016.prepid.courses.aap.org/script/march?req=201612122025133163&status=submit 1/3
12/13/2016 March
upon DNA analysis of the long arm of chromosome 7; two copies of a disease­causing mutation in the CFTR gene confirms
the diagnosis of CF. The Δ F508 allele or mutation (also known as Phe508del or F508del) is the most prevalent gene
mutation worldwide. In the United States, approximately 40% of patients with CF are homozygous for Δ F508 (have 2 copies)
and another 40% are heterozygous (have one ΔF508 and one other CF­causing mutation). Patients with at least one mild
allele have some low level of CFTR expression. They retain some degree of pancreatic function, have less severe lung
disease, and have sweat chloride concentrations that are borderline normal.

The exact mechanism by which altered salt and water transport leads to abnormal secretions in the respiratory tract,
pancreas, gastrointestinal tract, sweat glands, and other exocrine glands remains under intense investigation. The absence of
CFTR in the apical cellular membrane decreases the ability of cells to secrete chloride into the periciliary fluid. In the lung,
abnormal respiratory tract secretions lead to decreased airway surface liquid, decreased mucociliary clearance, and impaired
defenses to inhaled particulate matter and various microbial pathogens. CFTR inhibits the epithelial sodium channel; in its
absence, excessive absorption of sodium occurs. Other channels are available for secretion of chloride (ie, calcium­activated
chloride channel) in the respiratory epithelium; however, they cannot compensate for the loss of CFTR. Chloride also can
enter the cell through a chloride transporter. The net effect is increased absorption of sodium, chloride, and water, which
reduces the periciliary volume, alters the composition of mucins, and decreases mucociliary clearance. The reduction in the
periciliary fluid impairs ciliary movement, decreases mucus transport, markedly alters clearance, and sets the stage for airway
obstruction, infection, inflammation, and progressive lung destruction. Submucosal glands have high expression of CFTR.
Loss of CFTR function alters the composition of mucins produced by the submucosal glands, leading to ductular dilation with
mucus and obstruction. Mucus is tightly adhered to the respiratory epithelium and increases airway obstruction. Failure to
clear mucous plugs, continued mucin secretion, and adherent mucus to the airway surface provides the focus for infection. In
this environment, there is evidence that abnormal mucins enhance bacterial binding and epithelial cell receptors are altered,
both of which lead to impaired bacterial clearance and inactivation of epithelial­derived bactericidal activity.

Although very young infants with CF are considered to be free of lung disease, there is accumulated evidence to suggest that
the pulmonary manifestations of the disease have a very early onset, perhaps at birth itself. At the small airway level, the lung
pathology in young infants with CF points to focal changes at the small airway level, with the presence of mucous plugging
and dilation of the airway lumen, in addition to hyperplasia of the bronchial glands, as well as presence of active inflammation
with or without the presence of lower airway infection. In infants and young children with CF, early and subtle small airway
abnormalities are unlikely to produce any noticeable symptoms, thus creating a disconnect between the presence of
pathophysiological changes and symptomatology. More sensitive diagnostic methods are needed to detect the presence and
cause of early lower airway disease, and more importantly, to identify effective interventions in the youngest patients.

The respiratory airways of patients with CF become infected with bacterial and viral pathogens; chronic infection with
Pseudomonas aeruginosa develops in 80% of affected individuals by the time they reach 25 years of age. Pseudomonas
infection is associated with a more rapid decline in lung function and increased rates of mortality, and becomes the dominant
organism colonizing the CF airway. Initial isolates from CF infants and young children are unique and suggest an
environmental acquisition. Over time, one strain dominates and changes to a mucoid phenotype.

In the environment of low volume of airway surface liquid and abnormal salt and water transport, an anaerobic gradient
develops within the thickened mucous plugs, facilitating the growth of P aeruginosa. In anaerobic conditions, the production of
nitrate reductase allows P aeruginosa to cleave oxygen from nitrate, forming an alginate polysaccharide. Biofilm­containing
macrocolonies of Pseudomonas are then established, resisting antibiotics and host defense, thus facilitating chronic infection,
inflammation, and airway destruction to occur.

PREP Pearls
The amount of expression of the cystic fibrosis conductance transmembrane regulatory (CFTR) protein is important in the
pathogenesis of cystic fibrosis (CF).

The absence of CFTR in the apical cellular membrane decreases the ability of cells to secrete chloride into the periciliary
fluid. In the CF lung, abnormal respiratory tract secretions lead to decreased airway surface liquid, decreased mucociliary
clearance, and impaired defenses to inhaled particulate matter and various microbial pathogens.

In infants and young children with CF, early and subtle small airways abnormalities are unlikely to produce any noticeable
symptoms, thus creating a disconnect between the presence of pathophysiological changes and symptomatology.

The field of CF is undergoing dramatic and rapid change with the recent approvals of CFTR modulators, small molecular
weight drugs that target the basic function of the defective cystic fibrosis protein for a specific mutation.

American Board of Pediatrics Content Specification(s)
Know the pathophysiologic mechanisms that contribute to susceptibility to infection in the patient with cystic fibrosis

Suggested Readings
Cohen­Cymberknoh M, Kerem E, Ferkol T, Elizur A. Airway inflammation in cystic fibrosis: molecular mechanisms and clinical
implications. Thorax. 2013;68(12):1157­1162. DOI: http://dx.doi.org/10.1136/thoraxjnl­2013­203204

Milla CE. Cystic fibrosis in the era of genomic medicine. Curr Opin Pediatr. 2013;25(3):323­328. DOI:
http://dx.doi.org/10.1097/MOP.0b013e328360dbf5

Ramsey BW, Davies J, McElvaney NG, et al. A CFTR potentiator in patients with cystic fibrosis and the G551D mutation. N
Engl J Med. 2011;365(18):1663­1672. DOI: http://dx.doi.org/10.1056/NEJMoa1105185

Wainwright CE, Elborn JS, Ramsey BW, et al. Lumacaftor­Ivacaftor in Patients with Cystic Fibrosis Homozygous for
Phe508del CFTR. N Engl J Med. 2015;373(3):220­231. DOI: http://dx.doi.org/10.1056/NEJMoa1409547

Comment On This Question

Page 32 of 115

http://2016.prepid.courses.aap.org/script/march?req=201612122025133163&status=submit 2/3
12/13/2016 March

Contact Us
Links may open in separate window
Copyright © 2016   American Academy of Pediatrics. All rights reserved. Pop­up Blocker may need to be disabled

http://2016.prepid.courses.aap.org/script/march?req=201612122025133163&status=submit 3/3
12/13/2016 March

Welcome  mohammed alsaiary [ Logout ]

Home PREP Pearls FAQs My Bookmarks CME Information

Overview Home  > March

Editorial Board
March   Enter Keyword Search
January
Question View:   All (8) Jump to Question
February
Print this Page Add to my Bookmarks Page 33 of 115
March

April Assessment History Mode: Learner  Exam 

May ASSESSMENT PROGRESS:  Total Questions:  8  Questions Answered:  7  Correct Answers:  2


June

July You currently have 1 questions unanswered in this assessment

August

September
Question: 8
October
You are discussing the pathogenesis of Salmonella infection with a group of medical students.
November
Of the following, the MAJOR difference in the pathogenesis of Salmonella Typhi (S Typhi) and nontyphoidal Salmonella (NTS)
December infections is that

A. adhesion molecules play a key role in intestinal invasion in S Typhi infections
B. genomes that facilitates colonization, infection, and transmission vary between S Typhi and NTS strains
C. infectious dose of S Typhi is at least a 1,000­fold greater than that for other NTS infections

Claim Credit D. initial infection is characterized by a lack of mucosal inflammation in S Typhi as opposed to disease caused by NTS

Evaluation

My Learning Plan Submit   Reset  

Page 33 of 115

Contact Us
Links may open in separate window
Copyright © 2016   American Academy of Pediatrics. All rights reserved. Pop­up Blocker may need to be disabled

http://2016.prepid.courses.aap.org/script/march?q­page=8&req=201612122025137538 1/1
12/13/2016 March

Welcome  mohammed alsaiary [ Logout ]

Home PREP Pearls FAQs My Bookmarks CME Information

Overview Home  > March

Editorial Board
March   Enter Keyword Search
January
Question View:   All (8) Jump to Question
February
Print this Page Add to my Bookmarks Page 33 of 115
March

April Assessment History Mode: Learner  Exam 

May ASSESSMENT PROGRESS:  Total Questions:  8  Questions Answered:  8  Correct Answers:  2


June

July You currently have 2 required survey(s) that are incomplete. Please complete the survey(s) for the following
question(s) on this assessment:  Question 4, Question 8
August

September

October Question: 8
November You are discussing the pathogenesis of Salmonella infection with a group of medical students.

December Of the following, the MAJOR difference in the pathogenesis of Salmonella Typhi (S Typhi) and nontyphoidal Salmonella (NTS)
infections is that

A. adhesion molecules play a key role in intestinal invasion in S Typhi infections
B. genomes that facilitates colonization, infection, and transmission vary between S Typhi and NTS strains
C. infectious dose of S Typhi is at least a 1,000­fold greater than that for other NTS infections
Claim Credit
D. initial infection is characterized by a lack of mucosal inflammation in S Typhi as opposed to disease caused by NTS
Evaluation

My Learning Plan
Incorrect View Peer Results
Correct Answer: D Average Percent Correct:  34.52%

One of the key features in the pathogenesis of Salmonella Typhi infection that distinguishes it from
nontyphoidal Salmonella (NTS) infections is the lack of mucosal inflammation. Despite invasive
infection with bacteremia and systemic dissemination, the initial infection may be asymptomatic with no
evidence of an inflammatory or diarrheal response. This is described in greater detail in this critique. In
addition, S Typhi has an infectious dose that is at least a 1,000­fold less than other NTS infections.

Given that approximately 90% of the genome of S Typhi is related to sequences present in other
Salmonella serovars, it follows that a core Salmonella genome contributes to pathogenesis. Vi capsule expression and
modified gene expression likely play a bigger role in S Typhi, as opposed to NTS infections where a repertoire of adhesion
molecules play a key role in gastrointestinal invasion (Figure).

A member of the Enterobacteriaceae family, the genus Salmonella contains 2 main species: enterica and bongori. Salmonella
enterica subspecies enterica contains the overwhelming majority of all Salmonella serovars (serotypes) that affect humans,
other mammals, and birds. More than 2,500 serovars have been identified based on the O, H (flagellar) and Vi (capsular)
antigens. Most serovars that cause human disease belong to the O serogroups A through E. Salmonella enterica subspecies
enterica serovar Typhi, the causative organism for enteric fever (typhoid), is classified in O serogroup D along with serovars
Enteritidis and Dublin. Common serovars causing disease in humans in the United States include Enteritidis, Typhimurium,
Newport, and Javiana. These Salmonella serovars account for about half of culture­confirmed Salmonella isolates reported by
public health laboratories in the United States. The prevalence of different serovars varies by country.

Salmonella species are widely distributed in nature. The only reservoir for S Typhi and Paratyphi A is the human
gastrointestinal tract. Therefore, disease caused by these organisms is by fecal contamination of food or drink by individuals
who are ill, convalescing, or have asymptomatic infection. Direct person to person transmission is rare. Sexual transmission
has also been reported. The prevalence of typhoid and paratyphoid fever is uncommon in the United States, mainly seen in
travelers to and from endemic countries in south­central and southeast Asia, as well as southern Africa.

The major reservoir for NTS is the gastrointestinal tract of infected and often asymptomatic birds, mammals, and reptiles.
Chickens, turkeys, ducks, sheep, cows, pigs, turtles, snakes, lizards, iguanas, and hedgehogs are all commonly affected.
Infection can result from exposure to pets and animals in public settings and from environmental contamination without direct
contact with an infected animal. Even handling pet food treats containing dried beef has caused infection. Numerous
outbreaks of NTS infections related to contaminated fresh or thawed poultry, egg­containing products such as egg nog and
salad dressings, dairy products such as ice cream and mayonnaise, and fresh produce have been reported in the United
States. The prevalence of NTS also varies depending on water treatment, sewage disposal, and food handling practices.

http://2016.prepid.courses.aap.org/script/march?req=201612122026242696&status=submit 1/3
12/13/2016 March
The incubation period for NTS gastroenteritis is usually 12 to 36 hours (range 6 to 72 hours), while for enteric fever, it is 7 to
14 days (range 3 to 60 days). The infectious dose for NTS organisms in healthy adults is 106 to 108 organisms. In younger
children or immunocompromised hosts, a smaller inoculum size can produce disease. For enteric fever, the infectious dose
varies from 1,000 to 1 million organisms, which is about 1,000­fold higher than that associated with other Salmonella
infections. Once infection occurs, Salmonella organisms are excreted in the stool for a period of 5 weeks on average, but this
may be prolonged to as many as 20 weeks in 40% of children younger than 5 years of age. Antimicrobial therapy may prolong
excretion. Approximately 1% of adults continue to excrete NTS organisms for greater than 1 year.

Besides age and immunosuppression, several other factors may predispose to Salmonella infection. These include
achlorhydria, such as can be seen with the use of antacids because the organisms survive between a pH of 4 to 8. Antibiotic
therapy may not only prolong excretion of the organisms, but if given for long periods, may predispose to infection by altering
the normal gut flora. Gastrectomy, underlying gastrointestinal conditions such as inflammatory bowel disease, chronic
granulomatous disease, γ­interferon deficiency, and hemoglobinopathies, such as sickle cell disease, may also predispose to
infection.

The pathogenesis of Salmonella infection is extremely complex and still being understood. Multiple factors contribute to the
pathogenicity of organisms within this genus. Ninety percent of the genome of S Typhi is related to sequences also present in
other Salmonella serovars; therefore, it is likely that a single core Salmonella genome facilitates colonization, infection, and
transmission. However, genome sequencing for S Typhi has identified specific phage or Salmonella pathogenicity islands as,
well as S Typhi­specific gene clusters. The accumulation of “pseudogenes” in the genomes of S Typhi and S Paratyphi A is
another unique feature raising the possibility that new pathovars of the organism may be emerging.

S Typhi has a diverse repertoire of products that influence attachment to and invasion of human cells. These include pattern­
associated molecular patterns including lipopolysaccharide (LPS), flagella, fimbriae, and peptidoglycan. The Vi capsule has
anti­inflammatory properties and influences attachment likely by protecting surface components of S Typhi from host
receptors and complement components in vivo. This likely restricts immune activation and increases resistance to serum
killing. It is also released from the surface of S Typhi once the bacteria are within human cells. The Vi capsule may further
regulate interactions between LPS and toll­like receptor 4 (TLR4), a protein encoded by the TLR4 gene, and increases the
local concentrations of the immunoregulatory cytokine interleukin­10 in infected tissues.

After ingested and evasion of the acidic gastric milieu, Salmonella can be engulfed by phagocytes in the intestinal lumen that
facilitates invasion of the mucosal barrier. They can induce their own uptake, but usually traverse the epithelial barrier through
microfold cells that overlie intestinal Peyer patches. The bacterium can invade and survive within several mammalian cells,
including macrophages. The exception to this is neutrophils that rapidly clear the organisms. Intracellular survival within
macrophages is essential to surviving neutrophil­mediated killing and contributes to the pathogenesis of the organism. In
addition, transcriptional activation of genes in the organism helps remodel the bacterial cell surface that further facilitates
survival within the intracellular environment. Through a series of complex interactions with the macrophage, the organism also
forms the Salmonella­containing vacuole, yet another factor that helps survival of the organism.

The NTS strains elicit several changes in the intestinal epithelium including the infiltration of neutrophils and fluid into the
intestinal lumen, thus leading to inflammatory diarrhea. In contrast, due to limited neutrophil transmigration across the
intestinal epithelium and other factors specific to S Typhi, it does not elicit a similar response within the gastrointestinal tract
(Figure). This lack of intestinal inflammation and neutrophil transmigration are also thought to facilitate the invasion of typhoid
serovars into the gut and propagate dissemination to systemic sites. Following uptake of S Typhi by macrophages,
translocation to lymphoid follicles in the gut and lymph nodes occurs. A silent bacteremia with dissemination to the liver,
spleen, gall bladder, and bone marrow follows, with secondary bacteremia 7 to 14 days later. Secondary infection of the small
intestine can also occur via secretion of the organism in the bile through the enterohepatic cycle.

Clinically, infected individuals may develop fatigue during initial infection. However, they often remain asymptomatic with the
gradual onset of low­grade fever that begins to rise in a classic step­wise fashion until the disease becomes manifest in 7 to
14 days. The development of intestinal perforations is a well­known complication of S Typhi disease, and histologic
examinations have shown both acute and chronic inflammation at the sites of perforation, often with few bacteria at the actual
site.

* Required *
Take Survey  

PREP Pearls
There are multiple factors that contribute to the pathogenicity of Salmonella infections, particularly S Typhi: the Vi capsule,
lipopolysaccharide, fimbriae, peptidoglycan, pathogenicity islands, and development of a Salmonella­containing vacuole
are just some of the mechanisms contributing to the virulence of the organism.

Lack of inflammation of the intestinal mucosa is characteristic of S Typhi infection, which is in contrast to nontyphoidal
Salmonella infections where an intense inflammation of the intestinal mucosa with secretory and invasive diarrhea occurs.

Several other factors also contribute to the development of disease and include achlorhydria, gastric surgery, prior use of
antibiotics, as well as several underlying conditions such as sickle cell disease, chronic granulomatous disease, and
diseases resulting in impairment of cell­mediated immunity.

American Board of Pediatrics Content Specification(s)
Understand transmission and acquisition of Salmonella infections (fecal‑oral, environmental incubation, infectious dose,
gastric acidity)

Understand the pathophysiology of invasive Salmonella infection (small intestinal penetration, reticulo endothelial seeding,
intracellular foci, prolonged bacteremia)

Suggested Readings
American Academy of Pediatrics. Salmonella infections. In: Kimberlin DW, Brady MT, Jackson MA, Long SS, eds. Red Book:
2015 Report of the Committee on Infectious Diseases. 30th ed. Elk Grove Village, IL. American Academy of Pediatrics; 2015:
695­702.

http://2016.prepid.courses.aap.org/script/march?req=201612122026242696&status=submit 2/3
12/13/2016 March
Dougan G, Baker S. Salmonella enterica serovar Typhi and the pathogenesis of typhoid fever. Annu Rev Microbiol.
2014;68:317­336. DOI: http://dx.doi.org/10.1146/annurev­micro­091313­103739

Gal­Mor O, Boyle EC, Grassl GA. Same species, different diseases: how and why typhoidal and non­typhoidal Salmonella
enterica serovars differ. Front Microbiol. 2014;5:391. DOI: http://dx.doi.org/10.3389/fmicb.2014.00391

Reller ME. Salmonella species. In: Long SS, Pickering LK, Prober CG, eds. Principles and Practice of Pediatric Infectious
Diseases. 4th ed. Philadelphia, PA: Saunders Elsevier; 2012: 814­819.

Comment On This Question

Page 33 of 115

Contact Us
Links may open in separate window
Copyright © 2016   American Academy of Pediatrics. All rights reserved. Pop­up Blocker may need to be disabled

http://2016.prepid.courses.aap.org/script/march?req=201612122026242696&status=submit 3/3
12/13/2016 April

Welcome  mohammed alsaiary [ Logout ]

Home PREP Pearls FAQs My Bookmarks CME Information

Overview Home  > April

Editorial Board
April   Enter Keyword Search
January
Question View:   All (8) Jump to Question
February
Print this Page Add to my Bookmarks Page 35 of 115
March

April Assessment History

May ASSESSMENT PROGRESS:  Total Questions:  8  Questions Answered:  0  Correct Answers:  0

June

July You currently have 8 questions unanswered in this assessment

August

September Question: 1
October An 18­year­old young woman presents with a 4­day history of fever to 38.8°C, malaise, anorexia, and nausea. Examination reveals a
November thin, young woman with scleral icterus and jaundiced skin. She is mildly tachycardic, with a heart rate of 110 beats/min and a
respiratory rate of 12 breaths/min. Blood pressure and capillary refill time are normal. Laboratory results show:
December
Aspartate aminotransferase, 500 U/L (normal, 12­50 U/L)
Alanine aminotransferase, 1,000 U/L (normal, 12­50 U/L)
Alkaline phosphatase, 750 U/L (normal, 105­420 U/L)
Total bilirubin, 0.5 mg/dL (8.6 μmol/L) (normal, 0.0­1.2 mg/dL)
Hepatitis A immunoglobulin M, positive
Hepatitis B surface antibody (Anti­HBs), positive
Claim Credit
Hepatitis B surface antigen (HBsAg), negative
Evaluation Antibody to hepatitis B core antigen (Anti­HBc), negative
Antibody to hepatitis B e­antigen (Anti­HBe), negative
My Learning Plan
Hepatitis C immunoglobulin G, negative

Of the following, the MOST likely source of this patient’s infection is

A. after school job working in a daycare
B. eating at a fast food restaurant
C. receipt of a tattoo
D. volunteering at the local church blood drive

Submit   Reset  

Page 35 of 115

Contact Us
Links may open in separate window
Copyright © 2016   American Academy of Pediatrics. All rights reserved. Pop­up Blocker may need to be disabled

http://2016.prepid.courses.aap.org/script/april?q­page=1&req=201612122027050197 1/1
12/13/2016 April

Welcome  mohammed alsaiary [ Logout ]

Home PREP Pearls FAQs My Bookmarks CME Information

Overview Home  > April

Editorial Board
April   Enter Keyword Search
January
Question View:   All (8) Jump to Question
February
Print this Page Add to my Bookmarks Page 35 of 115
March

April Assessment History

May ASSESSMENT PROGRESS:  Total Questions:  8  Questions Answered:  1  Correct Answers:  0

June

July You currently have 7 questions unanswered in this assessment

August

September Question: 1
October An 18­year­old young woman presents with a 4­day history of fever to 38.8°C, malaise, anorexia, and nausea. Examination reveals a
November thin, young woman with scleral icterus and jaundiced skin. She is mildly tachycardic, with a heart rate of 110 beats/min and a
respiratory rate of 12 breaths/min. Blood pressure and capillary refill time are normal. Laboratory results show:
December
Aspartate aminotransferase, 500 U/L (normal, 12­50 U/L)
Alanine aminotransferase, 1,000 U/L (normal, 12­50 U/L)
Alkaline phosphatase, 750 U/L (normal, 105­420 U/L)
Total bilirubin, 0.5 mg/dL (8.6 μmol/L) (normal, 0.0­1.2 mg/dL)
Hepatitis A immunoglobulin M, positive
Hepatitis B surface antibody (Anti­HBs), positive
Claim Credit
Hepatitis B surface antigen (HBsAg), negative
Evaluation Antibody to hepatitis B core antigen (Anti­HBc), negative
Antibody to hepatitis B e­antigen (Anti­HBe), negative
My Learning Plan
Hepatitis C immunoglobulin G, negative

Of the following, the MOST likely source of this patient’s infection is

A. after school job working in a daycare
B. eating at a fast food restaurant
C. receipt of a tattoo
D. volunteering at the local church blood drive

Incorrect View Peer Results

Correct Answer: A Average Percent Correct:  57.14%

The patient in the vignette has acute hepatitis A infection. The most likely source of her infection is
from working in the daycare. Hepatitis A is transmitted through the fecal­oral route and unimmunized
young children are the most likely source of infection in adults. While eating at restaurants is a known
risk factor for infection, this is a less likely cause of infection in the United States owing to improved
food handling practices, health department inspections, and increasing rates of hepatitis A vaccination.
Receipt of a tattoo is a known risk factor for hepatitis B and hepatitis C infection, but not for hepatitis A
infection. Volunteering at the church blood drive does not pose a significant risk for development of
infection from viral hepatitis.

Hepatitis A is transmitted person­to­person from fecal contamination and subsequent oral ingestion of viable virus. Risk
factors for hepatitis A infection are listed in the Table. The source cannot be determined in approximately two­thirds of cases,
likely because of the spread of infection from asymptomatic individuals who are shedding virus. Young children are thought to
be the most likely source in these instances, as children younger than 6 years of age typically have asymptomatic infection,
and when symptoms do occur, they are nonspecific in nature. Large childcare centers that include young children in diapers
pose a particular risk of infection, and have been the source of community outbreaks. Hepatitis A is rarely transmitted from
mother to newborn via vertical transmission or from a blood transfusion. Peak infectivity occurs 2 weeks before symptom
onset and resolves 1 week following development of jaundice. There is no carrier state for hepatitis A infection.

Several different manifestations of hepatitis A virus infection exist. Infection may be asymptomatic with only viral shedding
present, asymptomatic with elevated liver enzymes (subclinical), symptomatic but without jaundice (anicteric), symptomatic
with jaundice (icteric), or fulminant infection. Less than 10% of children younger than 6 years of age with hepatitis A infection
develop jaundice. However, 76% to 97% of older children and adults develop symptomatic infection and 40% to 70% of those
develop jaundice. The most common clinical symptoms of hepatitis A infection are low­grade fever, myalgia, anorexia,
malaise, nausea, and vomiting. Signs of hepatic dysfunction develop several days later with dark urine, light­colored stools,

http://2016.prepid.courses.aap.org/script/april?req=201612122028438636&status=submit 1/2
12/13/2016 April
jaundice, and scleral icterus. In children who are symptomatic, 60% will develop diarrhea, in contrast to 20% of adults with
hepatitis A infection. Twenty percent of children exhibit upper respiratory symptoms (eg, cough, coryza, sore throat). Physical
examination findings may include jaundice, scleral icterus, hepatomegaly, splenomegaly, right upper quadrant tenderness, or
urticaria. Edema of the gallbladder and transient ascites may occur. Clinical manifestations of hepatitis A generally resolve
over a few weeks to 2 months. However, a small subset may have prolonged infection or relapsing signs and symptoms for
up to 6 months in duration.

PREP Pearls
Children younger than 6 years of age with hepatitis A infection uncommonly develop symptomatic infection, but serve as
the reservoir for older children and adults.

International travel and international adoption are known risk factors for acquisition of hepatitis A infection.

There is a wide range of symptomatology associated with hepatitis A infection including asymptomatic infection, subclinical
infection, anicteric infection, icteric infection, and fulminate infection.

American Board of Pediatrics Content Specification(s)
Know the usual means of transmission of hepatitis A virus

Recognize child‑care center contact as the probable source of hepatitis A when jaundice develops in an adult contact

Recognize the clinical characteristics and course of the hepatitis A virus infection (usually asymptomatic in children, rarely
causes acute fulminant hepatitis, does not cause chronic hepatitis, and is not associated with carrier state)

Suggested Readings
American Academy of Pediatrics. Hepatitis A. In: Kimberlin DW, Brady MT, Jackson MA, Long SS, eds. Red Book: 2015
Report of the Committee on Infectious Diseases. 30th ed. Elk Grove Village, IL: American Academy of Pediatrics; 2015: 391­
399.

Williams RE, Sharapov UM. Hepatitis A virus. In: Long S, Pickering LK, Prober CG, eds. Principles and Practice of Pediatric
Infectious Diseases. 4th ed. New York, NY: Saunders Elsevier; 2012: 1180­1185.

Comment On This Question

Page 35 of 115

Contact Us
Links may open in separate window
Copyright © 2016   American Academy of Pediatrics. All rights reserved. Pop­up Blocker may need to be disabled

http://2016.prepid.courses.aap.org/script/april?req=201612122028438636&status=submit 2/2
12/13/2016 April

Welcome  mohammed alsaiary [ Logout ]

Home PREP Pearls FAQs My Bookmarks CME Information

Overview Home  > April

Editorial Board
April   Enter Keyword Search
January
Question View:   All (8) Jump to Question
February
Print this Page Add to my Bookmarks Page 36 of 115
March

April Assessment History

May ASSESSMENT PROGRESS:  Total Questions:  8  Questions Answered:  1  Correct Answers:  0

June

July You currently have 7 questions unanswered in this assessment

August

September Question: 2
October A 15­year­old adolescent presents with a 9­day history of a painful ulcer on his penis and 3­day history of painful swelling in his left
November groin. On further questioning, he is sexually active with multiple male and female partners, and occasionally participates in
unprotected sexual encounters. His last sexual encounter was 14 days ago. He denies any systemic complaints, including fever,
December malaise, chills, night sweats, or weight loss. On physical examination, his temperature is 36.8°C, pulse is 72 beats/min, respiratory
rate is 18 breaths/min, and blood pressure is 110/70 mm Hg. He is not ill appearing and abnormalities are limited to the genital
examination. He is circumcised and has a 10 x 10 mm shallow, friable, tender ulceration on the penile shaft, plus a smaller one on
the glans, as seen in Figure 1. He has a 10 mm tender, firm lymph node present in the left inguinal region. Initial laboratory testing
shows nonreactive rapid plasma reagin and negative herpes simplex virus polymerase chain reaction of the lesion.

Administration of which of the following anti­infectives is most likely to result in resolution of his symptoms?
Claim Credit

Evaluation A. acyclovir
B. azithromycin
My Learning Plan
C. benzathine penicillin G
D. doxycycline

Submit   Reset  

Page 36 of 115

Contact Us
Links may open in separate window
Copyright © 2016   American Academy of Pediatrics. All rights reserved. Pop­up Blocker may need to be disabled

http://2016.prepid.courses.aap.org/script/april?q­page=2&req=201612122028443950 1/1
Figure 1.
12/13/2016 April

Welcome  mohammed alsaiary [ Logout ]

Home PREP Pearls FAQs My Bookmarks CME Information

Overview Home  > April

Editorial Board
April   Enter Keyword Search
January
Question View:   All (8) Jump to Question
February
Print this Page Add to my Bookmarks Page 36 of 115
March

April Assessment History

May ASSESSMENT PROGRESS:  Total Questions:  8  Questions Answered:  2  Correct Answers:  0

June

July You currently have 6 questions unanswered in this assessment

August

September Question: 2
October A 15­year­old adolescent presents with a 9­day history of a painful ulcer on his penis and 3­day history of painful swelling in his left
November groin. On further questioning, he is sexually active with multiple male and female partners, and occasionally participates in
unprotected sexual encounters. His last sexual encounter was 14 days ago. He denies any systemic complaints, including fever,
December malaise, chills, night sweats, or weight loss. On physical examination, his temperature is 36.8°C, pulse is 72 beats/min, respiratory
rate is 18 breaths/min, and blood pressure is 110/70 mm Hg. He is not ill appearing and abnormalities are limited to the genital
examination. He is circumcised and has a 10 x 10 mm shallow, friable, tender ulceration on the penile shaft, plus a smaller one on
the glans, as seen in Figure 1. He has a 10 mm tender, firm lymph node present in the left inguinal region. Initial laboratory testing
shows nonreactive rapid plasma reagin and negative herpes simplex virus polymerase chain reaction of the lesion.

Administration of which of the following anti­infectives is most likely to result in resolution of his symptoms?
Claim Credit

Evaluation A. acyclovir
B. azithromycin
My Learning Plan
C. benzathine penicillin G
D. doxycycline

Incorrect View Peer Results
Correct Answer: B Average Percent Correct:  52.38%

This individual most likely has chancroid, a sexually­transmitted disease caused by Haemophilus
ducreyi. The drug of choice is azithromycin, given as a single dose of 1 g orally; ceftriaxone,
ciprofloxacin, and erythromycin are alternatives. Acyclovir can be used for treatment of genital herpes
simplex virus (HSV) infections, another cause of painful genital ulcerating disease. Penicillin is the drug
of choice for syphilis, and although rapid plasma reagin may be nonreactive soon after infection, the
ulcer (or chancre) is typically painless. Doxycycline is the recommended treatment for
lymphogranuloma venereum (LGV), caused by Chlamydia trachomatis serovars L1, L2, and L3.
Lymphogranuloma venereum is a cause of genital ulcers, but again, the lesions are generally painless and also self­limiting.
Inguinal adenopathy generally appears much later and is usually painful.

It is useful to classify sexually transmitted genital lesions by type. Ulcerating lesions are most typically associated with HSV,
chancroid, primary syphilis, granuloma inguinale (donovaniasis), and LGV. The first 2 cause painful ulcers, while the latter 3
are usually associated with painless lesions.

The incidence of chancroid has been decreasing in the United States (Figure 2) and elsewhere in the world for many years;
in 2013, only 10 cases were reported to the US Centers for Disease Control and Prevention (CDC), although this might be an
underestimate because definitive diagnosis is difficult. The CDC considers definitive diagnosis as growth of H ducreyi from
culture of the lesion, though sensitivity is less than 80%, or a positive polymerase chain reaction (PCR) from the site.

Unfortunately, no US Food and Drug Administration­approved H ducreyi PCR is available, so the test is only possible if a
particular laboratory has validated an assay in compliance with the Clinical Laboratory Improvement Amendments. More
commonly, a probable diagnosis is made based on the presence of 4 features:

1. One or more painful genital ulcers
2. Typical features of ulcer and, if present, regional adenopathy
3. Darkfield examination or serologic test for syphilis (the latter at least 7 days after ulcer onset) is negative
4. Ulcer HSV PCR or culture is negative

Gram stain of an ulcer is no longer recommended for diagnosis; sensitivity and specificity are poor.

http://2016.prepid.courses.aap.org/script/april?req=201612122032152391&status=submit 1/2
12/13/2016 April
After initiation of treatment, symptoms usually improve within 3 days and the ulcer itself starts to resolve significantly by 7
days. Response of the inguinal buboes is slower and may occasionally require needle aspiration or incision and drainage for
complete resolution, even with effective antimicrobial treatment.

Follow­up at 3 to 7 days after therapy is recommended. Response is slower in uncircumcised men and in HIV­infected
individuals. Of course, testing for HIV and other sexually transmitted agents is recommended, and initial negative tests for
syphilis and HIV should be followed up by repeat testing in 3 months. Sexual partners should be evaluated and treated as
appropriate.

PREP Pearls
Chancroid is a sexually transmitted disease with decreasing incidence.

The differential diagnosis of painful genital ulcers includes chancroid and herpes simplex virus (HSV).

Diagnosis of chancroid is usually made clinically after ruling out related diseases such as HSV and primary syphilis.

American Board of Pediatrics Content Specification(s)
Recognize that Haemophilus ducreyi is a sexually transmitted disease (chancroid) of increasing incidence

Recognize the clinical manifestations of chancroid

Plan an appropriate treatment regimen for a patient with chancroid

Suggested Readings
American Academy of Pediatrics. Chancroid. In: Kimberlin DW, Brady MT, Jackson MA, Long SS, eds. Red Book: 2015
Report of the Committee on Infectious Diseases. 30th ed. Elk Grove Village, IL: American Academy of Pediatrics; 2015: 283­
284.

Trager JD. Sexually transmitted diseases causing genital lesions in adolescents. Adolesc Med Clin. 2004;15(2):323­352.

US Centers for Disease Control and Prevention. Sexually Transmitted Disease Surveillance 2013. Atlanta, GA: US
Department of Health and Human Services; 2014: 148 pp.

Comment On This Question

Page 36 of 115

Contact Us
Links may open in separate window
Copyright © 2016   American Academy of Pediatrics. All rights reserved. Pop­up Blocker may need to be disabled

http://2016.prepid.courses.aap.org/script/april?req=201612122032152391&status=submit 2/2
12/13/2016 April

Welcome  mohammed alsaiary [ Logout ]

Home PREP Pearls FAQs My Bookmarks CME Information

Overview Home  > April

Editorial Board
April   Enter Keyword Search
January
Question View:   All (8) Jump to Question
February
Print this Page Add to my Bookmarks Page 37 of 115
March

April Assessment History

May ASSESSMENT PROGRESS:  Total Questions:  8  Questions Answered:  2  Correct Answers:  0

June

July You currently have 6 questions unanswered in this assessment

August

September Question: 3
October A pediatric resident calls you to ask about catch­up immunizations for a 5­year, 6­month old­child. The resident gives you the child’s
November complete immunization record (Table 1).

December Of the following, the MOST likely immunization requirement for this child is

A. DTaP, IPV, PCV­13
B. DTaP, IPV, PCV­13, Rotavirus
C. IPV
D. IPV, now and in 6 months
Claim Credit
E. no further doses required
Evaluation

My Learning Plan
Submit   Reset  

Page 37 of 115

Contact Us
Links may open in separate window
Copyright © 2016   American Academy of Pediatrics. All rights reserved. Pop­up Blocker may need to be disabled

http://2016.prepid.courses.aap.org/script/april?q­page=3&req=201612122032156922 1/1
Table 1. Immunization Record for Child in the Vignette.
Vaccine Ages given
Hepatitis B Birth 2 months - 18 months -

DTaP - 2 months 4 months - -

DTaP-IPV/ Hib - - - 18 months 4 years

Hepatitis A - - - 18 months 4 years

MMRV - - - 18 months 4 years

PCV-13 - - - 18 months 4 years

Rotavirus - 2 months - - -

DTaP (Diphtheria, Tetanus, acellular Pertussis); IPV (inactivated poliovirus vaccine); Hib (Haemophilus 
influenza type b); MMRV (Measles, Mumps, Rubella, Varicella); PCV‐13 (Pneumococcal Conjugate Vaccine, 
serotype 13) 

Courtesy of K Moffett, MD

 
 
12/13/2016 April

Welcome  mohammed alsaiary [ Logout ]

Home PREP Pearls FAQs My Bookmarks CME Information

Overview Home  > April

Editorial Board
April   Enter Keyword Search
January
Question View:   All (8) Jump to Question
February
Print this Page Add to my Bookmarks Page 37 of 115
March

April Assessment History

May ASSESSMENT PROGRESS:  Total Questions:  8  Questions Answered:  3  Correct Answers:  0

June

July You currently have 5 questions unanswered in this assessment

August

September Question: 3
October A pediatric resident calls you to ask about catch­up immunizations for a 5­year, 6­month old­child. The resident gives you the child’s
November complete immunization record (Table 1).

December Of the following, the MOST likely immunization requirement for this child is

A. DTaP, IPV, PCV­13
B. DTaP, IPV, PCV­13, Rotavirus
C. IPV
D. IPV, now and in 6 months
Claim Credit
E. no further doses required
Evaluation

My Learning Plan
Incorrect View Peer Results

Correct Answer: C Average Percent Correct:  27.38%

The child in the vignette only needs a third inactivated poliovirus (IPV) dose to complete all of the
vaccines recommended for a child this age. A fourth dose against poliovirus is not necessary if the third
dose was given at 4 years of age or older and there was a minimum of 6 months after the second dose
(Table 2). Although this child only received 1 rotavirus vaccine dose, the maximum age for the final
dose of the rotavirus series is 8 months 0 days. Since the child received a 4th dose of DTaP at 4 years
of age, a 5th dose is not required. Since the child in the vignette received the 2nd dose of PCV­13
greater than 2 months after dose #1, and the child was > 24 months of age, no further conjugate
pneumococcal doses are required.

Polio vaccine is available worldwide in 2 forms, an inactivated form (IPV; given subcutaneously or intramuscularly by injection)
or in a live attenuated form (given orally [OPV]). The IPV vaccine is the only poliovirus vaccine available in the United States.
Both types of poliovirus vaccines consist of serotypes 1, 2, and 3. The IPV vaccine is also available in combination with other
childhood vaccines (DTaP­Hep B­IPV, DTaP­IPV, and DTaP­IPV/Hib).

The IPV vaccine is contraindicated for people who have experienced a hypersensitivity or anaphylactic reactions after a
previous dose of IPV or to streptomycin, neomycin, or polymyxin B. IPV vaccine is recommended for immunocompromised
patients and OPV vaccine should not be used. Household contacts of immunocompromised people or people with altered
immune states, immunosuppression attributable to therapy for other diseases, or known HIV should receive IPV vaccine. If
OPV vaccine inadvertently is introduced into a household of an immunocompromised or HIV­infected person, close contact
between the patient and the OPV­vaccine recipient should be minimized for approximately 4 to 6 weeks after immunization.
Breastfeeding and mild diarrhea are not contraindications to IPV or OPV vaccine administration.

* Required *
Take Survey  

PREP Pearls
The final dose in the inactivated poliovirus vaccine series should be administered at 4 years of age or older, regardless of
the previous number of doses.

A fourth dose is not necessary if the third dose was given at 4 years of age or older and a minimum of 6 months after the
second dose.

http://2016.prepid.courses.aap.org/script/april?req=201612122039436463&status=submit 1/2
12/13/2016 April

American Board of Pediatrics Content Specification(s)
Plan a routine schedule for diphtheria immunization, including age of the patient, number of doses and intervals and their
reasons, and recommendation if the schedule has been interrupted

Know the contraindications and precautions for poliovirus immunization, including administration during concurrent illness

Suggested Readings
American Academy of Pediatrics. Active immunization. In: Pickering LK, Baker CJ, Kimberlin DW, Long SS, eds. Red Book:
2012 Report of the Committee on Infectious Diseases. 29th ed. Elk Grove Village, IL: American Academy of Pediatrics;
2012:11­56.

US Centers for Disease Control and Prevention. Advisory Committee on Immunization Practices (ACIP) Recommended
Immunization Schedules for Persons Aged 0 Through 18 Years and Adults Aged 19 Years and Older — United States, 2013.
US Centers for Disease Control and Prevention website. http://www.cdc.gov/mmwr/pdf/other/su6201.pdf. Accessed
September 25, 2014

Comment On This Question

Page 37 of 115

Contact Us
Links may open in separate window
Copyright © 2016   American Academy of Pediatrics. All rights reserved. Pop­up Blocker may need to be disabled

http://2016.prepid.courses.aap.org/script/april?req=201612122039436463&status=submit 2/2
12/13/2016 April

Welcome  mohammed alsaiary [ Logout ]

Home PREP Pearls FAQs My Bookmarks CME Information

Overview Home  > April

Editorial Board
April   Enter Keyword Search
January
Question View:   All (8) Jump to Question
February
Print this Page Add to my Bookmarks Page 38 of 115
March

April Assessment History

May ASSESSMENT PROGRESS:  Total Questions:  8  Questions Answered:  3  Correct Answers:  0

June

July You currently have 5 questions unanswered in this assessment

August

September Question: 4
October A newborn was referred for his failed newborn hearing screen on the left ear, and confirmatory auditory brain stem evoked
November responses confirmed a left­sided, severe­to­profound unilateral sensorineural hearing loss. Testing for congenital cytomegalovirus
(CMV) was positive, with presence of CMV detected in the newborn’s urine at 7 days of age. The family is offered enrollment in a
December randomized, blinded, clinical trial evaluating whether or not an oral antiviral medication, valganciclovir, will reduce the risk of the
newborn experiencing a hearing loss progression. In this clinical trial, the baby will be randomized to receive either oral valganciclovir
solution at a dose of 16 mg/kg per dose every 12 hours for 6 months or a placebo solution every 12 hours for 6 months. The aim of
this study is to determine whether or not early treatment with valganciclovir will improve the baby’s hearing outcome.

Of the following, the type of clinical trial being offered to this newborn and his family is

Claim Credit A. phase I
Evaluation B. phase II
My Learning Plan C. phase III
D. phase IV

Submit   Reset  

Page 38 of 115

Contact Us
Links may open in separate window
Copyright © 2016   American Academy of Pediatrics. All rights reserved. Pop­up Blocker may need to be disabled

http://2016.prepid.courses.aap.org/script/april?q­page=4&req=201612122039441150 1/1
12/13/2016 April

Welcome  mohammed alsaiary [ Logout ]

Home PREP Pearls FAQs My Bookmarks CME Information

Overview Home  > April

Editorial Board
April   Enter Keyword Search
January
Question View:   All (8) Jump to Question
February
Print this Page Add to my Bookmarks Page 38 of 115
March

April Assessment History

May ASSESSMENT PROGRESS:  Total Questions:  8  Questions Answered:  4  Correct Answers:  0

June

July You currently have 4 questions unanswered in this assessment

August

September Question: 4
October A newborn was referred for his failed newborn hearing screen on the left ear, and confirmatory auditory brain stem evoked
November responses confirmed a left­sided, severe­to­profound unilateral sensorineural hearing loss. Testing for congenital cytomegalovirus
(CMV) was positive, with presence of CMV detected in the newborn’s urine at 7 days of age. The family is offered enrollment in a
December randomized, blinded, clinical trial evaluating whether or not an oral antiviral medication, valganciclovir, will reduce the risk of the
newborn experiencing a hearing loss progression. In this clinical trial, the baby will be randomized to receive either oral valganciclovir
solution at a dose of 16 mg/kg per dose every 12 hours for 6 months or a placebo solution every 12 hours for 6 months. The aim of
this study is to determine whether or not early treatment with valganciclovir will improve the baby’s hearing outcome.

Of the following, the type of clinical trial being offered to this newborn and his family is

Claim Credit A. phase I
Evaluation B. phase II
My Learning Plan C. phase III
D. phase IV

Incorrect View Peer Results

Correct Answer: C Average Percent Correct:  72.62%

The type of randomized clinical trial being offered to this family is a phase III randomized clinical trial,
where testing of an established therapeutic dose of a medication is planned, to assess efficacy,
effectiveness, and safety.

The testing of new drugs, new vaccines, or new diagnostic devices for medical use may be divided into
phases of preclinical and clinical research.

Preclinical studies usually begin with drug design or drug discovery, followed by in vitro testing and
animal testing, to determine whether or not a drug candidate has scientific merit for further clinical development as an
investigational new drug. If the preclinical studies are favorable, then clinical trials in human subjects will proceed. When a
new pharmacologic agent, such as the antiviral medication valganciclovir, is being evaluated for potential licensure, the
phases of clinical trials usually progress from phase I to phase III, potentially leading to approval of a national regulatory
authority for licensure and use in the general population.

Phase I clinical trials are clinical pharmacology and safety studies to determine dose ranges and maximum tolerated doses.
They usually include a small number of healthy subjects or patients with the condition under evaluation, all of whom receive
the medication, often in escalating doses, ranging from subtherapeutic to potentially therapeutic doses. Pharmacodynamics,
pharmacokinetics, and bioavailability and half­life studies of the drug may be performed to determine the most effective doses
of the medication. There is also careful determination of adverse effects and toxicities.

Phase II clinical trials are preliminary studies on efficacy and the relationship between a dose of the medication and the
efficacy or desired effect. Safety data will also be collected. All subjects receive the medication in the dose determined to be
safe and effective in previously conducted phase I clinical trials. The sample size for phase II clinical trials is usually larger
than phase I clinical trials. Usually 100 to 200 subjects are enrolled, but for more rare conditions, a smaller sample size may
be chosen. Some clinical trial designs may combine phase I and phase II trials and test efficacy, safety, and toxicities. The
percentage of phase II clinical trials that are favorable to proceed to phase III trials is less than 20%.

Phase III clinical trials are usually large scale, randomized control trials designed to evaluate efficacy (effect on a specific
clinical or laboratory endpoint), effectiveness (effect on controlling or curing the overall disease), as well as safety and
adverse effects of an established therapeutic dose from previous phase I and phase II clinical trials. Phase III clinical trials are
often conducted at multiple centers. The number of subjects enrolled is usually large, often over 1,000 subjects, but
enrollment also will depend on how common or rare is the condition under study. Phase III trials are designed to provide

http://2016.prepid.courses.aap.org/script/april?req=201612122040300370&status=submit 1/2
12/13/2016 April
definitive assessment of how effective a drug is, in comparison with a current “gold standard” or “reference standard”
treatment, or placebo if no current standard of treatment exists. Phase III trials are often expensive, time consuming, and may
have a relatively short follow­up period for evaluation, compared to the lifetime effects of the disease or condition. Goals of
phase III clinical trials include licensure by a regulatory agency for a new drug, or “label expansion” to show how the drug
works for additional types of patients or diseases beyond the original use for which the drug was approved or licensed.
Randomization is an important part of the study design for phase III clinical trials because it equitably allocates participants to
1 of 2 groups, a treatment group that receives the medication under investigation or a comparison/no treatment/placebo group
that does not. Blinding is another important component of many phase III clinical trials, which removes one form of bias in
interpretation of results. Blinding of the study participants means they do not know in which treatment group they are
randomized. Double­blinded studies also include blinded data collectors and data analysts to avoid introducing biases on
measuring the outcomes.

Phase IV trials are also known as postmarketing surveillance studies. These trials involve the safety surveillance and ongoing
technical and informational support of a drug after it receives licensure for use to treat patients. Phase IV studies may be
required by regulatory authorities as a condition for licensure, or may be undertaken voluntarily by the sponsoring
pharmaceutical company. A phase IV study is designed to detect rare or long­term adverse effects in a larger patient
population who receive the drug over a longer period of time than possible during the shorter phase I through III clinical trial
periods. If harmful effects are discovered during phase IV trial surveillance, then the drug may be taken off the market or its
use may be restricted.  

PREP Pearls
Drugs, vaccines, and medical devices undergo step­wise phases of clinical evaluation both before and after licensure for
medical use.

Phase I and II clinical trials are small and are designed to determine safe and appropriate dosing of a new drug or vaccine.

Phase III clinical trials determine efficacy, effectiveness, and safety of an established dose in large group of subjects, and
may lead to licensure if favorable results are obtained. Phase III is the only randomized phase of clinical trial development.

American Board of Pediatrics Content Specification(s)
Distinguish between Phase I, II, III, and IV clinical trials

Suggested Readings
Friedman LM, Furgerg CD, DeMets D. Fundamentals of Clinical Trials. 4th ed. New York, NY: Springer; 2010: 445 pp.

Kimberlin DW, Lin CY, Sánchez PJ, et al. Effect of ganciclovir therapy on hearing in symptomatic congenital cytomegalovirus
disease involving the central nervous system: a randomized, controlled trial. J Pediatr. 2003;143(1):16­25. DOI:
http://dx.doi.org/10.1016/S0022­3476(03)00192­6

Whitley RJ, Cloud G, Gruber W, et al. Ganciclovir treatment of symptomatic congenital cytomegalovirus infection: results of a
phase II study: National Institute of Allergy and Infectious Diseases Collaborative antiviral study group. J Infect Dis.
1997;175(5):1080­1086. DOI: http://dx.doi.org/10.1086/516445

Comment On This Question

Page 38 of 115

Contact Us
Links may open in separate window
Copyright © 2016   American Academy of Pediatrics. All rights reserved. Pop­up Blocker may need to be disabled

http://2016.prepid.courses.aap.org/script/april?req=201612122040300370&status=submit 2/2
12/13/2016 April

Welcome  mohammed alsaiary [ Logout ]

Home PREP Pearls FAQs My Bookmarks CME Information

Overview Home  > April

Editorial Board
April   Enter Keyword Search
January
Question View:   All (8) Jump to Question
February
Print this Page Add to my Bookmarks Page 39 of 115
March

April Assessment History

May ASSESSMENT PROGRESS:  Total Questions:  8  Questions Answered:  4  Correct Answers:  0

June

July You currently have 4 questions unanswered in this assessment

August

September Question: 5
October A 6­year­old boy with hereditary spherocytosis is hospitalized for severe anemia and parvovirus B19 infection. Associated signs and
November symptoms include a lack of energy, pallor, low­grade fever, splenomegaly, and a bright red rash on his cheeks. He has mild scleral
icterus. His hematologist explains that the patient is known to have a baseline mild anemia from his hematologic disorder, with
December hemoglobin concentrations between 9 g/dL and 11 g/dL (90­110 g/L) and reticulocyte counts typically between 8% and 12%. His
laboratory evaluation at the time of the present hospitalization showed a hemoglobin concentration of 5.8 g/dL (58 g/L), a reticulocyte
count of 0.3%, a total bilirubin of 2.8 mg/dL (47.9 µmol/L), and a direct bilirubin of 0.2 mg/dL (3.4 µmol/L).

Of the following, the BEST explanation for the decline in the patient’s hemoglobin concentration and reticulocyte count is that
parvovirus B19 infects

Claim Credit A. B­lymphocytes with production of anti­erythrocyte antibodies
Evaluation B. erythroid precursors, causing an interruption in erythropoiesis
My Learning Plan C. mature erythrocytes, causing cellular lysis
D. splenic T­lymphocytes with associated hypersplenism

Submit   Reset  

Page 39 of 115

Contact Us
Links may open in separate window
Copyright © 2016   American Academy of Pediatrics. All rights reserved. Pop­up Blocker may need to be disabled

http://2016.prepid.courses.aap.org/script/april?q­page=5&req=201612122040305057 1/1
12/13/2016 April

Welcome  mohammed alsaiary [ Logout ]

Home PREP Pearls FAQs My Bookmarks CME Information

Overview Home  > April

Editorial Board
April   Enter Keyword Search
January
Question View:   All (8) Jump to Question
February
Print this Page Add to my Bookmarks Page 39 of 115
March

April Assessment History

May ASSESSMENT PROGRESS:  Total Questions:  8  Questions Answered:  5  Correct Answers:  0

June

July You currently have 3 questions unanswered in this assessment

August

September Question: 5
October A 6­year­old boy with hereditary spherocytosis is hospitalized for severe anemia and parvovirus B19 infection. Associated signs and
November symptoms include a lack of energy, pallor, low­grade fever, splenomegaly, and a bright red rash on his cheeks. He has mild scleral
icterus. His hematologist explains that the patient is known to have a baseline mild anemia from his hematologic disorder, with
December hemoglobin concentrations between 9 g/dL and 11 g/dL (90­110 g/L) and reticulocyte counts typically between 8% and 12%. His
laboratory evaluation at the time of the present hospitalization showed a hemoglobin concentration of 5.8 g/dL (58 g/L), a reticulocyte
count of 0.3%, a total bilirubin of 2.8 mg/dL (47.9 µmol/L), and a direct bilirubin of 0.2 mg/dL (3.4 µmol/L).

Of the following, the BEST explanation for the decline in the patient’s hemoglobin concentration and reticulocyte count is that
parvovirus B19 infects

Claim Credit A. B­lymphocytes with production of anti­erythrocyte antibodies
Evaluation B. erythroid precursors, causing an interruption in erythropoiesis
My Learning Plan C. mature erythrocytes, causing cellular lysis
D. splenic T­lymphocytes with associated hypersplenism

Incorrect View Peer Results

Correct Answer: B Average Percent Correct:  95.24%

The primary cellular receptor for parvovirus B19 is blood group P antigen (also known as globoside). P
antigen is found on erythroid progenitor cells, erythroblasts, megakaryocytes and endothelial cells,
identifying these cell types as the major targets of the virus. When erythroid precursors are infected
with parvovirus B19, the cells undergo apoptosis resulting in a transient interruption of erythropoiesis.
Parvovirus B19 is not known to infect B­ or T­lymphocytes, and can only replicate in mitotically active
cells, so normoblasts and mature erythrocytes do not support infection. The patient in the vignette
shows a mild indirect hyperbilirubinemia, scleral icterus, and splenomegaly, which are all expected
findings in patients with forms of hemolytic anemia, including hereditary spherocytosis. Since parvovirus B19 does not infect
mature red blood cells, these findings are not secondary to virus­associated lysis of mature erythrocytes.

Healthy children who are infected with parvovirus B19 typically develop an acute, but self­limited cessation of erythrocyte
production with a corresponding decline in blood hemoglobin concentrations. The interruption lasts for approximately 1 week,
so healthy patients with normal erythroid cell turnover do not usually become anemic. In patients with higher red blood cell
turnover related to hemolysis or blood loss, the brief interruption in erythropoiesis can precipitate an aplastic crisis. Individuals
with underlying hematologic disorders that shorten the half life of circulating red blood cells rely on highly active erythroid
precursor cells to maintain their baseline red blood cell mass. Such patients are at risk for developing an aplastic crisis during
parvovirus B19 infection. Fetal infection is especially problematic because red blood cell turnover is very high during fetal
development. The risk of anemia­associated fetal hydrops is greatest when the mother and fetus are infected during the
second trimester of pregnancy.

Patients with moderate to severe congenital or acquired immune deficiencies may be unable to clear infection from parvovirus
B19. Newly formed bone marrow erythroid progenitor cells continue to be infected, leading to chronic, sometimes severe
anemia. Such patients do not produce detectable antiparvovirus B19 antibodies. Immune deficient patients who develop pure
red blood cell aplasia from the chronic parvovirus B19 infection demonstrate scattered giant pronormoblasts on bone marrow
examination (Figure 1). Administration of pooled immunoglobulin to assist in virus clearance can be beneficial, even curative.
Patients who do not clear infection may become red blood cell transfusion­dependent.

The nonhematologic manifestations seen with parvovirus B19 infection also depend on the underlying health and age of the
infected individual (Table). The classic clinical presentation of parvovirus B19 infection in otherwise healthy children is called
erythema infectiosum or “fifth disease,” but many infections in children are minimally symptomatic (or completely
asymptomatic) as individuals with detectable serum immunoglobulin G to parvovirus B19 often do not recall an illness

http://2016.prepid.courses.aap.org/script/april?req=201612122041132090&status=submit 1/2
12/13/2016 April
consistent with parvovirus B19 infection. For children with classic erythema infectiosum, a nonspecific prodrome of fever,
coryza, headache, and mild gastrointestinal distress is followed by the classic facial rash resembling “slapped cheeks”
(Figure 2). Some children go on to develop an erythematous maculopapular rash on the trunk and limbs that tends to have a
lacy appearance as it subsides. The rash may fade and reappear repeatedly over several weeks. Other dermatologic
manifestations of parvovirus B19 infection have been described including an uncommon, but classic syndrome referred to as
papular­purpuric gloves and socks syndrome, which is usually associated with the development of petechiae or purpura and
edema in the involved areas (Figure 3).

Another well­recognized nonhematologic manifestation of parvovirus B19 disease is the development of polyarthropathy. Joint
symptoms among children are not common, but a substantial proportion of adults (especially women) with recent acute
parvovirus B19 infection develop symmetrical small joint polyarthralgia that can progress to debilitating arthritis. When this
occurs, the polyarthritis can be problematic for months to years, mimicking rheumatoid arthritis.

The pathogenesis of the erythema infectiosum rash and polyarthropathy is thought to be immune complex­mediated. These
manifestations of disease present after the viremia has cleared, and are coincident with a detectable immune response to the
virus.

Infection is most common during childhood. By 15 years of age, approximately 50% of individuals show serologic evidence of
past infection. Women of childbearing age in the United States have an annual seroconversion rate of approximately 1%,
indicating the low but potential risk for developing infection during pregnancy, along with the potential complication of
parvovirus B19­associated fetal hydrops.

In temperate climates, parvovirus B19 infection is more common in the late winter, spring, and early summer. Humans are the
only known host. Community and household transmission is via respiratory droplets, so it is not surprising that the attack rate
in households and classrooms is 50% or higher for susceptible contacts. The usual age of infection and the high attack rate
explain why elementary and middle school outbreaks are seen regularly.

PREP Pearls
Parvovirus B19 infects erythroid progenitor cells, leading to a brief interruption in erythropoiesis.

Individuals with rapid red blood cell turnover, such as those with hemolytic anemia, are at risk for a parvovirus B19­
associated transient aplastic crisis.

Patients with moderate­to­severe immune deficiencies that interfere with their ability to clear parvovirus B19 infection are at
risk for developing chronic anemia.

American Board of Pediatrics Content Specification(s)
Know the epidemiology of parvovirus B19, including the age of acquisition, source, and means of transmission of
parvovirus

Understand the pathogenesis of parvovirus infection, including the site and type of cell infected

Suggested Readings
Brown KE, Anderson SM, Young NS. Erythrocyte P antigen: cellular receptor for B19 parvovirus. Science.
1993;262(5130):114­117. DOI: http://dx.doi.org/10.1126/science.8211117

Crabol Y, Terrier B, Rozenberg F, et al. Intravenous immunoglobulin therapy for pure red cell aplasia related to human
parvovirus b19 infection: a retrospective study of 10 patients and review of the literature. Clin Infect Dis. 2013;56(7):968­977.
DOI: http://dx.doi.org/10.1093/cid/cis1046

Gustafsson I, Kaldensjö T, Lindblom A, et al. Evaluation of parvovirus B19 infection in children with malignant or
hematological disorders. Clin Infect Dis. 2010;50(10):1426­1427. DOI: http://dx.doi.org/10.1086/652286

Comment On This Question

Page 39 of 115

Contact Us
Links may open in separate window
Copyright © 2016   American Academy of Pediatrics. All rights reserved. Pop­up Blocker may need to be disabled

http://2016.prepid.courses.aap.org/script/april?req=201612122041132090&status=submit 2/2
12/13/2016 April

Welcome  mohammed alsaiary [ Logout ]

Home PREP Pearls FAQs My Bookmarks CME Information

Overview Home  > April

Editorial Board
April   Enter Keyword Search
January
Question View:   All (8) Jump to Question
February
Print this Page Add to my Bookmarks Page 40 of 115
March

April Assessment History

May ASSESSMENT PROGRESS:  Total Questions:  8  Questions Answered:  5  Correct Answers:  0

June

July You currently have 3 questions unanswered in this assessment

August

September Question: 6
October A 12­year­old girl who underwent kidney transplantation 3 months prior for antineutrophil cytoplasmic autoantibody (ANCA)­positive
November microcytic polyangiitis is admitted for perianal pain and left lobar pneumonia. One month ago, she was started on prednisone 60 mg
orally twice daily for cellular rejection and she continues to receive cyclosporine. On admission, chest radiography reveals multilobar
December left­sided pneumonia (Figure), and intravenous cefepime and vancomycin are initiated. You are called because she continues to
have fever and hypoxemia 7 days into antibiotic treatment.

On physical examination, temperature is 39°C, heart rate is 124 beats/min, respiratory rate is 46 breaths/min, blood pressure is
123/76 mm Hg, and oxygen saturation is 98% on 8 L of oxygen via facemask. She is pale, tachypneic, and complains of substernal
chest pain. Breath sounds in the left upper and lower lobes are decreased, and she produces blood­tinged sputum during the
examination. Her perianal area appears indurated.
Claim Credit

Evaluation A complete blood cell count reveals 10,380 white blood cells/μL (10.4 x 109/L) (87% neutrophils, 3% bands, 10% lymphocytes),

My Learning Plan hemoglobin of 8.9 g/dL (89 g/L), and a platelet count of 191 x 103/μL (191 x 109/L). Serum galactomannan and (1,3)­β­D­glucan
assays are negative. Computed tomography of the chest reveals consolidative changes in the left upper lobe and portions of the left
lower lobe with pleural thickening.

Of the following, the BEST antimicrobial to include in this patient’s treatment regimen while additional studies are planned is

A. amphotericin
B. levofloxacin
C. linezolid
D. voriconazole

Submit   Reset  

Page 40 of 115

Contact Us
Links may open in separate window
Copyright © 2016   American Academy of Pediatrics. All rights reserved. Pop­up Blocker may need to be disabled

http://2016.prepid.courses.aap.org/script/april?q­page=6&req=201612122041139121 1/1
Figure. Chest radiograph showing multilobar left-sided pneumonia.

Courtesy of D Palazzi, MD
12/13/2016 April

Welcome  mohammed alsaiary [ Logout ]

Home PREP Pearls FAQs My Bookmarks CME Information

Overview Home  > April

Editorial Board
April   Enter Keyword Search
January
Question View:   All (8) Jump to Question
February
Print this Page Add to my Bookmarks Page 40 of 115
March

April Assessment History

May ASSESSMENT PROGRESS:  Total Questions:  8  Questions Answered:  6  Correct Answers:  0

June

July You currently have 2 questions unanswered in this assessment

August

September Question: 6
October A 12­year­old girl who underwent kidney transplantation 3 months prior for antineutrophil cytoplasmic autoantibody (ANCA)­positive
November microcytic polyangiitis is admitted for perianal pain and left lobar pneumonia. One month ago, she was started on prednisone 60 mg
orally twice daily for cellular rejection and she continues to receive cyclosporine. On admission, chest radiography reveals multilobar
December left­sided pneumonia (Figure), and intravenous cefepime and vancomycin are initiated. You are called because she continues to
have fever and hypoxemia 7 days into antibiotic treatment.

On physical examination, temperature is 39°C, heart rate is 124 beats/min, respiratory rate is 46 breaths/min, blood pressure is
123/76 mm Hg, and oxygen saturation is 98% on 8 L of oxygen via facemask. She is pale, tachypneic, and complains of substernal
chest pain. Breath sounds in the left upper and lower lobes are decreased, and she produces blood­tinged sputum during the
examination. Her perianal area appears indurated.
Claim Credit

Evaluation A complete blood cell count reveals 10,380 white blood cells/μL (10.4 x 109/L) (87% neutrophils, 3% bands, 10% lymphocytes),

My Learning Plan hemoglobin of 8.9 g/dL (89 g/L), and a platelet count of 191 x 103/μL (191 x 109/L). Serum galactomannan and (1,3)­β­D­glucan
assays are negative. Computed tomography of the chest reveals consolidative changes in the left upper lobe and portions of the left
lower lobe with pleural thickening.

Of the following, the BEST antimicrobial to include in this patient’s treatment regimen while additional studies are planned is

A. amphotericin
B. levofloxacin
C. linezolid
D. voriconazole

Incorrect View Peer Results
Correct Answer: A Average Percent Correct:  26.19%

The patient described in the vignette has impaired immunity due to therapy for renal transplantation
with rejection and is at increased risk for pulmonary infection from multiple pathogens (Table). The lack
of response to broad spectrum antibiotics, the presence of substernal chest pain with blood­tinged
sputum, and the finding of perianal induration suggest invasive mold infection. While Aspergillus is a
more common cause of pulmonary infection in transplant patients, mucormycosis is suggested by
negative galactomannan and (1,3)­β­D­glucan assays. Amphotericin B is the preferred initial treatment
for this patient in order to cover broadly for invasive fungal infection until additional data are available.

Mucormycosis generally refers to angioinvasive infections caused by fungi in the Mucorales order. These fungi are ubiquitous
in nature (eg, soil, decaying vegetation). Most human infections are caused by Rhizopus and Mucor species. Risk factors for
mucormycosis include uncontrolled diabetes, neutropenia, iron overload, immunosuppression (eg, steroids, tumor necrosis
factor­α blockers), graft­versus­host disease, and hematologic malignancy with opportunistic infections (eg, cytomegalovirus).

Rhino­orbital­cerebral mucormycosis is most common, but pulmonary, cutaneous, and gastrointestinal disease can also
occur. Disseminated disease usually begins in the lungs and spreads hematogenously to other sites, including the central
nervous system. Diagnosis is often difficult and delayed because there are no serum assays available for detection. For
immunocompromised patients with pulmonary mucormycosis or other difficult to diagnose pulmonary infections, invasive
procedures (eg, transbronchial, percutaneous, video­assisted thoracoscopic or open lung biopsy) are often required to obtain
tissue for histopathology and culture.

Bronchoalveolar lavage can be used in immunocompromised patients for the detection of routine bacterial pathogens,
Pneumocystis jirovecii, and mycobacteria using appropriate stains or cultures, but is not generally useful for the diagnosis of
mucormycosis. Broncoalveolar lavage fluid can be tested for Aspergillus galactomannan antigen and for the detection of

http://2016.prepid.courses.aap.org/script/april?req=201612122044217406&status=submit 1/2
12/13/2016 April
opportunistic viral pathogens (eg, cytomegalovirus) using culture or quantitative polymerase chain reaction assays. Routine
sputum samples (or tracheal samples for intubated patients) should be obtained whenever feasible for bacterial stains and
cultures. Nasal washings or swabs for rapid viral testing, viral cultures, and polymerase chain reactions can be used for
detecting influenza, respiratory syncytial virus, parainfluenza, adenovirus, and metapneumovirus. Urinary antigen testing can
be helpful for diagnosing endemic mycoses and Legionella. Serologic testing is of limited utility in immunocompromised
patients who may be unable to generate an immune response to new infection or who may have prior immunity to infection,
making negative or positive tests uninterpretable. Empirical therapy for immunocompromised patients with pneumonia should
be broad spectrum and include coverage against Gram­positive and Gram­negative bacterial pathogens as well as other
pathogens (eg, viruses, fungi), as indicated by the host’s underlying immunodeficiency, clinical presentation, and other risk
factors.

For patients with rhino­orbital­cerebral mucormycosis, aggressive surgical debridement is required. This approach also has
been employed for patients with unilobar pneumonia, but multilobar disease precludes surgical intervention. While the
organisms causing mucormycosis are variably susceptible to amphotericin B, it remains the drug of choice because
resistance to most other conventional antifungal agents is common. Posaconazole is an alternative agent. The mortality rate
for mucormycosis is high, but varies depending on the host, site of infection, and use of surgical intervention.

Voriconazole is the treatment of choice for invasive aspergillosis but it does not have activity against organisms in the
Mucorales order. Levofloxacin can be used for immunocompromised patients with pneumonia caused by Streptococcus
pneumoniae, Legionella, and susceptible Gram­negative pathogens (eg, Pseudomonas). Linezolid is appropriate for treating
pneumonia caused by methicillin­resistant Staphylococcus aureus and Streptococcus pneumoniae in patients who cannot
tolerate other agents.

Errata: Content revised for clarification. 10/2016

* Required *
Take Survey  

PREP Pearls
Empirical therapy for immunocompromised patients with pneumonia should be broad spectrum and include coverage
against Gram­positive and Gram­negative bacterial pathogens as well as other pathogens (eg, viruses, fungi), as indicated
by the host’s underlying immunodeficiency, clinical presentation, and other risk factors.

For immunocompromised patients with difficult to diagnose pulmonary infections, invasive procedures (eg, transbronchial,
percutaneous, video­assisted thoracoscopic or open lung biopsy) are often required to obtain tissue for histopathology and
culture.

Risk factors for mucormycosis include uncontrolled diabetes, neutropenia, iron overload, immunosuppression, graft­versus­
host disease, and hematologic malignancy with opportunistic infections.

Serum fungal assays (galactomannan and (1,3)­β­D­glucan) are negative in the setting of mucormycosis.

Amphotericin B is the initial treatment of choice for mucormycosis.

American Board of Pediatrics Content Specification(s)
Recognize specific predisposing factors (eg, immunologic, anatomic) for pneumonia in an immunocompromised host

Plan the initial diagnostic testing and treatment of pneumonia in an immunocompromised host

Suggested Readings
Chinen J, Kline MW, Shearer WT, et al. Infections of the compromised host. In: J Cherry, G Harrison, S Kaplan, W Steinbach,
P Hotez, eds. Feigin & Cherry's Textbook of Pediatric Infectious Diseases. 7th ed. Philadelphia, PA: Saunders Elsevier; 2013:
902­1094.

Ibrahim AS, Spellberg B, Walsh TJ, Kontoyiannis DP. Pathogenesis of mucromycosis. Clin Infect Dis. 2012;54 Suppl 1:S16­
S22. DOI: http://dx.doi.org/10.1093/cid/cir865

Rañó A, Agusti C, Sibila O, Torres A. Pulmonary infections in non­HIV­immunocompromised patients. Curr Opin Pulm Med.
2005;11(3):213­217.

Comment On This Question

Page 40 of 115

Contact Us
Links may open in separate window
Copyright © 2016   American Academy of Pediatrics. All rights reserved. Pop­up Blocker may need to be disabled

http://2016.prepid.courses.aap.org/script/april?req=201612122044217406&status=submit 2/2
Table. Examples of Pathogens Causing Pneumonia in Immunocompromised Patients.
Type or etiology of immune suppression Pathogens
Asplenia Encapsulated bacteria (eg, Streptococcus pneumoniae, Neisseria
meningitidis)
Autoimmune diseases (eg, lupus) Bacteria
Pneumocystis jirovecii pneumonia (PJP) with steroid use
Respiratory virus co-infection
B-cell defect Bacteria
Biologic agents and lympho-depletion
B lymphocyte Encapsulated bacteria
T lymphocyte Herpesviruses, molds
Tumor necrosis factor alpha inhibitors Mycobacteria, Legionella, viruses, fungi (PJP, Aspergillus, Cryptococcus,
Mucorales, endemic mycoses [eg, Histoplasma], Coccidioides)
HIV Bacteria
Viruses
Fungi (eg, PJP)
Mycobacteria (eg, Mycobacterium tuberculosis)
Parasites (eg, Toxoplasma, Strongyloides)
Neutropenia Bacteria (eg, Gram-positive including Nocardia and Gram-negative)
Fungi (eg, PJP, Aspergillus, other molds)
Herpesviruses and respiratory tract viruses
Nuclear factor-kappa-B (NF-kB) regulation Bacteria
defects Viruses
Nontuberculous mycobacteria
Prolonged intubation Bacteria
Respiratory viruses
Solid organ and bone marrow transplant Bacteria including Nocardia
Viruses (eg cytomegalovirus, human herpesvirus 6)
Fungi (PJP, Aspergillus, other molds)
Parasites (eg, Strongyloides)
Steroids Bacteria (eg, Gram-positive including Nocardia and Gram negative)
Fungi (eg, PJP, Aspergillus, other molds)
Viruses (eg, cytomegalovirus, varicella-zoster virus)
T-cell defect Viral infections including Epstein-Barr virus reactivation
Courtesy of D Palazzi, MD
12/13/2016 April

Welcome  mohammed alsaiary [ Logout ]

Home PREP Pearls FAQs My Bookmarks CME Information

Overview Home  > April

Editorial Board
April   Enter Keyword Search
January
Question View:   All (8) Jump to Question
February
Print this Page Add to my Bookmarks Page 41 of 115
March

April Assessment History

May ASSESSMENT PROGRESS:  Total Questions:  8  Questions Answered:  6  Correct Answers:  0

June

July You currently have 2 questions unanswered in this assessment

August

September Question: 7
October You are asked to evaluate an 8­year­old boy who recently arrived in the United States from Central America. He spent 7 days in a
November detainment camp before being allowed to relocate with relatives in your community. Over the past 2 days, he has developed a tactile
fever and maculopapular eruption beginning on his trunk that is spreading distally. He complains of chills, headache, diffuse
December achiness, nausea, and diffuse abdominal pain. He remembers a number of other people in the detainment camp having similar
symptoms while he was there.

On physical examination, he has a temperature of 39.9°C, heart rate of 104 beats/min, blood pressure of 108/60 mm Hg, and
respiratory rate of 22 breaths/min. The remainder of the physical examination is normal, with the exception of diffuse abdominal
tenderness and a generalized maculopapular rash that spares the palms and soles.

Claim Credit Of the following, the BEST test for confirming this child’s diagnosis is
Evaluation
A. blood for polymerase chain reaction for rickettsial DNA
My Learning Plan B. measles immunoglobulin M antibody titer
C. rapid plasma reagin
D. thick smear for malaria

Submit   Reset  

Page 41 of 115

Contact Us
Links may open in separate window
Copyright © 2016   American Academy of Pediatrics. All rights reserved. Pop­up Blocker may need to be disabled

http://2016.prepid.courses.aap.org/script/april?q­page=7&req=201612122044229750 1/1
12/13/2016 April

Welcome  mohammed alsaiary [ Logout ]

Home PREP Pearls FAQs My Bookmarks CME Information

Overview Home  > April

Editorial Board
April   Enter Keyword Search
January
Question View:   All (8) Jump to Question
February
Print this Page Add to my Bookmarks Page 41 of 115
March

April Assessment History

May ASSESSMENT PROGRESS:  Total Questions:  8  Questions Answered:  7  Correct Answers:  1

June

July You currently have 1 questions unanswered in this assessment

August

September Question: 7
October You are asked to evaluate an 8­year­old boy who recently arrived in the United States from Central America. He spent 7 days in a
November detainment camp before being allowed to relocate with relatives in your community. Over the past 2 days, he has developed a tactile
fever and maculopapular eruption beginning on his trunk that is spreading distally. He complains of chills, headache, diffuse
December achiness, nausea, and diffuse abdominal pain. He remembers a number of other people in the detainment camp having similar
symptoms while he was there.

On physical examination, he has a temperature of 39.9°C, heart rate of 104 beats/min, blood pressure of 108/60 mm Hg, and
respiratory rate of 22 breaths/min. The remainder of the physical examination is normal, with the exception of diffuse abdominal
tenderness and a generalized maculopapular rash that spares the palms and soles.

Claim Credit Of the following, the BEST test for confirming this child’s diagnosis is
Evaluation
A. blood for polymerase chain reaction for rickettsial DNA
My Learning Plan B. measles immunoglobulin M antibody titer
C. rapid plasma reagin
D. thick smear for malaria

Correct View Peer Results
Average Percent Correct:  53.57%

The recent experience of the child being in a detainment camp with crowded conditions in association
with a clinical illness characterized by abrupt onset of high fever, chills, a maculopapular eruption
beginning on the trunk that is spreading to the extremities with sparing of the palms and soles,
myalgias, abdominal pain, and headache is most consistent with a diagnosis of epidemic typhus
transmitted in crowded settings with poor sanitary conditions by the human body louse, Pediculus
humanus corporis.  

The other responses are tests for diagnoses that are not consistent with the clinical illness presented in this vignette.
Secondary syphilis may present with a diffuse maculopapular rash, however, the palms and soles are typically involved in this
condition. Additionally, high fever and the systemic signs and symptoms described for this child are not consistent with
syphilis, making a rapid plasma reagin incorrect. Measles may present with high fever, constitutional symptoms, and a rash.
However, the rash begins on the head and neck, and progresses downward, following a 4­ to 5­day prodrome of fever, cough,
coryza, and conjunctivitis. High fever, rigor, and headaches may be part of the clinical presentation for malaria, but extensive
rash is not typical. Furthermore, Plasmodium vivax would be the most likely cause of malaria in this geographical location,
whereas the most severe systemic illness from malaria is associated with Plasmodium falciparum.

Although rare in the United States, epidemic typhus occurs throughout the world, including Central and South America, Africa,
the colder mountain regions of Asia, and some parts of Europe. Rare transmission of Rickettsia prowazekii from flying
squirrels (Glaucomys volans) has also been reported in the United States.

The causative organism R prowazekii may be detected within the first week of illness in blood or serum specimens by
polymerase chain reaction (PCR) assay. If PCR is not available, acute and convalescent titers obtained 2 to 6 weeks apart
measured by an indirect fluorescent antibody (IFA) test detecting a 4­fold rise in immunoglobulin G antibodies confirms the
diagnosis. Both tests may yield false­negative results. Cross­reactivity with Rickettsia rickettsii and other spotted fever group
rickettsiae may occur. Doxycycline is the treatment of choice for epidemic typhus and other rickettsial infections. The PCR
testing should be obtained prior to or within 48 hours of therapy. Untreated epidemic typhus has a mortality rate of up to 30%.
Recrudescence of epidemic typhus occurring years after the initial episode is called Brill­Zinsser disease and is generally
milder than the original infection.

http://2016.prepid.courses.aap.org/script/april?req=201612122046073346&status=submit 1/2
12/13/2016 April
Endemic typhus (scrub typhus), caused by Rickettsia typhi, presents with a similar clinical illness, although it is less abrupt in
onset and tends to be milder than epidemic typhus. The primary host for R typhi is the rat and the rat flea, Xenopsylla, the
vector for transmission to humans. Although rare in the United States, cases of scrub typhus have been reported primarily in
southern California, the southeastern Gulf Coast, southern Texas, and Hawaii. Serologic testing for antibodies to R typhi is
available using IFA, enzyme immunoassay, or latex agglutination techniques.

PREP Pearls
Endemic and epidemic typhus are vector­borne rickettsial infections transmitted by the rat flea and human louse,
respectively.

Diagnosis is classically confirmed by 4­fold rise in immunoglobulin G antibodies detected in acute and convalescent
specimens; although a polymerase chain reaction on blood or serum for Rickettsia prowazekii (epidemic typhus), if
available, can make the diagnosis during the acute illness.

American Board of Pediatrics Content Specification(s)
Plan appropriate diagnostic testing for a patient in whom endemic typhus is suspected

Recognize the setting, vector, and clinical manifestations of epidemic typhus

Plan appropriate diagnostic testing for a patient in whom epidemic typhus is suspected

Suggested Readings
American Academy of Pediatrics. Endemic typhus. In: Kimberlin DW, Brady MT, Jackson MA, Long SS, eds. Red Book: 2015
Report of the Committee on Infectious Diseases. 30th ed. Elk Grove Village, IL: American Academy of Pediatrics; 2015: 841­
842.

American Academy of Pediatrics. Epidemic typhus. In: Kimberlin DW, Brady MT, Jackson MA, Long SS, eds. Red Book: 2015
Report of the Committee on Infectious Diseases. 30th ed. Elk Grove Village, IL: American Academy of Pediatrics; 2015: 843­
845.

McQuiston J. Rickettsial (spotted and typhus fevers) and related infections (anaplasmosis & ehrlichiosis). US Centers of
Disease Control and Prevention website. http://wwwnc.cdc.gov/travel/yellowbook/2016/infectious­diseases­related­to­
travel/rickettsial­spotted­typhus­fevers­related­infections­anaplasmosis­ehrlichiosis. Updated July 10, 2015

Comment On This Question

Page 41 of 115

Contact Us
Links may open in separate window
Copyright © 2016   American Academy of Pediatrics. All rights reserved. Pop­up Blocker may need to be disabled

http://2016.prepid.courses.aap.org/script/april?req=201612122046073346&status=submit 2/2
12/13/2016 April

Welcome  mohammed alsaiary [ Logout ]

Home PREP Pearls FAQs My Bookmarks CME Information

Overview Home  > April

Editorial Board
April   Enter Keyword Search
January
Question View:   All (8) Jump to Question
February
Print this Page Add to my Bookmarks Page 42 of 115
March

April Assessment History

May ASSESSMENT PROGRESS:  Total Questions:  8  Questions Answered:  7  Correct Answers:  1

June

July You currently have 1 questions unanswered in this assessment

August

September Question: 8
October A 15­year­old adolescent boy experienced an acute febrile illness with pharyngitis about 3 weeks prior to the onset of neuropathy of
November multiple peripheral nerves. He was hospitalized briefly in another hospital and is now being evaluated by a neurologist, who is
awaiting transfer of records. She is consulting you to discuss work­up for possible infectious causes of polyneuropathy, and is unable
December to furnish you with an accurate history of exposure to infectious agents at this time.

Of the following, the infectious agent that is MOST commonly associated with pharyngitis and polyneuropathy is

A. Borrelia burgdorferi
B. Corynebacterium diphtheriae
Claim Credit C. Cytomegalovirus

Evaluation D. Epstein­Barr virus

My Learning Plan E. herpes simplex virus

Submit   Reset  

Page 42 of 115

Contact Us
Links may open in separate window
Copyright © 2016   American Academy of Pediatrics. All rights reserved. Pop­up Blocker may need to be disabled

http://2016.prepid.courses.aap.org/script/april?q­page=8&req=201612122046080221 1/1
12/13/2016 April

Welcome  mohammed alsaiary [ Logout ]

Home PREP Pearls FAQs My Bookmarks CME Information

Overview Home  > April

Editorial Board
April   Enter Keyword Search
January
Question View:   All (8) Jump to Question
February
Print this Page Add to my Bookmarks Page 42 of 115
March

April Assessment History

May ASSESSMENT PROGRESS:  Total Questions:  8  Questions Answered:  8  Correct Answers:  1

June

July You currently have 2 required survey(s) that are incomplete. Please complete the survey(s) for the following
question(s) on this assessment:  Question 3, Question 6
August

September

October Question: 8
November A 15­year­old adolescent boy experienced an acute febrile illness with pharyngitis about 3 weeks prior to the onset of neuropathy of
multiple peripheral nerves. He was hospitalized briefly in another hospital and is now being evaluated by a neurologist, who is
December awaiting transfer of records. She is consulting you to discuss work­up for possible infectious causes of polyneuropathy, and is unable
to furnish you with an accurate history of exposure to infectious agents at this time.

Of the following, the infectious agent that is MOST commonly associated with pharyngitis and polyneuropathy is

A. Borrelia burgdorferi
B. Corynebacterium diphtheriae
Claim Credit
C. Cytomegalovirus
Evaluation
D. Epstein­Barr virus
My Learning Plan
E. herpes simplex virus

Incorrect View Peer Results
Correct Answer: B Average Percent Correct:  36.90%

Peripheral neuropathy syndromes can follow acute infectious illnesses. Most of these are
polyneuropathies. Of the infectious agents listed for the patient with pharyngitis in the vignette,
Corynebacterium diphtheriae is the one most commonly associated with polyneuropathy. Epstein­Barr
virus and Cytomegalovirus (CMV) are associated with post­infectious polyneuropathy, including
Guillain­Barré syndrome, but at a much lower rate. A causative role for herpes simplex virus infection in
Bell’s palsy is proposed based on studies that detected virus DNA in endoneurial fluid, but the virus is
not associated with polyneuropathy, and the most common neuropathy associated with Borrelia
burgdorferi is a seventh cranial nerve mononeuropathy (Lyme facial palsy).

Twenty percent to 70% of patients with diphtheria develop a peripheral polyneuropathy following resolution of the acute
infection. Three to 4 weeks after the illness, cranial nerve paresis results in decreased pharyngeal sensation, dysphagia,
dysarthria, hoarseness, or blurred vision. Two to 3 months later, a generalized polyneuropathy with sensory and motor
deficits, paresthesias and respiratory failure can follow. The pathogenesis is a diffuse axonal sensorimotor neuropathy caused
by an exotoxin of C diphtheriae, which usually is self­limited after several months.

Acute manifestations of peripheral nervous system involvement in B burgdorferi infection (Lyme disease) include, cranial
neuropathy, mononeuropathy multiplex, and Bannwarth syndrome. Unilateral or bilateral facial palsy is common in early Lyme
disease. Other cranial nerve involvement is rare, but in children, the optic nerve may be affected because of inflammation or
increased intracranial pressure, which may lead to blindness. Acute painful meningoradiculitis (Bannwarth syndrome)
develops several weeks after the tick bite, with multiple spinal or cranial nerve involvement. An asymmetrical sensorimotor
radiculoneuropathy with weakness may follow in some patients. Electrophysiology demonstrates predominantly axonal
degeneration and occasionally demyelination. Late Lyme disease­associated peripheral polyneuropathy is uncommon; it
presents as a mild, diffuse, intermittent “stocking glove” limb paresthesias, and reduced vibratory sensation of the distal lower
extremities. In acrodermatitis chronica atrophicans, a chronic manifestation of Lyme disease that occurs in Europe, peripheral
neuropathy in the affected extremity is common, manifesting primarily as loss of sensation.

Neuropathies are common in Mycobacterium leprae infection (leprosy or Hansen disease), being a major clinical feature of
the tuberculoid and borderline forms. Cutaneous nerves of the ears and distal limbs are most commonly involved. The pattern
may be mononeuropathy, mononeuropathy multiplex, or a slowly progressive, symmetric, sensorimotor peripheral
polyneuropathy. Diagnosis is established by skin lesion biopsy showing granulomas, with a paucity of bacilli in tuberculoid
form. If there are no obvious skin lesions, nerve biopsy can be diagnostic.

http://2016.prepid.courses.aap.org/script/april?req=201612122047114597&status=submit 1/2
12/13/2016 April
Peripheral neuropathy is common in HIV­infected individuals because of several mechanisms. Presentations include distal
symmetric polyneuropathy (immune­mediated), inflammatory demyelinating polyneuropathy (either acute or chronic), multiple
mononeuropathies (vasculitis or CMV­related), polyradiculopathy (usually CMV­related), autonomic neuropathy, sensory
neuronopathy due to dorsal root ganglionitis, and neurotoxicity secondary to antiviral medications (eg, dideoxycytidine,
dideoxyinosine, stavudine).

An anatomical approach (location and type of neurologic deficits) is needed to establish the cause of a peripheral neuropathy.
Physical diagnosis should include examination for motor function, atrophy of muscles, tendon reflexes, sensory loss,
paresthesias, pain, dysesthesias, deformity and trophic changes, autonomic dysfunction (especially anhidrosis and orthostatic
hypotension), sensory ataxia, tremor, proprioception, fasciculations, cramps, and spasms. Nerve conduction studies and
electromyogram help determine whether the neuropathy is demyelinating or axonal, or whether the patient has a primary or
secondary myopathy. The table lists infectious causes by type of peripheral neuropathy (Table).

PREP Pearls
The clinical evaluation of peripheral neuropathy includes the location and type of neurologic deficits, the time course, and
the presence of underlying conditions.

Infectious causes of peripheral neuropathy are uncommon, but can be recognized by specific clinical features.

American Board of Pediatrics Content Specification(s)
Recognize the clinical manifestations of peripheral neuropathy/neuritis

Identify the organism most commonly associated with peripheral neuropathy/neuritis (eg, Borrelia, virus)

Suggested Readings
Amato AA, Barohn RJ. Peripheral neuropathy. In: Longo DL, Kasper DL, Jameson JL, Fauci AS, Hauser SL, Loscalzo J, eds.
Harrison's Principles Of Internal Medicine. 18th ed. New York, NY: McGraw­Hill; 2011: 3448­3472.

Dyck PJ, Thomas PK. Neuropathy associated with infections. Peripheral Neuropathy. 4th ed. Philadelphia, PA: Saunders
Elsevier; 2005: 2081­2176.

Ropper AH, Samuels MA. Diseases of the peripheral nerves. Adams and Victor's Principles of Neurology. 9th ed. New York,
NY: McGraw­Hill; 2009: 1572 pp.

Yuki N, Hartung HP. Guillain­Barré syndrome. N Engl J Med. 2012;366(24):2294­2304. DOI:
http://dx.doi.org/10.1056/NEJMra1114525

Comment On This Question

Page 42 of 115

Contact Us
Links may open in separate window
Copyright © 2016   American Academy of Pediatrics. All rights reserved. Pop­up Blocker may need to be disabled

http://2016.prepid.courses.aap.org/script/april?req=201612122047114597&status=submit 2/2
12/13/2016 May

Welcome  mohammed alsaiary [ Logout ]

Home PREP Pearls FAQs My Bookmarks CME Information

Overview Home  > May

Editorial Board
May   Enter Keyword Search
January
Question View:   All (8) Jump to Question
February
Print this Page Add to my Bookmarks Page 44 of 115
March

April Assessment History

May ASSESSMENT PROGRESS:  Total Questions:  8  Questions Answered:  0  Correct Answers:  0

June

July You currently have 8 questions unanswered in this assessment

August

September Question: 1
October A 6­year­old girl with a history or recurrent urinary tract infections presents with a 2­month history of abdominal pain, weight loss,
November anorexia, and intermittent dysuria. Physical examination reveals a temperature of 38.6°C, diffuse abdominal tenderness, and a
palpable tender mass on the right flank. Urinalysis shows pyuria, hematuria, and presence of bacteria. Culture of the urine grows
December greater than 100,000 colony­forming units per mL of Escherichia coli that is resistant to ampicillin and cephalexin but susceptible to
cefotaxime and trimethoprim­sulfamethoxazole. After 5 days of treatment with intravenous cefotaxime, the child's symptoms persist,
and a repeat urine culture is still positive for the same E coli.

Computed tomography of the kidneys shows an enlarged right kidney, multiple ring­enhancing lesions, and the presence of renal
calculi. Ultrasound­guided percutaneous biopsy of one of the lesions shows chronic granulomatous inflammation with lipid­laden
macrophages, and Gram­negative bacteria on Gram stain, proven by culture to be E coli.
Claim Credit

Evaluation Of the following, the BEST description of this child's condition is

My Learning Plan A. chronic pyelonephritis
B. perinephric abscess
C. renal abscess
D. xanthogranulomatous pyelonephritis

Submit   Reset  

Page 44 of 115

Contact Us
Links may open in separate window
Copyright © 2016   American Academy of Pediatrics. All rights reserved. Pop­up Blocker may need to be disabled

http://2016.prepid.courses.aap.org/script/may?q­page=1&req=201612122047441005 1/1
12/13/2016 May

Welcome  mohammed alsaiary [ Logout ]

Home PREP Pearls FAQs My Bookmarks CME Information

Overview Home  > May

Editorial Board
May   Enter Keyword Search
January
Question View:   All (8) Jump to Question
February
Print this Page Add to my Bookmarks Page 44 of 115
March

April Assessment History

May ASSESSMENT PROGRESS:  Total Questions:  8  Questions Answered:  1  Correct Answers:  0

June

July You currently have 7 questions unanswered in this assessment

August

September Question: 1
October A 6­year­old girl with a history or recurrent urinary tract infections presents with a 2­month history of abdominal pain, weight loss,
November anorexia, and intermittent dysuria. Physical examination reveals a temperature of 38.6°C, diffuse abdominal tenderness, and a
palpable tender mass on the right flank. Urinalysis shows pyuria, hematuria, and presence of bacteria. Culture of the urine grows
December greater than 100,000 colony­forming units per mL of Escherichia coli that is resistant to ampicillin and cephalexin but susceptible to
cefotaxime and trimethoprim­sulfamethoxazole. After 5 days of treatment with intravenous cefotaxime, the child's symptoms persist,
and a repeat urine culture is still positive for the same E coli.

Computed tomography of the kidneys shows an enlarged right kidney, multiple ring­enhancing lesions, and the presence of renal
calculi. Ultrasound­guided percutaneous biopsy of one of the lesions shows chronic granulomatous inflammation with lipid­laden
macrophages, and Gram­negative bacteria on Gram stain, proven by culture to be E coli.
Claim Credit

Evaluation Of the following, the BEST description of this child's condition is

My Learning Plan A. chronic pyelonephritis
B. perinephric abscess
C. renal abscess
D. xanthogranulomatous pyelonephritis

Incorrect View Peer Results

Correct Answer: D Average Percent Correct:  82.50%

This child has xanthogranulomatous pyelonephritis (XGP), a rare variant of chronic refractory
pyelonephritis, characterized by a classic triad of palpable flank mass, renal calculi, and a unilateral
enlarged, nonfunctioning kidney. It is usually an illness of adults, however, it has rarely been reported in
infants and children. The typical initial presentation is fever, abdominal pain, weight loss, and anorexia.
Physical examination may reveal a palpable flank mass. Laboratory evaluation may show elevated
sedimentation rate, anemia, and leukocytosis. Urinalysis is often abnormal with pyuria and hematuria,
and the urine culture may be positive. The illness is usually caused by a chronic renal infection with
Gram­negative bacteria such as Escherichia coli or Proteus mirabilis. Imaging classically shows a unilateral large non­ or
poorly functioning kidney and a renal pelvic stone or staghorn calculus. The presence of a large kidney helps differentiate
XGP from the more classic form of chronic pyelonephritis, in which the kidney is often shrunken in size. A "bear's claw sign"
showing dilated calyces with a contracted renal pelvis, along with focal or multiple hypodense ring­enhancing lesions, is
usually seen on computed tomography of the kidney in XGP. Psoas abscess formation may complicate this infection. The
process may involve the entire kidney or may be focal. In children, the focal form appears to be more common in some case
series. Renal biopsy shows pathognomonic histologic findings of chronic granulomatous inflammation with lipid­laden
macrophages that coalesce into discrete yellow (xanthochromic) nodules, and established the diagnosis in the patient in the
vignette. Small abscess cavities may involve a focal area of the kidney, but are often seen throughout the kidney. Treatment
of the infection with systemic antibiotics should be initiated, but renal destruction is often so severe and irreversible that partial
or complete nephrectomy is necessary.

Urinary tract infections (UTIs) include asymptomatic bacteriuria and uncomplicated cystitis if only the lower tract is involved.
However, involvement of the upper urinary tract may complicate lower tract disease, causing more severe disease. These
complications include pyelonephritis and renal abscesses.

Pyelonephritis involves the kidney and may present in patients with high spiking fevers, chills, back pain, and vomiting, along
with lower urinary tract signs and symptoms. Pyelonephritis may progress to acute focal bacterial nephritis or acute lobar
nephropathy, which may then, in turn, progress to a localized renal abscess; pyelonephritis may also progress to a
perinephric abscess. Renal abscess is usually a focal lesion, not the multiple ring­enhancing lesions on imaging
demonstrated by this patient. A perinephric space abscess or psoas abscess may complicate XGP.

http://2016.prepid.courses.aap.org/script/may?req=201612122048426944&status=submit 1/2
12/13/2016 May
Most urinary tract infections are caused by Gram­negative bacteria, members of Enterobacteriaceae. These organisms
include E coli, Klebsiella, Proteus, Enterobacter, and Citrobacter. Pseudomonas and many other Gram­negative bacteria also
may cause UTIs. Gram­positive bacteria, such as Staphylococcus saprophyticus, may also cause UTIs.

Primary renal abscess occurs by a hematogenous spread of bacteria from the blood stream to the kidney and usually results
in a single cortical renal abscess. Renal abscesses resulting from a complication of UTI are more often caused by Gram­
negative organisms. The most common organism associated with renal abscess are Staphylococcus aureus or group A
streptococcus. Unusual organisms isolated from children with pyelonephritis include anaerobes, Bartonella henselae, and
Mycobacterium tuberculosis. In immune­compromised children, fungal organisms, especially Aspergillus, may be involved.

Cultures of blood, urine, abscess material, and in some cases, biopsy of tissue from the involved kidney, will help establish
the diagnosis of a complicated UTI, and help guide appropriate antimicrobial treatment.

Imaging procedures to assess renal anatomy in children with complicated UTIs include renal ultrasonography for detection of
duplication or dilation of ureters, presence of ureteroceles, and horseshoe kidney or absent kidney; renal scintigraphy or
magnetic resonance imaging for acute pyelonephritis; voiding cystourethrogram for vesicoureteral reflux; and computed
tomography for focal nephronia, abscess, or XGP. Intravenous pyelography is now rarely used in contemporary management
of complicated UTIs.

PREP Pearls
Renal abscesses resulting from a complication of a urinary tract infection are more often caused by Gram­negative
organisms, whereas renal abscesses from hematogenous spread are more often caused by Staphylococcus aureus.

Xanthogranulomatous pyelonephritis is a rare but serious variant of chronic refractory pyelonephritis, often requiring partial
or total nephrectomy for cure.

American Board of Pediatrics Content Specification(s)
Recognize the complications of urinary tract infection (renal abscess, perinephric abscess, reflux uropathy),
xanthogranulomatous pyelonephritis

Suggested Readings
Bingol­Kologlu M, Ciftci A, Senocak M, Tanyel FC, Karnak I, Büyükpamukçu N. Xanthogranulomatous pyelonephritis in
children: diagnostic and therapeutic aspects. Eur J Pediatr Surg. 2002;12(1):42­48. DOI: http://dx.doi.org/10.1055/s­2002­
25085

Gonzales E, Kaplan S. Renal abscess. In: Cherry J, Demmler­Harrison GJ, Kaplan SL, Steinbach WJ, Hotez P, eds. Feigin
and Cherry's Textbook of Pediatric Infectious Diseases. 7th ed. Philadelphia, PA: Saunders Elsevier; 2014: 553­558.

Hussein N, Osman Y, Sarhan O, el­Diasty T, Dawaba M. Xanthogranulomatous pyelonephritis in pediatric patients: effect of
surgical approach. Urology. 2009;73(6):1247­1250. DOI: http://dx.doi.org/10.1016/j.urology.2009.01.014

Zugor V, Schott G, Labanaris AP. Xanthogranulomatous pyelonephritis in childhood: a critical analysis of 10 cases and of the
literature. Urology. 2007;70(1):157­160. DOI: http://dx.doi.org/10.1016/j.urology.2007.02.068

Comment On This Question

Page 44 of 115

Contact Us
Links may open in separate window
Copyright © 2016   American Academy of Pediatrics. All rights reserved. Pop­up Blocker may need to be disabled

http://2016.prepid.courses.aap.org/script/may?req=201612122048426944&status=submit 2/2
12/13/2016 May

Welcome  mohammed alsaiary [ Logout ]

Home PREP Pearls FAQs My Bookmarks CME Information

Overview Home  > May

Editorial Board
May   Enter Keyword Search
January
Question View:   All (8) Jump to Question
February
Print this Page Add to my Bookmarks Page 45 of 115
March

April Assessment History

May ASSESSMENT PROGRESS:  Total Questions:  8  Questions Answered:  1  Correct Answers:  0

June

July You currently have 7 questions unanswered in this assessment

August

September Question: 2
October A 20­year­old man presents with a 10­day history of constant and severe headaches, intermittent high­grade fever with chills, painful
November lumps on the back of his neck, a 10­lb weight loss, and myalgias. About 3 months ago, he returned after a 3­week long safari trip to
east Africa. While on his trip, he experienced numerous “bug bites” on his extremities. He recalls some of the bites on his legs as
December being hard and painful for several weeks after his return. On examination, he is moderately ill­appearing with normal vital signs other
than a temperature of 38.9°C. He has 10 enlarged tender posterior cervical lymph nodes with the largest measuring 2 cm. He has no
rash, but there are numerous healing insect bites on his extremities. The rest of his examination is within normal limits.

You are concerned that he has an infection acquired in east Africa.

Of the following, the MOST likely etiology of this man’s illness is
Claim Credit
A. Francisella tularensis
Evaluation
B. Rickettsia africae
My Learning Plan
C. Rickettsia slovaca
D. Trypanosoma b. gambiense

Submit   Reset  

Page 45 of 115

Contact Us
Links may open in separate window
Copyright © 2016   American Academy of Pediatrics. All rights reserved. Pop­up Blocker may need to be disabled

http://2016.prepid.courses.aap.org/script/may?q­page=2&req=201612122048431630 1/1
12/13/2016 May

Welcome  mohammed alsaiary [ Logout ]

Home PREP Pearls FAQs My Bookmarks CME Information

Overview Home  > May

Editorial Board
May   Enter Keyword Search
January
Question View:   All (8) Jump to Question
February
Print this Page Add to my Bookmarks Page 45 of 115
March

April Assessment History

May ASSESSMENT PROGRESS:  Total Questions:  8  Questions Answered:  2  Correct Answers:  0

June

July You currently have 6 questions unanswered in this assessment

August

September Question: 2
October A 20­year­old man presents with a 10­day history of constant and severe headaches, intermittent high­grade fever with chills, painful
November lumps on the back of his neck, a 10­lb weight loss, and myalgias. About 3 months ago, he returned after a 3­week long safari trip to
east Africa. While on his trip, he experienced numerous “bug bites” on his extremities. He recalls some of the bites on his legs as
December being hard and painful for several weeks after his return. On examination, he is moderately ill­appearing with normal vital signs other
than a temperature of 38.9°C. He has 10 enlarged tender posterior cervical lymph nodes with the largest measuring 2 cm. He has no
rash, but there are numerous healing insect bites on his extremities. The rest of his examination is within normal limits.

You are concerned that he has an infection acquired in east Africa.

Of the following, the MOST likely etiology of this man’s illness is
Claim Credit
A. Francisella tularensis
Evaluation
B. Rickettsia africae
My Learning Plan
C. Rickettsia slovaca
D. Trypanosoma b. gambiense

Incorrect View Peer Results
Correct Answer: D Average Percent Correct:  62.50%

The man in the vignette presents with an acute constitutional illness within the incubation period for
trypanosomal infections acquired in east Africa. A Giemsa stain of fresh whole blood was examined for
trypanosomes (Figure 1), indicating that he has Trypanosoma b. gambiense.

He has the characteristic “Winterbottom sign” of posterior cervical lymphadenopathy seen in human
African trypanosomiasis (HAT). Serum antibodies to Rickettsia rickettsii would be the test for Rickettsia
africae, the causative agent of African tick­bite fever, which presents with fever, maculopapular rash,
and an eschar. Tickborne tularemia is not common in Uganda; additionally, a posterior cervical location of lymphadenitis
would be associated with a tick bite on the scalp. Rickettsia slovaca infection, the rickettsial organism associated with
tickborne lymphadenopathy (TIBOLA) has been observed only in Europe.

Human African trypanosomiasis, popularly known as “sleeping sickness,” is caused by 2 morphologically identical subspecies
of trypanosomes, which are transmitted to humans by tsetse flies (Glossina species). Rare cases of transmission by blood
transfusion and congenital transmission have been reported. Trypanosomes were first identified in the blood of cattle with
wasting disease (nagana) in Zululand by David Bruce, and the species is named Trypanosoma brucei in recognition. This
infection occurs in 36 sub­Saharan Africa countries, as a Gambian form that is seen in the western and central parts of
tropical Africa (causative agent Trypanosoma b. gambiense), and as a more severe Rhodesian form in East Africa (causative
agent Trypanosoma b. rhodesiense) (Figure 2).

Both are zoonoses with mammalian reservoirs, but Trypanosoma b. gambiense is an anthroponotic infection because
humans are more susceptible to T. b. gambiense and maintain a high level parasitemia that can sustain the fly­human­fly
cycle (Figure 3).

In contrast, humans are an occasional or facultative host for T. b. rhodesiense. Infection is maintained in wild mammals while
acutely ill humans are removed from the cycle as an infective source. In the last 10 years, over 70% of reported cases
occurred in the Democratic Republic of Congo (DRC), which is the only country reporting more than 1,000 new cases
annually. Fewer than 200 new cases occurred in 2013 in Chad, South Sudan, Central African Republic, Angola, Cameroon,
Congo, Côte d'Ivoire, Equatorial Guinea, Gabon, Ghana, Guinea, Kenya, Malawi, Nigeria, Uganda (the only country reporting
both forms), United Republic of Tanzania, Zambia, and Zimbabwe. Ninety­eight percent of HAT cases are the Gambian form.
Table 1 summarizes epidemiological differences between the 2 forms.

http://2016.prepid.courses.aap.org/script/may?req=201612122049242569&status=submit# 1/2
12/13/2016 May
African trypanosomiasis is seen primarily in residents of rural areas. Infection of US travelers is seen at an average rate of 1
case per year, mostly in individuals returning from safari in East Africa. There can be a hard and painful chancre at the site of
the tsetse fly bite, about a week after infection, which can last for weeks. Trypanosomiasis then progresses in 2 stages. In the
early or hemolymphatic stage, after a period of asymptomatic parasitemia, trypanosomes multiply in subcutaneous tissues,
lymph, and blood. Initial symptoms consist of intermittent fever, weight loss, and constitutional symptoms in a relapsing
course driven by antigenic variation. Posterior cervical lymphadenopathy, known as Winterbottom sign, is characteristic. The
second or late stage is central nervous system (CNS) invasion. As early as 3 weeks after onset of the initial illness, the patient
presents with signs and symptoms of meningoencephalitis, such as hallucinations, delusions, diurnal somnolence, nocturnal
insomnia, constant headaches, and personality changes. The time course of progression can be months or years with
Gambian HAT, but only weeks with Rhodesian HAT. In addition, in Rhodesian HAT, the acute phase can be complicated by
hepatitis, myocarditis, thrombocytopenia, anemia, and disseminated intravascular coagulopathy. Untreated disease has a
high fatality rate in both forms. The diagnosis may be difficult in young children, often being identified upon a work­up for
malnutrition, encephalopathy, seizures, or psychomotor retardation.

The conventional diagnostic method for HAT is identification of trypanosomes in blood, cerebrospinal fluid (CSF), or lymph
node aspirate. Blood films may need to be examined for 10 consecutive days to account for the periodicity of the parasitemia.
Giemsa staining of the buffy coat layer and double­centrifugation of CSF may be employed to enhance the sensitivity,
especially for T. b. rhodesiense, because of a lower blood density of organism than with T. b. gambiense infection. A
screening card agglutination test with freeze­dried trypanosomal antigen for detection from a finger­prick drop of blood has
high sensitivity and specificity. Availability of serologic testing is very limited, and only for T. b. gambiense.

Intravenous (IV) or intramuscular pentamidine and IV suramin are drugs of choice for the hemolymphatic stage of Gambian
and Rhodesian HAT, respectively. Intravenous eflornithine and IV melarsoprol are drugs of choice for the CNS stage of
Gambian and Rhodesian HAT, respectively. Duration of treatment ranges from 7 to 21 days (Table 2). Some of these drugs
can only be obtained from the US Centers for Disease Control and Prevention (available at phone number 404­718­4745 ).
Patients with CNS involvement should be monitored with CSF examinations every 6 months for 2 years because of the risk of
relapse.

PREP Pearls
Human African trypanosomiasis occurs in sub­Saharan Africa, in Gambian and Rhodesian forms.

Human African trypanosomiasis occurs in 2 stages: early hemolymphatic, and central nervous system invasion, which is
the “sleeping sickness.”

The Rhodesian form is more fulminant and often fatal if untreated.

American Board of Pediatrics Content Specification(s)
Recognize the epidemiologic and clinical features of African Trypanosomiasis

Plan the appropriate therapy for a patient with African Trypanosomiasis

Suggested Readings
American Academy of Pediatrics. African trypanosomiasis (African sleeping sickness). In: Kimberlin DW, Brady MT, Jackson
MA, Long SS, eds. Red Book: 2015 Report of the Committee on Infectious Diseases. 30th ed. Elk Grove Village, IL: American
Academy of Pediatrics; 2015: 801­803.

Kennedy PGE. Human African trypanosomiasis of the CNS: current issues and challenges. J Clin Invest. 2004;113(4):496­
504. DOI: http://dx.doi.org/10.1172/JCI200421052

Lindner AK, Priotto G. The unknown risk of vertical transmission in sleeping sickness­­a literature review. PLoS Negl Trop Dis.
2010;4(12):e783. DOI: http://dx.doi.org/10.1371/journal.pntd.0000783

Ross LA, Garcia LS, Wittner M, Tanowitz HB. Trypanosomiasis. In: Cherry J, Demmler­Harrison GJ, Kaplan SK, Steinbach
WJ, Hotez P, eds. Feigin and Cherry's Textbook of Pediatric Infectious Diseases. 7th ed. Philadelphia, PA: Saunders Elsevier;
2013: 2959­2974.

Stamatoiu A. Membrane proteins as mechanisms of immune system evasion in Trypanosoma brucei. Microbe wiki, Kenyon
College website
https://microbewiki.kenyon.edu/index.php/Membrane_proteins_as_mechanisms_of_immune_system_evasion_in_Trypanosoma_brucei

World Health Organization. Trypanosomiasis, human African (sleeping sickness). World Health Organization website.
http://www.who.int/mediacentre/factsheets/fs259/en/

Comment On This Question

Page 45 of 115

Contact Us
Links may open in separate window
Copyright © 2016   American Academy of Pediatrics. All rights reserved. Pop­up Blocker may need to be disabled

http://2016.prepid.courses.aap.org/script/may?req=201612122049242569&status=submit# 2/2
12/13/2016 May

Welcome  mohammed alsaiary [ Logout ]

Home PREP Pearls FAQs My Bookmarks CME Information

Overview Home  > May

Editorial Board
May   Enter Keyword Search
January
Question View:   All (8) Jump to Question
February
Print this Page Add to my Bookmarks Page 46 of 115
March

April Assessment History

May ASSESSMENT PROGRESS:  Total Questions:  8  Questions Answered:  2  Correct Answers:  0

June

July You currently have 6 questions unanswered in this assessment

August

September Question: 3
October You are seeing an 18­year­old young woman with acute myeloid leukemia who is now 30 days post­stem cell transplant. Over the
November last 12 hours, she has developed fevers to 38.5˚C, cough, shortness of breath, and increased work of breathing. Her O2 saturation
on room air by pulse oximetry is 84%. Physical examination is significant for a very tired­appearing young woman in moderate
December
respiratory distress. She has a respiratory rate of 28 breaths/min, nasal flaring, tracheal tugging, subcostal retractions with use of
abdominal accessory muscles, and diffuse crackles in all lung fields. Chest radiograph is significant for bilateral diffuse interstitial
infiltrates. The patient has a history of anaphylaxis with trimethoprim­sulfamethoxazole and has been receiving pentamidine
prophylaxis.

In addition to corticosteroids, the MOST appropriate treatment for severe Pneumocystis jirovecii pneumonia in this patient is
Claim Credit
A. aerosolized pentamidine
Evaluation B. amphotericin B
My Learning Plan C. clindamycin with primaquine
D. trimethoprim with dapsone

Submit   Reset  

Page 46 of 115

Contact Us
Links may open in separate window
Copyright © 2016   American Academy of Pediatrics. All rights reserved. Pop­up Blocker may need to be disabled

http://2016.prepid.courses.aap.org/script/may?q­page=3&req=201612122049247256 1/1
12/13/2016 May

Welcome  mohammed alsaiary [ Logout ]

Home PREP Pearls FAQs My Bookmarks CME Information

Overview Home  > May

Editorial Board
May   Enter Keyword Search
January
Question View:   All (8) Jump to Question
February
Print this Page Add to my Bookmarks Page 46 of 115
March

April Assessment History

May ASSESSMENT PROGRESS:  Total Questions:  8  Questions Answered:  3  Correct Answers:  0

June

July You currently have 5 questions unanswered in this assessment

August

September Question: 3
October You are seeing an 18­year­old young woman with acute myeloid leukemia who is now 30 days post­stem cell transplant. Over the
November last 12 hours, she has developed fevers to 38.5˚C, cough, shortness of breath, and increased work of breathing. Her O2 saturation
on room air by pulse oximetry is 84%. Physical examination is significant for a very tired­appearing young woman in moderate
December
respiratory distress. She has a respiratory rate of 28 breaths/min, nasal flaring, tracheal tugging, subcostal retractions with use of
abdominal accessory muscles, and diffuse crackles in all lung fields. Chest radiograph is significant for bilateral diffuse interstitial
infiltrates. The patient has a history of anaphylaxis with trimethoprim­sulfamethoxazole and has been receiving pentamidine
prophylaxis.

In addition to corticosteroids, the MOST appropriate treatment for severe Pneumocystis jirovecii pneumonia in this patient is
Claim Credit
A. aerosolized pentamidine
Evaluation B. amphotericin B
My Learning Plan C. clindamycin with primaquine
D. trimethoprim with dapsone

Incorrect View Peer Results
Correct Answer: C Average Percent Correct:  65.00%

The patient in the vignette has Pneumocystis pneumonia. Pneumocystis encompasses a genus of
opportunistic fungal pathogens that cause potentially fatal pneumonia in immunocompromised hosts.
Pneumocystis jirovecii is a species that is capable of only infecting humans and is a major cause of
infection.

Historically, the mainstay of treatment for Pneumocystis jirovecii pneumonia (PJP) has been
trimethoprim­sulfamethoxazole (TMP­SMX), which is considered to be the first­line therapy for patients
with mild, moderate, and severe disease. The standard dose for both the pediatric (older than 2 months of age) and adult
populations is 15 to 20 mg/kg per day of TMP and 75 to 100 mg/kg per day of SMX administered in divided doses. For severe
cases, the intravenous (IV) form is preferred over the oral formulation. However, IV can be switched to oral once clinical
improvement is achieved. Dose adjustments are necessary for patients with renal and liver failure because SMX is
extensively metabolized in the liver and is renally excreted. For patients that have severe allergy to sulfonamides, alternative
treatment regimens include:

1. Combination of clindamycin with primaquine (for mild, moderate, and severe disease)
2. Atovaquone (for mild to moderate disease)
3. TMP with dapsone (for mild to moderate disease)
4. Pentamidine IV (for severe disease)
5. Trimetrexate with leucovorin (for mild to moderate disease)

Aerosolized pentamidine is not recommended for the treatment of PJP because of low efficacy and increased likelihood of
relapse of PJP. Although TMP with dapsone is an option for mild­to­moderate disease, clindamycin plus primaquine is the
combination drug of choice for this patient with severe disease. Amphotericin B does not have effective activity against P
jirovecii.

Another critical aspect of the treatment of Pneumocystis infection is the use of corticosteroids, including prednisone, as
adjunctive therapy. Studies have demonstrated that the use of corticosteroids reduces the risk of respiratory failure and death
in hypoxemic HIV­infected patients with moderate­to­severe pneumonia; however, the data for non­HIV patients are less
clear. Therefore, the recommendation for use of adjunctive corticosteroid therapy in non­HIV patients should be
individualized. For these patients, adjunctive therapy should not be used in those with mild­to­moderate disease. However,

http://2016.prepid.courses.aap.org/script/may?req=201612122050045694&status=submit 1/3
12/13/2016 May
adjunctive corticosteroids should be seriously considered in non­HIV­infected patients with severe PJP, especially if
hypoxemia is present.

The route of Pneumocystis infection is not completely understood. The most commonly accepted theory is that infectious
organisms are transmitted as aerosolized particles from host to host. Once inhaled, Pneumocystis has a tropism for the lung
and rarely disseminates to other organs. When the infective form reaches the alveoli, it binds tightly to alveolar epithelial cells
and extracellular matrix proteins. This triggers several signaling pathways which mediate proliferation.  

The persons at highest risk for P jirovecii infection include: patients infected with HIV; those with autoimmune diseases on
immunosuppressive medications (eg, connective tissue and rheumatologic disorders); persons post­hematopoietic stem cell
and solid organ transplantation; those with hematological and solid malignancies; persons with inflammatory bowel disease;
any person with impaired cellular immunity such as those receiving moderate doses of oral steroids (≥ 20 mg/day) for greater
than 4 weeks; or those receiving other immunosuppressive medications. Table 1 shows the immunosuppressive agents that
have been associated with the development of PJP.

The clinical presentation of patients affected with PJP differs among HIV and non­HIV immunosuppressed patients. HIV
patients tend to present with a subacute onset of progressive dyspnea, nonproductive or minimally productive cough, low­
grade fever, and malaise. Up to 7% of these patients can be asymptomatic. In contrast, patients that are
immunocompromised but HIV­negative usually present acutely, with substantial dyspnea, fever, dry cough, chills, and some
may present in fulminant respiratory failure requiring mechanical ventilation. Respiratory failure in this group of patients has
an overall mortality rate of 40%. Death rates range from 10% in renal transplant patients to 50% in cancer patients,
suggesting that mortality is related to the degree of underlying immunosuppression. HIV patients usually have significantly
more organisms in the bronchoalveolar lavage that non­HIV patients, but this does not seem to correlate with worse
oxygenation. Also, patients with HIV­associated PJP tend to have higher arterial oxygen tension and a lower alveolar­arterial
oxygen gradient compared to non­HIV patients. Neutrophil and inflammatory cell numbers, rather than organism burden,
appear to correlate with the degree of oxygenation during PJP, suggesting that patients that mount a more robust
inflammatory response are at higher risk of developing respiratory failure, compared to those who exhibit lower levels of lung
inflammation.

The lung examination varies depending on the degree of illness, ranging from an essentially normal examination to diffuse
crackles on auscultation. Extrapulmonary manifestations may occur, but are not common, and may include: retinitis,
thyroiditis, bone lesions, and pneumocystosis of brain, liver, spleen, and kidney. These findings occur more frequently in
patients who have been on prophylaxis with aerosolized pentamidine or who are extremely immunocompromised from
advanced AIDS.

Radiological findings of PJP are generally not specific and may range from a normal chest radiograph to different degrees of
bilateral and usually symmetrical pulmonary infiltrates. Less common reported patterns that may be seen include: lobar
infiltrates, pulmonary nodules, pneumatoceles and other cystic changes, and pneumothorax. In cases where clinical
symptomatology occurs in the absence of significant chest radiograph findings, more sensitive radiological studies such as
high­resolution chest computed tomography (CT) may be needed. The most typical findings on chest CT are bilateral ground­
glass opacities. Other findings that may be seen include reticular, granular, and cystic lesions. The differential diagnosis of a
patient presenting with these clinical and radiographic findings includes: acute respiratory distress syndrome, cytomegalovirus
infection, lymphocytic interstitial pneumonia, mycoplasma infection, pulmonary embolism, legionellosis, tuberculosis, and
mycobacterium avium complex infection. The combination of clinical history and diagnostic testing will help to distinguish
between the different entities.

The single most important diagnostic tool for Pneumocystis infection is a high clinical suspicion. In the appropriate clinical
setting, an immunosuppressed patient with new onset dyspnea or new symptoms of pneumonia, with or without radiological
findings, should prompt further evaluation, especially if they are not receiving chemoprophylaxis. The gold standard for
diagnosis is microscopic visualization of the organism. Traditionally, different stains have been used to identify either the
trophic form (Gram­Weigert, Wright­Giemsa, or modified Papanicolaou stains) or the cyst forms (calcofluor white, cresyl echt
violet, Gomori methenamine silver, or toluidine blue). However, the most common technique used currently in the majority of
laboratories is fluorescein­conjugated monoclonal antibody staining. Sensitivity of these assays depends on several factors
such as the type and quality of the sample, the number of organisms present, and the experience of the laboratory staff with
the particular assay. Molecular methods for identification (eg, real­time polymerase chain reaction) are now available and
have increased sensitivity and specificity. Measurement of serum or plasma (1­3) β­D­glucan (BG), a component of many
fungal cell walls, is increasingly being used as an adjunctive diagnostic tool for the diagnosis of PJP. The levels of BGs are
higher among both HIV­infected and HIV­uninfected patients with PJP, when compared with symptomatic patients with
confirmed alternative diagnoses, including aspergillosis and histoplasmosis. Table 2 shows the advantages and
disadvantages of these different tests.

PREP Pearls
The gold standard for diagnosis of Pneumocystis jirovecii is microscopic visualization of the organism.

The persons at highest risk for Pneumocystis jirovecii infection include:

patients infected with HIV;

those with autoimmune diseases on immunosuppressive medications (eg. connective tissue and rheumatologic
disorders);

persons post­hematopoietic stem cell and solid organ transplantation;

those with hematological and solid malignancies;

individuals with inflammatory bowel disease;

any person with impaired cellular immunity such as those receiving moderate doses of oral steroids (≥ 20 mg/day) for
greater than 4 weeks;

or those receiving other immunosuppressive medications.

The mainstay of treatment for Pneumocystis jirovecii pneumonia is trimethoprim­sulfamethoxazole, which is considered to
be the first­line of therapy for patients with mild, moderate, and severe disease. Adjunctive corticosteroid therapy is used in
conjunction with antimicrobial therapy.

http://2016.prepid.courses.aap.org/script/may?req=201612122050045694&status=submit 2/3
12/13/2016 May

American Board of Pediatrics Content Specification(s)
Recognize the predisposing conditions and characteristic clinical and chest radiographic manifestations of Pneumocystis
jiroveci infection

Plan the diagnostic evaluation for a patient with suspected Pneumocystis jiroveci pneumonia infection and differentiate it
from other potential causes of lung disease (eg, interstitial pneumonia in infants, lymphocytic interstitial pneumonitis in a
patient with AIDS

Plan the treatment for Pneumocystis jiroveci pneumonia, including the use of corticosteroids

Suggested Readings
Carmona EM, Limper AH. Update on the diagnosis and treatment of Pneumocystis pneumonia. Ther Adv Respir Dis.
2011;5(1):41­59. DOI: http://dx.doi.org/10.1177/1753465810380102

Huang L, Cattamanchi A, Davis JL, et al. HIV­associated Pneumocystis pneumonia. Proc Am Thorac Soc. 2011;8(3):294­300.
DOI: http://dx.doi.org/10.1513/pats.201009­062WR

Neumann S, Krause SW, Maschmeyer G, et al. Primary prophylaxis of bacterial infections and Pneumocystis jiroveccii
pneumonia in patients with hematological malignancies and solid tumors. Ann Hematol. 2013;92(4):433­442. DOI:
http://dx.doi.org/10.1007/s00277­013­1698­0

Reid AB, Sharon C­AC, Worth LJ. Pneumocystis jiroveccii in non­HIV patients: new risks and diagnostic tools. Curr Opin
Infect Dis. 2011;24(6):534­544. DOI: http://dx.doi.org/10.1097/QCO.0b013e32834cac17

Roux A, Gonzalez F, Roux M, et al. Update on pulmonary Pneumocystis jiroveccii infection in non­HIV patients. Med Mal
Infect. 2014;44(5):185­198. DOI: http://dx.doi.org/10.1016/j.medmal.2014.01.007

Comment On This Question

Page 46 of 115

Contact Us
Links may open in separate window
Copyright © 2016   American Academy of Pediatrics. All rights reserved. Pop­up Blocker may need to be disabled

http://2016.prepid.courses.aap.org/script/may?req=201612122050045694&status=submit 3/3
12/13/2016 May

Welcome  mohammed alsaiary [ Logout ]

Home PREP Pearls FAQs My Bookmarks CME Information

Overview Home  > May

Editorial Board
May   Enter Keyword Search
January
Question View:   All (8) Jump to Question
February
Print this Page Add to my Bookmarks Page 47 of 115
March

April Assessment History

May ASSESSMENT PROGRESS:  Total Questions:  8  Questions Answered:  3  Correct Answers:  0

June

July You currently have 5 questions unanswered in this assessment

August

September Question: 4
October You are called by a general pediatrician for advice regarding the most appropriate immunizations for a 12­month­old infant who was
November treated for Kawasaki disease at 9 months of age. The infant was treated with 1 dose of intravenous immune globulin (2 g/kg) and
high­dose aspirin; he has done well. He now presents to his pediatrician for his 12­month well visit. The infant has received all age­
December appropriate vaccines to date and is due to receive the following vaccines at this visit:

First dose of hepatitis A
Fourth dose of Haemophilus influenzae type b (Hib)
Third dose of pneumococcal conjugate (PCV13)
First dose of measles, mumps, rubella (MMR)
First dose of varicella
Claim Credit

Evaluation Your advice at this time is that, of the following, it is MOST appropriate to administer

My Learning Plan A. all scheduled vaccines except MMR
B. all scheduled vaccines except MMR and varicella
C. all scheduled vaccines except MMR, varicella, and hepatitis A
D. no scheduled vaccines

Submit   Reset  

Page 47 of 115

Contact Us
Links may open in separate window
Copyright © 2016   American Academy of Pediatrics. All rights reserved. Pop­up Blocker may need to be disabled

http://2016.prepid.courses.aap.org/script/may?q­page=4&req=201612122050052726 1/1
12/13/2016 May

Welcome  mohammed alsaiary [ Logout ]

Home PREP Pearls FAQs My Bookmarks CME Information

Overview Home  > May

Editorial Board
May   Enter Keyword Search
January
Question View:   All (8) Jump to Question
February
Print this Page Add to my Bookmarks Page 47 of 115
March

April Assessment History

May ASSESSMENT PROGRESS:  Total Questions:  8  Questions Answered:  4  Correct Answers:  1

June

July You currently have 4 questions unanswered in this assessment

August

September Question: 4
October You are called by a general pediatrician for advice regarding the most appropriate immunizations for a 12­month­old infant who was
November treated for Kawasaki disease at 9 months of age. The infant was treated with 1 dose of intravenous immune globulin (2 g/kg) and
high­dose aspirin; he has done well. He now presents to his pediatrician for his 12­month well visit. The infant has received all age­
December appropriate vaccines to date and is due to receive the following vaccines at this visit:

First dose of hepatitis A
Fourth dose of Haemophilus influenzae type b (Hib)
Third dose of pneumococcal conjugate (PCV13)
First dose of measles, mumps, rubella (MMR)
First dose of varicella
Claim Credit

Evaluation Your advice at this time is that, of the following, it is MOST appropriate to administer

My Learning Plan A. all scheduled vaccines except MMR
B. all scheduled vaccines except MMR and varicella
C. all scheduled vaccines except MMR, varicella, and hepatitis A
D. no scheduled vaccines

Correct View Peer Results

Average Percent Correct:  88.75%

The administration of immune globulin intravenous (IGIV) diminishes the immune response to live virus
vaccines such as measles and rubella, and possibly varicella. Thus, administration of these vaccines
following receipt of IGIV products should be delayed. Although the effect of IGIV on the response to
varicella vaccine is not known, administration of this vaccine should be delayed, as is the case for
measles vaccine, given the possible decreased response and until additional data are available. The
suggested intervals between IGIV administration and these live virus vaccines depends on the type
and dose of IGIV administered, as summarized in the Table. For the infant in the vignette, who
received 2 g/kg of IGIV, measles, mumps, rubella (MMR) and varicella vaccines should be deferred for 11 months after receipt
of IGIV. Administration of IGIV does not significantly interfere with the immune response to inactivated vaccines or toxoids.
Thus, the infant in the vignette should receive all scheduled vaccines except the live vaccines, MMR and varicella.

Immune globulin intravenous does not appear to interfere with the response to other live virus vaccines, such as yellow fever,
oral poliovirus, oral rotavirus, and live­attenuated influenza vaccines.

Immune globulin intravenous can interfere with the immune response when given within 14 days after administration of
measles or varicella vaccines. In these situations, children should be revaccinated with these vaccines after the interval
specified in the Table, unless serologic testing after the appropriate interval after IGIV administration documents
seroconversion.

In children who have received immunoglobulin products, measles vaccine may be given at an interval shorter than that
recommended if exposure to measles is imminent. In this circumstance, the child should be reimmunized at the appropriate
interval according to the Table (and at least 28 days after the earlier immunization), unless serologic testing indicates
immunity.

* Required *
Take Survey  

http://2016.prepid.courses.aap.org/script/may?req=201612122051544134&status=submit 1/2
12/13/2016 May

PREP Pearls
Immune globulin diminishes the immune response to measles, rubella, and possibly varicella vaccines.

Administration of measles, mumps, rubella (MMR) and varicella vaccines should be delayed after receipt of immune
globulin; the duration of delay depends on the type of immune globulin and dose given.

American Board of Pediatrics Content Specification(s)
Know which vaccines are contraindicated in a child who has recently received immune globulin and for how long

Suggested Readings
American Academy of Pediatrics. Active immunization of people who recently received immune globulin and other blood
products. In: Kimberlin DW, Brady MT, Jackson MA, Long SS, eds. Red Book: 2015 Report of the Committee on Infectious
Diseases. 30th ed. Elk Grove Village, IL: American Academy of Pediatrics; 2015: 38­40.

American Academy of Pediatrics. Measles. In: Kimberlin DW, Brady MT, Jackson MA, Long SS, eds. Red Book: 2015 Report
of the Committee on Infectious Diseases. 30th ed. Elk Grove Village, IL: American Academy of Pediatrics; 2015: 535­547.

US Centers for Disease Control and Prevention. General recommendations on immunization: recommendations of the
Advisory Committee on Immunization Practices (ACIP). MMWR Recomm Rep. 2011;60(2):1­64.
http://www.cdc.gov/mmwr/preview/mmwrhtml/rr6002a1.htm

Comment On This Question

Page 47 of 115

Contact Us
Links may open in separate window
Copyright © 2016   American Academy of Pediatrics. All rights reserved. Pop­up Blocker may need to be disabled

http://2016.prepid.courses.aap.org/script/may?req=201612122051544134&status=submit 2/2
Table. Suggested Intervals Between Immune Globulin Administration and Measles Immunization (MMR
or MMRV)

MMR indicates measles-mumps-rubella; MMRV, measles-mumps-rubella-varicella; RSV, respiratory


syncytial virus; IM, intramuscular; TIG, Tetanus Immune Globulin; IG, Immune Globulin; HBIG,
Hepatitis B Immune Globulin; RIG, Rabies Immune Globulin; VariZIG, Varicella-Zoster Immune
Globulin; IV, intravenous; RBCs, Red Blood Cells; IGIV, Immune Globulin Intravenous; ITP, immune
(formerly termed “idiopathic”) thrombocytopenic purpura.

a These intervals should provide sufficient time for decreases in passive antibodies in all children to
allow for an adequate response to measles vaccine. Physicians should not assume that children are
protected fully against measles during these intervals. Additional doses of IG or measles vaccine may
be indicated after exposure to measles.

b RSV monoclonal antibody (palivizumab) does not interfere with the immune response to vaccines.

Reprinted with permission from the American Academy of Pediatrics. Active Immunization of People Who
Recently Received Immune Globulin and Other Blood Products. In: Pickering LK, Baker CJ, Kimberlin DW, Long SS,
eds. Red Book: 2015 Report of the Committee on Infectious Diseases. 30th ed. Elk Grove Village, IL: American
Academy of Pediatrics; 2015: 38-40.
12/13/2016 May

Welcome  mohammed alsaiary [ Logout ]

Home PREP Pearls FAQs My Bookmarks CME Information

Overview Home  > May

Editorial Board
May   Enter Keyword Search
January
Question View:   All (8) Jump to Question
February
Print this Page Add to my Bookmarks Page 48 of 115
March

April Assessment History

May ASSESSMENT PROGRESS:  Total Questions:  8  Questions Answered:  4  Correct Answers:  1

June

July You currently have 4 questions unanswered in this assessment

August

September Question: 5
October A couple becomes ill with sudden onset of fever, severe constitutional symptoms, pneumonia, septicemia, and painful inguinal lymph
November nodes 2 days into a summer visit to New York City. They report no rodent or other zoonotic exposure while in the city, but had animal
contact and frequent insect bites while on their ranch in their home state of New Mexico.
December
Of the following, the MOST likely mode of acquisition of this infection is

A. contact with an animal carcass
B. eating infected meat
C. inhalation of droplets from a farm animal
Claim Credit
D. inoculation from flea bites
Evaluation

My Learning Plan
Submit   Reset  

Page 48 of 115

Contact Us
Links may open in separate window
Copyright © 2016   American Academy of Pediatrics. All rights reserved. Pop­up Blocker may need to be disabled

http://2016.prepid.courses.aap.org/script/may?q­page=5&req=201612122054462888 1/1
12/13/2016 May

Welcome  mohammed alsaiary [ Logout ]

Home PREP Pearls FAQs My Bookmarks CME Information

Overview Home  > May

Editorial Board
May   Enter Keyword Search
January
Question View:   All (8) Jump to Question
February
Print this Page Add to my Bookmarks Page 48 of 115
March

April Assessment History

May ASSESSMENT PROGRESS:  Total Questions:  8  Questions Answered:  5  Correct Answers:  1

June

July You currently have 3 questions unanswered in this assessment

August

September Question: 5
October A couple becomes ill with sudden onset of fever, severe constitutional symptoms, pneumonia, septicemia, and painful inguinal lymph
November nodes 2 days into a summer visit to New York City. They report no rodent or other zoonotic exposure while in the city, but had animal
contact and frequent insect bites while on their ranch in their home state of New Mexico.
December
Of the following, the MOST likely mode of acquisition of this infection is

A. contact with an animal carcass
B. eating infected meat
C. inhalation of droplets from a farm animal
Claim Credit
D. inoculation from flea bites
Evaluation

My Learning Plan
Incorrect View Peer Results

Correct Answer: D Average Percent Correct:  76.25%

The zoonotic Gram­negative bacterium Yersinia pestis is the cause of bubonic, pneumonic, septicemic,
and meningeal plague. In the United States, just over a thousand confirmed or probable human plague
cases have occurred since it was first introduced into the country in 1900 on rat­infested ships. Urban
epidemics occurred in port cities until 1925, but since then, most cases in the United States have been
sporadic or in small clusters. During recent decades, an average of 7 human plague cases have been
reported each year (range: 1–17), in people of all ages, with over 80% in the bubonic form. Human
infection most often occurs following the bite of an infected flea, and less often, contact transmission
can occur during handling of animals or eating infected meat, or from inhalation of droplets from humans or animals with
pneumonic plague. Based upon knowledge of the modern epidemiology of plague in the United States, the couple in the
vignette who had bubonic and septicemic plague were assumed to have acquired the infection from flea bites, and not from
the other modes of transmission listed. Fleas from a woodrat found on their property tested positive for Y pestis.

Y pestis is primarily harbored by rodents (especially domestic or wild rats) and fleas, which serve as long­term reservoirs in
an enzootic cycle (Figure 1).

Y pestis also has a reservoir in more than 200 other mammalian species, notably ground squirrels, prairie dogs, field mice,
bobcats, cats, rabbits, chipmunks, and camels. If a wider variety of rodents get infected in an outbreak among animals, called
a sylvatic plague epizootic, there is increased risk to humans. Epizootics tend to occur in the summer, and most rural cases
occur between April and September. Pet dogs and cats can bring fleas into the home. Currently, plague occurs in rural areas
of 2 regions of the Western United States: northern New Mexico, northern Arizona, and southern Colorado; and California,
southern Oregon, and far western Nevada (Figure 2). Worldwide, most human cases occur in rural areas in developing
nations, with 90% of reported cases occurring in Africa (Figure 3).

Yersinia enterocolitica, the cause of enterocolitis and septicemic infection, also has a zoonotic reservoir, principally in swine.
In the United States, the US Centers for Disease Control and Prevention DC Foodborne Diseases Active Surveillance
Network (FoodNet) database indicates there is approximately one culture­confirmed Y enterocolitica infection per 100,000
persons annually, with incidence peaking in the winter. Most infections reported to FoodNet are among young black children,
albeit now with an annually decreasing trend attributed to the educational efforts of appropriate handling of raw pork products.
Conversely, a study in 9 non­FoodNet states identified an increase in yersiniosis (which included Y enterocolitica and Yersinia
pseudotuberculosis) in non­black populations, with an April to June seasonality among white individuals. Up to a quarter of
children with Y enterocolitica infection are hospitalized. Contamination of food and fomites such as bottles or pacifiers during
the preparation of pork intestines (chitterlings) is chiefly implicated. Less common modes of transmission are drinking
contaminated unpasteurized milk or water, eating contaminated tofu, contact with infected animals, transfusion with packed
red blood cells, and rarely person­to­person. Y enterocolitica is also a travel­related cause of diarrhea in some temperate
countries, notably in Scandinavia, Japan, and Canada. The illness has a short incubation period of under 2 weeks. The

http://2016.prepid.courses.aap.org/script/may?req=201612122056566172&status=submit 1/2
12/13/2016 May
organism may be excreted asymptomatically for up to 3 months in untreated persons. Y enterocolitica strains are typed by
biotype (1A, 1B, 2, 3, 4, or 5) based upon chemical and fermentation reactions, as well as by serotype of somatic O antigen.
Bioserotypes most often associated with human illness are 1B/O:8, 2/O:5,27, 2/O:9, 3/O:3, and 4/O:3; the latter is currently
the predominant bioserotype in the United States.

PREP Pearls
Plague is currently restricted to a few areas in the western United States.

Human infection occurs sporadically in rural areas with diverse rodent populations in the warmer months, and is mostly
bubonic.

The reservoir of Yersinia enterocolitica is principally swine, and the mode of transmission is indirect fecal­oral.

American Board of Pediatrics Content Specification(s)
Know the epidemiologic features and mode of transmission of Yersinia enterocolitica

Recognize the epidemiologic features, including vectors and reservoirs, of Yersinia pestis

Suggested Readings
Chakraborty A, Komatsu K, Roberts M, et al. The descriptive epidemiology of yersiniosis: a multistate study, 2005­2011.
Public Health Rep. 2015;130(3):269­277.

Hartocollis A. Plague survivors are back in hospital, to say thanks. The New York Times. February 11, 2004.
http://www.nytimes.com/2004/02/11/nyregion/plague­survivors­are­back­in­hospital­to­say­thanks.html. Accessed August 10,
2015

Long C, Jones TF, Vugia DJ, et al. Yersinia pseudotuberculosis and Y. enterocolitica infections, FoodNet, 1996–2007. Emerg
Infect Dis. 2010;16(3):566­567. DOI: http://dx.doi.org/10.3201/eid1603.091106

Longenberger AH, Gronostaj MP, Yee GY, et al. Yersinia enterocolitica infections associated with improperly pasteurized milk
products: southwest Pennsylvania, March­August, 2011. Epidemiol Infect. 2014;142(8):1640­1650. DOI:
http://dx.doi.org/10.1017/S0950268813002616

Overturf GD. Plague (Yersinia pestis). In: Feigin and Cherry's Textbook Of Pediatric Infectious Diseases. 7th ed. Philadelphia,
PA: Saunders Elsevier; 2014: 1509­1514.

US Centers for Disease Control and Prevention. Plague: maps and statistics. US Centers for Disease Control and Prevention
website. http://www.cdc.gov/plague/maps/index.html. Updated August 24, 2015

Comment On This Question

Page 48 of 115

Contact Us
Links may open in separate window
Copyright © 2016   American Academy of Pediatrics. All rights reserved. Pop­up Blocker may need to be disabled

http://2016.prepid.courses.aap.org/script/may?req=201612122056566172&status=submit 2/2
12/13/2016 May

Welcome  mohammed alsaiary [ Logout ]

Home PREP Pearls FAQs My Bookmarks CME Information

Overview Home  > May

Editorial Board
May   Enter Keyword Search
January
Question View:   All (8) Jump to Question
February
Print this Page Add to my Bookmarks Page 49 of 115
March

April Assessment History

May ASSESSMENT PROGRESS:  Total Questions:  8  Questions Answered:  5  Correct Answers:  1

June

July You currently have 3 questions unanswered in this assessment

August

September Question: 6
October You are speaking to a group of medical students about Mycoplasma pneumoniae, the epidemiology of disease, and the different
November types of infections that it may cause.

December Of the following, the MOST accurate statement regarding M pneumoniae is

A. causes disease in humans and animals
B. cold agglutinin autoantibodies are immunoglobulin G antibodies
C. incubation period is 2 to 3 weeks
D. most common clinical syndrome is atypical pneumonia
Claim Credit

Evaluation
Submit   Reset  
My Learning Plan

Page 49 of 115

Contact Us
Links may open in separate window
Copyright © 2016   American Academy of Pediatrics. All rights reserved. Pop­up Blocker may need to be disabled

http://2016.prepid.courses.aap.org/script/may?q­page=6&req=201612122056570859 1/1
12/13/2016 May

Welcome  mohammed alsaiary [ Logout ]

Home PREP Pearls FAQs My Bookmarks CME Information

Overview Home  > May

Editorial Board
May   Enter Keyword Search
January
Question View:   All (8) Jump to Question
February
Print this Page Add to my Bookmarks Page 49 of 115
March

April Assessment History

May ASSESSMENT PROGRESS:  Total Questions:  8  Questions Answered:  6  Correct Answers:  1

June

July You currently have 2 questions unanswered in this assessment

August

September Question: 6
October You are speaking to a group of medical students about Mycoplasma pneumoniae, the epidemiology of disease, and the different
November types of infections that it may cause.

December Of the following, the MOST accurate statement regarding M pneumoniae is

A. causes disease in humans and animals
B. cold agglutinin autoantibodies are immunoglobulin G antibodies
C. incubation period is 2 to 3 weeks
D. most common clinical syndrome is atypical pneumonia
Claim Credit

Evaluation
Incorrect View Peer Results
My Learning Plan
Correct Answer: C Average Percent Correct:  33.75%

Transmission of Mycoplasma occurs person­to­person by respiratory droplets through coughing during
close contact with a symptomatic person. The incubation period is usually 2 to 3 weeks with a range of
1 to 4 weeks. Asymptomatic carriage after infection may occur for weeks to months. The disease can
relapse and the organism can be transmitted even after treatment of patients with effective antibiotics.
Because of prolonged carriage, spread of the disease may be extensive and outbreaks have been
reported in hospitals, military bases, colleges, boarding schools, and summer camps.

Mycoplasma species represent the smallest self­replicating organisms capable of cell­free existence, both in cellular
dimensions and genome size. There are more than 200 Mycoplasma species that have been identified in humans, animals,
plants, and arthropods, with the best known being Mycoplasma pneumoniae. M pneumoniae causes disease only in humans,
and infections can involve both the upper and lower respiratory tract. Infections occur both endemically and epidemically
worldwide in persons of all ages, in any season, and in all geographic settings, with epidemics occurring every 4 to 7 years.
Even though infection has been described at all ages, it is primarily a disease of childhood and adolescence, with the peak
incidence of infection between 5 and 15 years of age. M pneumoniae infections are mostly mild and self­limited, with the most
common clinical syndromes being pharyngitis, sinus congestion, and acute bronchitis. However, disease may be more
severe. M pneumoniae is responsible for up to 40% of community­acquired pneumonias in children older than 5 years of age,
but is an infrequent cause of infection in preschool­aged children younger than 5 years of age. In addition to respiratory tract
infections, M pneumoniae can also cause extrapulmonary manifestations that may involve every organ system; this may be
caused by the direct effects of M pneumoniae after dissemination of the bacteria throughout the body or from autoimmune
mechanisms. Table 1 shows the different manifestations of M pneumoniae infection.

Serology has historically been the most common laboratory method for diagnosis of M pneumoniae infections. M pneumoniae
has both lipid and protein antigens that elicit antibody responses which can be detected after about 1 week of illness, peaking
at 3 to 6 weeks, followed by a gradual decline. A 4­fold increase in antibody titer in acute and convalescent sera is considered
the “gold standard” for diagnosis of acute M pneumoniae respiratory infection. Serology is a useful epidemiologic tool in
circumstances where the likelihood of Mycoplasma disease is high, but is less suited for assessment of individual patients.
The main disadvantage is the need for both acute and convalescent paired sera collected 2 to 3 weeks apart that are tested
simultaneously for immunoglobulin M (IgM) and immunoglobulin G to confirm seroconversion. This is an issue especially in
adults older than 40 years of age who may not mount an IgM response, presumably because of reinfection. A single
measurement of IgM may detect an acute infection if the test is performed after at least 7 days following symptom onset, but
the result may be negative if the test is performed sooner than this. Immunoglobulin M antibodies may persist for several
weeks to months. Therefore, basing the diagnosis of an acute Mycoplasma respiratory infection on a single assay for IgM
alone may be misleading. Antibody production may be delayed in some infections, and may be absent if the patient is
immunosuppressed. False­negative tests can also occur if the serum is collected after antibiotics are administered. The best
commercial serological test for individual patient diagnosis depends on the age of the patient, timing of serum collection,
whether paired sera are obtained, availability of appropriate equipment, and experience of the laboratory personnel.

http://2016.prepid.courses.aap.org/script/may?req=201612122057386016&status=submit 1/2
12/13/2016 May
Diagnosis of M pneumoniae disease is best achieved by acute and convalescent serology and polymerase chain reaction.
Cold agglutinin autoantibodies based on IgM testing are nonspecific and should not be used. Table 2 shows the diagnostic
tests available for M pneumoniae.

* Required *
Take Survey  

PREP Pearls
Mycoplasma pneumoniae causes disease only in humans and is primarily a disease of childhood and adolescence, with
the peak incidence of infection between 5 and 15 years of age.

Infections can involve both the upper and lower respiratory tract, as well as having extrapulmonary manifestations that may
involve every organ system.

Incubation period is 2 to 3 weeks and asymptomatic carriage after infection may last for weeks to months.

American Board of Pediatrics Content Specification(s)
Know the epidemiology of Mycoplasma pneumoniae infection, including mode of transmission, ages of infection and
disease, and incubation period

Know the available diagnostic tests for Mycoplasma pneumoniae infection and their relative advantages and disadvantages

Suggested Readings
Atkinson TP, Balish MF, Waites KB. Epidemiology, clinical manifestations, pathogenesis and laboratory detection of
Mycoplasma pneumoniae infections. FEMS Microbiol Rev. 2008;32(6):956­973. DOI: http://dx.doi.org/10.1111/j.1574­
6976.2008.00129.x

Biondi E, McCulloh R, Alverson B, Klein A, Dixon A, Ralston S. Treatment of Mycoplasma pneumonia: a systemic review.
Pediatrics. 2014;133(6):1081­1090. DOI: http://dx.doi.org/10.1542/peds.2013­3729

Chang HY, Chang LY, Shao PL, et al. Comparison of real­time polymerase chain reaction and serological tests for the
confirmation of Mycoplasma pneumoniae infection in children with clinical diagnosis of atypical pneumonia. J Microbiol
Immunol Infect. 2014;47(2):137­144. DOI: http://dx.doi.org/10.1016/j.jmii.2013.03.015

Meyer Sauteur PM, van Rossum AM, Vink C. Mycoplasma pneumoniae in children: carriage, pathogenesis, and antibiotic
resistance. Curr Opin Infect Dis. 2014;27(3):220­227. DOI: http://dx.doi.org/10.1097/QCO.0000000000000063

Waites KB, Balish MF, Atkinson TP. New insights into the pathogenesis and detection of Mycoplasma pneumoniae infections.
Future Microbiol. 2008;3(6):635­648. DOI: http://dx.doi.org/10.2217/17460913.3.6.635

Comment On This Question

Page 49 of 115

Contact Us
Links may open in separate window
Copyright © 2016   American Academy of Pediatrics. All rights reserved. Pop­up Blocker may need to be disabled

http://2016.prepid.courses.aap.org/script/may?req=201612122057386016&status=submit 2/2
12/13/2016 May

Welcome  mohammed alsaiary [ Logout ]

Home PREP Pearls FAQs My Bookmarks CME Information

Overview Home  > May

Editorial Board
May   Enter Keyword Search
January
Question View:   All (8) Jump to Question
February
Print this Page Add to my Bookmarks Page 50 of 115
March

April Assessment History

May ASSESSMENT PROGRESS:  Total Questions:  8  Questions Answered:  6  Correct Answers:  1

June

July You currently have 2 questions unanswered in this assessment

August

September Question: 7
October You are speaking to a group of medical students about Epstein­Barr virus­associated infectious mononucleosis and the various
November laboratory tests that may be used to make the diagnosis in a patient.

December Of the following, the MOST accurate statement regarding the heterophile antibody test is that

A. antibodies are detectable for short period of time after onset of Epstein­Barr virus infection
B. children 4 years of age or younger may not develop heterophile antibody to Epstein­Barr virus infection
C. it is based on observed agglutination of swine red blood cells
D. presence of antibody is specific for acute Epstein­Barr virus infection
Claim Credit

Evaluation
Submit   Reset  
My Learning Plan

Page 50 of 115

Contact Us
Links may open in separate window
Copyright © 2016   American Academy of Pediatrics. All rights reserved. Pop­up Blocker may need to be disabled

http://2016.prepid.courses.aap.org/script/may?q­page=7&req=201612122057393048 1/1
12/13/2016 May

Welcome  mohammed alsaiary [ Logout ]

Home PREP Pearls FAQs My Bookmarks CME Information

Overview Home  > May

Editorial Board
May   Enter Keyword Search
January
Question View:   All (8) Jump to Question
February
Print this Page Add to my Bookmarks Page 50 of 115
March

April Assessment History

May ASSESSMENT PROGRESS:  Total Questions:  8  Questions Answered:  7  Correct Answers:  1

June

July You currently have 1 questions unanswered in this assessment

August

September Question: 7
October You are speaking to a group of medical students about Epstein­Barr virus­associated infectious mononucleosis and the various
November laboratory tests that may be used to make the diagnosis in a patient.

December Of the following, the MOST accurate statement regarding the heterophile antibody test is that

A. antibodies are detectable for short period of time after onset of Epstein­Barr virus infection
B. children 4 years of age or younger may not develop heterophile antibody to Epstein­Barr virus infection
C. it is based on observed agglutination of swine red blood cells
D. presence of antibody is specific for acute Epstein­Barr virus infection
Claim Credit

Evaluation
Incorrect View Peer Results
My Learning Plan
Correct Answer: B Average Percent Correct:  92.50%

Infectious mononucleosis (IM) is a clinical entity characterized by fever, pharyngitis, cervical
lymphadenopathy, hepatosplenomegaly, fatigue, malaise, and headache. The disease occurs
worldwide with no seasonal predilection. It is most frequently recognized in adolescents and young
adults from developed countries because the age at acquisition of primary Epstein­Barr virus (EBV)
infection is older than it is in the developing world. Most young adults develop IM after primary EBV
infection following an incubation period of 6 weeks. The most frequent signs and symptoms are shown
in Table 1.

Hepatitis occurs in 75% of cases, but is usually subclinical with elevation of liver transaminase levels without jaundice or
abdominal pain. The incidence of rash has historically been reported to occur in 80% to 100% of patients who had received
penicillin derivatives and is postulated to be secondary to the development of a transient delayed type aminopenicillin
hypersensitive reaction, although the exact mechanism is unknown. The development of this aminopenicillin­related rash is
not predictive of future intolerance to these agents. The current incidence of rash occurring in patients with IM who receive an
aminopenicillin may be lower than that historically reported. A recent study in pediatric patients with IM who were treated with
an aminopenicillin had an observed incidence of rash of 29.5%, which is much lower than the rates that were originally
reported. However, further studies are needed to confirm this observed decrease in rash occurrence.

During the 6­week incubation period of primary EBV infection, viral replication is first detected in the oral cavity. In the oral
cavity, EBV infects both B cells and tonsillar epithelial cells. The virus transitions from the oral cavity to the peripheral blood at
some point during the incubation period; however, how and when this transition occurs is not well understood. Studies have
shown that copies of EBV genome can be detected in peripheral blood up to 2 weeks before the onset of symptoms. In
addition, gene expression profiling has shown that, 2 weeks before symptom onset, a systemic type 1 interferon response can
be detected in some individuals who subsequently develop IM. The onset of the acute illness is marked by high viral loads in
both the oral cavity and blood. This is accompanied by the production of immunoglobulin M (IgM) antibodies against EBV viral
capsid antigen (VCA) and a robust expansion of CD8+ T lymphocytes. Acute IM is characterized by abnormally high numbers
of circulating CD8+ T cells. Of these cells, many are specific for EBV antigens derived from the immediate early and early
states of lytic infection, with a marked bias toward the immediate early stage. Late lytic antigens also generate a specific
CD8+ T­cell response.

Serological tests confirm primary infection and document remote infection (Figure). The most widely used serological assay,
the heterophile antibody test (Monospot test), was first introduced in 1932. Heterophile antibodies are antibodies produced
against poorly defined antigens and are generally weak antibodies with multispecific activities. The original heterophile test
was based on the discovery that serum or plasma from patients with IM could agglutinate horse or sheep erythrocytes.
Modern variants of the test detect serum­mediated agglutination of latex beads coated by bovine heterophile antigens. A titer
of 40 or greater along with a compatible clinical presentation is strong evidence for IM. Heterophile antibodies typically appear
during the second week of illness. A delayed appearance of heterophile antibodies may be associated with a more prolonged

http://2016.prepid.courses.aap.org/script/may?req=201612122058128048&status=submit# 1/2
12/13/2016 May
convalescence. Horse red blood cell agglutinins persist for a year after diagnosis in 75% of the cases, whereas sheep cell
agglutinins fall to titers of less than 40 by a year in 70% of cases. False­positive titers greater than 40 of sheep and horse
erythrocyte agglutinins have been found in 12% and 6.7% of sera, respectively. There are limitations with the heterophile
antibody testing, which include:

1. Approximately 40% of children 4 years of age and younger do not develop heterophile antibodies following a primary
EBV infection. In these cases, if the heterophile is the only test ordered, the diagnosis will be missed.
2. Heterophile antibodies are nonspecific and may be present in infections caused by other pathogens, malignancies,
and autoimmune diseases.
3. Heterophile antibodies may persist for a year or more and are therefore not always diagnostic of an acute EBV
infection.

The most useful specific antibody tests for the diagnosis of IM are VCA IgM, VCA immunoglobulin G (IgG), and EBV nuclear
antigen (EBNA)­1 IgG, which are usually measured using an enzyme immunoassay platform. The VCA IgM antibodies are
present in 75% of patients during the acute illness. However, false­positive results have been reported especially with
cytomegalovirus infection. All patients with IM develop IgG antibodies to VCA, and this is the best laboratory test to document
a previous EBV infection. Antibodies against EBNA­1 develop slowly and usually are not detectable until 90 days or longer
after onset of illness. Therefore, the presence of EBNA­1 antibodies during an acute illness rules out acute primary EBV
infection. VCA IgG and EBNA­1 remain positive for life. In general, the vast majority of EBV infections can be staged by
measuring VCA IgM, VCA IgG, and EBNA­1 IgG serum antibodies, as shown in Table 2. Early antigen IgG antibodies are not
diagnostic of primary EBV infection because only 60% to 80% of patients are positive during the acute illness and these
antibodies can be found in 20% of healthy individuals.

PREP Pearls
The most widely used serological assay is the heterophile antibody test (monospot test). Heterophile antibodies may be
seen at the onset of illness or may appear later in the course of the illness.

The most useful specific antibody tests for the diagnosis of infectious mononucleosis are viral capsid antigen (VCA)
immunoglobulin M, VCA immunoglobulin G (IgG), and EBV nuclear antigen (EBNA)­1 IgG, which are usually measured
using an enzyme immunoassay platform.

Rash may be seen in patients who have received penicillin derivatives and is postulated to be secondary to the
development of a transient delayed type aminopenicillin hypersensitive reaction, although the exact mechanism is
unknown. The development of this aminopenicillin­related rash is not predictive of future intolerance to these agents.

American Board of Pediatrics Content Specification(s)
Understand the basis for and interpret results of the rapid slide agglutination (Monospot) tests and Epstein Barr virus
serologic tests (viral capsid antigen {VCA}, early antigen {EA}, and Epstein Barr nuclear antigen {EBNA}) according to
manifestations and clinical course

Recognize the association of rashes with ampicillin in infectious Mononucleosis

Suggested Readings
Balfour HH Jr, Dunmire SK, Hogquist KA. Infectious monomucleosis. Clin Trans Immunol. 2015;4(2):e33. DOI:
http://dx.doi.org/10.1038/cti.2015.1

Chovel­Sella A, Tov AB, Lahav E, et al. Incidence of rash after amoxicillin treatment in children with infectious mononucleosis.
Pediatrics. 2013;131(5):e1424­e1427. DOI: http://dx.doi.org/10.1542/peds.2012­1575

Devanthery O, Meylan P. Comparison of a multiplexed bead­based assay with an immunofluorescence and an enzyme­
immuno assay for the assessment of Epstein­Barr virus serologic status. Clin Microbiol Infect. 2010;16(12):1776­1782. DOI:
http://dx.doi.org/10.1111/j.1469­0691.2010.03204.x

Eminger LA, Hall LD, Hesterman KS, Heymann WR. Epstein­Barr virus: dermatologic associations and implications. J Am
Acad Dermatol. 2015;72(1):21­34. DOI: http://dx.doi.org/10.1016/j.jaad.2014.07.035

Gulley ML, Tang W. Laboratory assays for Epstein­Barr virus­related disease. J Mol Diagn. 2008;10(4):279­292. DOI:
http://dx.doi.org/10.2353/jmoldx.2008.080023

Jappe U. Amoxicillin induced exanthema in patients with infectious mononucleosis: allergy or transient immunostimulation?
Allergy. 2007;62(12):1474­1475. DOI: http://dx.doi.org/10.1111/j.1398­9995.2007.01518.x

Vouloumanou EK, Rafailidis PI, Falagas ME. Current diagnosis and management of infectious mononucleosis. Curr Opin
Hematol. 2012;19(1):14­20. DOI: http://dx.doi.org/10.1097/MOH.0b013e32834daa08

Comment On This Question

Page 50 of 115

Contact Us
Links may open in separate window
Copyright © 2016   American Academy of Pediatrics. All rights reserved. Pop­up Blocker may need to be disabled

http://2016.prepid.courses.aap.org/script/may?req=201612122058128048&status=submit# 2/2
12/13/2016 May

Welcome  mohammed alsaiary [ Logout ]

Home PREP Pearls FAQs My Bookmarks CME Information

Overview Home  > May

Editorial Board
May   Enter Keyword Search
January
Question View:   All (8) Jump to Question
February
Print this Page Add to my Bookmarks Page 51 of 115
March

April Assessment History

May ASSESSMENT PROGRESS:  Total Questions:  8  Questions Answered:  7  Correct Answers:  1

June

July You currently have 1 questions unanswered in this assessment

August

September Question: 8
October A 5­year­old boy is referred to your pediatric infectious disease clinic for recommendations about vaccines. The child was born at
November term and has had normal growth and development. He has had 2 hospitalizations, one at 6 months of age for respiratory syncytial
virus (RSV) bronchiolitis, and one for wheezing at 3 years of age. He has been diagnosed with allergic rhinitis, multiple seasonal
December allergies, eczema, and asthma; all are currently under good control. Table 1 shows a list of his current medications.

Of the following, the vaccine that is contraindicated in this child is

A. inactivated polio
B. live­attenuated influenza (LAIV)
Claim Credit C. measles, mumps, rubella (MMR)

Evaluation D. varicella

My Learning Plan

Submit   Reset  

Page 51 of 115

Contact Us
Links may open in separate window
Copyright © 2016   American Academy of Pediatrics. All rights reserved. Pop­up Blocker may need to be disabled

http://2016.prepid.courses.aap.org/script/may?q­page=8&req=201612122058132893 1/1
12/13/2016 May

Welcome  mohammed alsaiary [ Logout ]

Home PREP Pearls FAQs My Bookmarks CME Information

Overview Home  > May

Editorial Board
May   Enter Keyword Search
January
Question View:   All (8) Jump to Question
February
Print this Page Add to my Bookmarks Page 51 of 115
March

April Assessment History

May ASSESSMENT PROGRESS:  Total Questions:  8  Questions Answered:  8  Correct Answers:  1

June

July You currently have 2 required survey(s) that are incomplete. Please complete the survey(s) for the following
question(s) on this assessment:  Question 4, Question 6
August

September

October Question: 8
November A 5­year­old boy is referred to your pediatric infectious disease clinic for recommendations about vaccines. The child was born at
term and has had normal growth and development. He has had 2 hospitalizations, one at 6 months of age for respiratory syncytial
December virus (RSV) bronchiolitis, and one for wheezing at 3 years of age. He has been diagnosed with allergic rhinitis, multiple seasonal
allergies, eczema, and asthma; all are currently under good control. Table 1 shows a list of his current medications.

Of the following, the vaccine that is contraindicated in this child is

A. inactivated polio
B. live­attenuated influenza (LAIV)
Claim Credit
C. measles, mumps, rubella (MMR)
Evaluation
D. varicella
My Learning Plan

Incorrect View Peer Results
Correct Answer: B Average Percent Correct:  90.00%

The live­attenuated influenza vaccine should not be administered to people with asthma or children
younger than 5 years of age with recurrent wheezing. The child in the vignette can safely receive the
measles, mumps, rubella (MMR) vaccine, as well as the varicella and inactivated polio vaccines. For
patients with asthma who have received high doses of corticosteroids (≥ 2 mg/kg of body weight or ≥
20 mg/day of prednisone or its equivalent for people who weigh > 10 kg) for 14 days or more and who
otherwise are not immunocompromised, the recommended interval between stopping the
corticosteroids and live virus immunization (ie, MMR and/or varicella vaccine) is at least 1 month. In
general, inhaled steroids do not cause immunosuppression and are not a contraindication to live virus immunization.

Influenza virus infection is a common cause of febrile respiratory tract disease and school absenteeism. Annual influenza
vaccination is the most effective method for preventing seasonal influenza virus infection and its complications. All individuals
6 months of age and older are recommended for annual influenza vaccination. Particular focus should be on the
administration of inactivated influenza virus (IIV) vaccine for all children and adolescents with underlying medical conditions
associated with an elevated risk of complications from influenza, including asthma or other chronic pulmonary diseases.
During the 2009 seasonal influenza and influenza A (pH1N1) pandemic, an asthma diagnosis in children was highly predictive
of an intensive care unit admission with influenza­like illness (ILI)­related infection, highlighting the future importance of
influenza virus infection prevention.

Chemoprophylaxis should be considered for any child with asthma potentially exposed to influenza, but should not be
considered a substitute for immunization (Table 2). Influenza vaccine always should be offered if not contraindicated, even
after influenza virus has begun circulating in the community. Antiviral medications are effective for the prevention of influenza,
and when used for treatment, can reduce the duration and severity of illness. Early antiviral treatment can reduce the risk for
severe illness or death related to influenza. Antiviral medications currently licensed are important adjuncts to influenza
immunization for control and prevention of influenza disease. Since there are high rates of resistance of 2009 pandemic
influenza A (H1N1), influenza A (H3N2), and influenza B strains to amantadine or rimantadine, only oseltamivir or zanamivir
are currently recommended. Zanamivir is associated with possible bronchospasm and should be used with caution in a
known asthmatic. However, recommendations for use of these drugs for chemoprophylaxis may vary by location and season,
depending on susceptibility patterns. The selection of antiviral medications should be considered in the context of any
available information about surveillance data on influenza antiviral resistance patterns among circulating influenza viruses,
local, state, and national influenza surveillance information on influenza virus type or influenza A virus subtype, the
characteristics of the person who is ill, and results of influenza testing if testing is done. Pediatricians should inform recipients
of antiviral chemoprophylaxis that the risk of influenza is lowered, but still remains while taking medication, and susceptibility
to influenza returns when medication is discontinued. Oseltamivir use is not a contraindication to immunization with IIV.

http://2016.prepid.courses.aap.org/script/may?req=201612122058489768&status=submit 1/2
12/13/2016 May
Corticosteroids administered for brief periods or every other day in the treatment of asthma seem to have a minimal effect on
antibody response to influenza vaccine. Prolonged administration of high doses of corticosteroids (ie, a dose of prednisone of
either 2 mg/kg or greater than a total of 20 mg/day or greater of an equivalent) may impair antibody response. Influenza
immunization can be deferred temporarily during the time of receipt of high­dose corticosteroids, provided deferral does not
compromise the likelihood of immunization before the start of influenza season.

PREP Pearls
The live­attenuated influenza vaccine should not be administered to people with asthma or children younger than 5 years of
age with recurrent wheezing.

Chemoprophylaxis for influenza should not be considered a substitute for immunization.

Influenza immunization can be deferred temporarily during the time of receipt of high­dose corticosteroids, provided deferral
does not compromise the likelihood of immunization before the start of influenza season.

During the 2009 seasonal influenza and influenza A (pH1N1) pandemic, an asthma diagnosis in children was highly
predictive of an intensive care unit admission with influenza­like illness­related infection, highlighting the future importance
of influenza virus prevention.

American Board of Pediatrics Content Specification(s)
Know specific measures, and their effectiveness for immuno‑ and chemoprophylaxis, of infection in patients with asthma
(eg, influenza vaccine)

Suggested Readings
American Academy of Pediatrics. Influenza. In: Kimberlin DW, Brady MT, Jackson MA, Long SS, eds. Red Book: 2015 Report
of the Committee on Infectious Diseases. 30th ed. Elk Grove Village, IL: American Academy of Pediatrics; 2015: 476­493.

Fiore AE, Fry A, Shay D, Gubareva L, Bresee JS, Uyeki TM. Antiviral agents for the treatment and chemoprophylaxis of
influenza: recommendations of the Advisory Committee on Immunization Practices (ACIP). MMWR Morbid Mortal Wkly Rep.
2011;60(RR01):1­24. http://www.cdc.gov/mmwr/preview/mmwrhtml/rr6001a1.htm

National Asthma Education and Prevention Program. Expert Panel Report 3 (EPR­3): guidelines for the diagnosis and
management of asthma­summary report 2007. J Allergy Clin Immunol. 2007;120(5 Suppl):S94­S138. DOI:
http://dx.doi.org/10.1016/j.jaci.2007.09.029

Placzek HE, Madoff LC. Association of age and comorbidity on 2009 influenza A pandemic H1N1­related intensive care unit
stay in Massachusetts. Am J Public Health. 2014;104(11):e118­e125. DOI: http://dx.doi.org/10.2105/AJPH.2014.302197

Comment On This Question

Page 51 of 115

Contact Us
Links may open in separate window
Copyright © 2016   American Academy of Pediatrics. All rights reserved. Pop­up Blocker may need to be disabled

http://2016.prepid.courses.aap.org/script/may?req=201612122058489768&status=submit 2/2
12/13/2016 June

Welcome  mohammed alsaiary [ Logout ]

Home PREP Pearls FAQs My Bookmarks CME Information

Overview Home  > June

Editorial Board
June   Enter Keyword Search
January
Question View:   All (8) Jump to Question
February
Print this Page Add to my Bookmarks Page 53 of 115
March

April Assessment History Mode: Learner  Exam 

May ASSESSMENT PROGRESS:  Total Questions:  8  Questions Answered:  0


June

July You currently have 8 questions unanswered in this assessment

August

September
Question: 1
October
A 10­year­old girl, 20 days postallogeneic hematopoietic stem cell transplantation from a human leukocyte antigen­matched but
November unrelated male donor, for acute leukemia, presents with a 5­day history of fever, cough, chest pain, diarrhea, and an evolving skin
condition. She experienced pain, itching, and burning of her skin, and a rash that subsequently developed on her palms, soles, and
December
legs. Examination reveals a fever of 38°C, and otherwise normal vital signs. She is thin, but well hydrated, and mild jaundice is
present. Alopecia is present.

There are no oral lesions. Her cardiovascular examination was normal, and lung examination revealed decreased breath sounds on
the left base. Her liver is palpable 3 cm below the right costal margin. There are multiple erythematous blanching macules present on
her palms, soles, and in her pretibial area. Skin punch biopsy showed lymphocytic infiltrates at the dermal epidermal junction and
Claim Credit perivascular lymphocytic infiltration. No organisms were seen. Complete blood cell count showed a total white blood cell count of
2,500/μL (2.5 x 109/L) with 20% neutrophils, 40% lymphocytes, and 40% monocytes, and liver function tests showed an alanine
Evaluation
aminotransferase of 200 U/L (3.34 μkat/L), aspartate aminotransferase of 315 U/L (5.26 μkat/L), and a total bilirubin level of 6.5
My Learning Plan mg/dL (111.2 μmol/L). The cytomegalovirus (CMV) immunoglobulin G (IgG) antibody level was positive, and quantitative CMV DNA
polymerase chain reaction (PCR) of plasma showed 500 copies/mL. Epstein­Barr virus (EBV) viral capsid antigen (VCA) IgG
antibody level was positive, EBV VCA immunoglobulin M antibody level was negative, and EBV Epstein­Barr nuclear antigen
antibody was positive. Quantitative EBV DNA PCR of plasma showed 100 copies/mL.

Computed tomography of the chest showed a localized nodular pneumonia in the lower lobe of the left lung. High­dose systemic
corticosteroids were administered, with partial resolution of her symptoms and worsening of her nodular pneumonia.

Of the following infections, the MOST likely cause of her pneumonia is

A. Aspergillus fumigatus
B. cytomegalovirus
C. Epstein­Barr virus
D. Pneumocystis jirovecii

Submit   Reset  

Page 53 of 115

Contact Us
Links may open in separate window
Copyright © 2016   American Academy of Pediatrics. All rights reserved. Pop­up Blocker may need to be disabled

http://2016.prepid.courses.aap.org/script/june?req=201612122123320580 1/1
12/13/2016 June

Welcome  mohammed alsaiary [ Logout ]

Home PREP Pearls FAQs My Bookmarks CME Information

Overview Home  > June

Editorial Board
June   Enter Keyword Search
January
Question View:   All (8) Jump to Question
February
Print this Page Add to my Bookmarks Page 53 of 115
March

April Assessment History Mode: Learner  Exam 

May ASSESSMENT PROGRESS:  Total Questions:  8  Questions Answered:  1  Correct Answers:  1


June

July You currently have 7 questions unanswered in this assessment

August

September
Question: 1
October
A 10­year­old girl, 20 days postallogeneic hematopoietic stem cell transplantation from a human leukocyte antigen­matched but
November unrelated male donor, for acute leukemia, presents with a 5­day history of fever, cough, chest pain, diarrhea, and an evolving skin
condition. She experienced pain, itching, and burning of her skin, and a rash that subsequently developed on her palms, soles, and
December
legs. Examination reveals a fever of 38°C, and otherwise normal vital signs. She is thin, but well hydrated, and mild jaundice is
present. Alopecia is present.

There are no oral lesions. Her cardiovascular examination was normal, and lung examination revealed decreased breath sounds on
the left base. Her liver is palpable 3 cm below the right costal margin. There are multiple erythematous blanching macules present on
her palms, soles, and in her pretibial area. Skin punch biopsy showed lymphocytic infiltrates at the dermal epidermal junction and
Claim Credit perivascular lymphocytic infiltration. No organisms were seen. Complete blood cell count showed a total white blood cell count of
2,500/μL (2.5 x 109/L) with 20% neutrophils, 40% lymphocytes, and 40% monocytes, and liver function tests showed an alanine
Evaluation
aminotransferase of 200 U/L (3.34 μkat/L), aspartate aminotransferase of 315 U/L (5.26 μkat/L), and a total bilirubin level of 6.5
My Learning Plan mg/dL (111.2 μmol/L). The cytomegalovirus (CMV) immunoglobulin G (IgG) antibody level was positive, and quantitative CMV DNA
polymerase chain reaction (PCR) of plasma showed 500 copies/mL. Epstein­Barr virus (EBV) viral capsid antigen (VCA) IgG
antibody level was positive, EBV VCA immunoglobulin M antibody level was negative, and EBV Epstein­Barr nuclear antigen
antibody was positive. Quantitative EBV DNA PCR of plasma showed 100 copies/mL.

Computed tomography of the chest showed a localized nodular pneumonia in the lower lobe of the left lung. High­dose systemic
corticosteroids were administered, with partial resolution of her symptoms and worsening of her nodular pneumonia.

Of the following infections, the MOST likely cause of her pneumonia is

A. Aspergillus fumigatus
B. cytomegalovirus
C. Epstein­Barr virus
D. Pneumocystis jirovecii

Correct View Peer Results
Average Percent Correct:  62.86%

This patient has acute graft­versus­host disease (GvHD), a common complication of hematopoietic
stem cell transplantation (HSCT). Her early postengraftment course is complicated by a fungal nodular
pneumonia caused by Aspergillus fumigatus, which progressed after administration of high dose
steroids administered to treat her acute GvHD. Nodular pneumonia during the early post engraftment
period after HSCT is most often caused by Gram­positive or Gram­negative bacteria or fungal
organisms, including Aspergillus. Cytomegalovirus (CMV) usually causes a diffuse interstitial
pneumonitis, especially in the early postengraftment period, but it is not a likely cause of nodular
pneumonia. Epstein­Barr virus (EBV) is associated with post­transplant lymphoproliferative disease. While it may cause a
nodular pneumonia in some patients, it most often manifests as lymphadenopathy, and usually presents between 3 and 6
months post­HSCT, during the late postengraftment period after HSCT. Pneumocystis pneumonia may present in the
immediate postengraftment period, but usually presents with high fever, dry cough, hypoxemia, and interstitial rather than
nodular pneumonitis. It also is uncommon in HSCT patients, who usually receive prophylaxis with trimethoprim­
sulfamethoxazole or aerosolized pentamidine.

Patients who undergo HSCT are at high risk for viral, bacterial, fungal, and parasitic infections. The organ dysfunction caused
by GvHD as well as the immunosuppressive regimens, such as steroids, used to treat GvHD further increase the infection risk
in these patients, through ongoing suppression of T lymphocyte function. Graft­versus­host disease may be acute, when it

http://2016.prepid.courses.aap.org/script/june?req=201612122124302144# 1/2
12/13/2016 June
occurs within 100 days post­HSCT, or chronic, if it persists more than 100 days after HSCT. Severity of GvHD ranges from 0
through IV, with severities of II through IV most often associated with poor prognosis for quality of life and survival.

To assess a patient’s risk for GvHD and subsequent risk for infection, consideration should be given to the patient’s primary
underlying illness (cancer stage), the conditioning regimen received in preparation for transplant, the type of transplant
received (allogenic matched or unmatched, related or unrelated versus autologous), the immune suppression regimen
administered to decrease risk for GvHD (pharmacologic agents administered 2 days prior and for 100 days or more after
transplant to suppress T lymphocyte function, pretransplant conditioning, or T­cell depletion of transplanted stem cells), and
evidence of prior infections prone to recurrence during immune suppression (toxoplasmosis, herpes simplex virus, CMV, EBV,
adenovirus).

During the late postengraftment period, 100 days after HSCT, the new T lymphocytes from the successfully engrafted donor
cells constitute parts of a functioning immune system, with subsequent reduction of infection risk. However, late infectious
complications may be seen, especially if chronic GvHD is present, because these patients may have delayed immune
reconstitution caused by cellular and humoral dysfunction, including hypogammaglobulinemia and poor opsonization, as well
as hyposplenism. If chronic GvHD is present, then glucocorticoids and other immunosuppressants administered long term for
treatment of this condition further comprise immune function. Infections in the late postengraftment period may be caused by
viral infections, such as CMV, which may cause colitis or hepatitis, EBV, the cause of post­transplant lymphoproliferative
disease, or varicella zoster virus, the cause of zoster and disseminated zoster. Nodular pneumonias, caused by Gram­
positive or Gram­negative bacteria or fungal organisms, especially Aspergillus species or Nocardia, may occur. Bacterial
infections from encapsulated bacteria (Streptococcus pneumoniae, Haemophilus influenzae, and Neisseria meningitidis) are
also common. In addition, staphylococci and Gram­negative bacteria, including Pseudomonas species, may cause serious
life­threatening bacteremias and other types of infections.

PREP Pearls
Aspergillus may cause nodular pneumonia after hematopoietic stem cell transplantation.

Hematopoietic stem cell transplantation may be complicated by graft­versus­host disease, which further increases the risk
for infection post­transplant.

Treatments such as high dose steroids administered for graft versus host disease further increase the infection risks in
these patients

American Board of Pediatrics Content Specification(s)
Understand the immunocompromised status of patients receiving therapy for graft versus host disease

Suggested Readings
Dvorak CC, Steinbach WJ. Opportunistic infections in hematopoietic stem cell transplantation. In: Cherry J, Demmler­Harrison
GJ, Kaplan SL, Steinbach WJ, Hotez P, eds. Feigin and Cherry's Textbook of Pediatric Infectious Diseases. 7th ed.
Philadelphia, PA: Saunders Elsevier; 2013:933­944.

Lujan­Zilbermann J. Infections in hematopoietic stem cell transplant recipients. In: Long SS, Pickering LK, Prober CG, eds.
Principals and Practices of Pediatric Infectious Diseases. 4th ed. Philadelphia, PA: Saunders Elsevier; 2012:562­566.

Comment On This Question

Page 53 of 115

Contact Us
Links may open in separate window
Copyright © 2016   American Academy of Pediatrics. All rights reserved. Pop­up Blocker may need to be disabled

http://2016.prepid.courses.aap.org/script/june?req=201612122124302144# 2/2
12/13/2016 June

Welcome  mohammed alsaiary [ Logout ]

Home PREP Pearls FAQs My Bookmarks CME Information

Overview Home  > June

Editorial Board
June   Enter Keyword Search
January
Question View:   All (8) Jump to Question
February
Print this Page Add to my Bookmarks Page 54 of 115
March

April Assessment History Mode: Learner  Exam 

May ASSESSMENT PROGRESS:  Total Questions:  8  Questions Answered:  1  Correct Answers:  1


June

July You currently have 7 questions unanswered in this assessment

August

September
Question: 2
October
A 9­month­old daycare attendee presents to the emergency department with fever and bloody, mucoid diarrhea. On admission, his
November white blood cell count is 52,000/µL (52 x 109/L) with 82% neutrophils. A stool culture is resulted as positive for a Shiga toxin­
December producing organism.

Of the following, the factor that plays the MOST important role in triggering this leukemoid reaction is

A. complement
B. cytoplasmic granules
C. granulocyte colony­stimulating factor
Claim Credit
D. interferon­ γ
Evaluation

My Learning Plan
Submit   Reset  

Page 54 of 115

Contact Us
Links may open in separate window
Copyright © 2016   American Academy of Pediatrics. All rights reserved. Pop­up Blocker may need to be disabled

http://2016.prepid.courses.aap.org/script/june?q­page=2&req=201612122124306989 1/1
12/13/2016 June

Welcome  mohammed alsaiary [ Logout ]

Home PREP Pearls FAQs My Bookmarks CME Information

Overview Home  > June

Editorial Board
June   Enter Keyword Search
January
Question View:   All (8) Jump to Question
February
Print this Page Add to my Bookmarks Page 54 of 115
March

April Assessment History Mode: Learner  Exam 

May ASSESSMENT PROGRESS:  Total Questions:  8  Questions Answered:  2  Correct Answers:  1


June

July You currently have 6 questions unanswered in this assessment

August

September
Question: 2
October
A 9­month­old daycare attendee presents to the emergency department with fever and bloody, mucoid diarrhea. On admission, his
November white blood cell count is 52,000/µL (52 x 109/L) with 82% neutrophils. A stool culture is resulted as positive for a Shiga toxin­
December producing organism.

Of the following, the factor that plays the MOST important role in triggering this leukemoid reaction is

A. complement
B. cytoplasmic granules
C. granulocyte colony­stimulating factor
Claim Credit
D. interferon­ γ
Evaluation

My Learning Plan
Incorrect View Peer Results

Correct Answer: C Average Percent Correct:  57.14%

A leukemoid reaction is a reactive increase in the white blood cell count (WBC), which can mimic
leukemia. The reaction is actually caused by an infection or another disease, and is not a sign of
cancer. Acute infection should be particularly suspected in patients with a total WBC greater than
25,000/μL (25 x 109/L). Certain bacteria (eg, pneumococcus, Staphylococcus, clostridial species) may
cause particularly high leukocyte counts. Certain infections such as Shigella, Clostridium difficile, and
disseminated fungal infections can lead to leukemoid reactions of over 50,000/μL (50 x 109/L) WBC
being seen on the peripheral blood. A leukemoid reaction is also seen in Bordetella pertussis infections, but with a
predominance of lymphocytes rather than neutrophils. Blood cell counts usually return to normal when the underlying
condition is treated. In contrast, overwhelming sepsis can lead to neutropenia through consumption of the neutrophil marrow
reserve.

Neutrophils are leukocytes responsible for phagocytosis and killing of bacteria through the process of respiratory burst. In
response to an acute infectious and inflammatory process, the body upregulates the production of neutrophils 5 to 10 times
the normal rate and stimulates the release of these infection­fighting cells from the bone marrow into the bloodstream and
then to infected tissue. Factors such as interleukin­3, interleukin­6, and granulocyte colony­stimulating factor (G­CSF) and
granulocyte macrophage colony­stimulating factor (GM­CSF) bind to target cells to increase the mitotic pool of promyelocytes
and myelocytes, the precursors of mature neutrophils that are found in the bone marrow. These factors also serve to shorten
the transit time of neutrophils in the bone marrow. It is for these reasons that injectable G­CSF is prescribed to neutropenic
patients. Complement enhances phagocytosis by neutrophils, but does not promote their release from the bone marrow.
Interferon­γ is a cytokine that is produced by natural killer cells and cytotoxic T cells. Cytoplasmic granules are released by
neutrophils and help eliminate bacteria. Once released from the bone marrow, the life span of a neutrophil is 8 to 12 hours.
The neutrophils then undergo the steps of margination (attachment to the lining of blood vessels), then diapedesis or
chemotaxis and migration to infected tissue. Stimuli that lead to the release of neutrophils from the bone marrow include
bacterial and fungal cell wall components, which also comprise of exotoxins and endotoxins.

Any disruption of neutrophil adherence and chemotaxis can lead to recurrent and sometimes severe infections. An example of
this is leukocyte adhesion deficiency I, which involves a defect of the integrin CD18 on polymorphonuclear cells that leads to
impaired adherence to the vascular endothelium with resulting poor chemotaxis and phagocytosis. Severe periodontal
disease, delayed umbilical cord separation, poor wound healing, and recurrent necrotizing skin, bowel, and perirectal
infections with organisms such as Staphylococcus aureus and Pseudomonas aeruginosa are seen. The lack of marginated
neutrophils leads to persistent leukocytosis typically of above 15,000/µL (15x 109/L). This immunodeficiency illustrates the
important role of polymorphonuclear neutrophil release and activity in host defense.

http://2016.prepid.courses.aap.org/script/june?req=201612122125550895&status=submit 1/2
12/13/2016 June

PREP Pearls
A leukemoid reaction can indicate certain infections including Shigella, Clostridium difficile, serious bacterial infections, or
disseminated fungal infections.

Granulocyte colony­stimulating factor may serve to shorten transit time of neutrophils in the bone marrow.

Leukocyte adhesion deficiency is an example of impaired neutrophil mobility that leads to recurrent infections.

American Board of Pediatrics Content Specification(s)
Understand the significance of release of PMNs from bone marrow and how it affects host defense

Suggested Readings
de la Morena M. Immunologic developments and susceptibility to infection. In: Long SS, Pickering LK, Prober CG, eds.
Principles and Practice of Pediatric Infectious Diseases: Expert Consult. 4th ed. New York, NY: Churchill Livingstone Elsevier;
2012: 83­90.

Glougauer M. Disorders of phagocyte function. In: Goldman L, Schafer AI, eds. Goldman’s Cecil Medicine. 24th ed.
Philadelphia, PA: Saunders Elsevier; 2012: 1111­1117.

Hohl TM. Cell­mediated defense against infection. In: Bennett JE, Dolin R, Blaser MJ, eds. Mandell, Douglas, and Bennett's
Principles and Practice of Infectious Diseases. 8th ed. Philadelphia, PA: Saunders Elsevier; 2015:50­69.

Comment On This Question

Page 54 of 115

Contact Us
Links may open in separate window
Copyright © 2016   American Academy of Pediatrics. All rights reserved. Pop­up Blocker may need to be disabled

http://2016.prepid.courses.aap.org/script/june?req=201612122125550895&status=submit 2/2
12/13/2016 June

Welcome  mohammed alsaiary [ Logout ]

Home PREP Pearls FAQs My Bookmarks CME Information

Overview Home  > June

Editorial Board
June   Enter Keyword Search
January
Question View:   All (8) Jump to Question
February
Print this Page Add to my Bookmarks Page 55 of 115
March

April Assessment History Mode: Learner  Exam 

May ASSESSMENT PROGRESS:  Total Questions:  8  Questions Answered:  2  Correct Answers:  1


June

July You currently have 6 questions unanswered in this assessment

August

September
Question: 3
October
A 10­year­old boy presents with a 4­day history of fever of 39°C, along with intense headache, myalgias, abdominal pain, nausea,
November and vomiting. History reveals that he attended boy scout camp in the southeastern United States 2 weeks ago, where he went
fishing, kayaking, and took long nature walks with his troop. He reports that that they did not eat any undercooked or raw meats at
December
camp, and that they drank filtered water. His temperature is 39.3°C, heart rate is 120 beats/min, respiratory rate is 15 breaths/min,
and blood pressure is 112/70 mm Hg. Examination reveals a mildly dehydrated male who is ill appearing. He has tachycardia without
murmur and normal lung findings. His abdomen is flat, but uncomfortable on palpation, and his liver is 2 cm below the costal margin.
Laboratory evaluation reveals a white blood cell count of 8,000/μL (8 x 109/L) with 75% segmented neutrophils, hemoglobin of 12
g/dL (120 g/L), platelet count of 90 x 103/μL (90 x 109/L), aspartate aminotransferase of 200 U/L (3.34 μkat/L) (normal, 12­50 U/L),
alanine aminotransferase of 300 U/L (5.01 μkat/L) (normal, 12­50 U/L), and total bilirubin of 0.5 mg/dL (8.6 μmol/L) (normal, 0.0­1.2
Claim Credit
mg/dL).
Evaluation
Of the following, the MOST likely diagnosis in this patient is
My Learning Plan
A. babesiosis
B. Colorado tick fever
C. Rocky Mountain spotted fever
D. tularemia

Submit   Reset  

Page 55 of 115

Contact Us
Links may open in separate window
Copyright © 2016   American Academy of Pediatrics. All rights reserved. Pop­up Blocker may need to be disabled

http://2016.prepid.courses.aap.org/script/june?q­page=3&req=201612122125555427 1/1
12/13/2016 June

Welcome  mohammed alsaiary [ Logout ]

Home PREP Pearls FAQs My Bookmarks CME Information

Overview Home  > June

Editorial Board
June   Enter Keyword Search
January
Question View:   All (8) Jump to Question
February
Print this Page Add to my Bookmarks Page 55 of 115
March

April Assessment History Mode: Learner  Exam 

May ASSESSMENT PROGRESS:  Total Questions:  8  Questions Answered:  3  Correct Answers:  1


June

July You currently have 5 questions unanswered in this assessment

August

September
Question: 3
October
A 10­year­old boy presents with a 4­day history of fever of 39°C, along with intense headache, myalgias, abdominal pain, nausea,
November and vomiting. History reveals that he attended boy scout camp in the southeastern United States 2 weeks ago, where he went
fishing, kayaking, and took long nature walks with his troop. He reports that that they did not eat any undercooked or raw meats at
December
camp, and that they drank filtered water. His temperature is 39.3°C, heart rate is 120 beats/min, respiratory rate is 15 breaths/min,
and blood pressure is 112/70 mm Hg. Examination reveals a mildly dehydrated male who is ill appearing. He has tachycardia without
murmur and normal lung findings. His abdomen is flat, but uncomfortable on palpation, and his liver is 2 cm below the costal margin.
Laboratory evaluation reveals a white blood cell count of 8,000/μL (8 x 109/L) with 75% segmented neutrophils, hemoglobin of 12
g/dL (120 g/L), platelet count of 90 x 103/μL (90 x 109/L), aspartate aminotransferase of 200 U/L (3.34 μkat/L) (normal, 12­50 U/L),
alanine aminotransferase of 300 U/L (5.01 μkat/L) (normal, 12­50 U/L), and total bilirubin of 0.5 mg/dL (8.6 μmol/L) (normal, 0.0­1.2
Claim Credit
mg/dL).
Evaluation
Of the following, the MOST likely diagnosis in this patient is
My Learning Plan
A. babesiosis
B. Colorado tick fever
C. Rocky Mountain spotted fever
D. tularemia

Incorrect View Peer Results
Correct Answer: C Average Percent Correct:  71.43%

There is much clinical overlap of tickborne diseases, making history, examination, and laboratory
evaluation critical in developing the differential diagnosis. The clinical and epidemiologic features of the
patient described above are most consistent with Rocky Mountain spotted fever (RMSF) caused by the
organism Rickettsia rickettsii. High­spiking fevers, headache, myalgias, nausea, and vomiting are
common clinical features of RMSF. Abdominal pain is also common and can lead to a delay in the
diagnosis. Hepatomegaly and splenomegaly occur in approximately one­third of patients and
transaminitis occurs frequently, worsening throughout the course of disease until appropriate treatment
is initiated. While 80% of patients exhibit the characteristic macular rash that progresses to petechiae on the wrists and ankles
with subsequent spread to palms/soles and trunk, 20% will manifest an atypical rash or no rash at all. Lack of the
characteristic rash may also lead to diagnostic delays and worse outcomes. The clinical features of babesiosis overlap with
RMSF, although patients with babesiosis typically manifest hemolytic anemia and jaundice, which this patient does not have
(Table). Patients with Colorado tick fever generally exhibit a biphasic febrile illness with myalgias. Leukopenia with atypical
lymphocytosis and thrombocytopenia are common, but maculopapular and petechial rashes occur in only 5% to 12% of
patients. Colorado tick fever is seen in the western United States and southern Canada, but not in the southeastern United
States. In contrast, tularemia is seen all over the United States, except for Hawaii. The typhoidal form of tularemia manifests
with signs and symptoms including high fever, chills, malaise, headache, vomiting, myalgias, and hepatosplenomegaly, which
overlaps with the clinical features of RMSF. However, the typhoidal form of tularemia is much less common than RMSF, as
well as other forms of tularemia (eg, glandular and ulceroglandular), which are transmitted by ticks. Typhoidal tularemia is
generally caused by ingestion of contaminated foods including undercooked meat or water, which is not consistent with the
history given for the patient in the vignette.

There are several other tickborne diseases that are clinically indistinguishable to RMSF, including ehrlichiosis and
anaplasmosis, which manifest with fever, myalgias, headache, nausea, vomiting, and diarrhea in most children. However,
some differences exist on physical examination. In contrast to RMSF, children with ehrlichiosis often exhibit a maculopapular
rash on the trunk with sparing of the hands and feet; children with anaplasmosis are unlikely to develop rash during the
course of their illness. Further, anaplasmosis occurs in the northeastern and upper midwestern states, and northern
California, where RMSF is not endemic.

http://2016.prepid.courses.aap.org/script/june?req=201612122126375584&status=submit# 1/2
12/13/2016 June
The mortality rate of RMSF was as high as 20% historically. With the advent of effective therapy, the overall mortality has
decreased to around 1%. However, mortality rates as high as 3% have been seen in some years, and up to 10% in some
outbreak settings. The keys to reducing morbidity and mortality include considering the diagnosis in the differential, obtaining
appropriate serology, and initiating supportive care with intravenous fluids and antibiotic treatment based on suspicion without
waiting for diagnostic confirmation. Doxycycline is the recommended treatment for RMSF regardless of patient age and is
preferred over other tetracycline agents owing to reduced risk of phototoxicity and lack of risk of dental staining when used in
durations to treat tickborne diseases. Typical treatment duration is 3 days beyond clinical improvement with defervescence, or
7 to 10 days total. The recommended dose is 4 mg/kg per day divided every 12 hours (max 100 mg/dose) in both intravenous
and oral formulations.

* Required *
Take Survey  

PREP Pearls
Twenty percent of patients with Rocky Mountain spotted fever (RMSF) manifest an atypical rash or absence of rash.

Patients with an atypical presentation of RMSF are more likely to have a delay in the diagnosis and potentially worse
outcomes.

There is much clinical overlap of tickborne diseases, making history, examination, and laboratory evaluation critical in
developing the differential diagnosis.

American Board of Pediatrics Content Specification(s)
Formulate the differential diagnosis in a patient in whom Rocky Mountain spotted fever is suspected

Plan the management of a patient with Rocky Mountain spotted fever (fluid therapy, effective antibiotics)

Suggested Readings
American Academy of Pediatrics. Rocky Mountain spotted fever. In: Kimberlin DW, Brady MT, Jackson MA, Long SS, eds.
Red Book: 2015 Report of the Committee on Infectious Diseases. 30th ed. Elk Grove Village, IL: American Academy of
Pediatrics; 2015: 682­684.

McQuiston JH, Regan JJ, Paddock CD. Rickettsia rickettsii (Rocky Mountain Spotted Fever). In: Long S, Pickering LK, Prober
CG, eds. Principles and Practice of Pediatric Infectious Diseases. 4th ed. New York, NY: Saunders Elsevier; 2012:926­929.

Comment On This Question

Page 55 of 115

Contact Us
Links may open in separate window
Copyright © 2016   American Academy of Pediatrics. All rights reserved. Pop­up Blocker may need to be disabled

http://2016.prepid.courses.aap.org/script/june?req=201612122126375584&status=submit# 2/2
12/13/2016 June

Welcome  mohammed alsaiary [ Logout ]

Home PREP Pearls FAQs My Bookmarks CME Information

Overview Home  > June

Editorial Board
June   Enter Keyword Search
January
Question View:   All (8) Jump to Question
February
Print this Page Add to my Bookmarks Page 56 of 115
March

April Assessment History Mode: Learner  Exam 

May ASSESSMENT PROGRESS:  Total Questions:  8  Questions Answered:  3  Correct Answers:  1


June

July You currently have 5 questions unanswered in this assessment

August

September
Question: 4
October
A 10­month­old full term, previously healthy male infant presents to the emergency room with increasing lethargy and poor feeding
November over the past 2 weeks. The patient lives with his parents and 5­year­old brother. His parents recently immigrated to the United States
several years ago from a resource­limited area. The mother has a chronic cough. Vital signs on physical examination revealed a
December
temperature of 37.3°C, respiratory rate of 65 breaths/min, heart rate of 170 beats/min, and an oxygen saturation of 95% on room air.
His weight is less than fifth percentile and his height is 50th percentile for age. The infant is lethargic but responsive to painful stimuli,
and has a bulging anterior fontanelle, bilateral crackles on lung examination, hepatosplenomegaly, and lower extremity weakness. A
chest radiograph revealed increased interstitial markings and bilateral hilar adenopathy (Figure). The tuberculin skin test using
purified protein derivative was read as 4 mm of induration.

Claim Credit Of the following, the laboratory test result that would be the MOST useful for guiding antituberculous therapy is

Evaluation A. mother’s sputum culture

My Learning Plan B. mother’s tuberculin skin test
C. patient’s gastric aspirate
D. patient’s interferon ­ γ release assay

Submit   Reset  

Page 56 of 115

Contact Us
Links may open in separate window
Copyright © 2016   American Academy of Pediatrics. All rights reserved. Pop­up Blocker may need to be disabled

http://2016.prepid.courses.aap.org/script/june?q­page=4&req=201612122126384646 1/1
12/13/2016 June

Welcome  mohammed alsaiary [ Logout ]

Home PREP Pearls FAQs My Bookmarks CME Information

Overview Home  > June

Editorial Board
June   Enter Keyword Search
January
Question View:   All (8) Jump to Question
February
Print this Page Add to my Bookmarks Page 56 of 115
March

April Assessment History Mode: Learner  Exam 

May ASSESSMENT PROGRESS:  Total Questions:  8  Questions Answered:  4  Correct Answers:  2


June

July You currently have 4 questions unanswered in this assessment

August

September
Question: 4
October
A 10­month­old full term, previously healthy male infant presents to the emergency room with increasing lethargy and poor feeding
November over the past 2 weeks. The patient lives with his parents and 5­year­old brother. His parents recently immigrated to the United States
several years ago from a resource­limited area. The mother has a chronic cough. Vital signs on physical examination revealed a
December
temperature of 37.3°C, respiratory rate of 65 breaths/min, heart rate of 170 beats/min, and an oxygen saturation of 95% on room air.
His weight is less than fifth percentile and his height is 50th percentile for age. The infant is lethargic but responsive to painful stimuli,
and has a bulging anterior fontanelle, bilateral crackles on lung examination, hepatosplenomegaly, and lower extremity weakness. A
chest radiograph revealed increased interstitial markings and bilateral hilar adenopathy (Figure). The tuberculin skin test using
purified protein derivative was read as 4 mm of induration.

Claim Credit Of the following, the laboratory test result that would be the MOST useful for guiding antituberculous therapy is

Evaluation A. mother’s sputum culture

My Learning Plan B. mother’s tuberculin skin test
C. patient’s gastric aspirate
D. patient’s interferon ­ γ release assay

Correct View Peer Results

Average Percent Correct:  64.29%

The patient’s clinical and radiographic findings are consistent with miliary tuberculosis (TB). Miliary TB
is the disseminated form of the disease that occurs when hematogenous spread of bacilli leads to
disease in 2 or more organs. In this case, the patient’s lungs, liver, spleen, and central nervous system
are involved. The transmission to infants is typically from an adult source case. In this scenario, one of
the patient’s parents is the likely source. An investigation to identify the source case should be
conducted by the local department of health. Susceptibilities from the source case (mother’s sputum)
would reveal resistance patterns and therefore guide antimycobacterial choice. Although culture from
the patient’s gastric aspirate would be useful, it is difficult to obtain in infants, and the overall yield of gastric aspirates is less
than 40% in this age group. Interferon­ γ release assays (IGRA) are used to diagnose TB, but do not provide susceptibilities
and thus cannot guide therapy. Further, they are relatively unstudied in young children.

Tuberculin skin test (or IGRA) screening is indicated in high­risk patients, including recent immigrants, international adoptees
and refugees, individuals that have traveled to high­prevalence areas, the homeless, and those exposed to prisons or
shelters. Other factors that increase risk of TB disease include immunocompromised status (lymphoma, HIV), malnutrition,
intravenous drug use, diabetes mellitus, and chronic renal failure. The test is also indicated for patients prior to the start of
tumor necrosis factor­a antagonists or prolonged systemic steroid therapy. It is important to note that the patient’s tuberculin
skin test is not considered positive based on established guidelines (Table 1). This does not exclude the diagnosis of active
TB as skin testing can be negative in cases of disseminated disease, in infants, immunocompromised hosts, and those with
failure to thrive or malnutrition (Table 2). Up to 10% of immunocompetent children with culture­positive disease do not have
an initial reactive tuberculin skin test, and that proportion increases to 30% in those with miliary disease. A previous history of
Bacillus Calmette­Guérin (BCG) vaccination should be disregarded when interpreting tuberculin skin test results. The effect of
the BCG vaccine on tuberculin skin test results typically wanes within 5 years of vaccination. Tuberculin skin test results can
also be false­positive, as there is potential for cross­reactivity with nontuberculous mycobacteria. The IGRAs demonstrate
improved specificity with no false­positives due to BCG vaccination and decreased cross reactivity to non­TB mycobacteria,
although there is a still a potential but rare risk of false­positives secondary to Mycobacterium kansasii, Mycobacterium
szulgai, and Mycobacterium marinum. Further, IGRAs are not more sensitive than tuberculin skin tests in the
immunocompromised host.

http://2016.prepid.courses.aap.org/script/june?req=201612122127106053&status=submit 1/2
12/13/2016 June
Miliary disease caused by TB develops within 2 to 6 months of primary disease, typically in infants and young children. The
onset is often indolent, with symptoms developing over days to weeks. The organs most commonly involved are the lungs,
spleen, liver, and bone marrow. A liver or bone marrow biopsy can confirm the diagnosis. Early symptoms include malaise,
weight loss, and low­grade fever, followed by generalized lymphadenopathy, high fevers, and hepatosplenomegaly. Late
symptoms include respiratory distress and positive chest radiograph findings. With appropriate treatment, the prognosis is
excellent and in most cases, clinical improvement is seen within 2 weeks of therapy.

PREP Pearls
Look for a source case when an infant or young child is diagnosed with tuberculosis.

Infants are at highest risk of progressing to active disease.

Tuberculin skin testing can be negative in infants, especially in cases of miliary tuberculosis.

American Board of Pediatrics Content Specification(s)
Recognize the history and clinical manifestations of miliary tuberculosis

Understand the indications for and interpretation of tuberculin skin tests, including differential interpretation based on risk
factors in the host

Suggested Readings
American Academy of Pediatrics. Tuberculosis. In: Pickering LK, Baker CJ, Kimberlin DW, Long SS, eds. Red Book: 2015
Report of the Committee on Infectious Diseases. 30th ed. Elk Grove Village, IL: American Academy of Pediatrics; 2015:805­
831.

Mazurek GH, Jereb J, Vernon A, LoBue P, Goldberg S, Castro K. Updated guidelines for using interferon gamma release
assays to detect Mycobacterium tuberculosis infection ­­­ United States, 2010. MMWR Morbid Mortal Wkly Rep.
2010;59(RR05):1­25. http://www.cdc.gov/mmwr/preview/mmwrhtml/rr5905a1.htm

Starke JR, Committee on Infectious Diseases. Interferon ­ γ release assays for diagnosis of tuberculosis infection and disease
in children. Pediatrics. 2014;134(6):e1763­e1773. DOI: http://dx.doi.org/10.1542/peds.2014­2983

Starke JR. Mycobacterium tuberculosis. In: Long SS, Pickering LK, Prober CG, eds. Principles and Practice of Pediatric
Infectious Diseases. 4th ed. Philadelphia, PA: Saunders Elsevier; 2012:771­785.

Comment On This Question

Page 56 of 115

Contact Us
Links may open in separate window
Copyright © 2016   American Academy of Pediatrics. All rights reserved. Pop­up Blocker may need to be disabled

http://2016.prepid.courses.aap.org/script/june?req=201612122127106053&status=submit 2/2
12/13/2016 June

Welcome  mohammed alsaiary [ Logout ]

Home PREP Pearls FAQs My Bookmarks CME Information

Overview Home  > June

Editorial Board
June   Enter Keyword Search
January
Question View:   All (8) Jump to Question
February
Print this Page Add to my Bookmarks Page 57 of 115
March

April Assessment History Mode: Learner  Exam 

May ASSESSMENT PROGRESS:  Total Questions:  8  Questions Answered:  4  Correct Answers:  2


June

July You currently have 4 questions unanswered in this assessment

August

September
Question: 5
October
You are asked to see a 17­year­old adolescent boy who is hospitalized for a febrile illness. He developed fever, diffuse myalgias,
November headache, and conjunctival injection 8 days ago. His fevers resolved 4 days ago and his constitutional symptoms improved. Two
days ago, his fever recurred and he was admitted to the hospital. He complains of a severe headache, neck pain, diffuse myalgias,
December
nausea, vomiting, and nonspecific abdominal pain. On questioning, you find out that the adolescent has been spending his summer
working with his uncle in in a rural area. The patient had direct contact with cattle and pigs, and also spent time helping his uncle in a
slaughterhouse. On physical examination, the patient is ill appearing, has a temperature of 38.7°C, pulse of 120 beats/min,
respiratory rate of 20 breaths/min, and blood pressure of 110/60 mm Hg. He has bilateral, nonpurulent, conjunctival injection and a
diffuse maculopapular and petechial rash on his trunk and extremities. His mental status is normal, but he has nuchal rigidity. His
abdominal examination is benign without organomegaly and he is not icteric. Laboratory results reveal a white blood cell count of
Claim Credit 18,000/μL (18 x 109/L) with a predominance of neutrophils. The rest of his complete blood cell count is normal and his metabolic
Evaluation profile is normal, except for mildly elevated hepatic transaminases.

My Learning Plan Of the following, the MOST likely diagnosis in the patient is

A. brucellosis
B. Kawasaki disease
C. leptospirosis
D. tularemia

Submit   Reset  

Page 57 of 115

Contact Us
Links may open in separate window
Copyright © 2016   American Academy of Pediatrics. All rights reserved. Pop­up Blocker may need to be disabled

http://2016.prepid.courses.aap.org/script/june?q­page=5&req=201612122127110428 1/1
12/13/2016 June

Welcome  mohammed alsaiary [ Logout ]

Home PREP Pearls FAQs My Bookmarks CME Information

Overview Home  > June

Editorial Board
June   Enter Keyword Search
January
Question View:   All (8) Jump to Question
February
Print this Page Add to my Bookmarks Page 57 of 115
March

April Assessment History Mode: Learner  Exam 

May ASSESSMENT PROGRESS:  Total Questions:  8  Questions Answered:  5  Correct Answers:  2


June

July You currently have 3 questions unanswered in this assessment

August

September
Question: 5
October
You are asked to see a 17­year­old adolescent boy who is hospitalized for a febrile illness. He developed fever, diffuse myalgias,
November headache, and conjunctival injection 8 days ago. His fevers resolved 4 days ago and his constitutional symptoms improved. Two
days ago, his fever recurred and he was admitted to the hospital. He complains of a severe headache, neck pain, diffuse myalgias,
December
nausea, vomiting, and nonspecific abdominal pain. On questioning, you find out that the adolescent has been spending his summer
working with his uncle in in a rural area. The patient had direct contact with cattle and pigs, and also spent time helping his uncle in a
slaughterhouse. On physical examination, the patient is ill appearing, has a temperature of 38.7°C, pulse of 120 beats/min,
respiratory rate of 20 breaths/min, and blood pressure of 110/60 mm Hg. He has bilateral, nonpurulent, conjunctival injection and a
diffuse maculopapular and petechial rash on his trunk and extremities. His mental status is normal, but he has nuchal rigidity. His
abdominal examination is benign without organomegaly and he is not icteric. Laboratory results reveal a white blood cell count of
Claim Credit 18,000/μL (18 x 109/L) with a predominance of neutrophils. The rest of his complete blood cell count is normal and his metabolic
Evaluation profile is normal, except for mildly elevated hepatic transaminases.

My Learning Plan Of the following, the MOST likely diagnosis in the patient is

A. brucellosis
B. Kawasaki disease
C. leptospirosis
D. tularemia

Incorrect View Peer Results

Correct Answer: C Average Percent Correct:  62.86%

The biphasic course of illness, manifesting with fever, headache, conjunctival suffusion, and myalgias
in the first phase, followed by recurrence of fever along with aseptic meningitis, rash, conjunctival
suffusion, and the history of potential contact with infected animals, is most characteristic of
leptospirosis, most commonly caused by Leptospira interrogans. Although asymptomatic infection is
common, the biphasic course described for the adolescent in the vignette is characteristic of the
anicteric form of leptospirosis, which accounts for 90% of cases of leptospirosis. The first phase of the
illness, characterized by fever and viral­like illness symptoms, has a wide spectrum of severity, and
correlates with the presence of the organism in the blood. The second phase of the illness in the anicteric form is the
immunologic phase of the illness, may not be present in some patients, and correlates with isolation of the organism in the
urine. Other less common manifestations of the anicteric form of leptospirosis include other central nervous system
manifestations, such as Guillain­Barré syndrome, radiculomyelitis, and encephalitis, and varying severity of renal dysfunction.
The manifestations of the anicteric form are self­limited and most patients recover within 2 weeks of the onset of symptoms.
Brucellosis is unlikely to present with a biphasic course of illness, conjunctival involvement, or rash. Kawasaki disease can
mimic leptospirosis, but does not typically have a biphasic course of illness and would be less likely in a 17­year­old patient.
Tularemia can present with a prolonged febrile illness, but most commonly presents with an ulceroglandular lesion with a site
of inoculation from a tick bite or a history of direct exposure to rodents.

A more severe icteric form of leptospirosis (Weil syndrome) occurs in about 10% of all patients with leptospirosis. This form is
characterized by the development of jaundice, azotemia, and hemorrhage, which usually becomes apparent 5 to 6 days into
the illness, along with persistent fever. Shock, circulatory collapse, and hemoglobinemia from hemolysis can lead to
significant renal dysfunction. Weil syndrome has been associated with a mortality rate of 5% to 10%. Leptospirosis is a
zoonotic illness, most commonly acquired through contact with the urine of infected animals. In the United States, dogs and
farm animals, including pigs and cattle, are the most important reservoirs of infection. Rats and other rodents can also serve
as reservoirs. People who work on farms (especially cattle and dairy and pig farms), abattoir (slaughterhouse) workers,
veterinarians, and field workers are at increased risk for infection. Recreational water exposures such as sporting events,
such as triathlons, boating, swimming, have also been reported.

http://2016.prepid.courses.aap.org/script/june?req=201612122127573242&status=submit 1/2
12/13/2016 June

PREP Pearls
The most common form of leptospirosis is the anicteric form, which presents with a biphasic illness featuring a flu­like viral
illness early, followed by an immune­mediated phase characterized by aseptic meningitis, rash, uveitis, and fever.

A more severe but less common form of leptospirosis is the icteric form, which is characterized by fever, jaundice,
hemorrhage, and renal failure.

Leptospirosis is a zoonotic illness, most commonly acquired through contact with the urine of infected animals.

American Board of Pediatrics Content Specification(s)
Recognize the clinical manifestations suggestive of leptospirosis

Know the modes of acquisition and epidemiology of leptospirosis

Suggested Readings
Desai S, van Treeck U, Lierz M, et al. Resurgence of field fever in a temperate country: an epidemic of leptospirosis among
seasonal strawberry harvesters in Germany in 2007. Clin Infect Dis. 2009;48(6):691­697. DOI:
http://dx.doi.org/10.1086/597036

Shapiro ED. Leptospira species (leptospirosis). In: Long SS, Pickering LK, Prober CG, eds. Principles and Practice of
Pediatric Infectious Diseases. 4th ed. Philadelphia, PA: Churchill Livingstone Elsevier; 2012:949­952.

Comment On This Question

Page 57 of 115

Contact Us
Links may open in separate window
Copyright © 2016   American Academy of Pediatrics. All rights reserved. Pop­up Blocker may need to be disabled

http://2016.prepid.courses.aap.org/script/june?req=201612122127573242&status=submit 2/2
12/13/2016 June

Welcome  mohammed alsaiary [ Logout ]

Home PREP Pearls FAQs My Bookmarks CME Information

Overview Home  > June

Editorial Board
June   Enter Keyword Search
January
Question View:   All (8) Jump to Question
February
Print this Page Add to my Bookmarks Page 58 of 115
March

April Assessment History Mode: Learner  Exam 

May ASSESSMENT PROGRESS:  Total Questions:  8  Questions Answered:  5  Correct Answers:  2


June

July You currently have 3 questions unanswered in this assessment

August

September
Question: 6
October
A 5­year­old child is being seen by her pediatrician after sustaining a bite on her hand when you receive a call about whether this
November child requires rabies post­exposure prophylaxis.

December Of the following, the mammal MOST likely to be at risk for transmission of rabies to this child is a

A. family dog
B. goat
C. rabbit

Claim Credit D. skunk
E. squirrel
Evaluation

My Learning Plan
Submit   Reset  

Page 58 of 115

Contact Us
Links may open in separate window
Copyright © 2016   American Academy of Pediatrics. All rights reserved. Pop­up Blocker may need to be disabled

http://2016.prepid.courses.aap.org/script/june?q­page=6&req=201612122127578242 1/1
12/13/2016 June

Welcome  mohammed alsaiary [ Logout ]

Home PREP Pearls FAQs My Bookmarks CME Information

Overview Home  > June

Editorial Board
June   Enter Keyword Search
January
Question View:   All (8) Jump to Question
February
Print this Page Add to my Bookmarks Page 58 of 115
March

April Assessment History Mode: Learner  Exam 

May ASSESSMENT PROGRESS:  Total Questions:  8  Questions Answered:  6  Correct Answers:  3


June

July You currently have 2 questions unanswered in this assessment

August

September
Question: 6
October
A 5­year­old child is being seen by her pediatrician after sustaining a bite on her hand when you receive a call about whether this
November child requires rabies post­exposure prophylaxis.

December Of the following, the mammal MOST likely to be at risk for transmission of rabies to this child is a

A. family dog
B. goat
C. rabbit

Claim Credit D. skunk
E. squirrel
Evaluation

My Learning Plan
Correct View Peer Results
Average Percent Correct:  90.00%

In the United States, all mammals are believed to be susceptible to rabies, but bats, raccoons, skunks,
and foxes are more likely to be infected than other animals. Coyotes, cattle, dogs, cats, goats, and
ferrets occasionally are infected. Bites of rodents (such as squirrels, mice, and rats) or lagomorphs
(rabbits, hares, and pikas) rarely require prophylaxis. Additional factors must be considered when
deciding if post­exposure prophylaxis (PEP) is required. An unprovoked attack may be more
suggestive of a rabid animal than bites that occur when trying to feed, pet, or handle an animal.
Properly immunized dogs, cats, and ferrets have only a minimal chance of developing rabies, however,
in rare instances, rabies has developed in properly immunized animals. The decision to immunize a potentially exposed
person to rabies depends upon the risk of rabies in a particular area for each species of animal (Table 1).

Exposure to rabies results from a break in the skin caused by the teeth of a rabid animal or from contamination of a scratch,
abrasion, open wound, or mucous membrane with saliva or other potentially infectious material (eg, brain tissue) from a rabid
animal (or person). The injury from a bat bite or scratch may be small and not readily evident, or the circumstances of contact
may preclude accurate recall (eg, bat in a room of a sleeping person or unattended child), therefore prophylaxis may be
indicated for situations in which a bat physically is present and a bite cannot be excluded, unless prompt testing of the bat
excludes rabies infection.

Post­exposure prophylaxis for rabies is recommended for all people bitten by wild mammalian carnivores or bats that are not
available for testing or by high risk domestic animals that may be infected. Prophylaxis should be initiated as soon as possible
after bites by known or suspected rabid animals. This consists of a regimen of 1 dose of rabies immune globulin (RIG) and 4
doses of human rabies vaccine over a 14­day period (Table 2). Rabies immune globulin and the first dose of rabies vaccine
should be given as soon as possible after exposure. Additional doses of rabies vaccine should be given on days 3, 7, and 14
after the first vaccination.

Immunosuppressive agents should not be administered during post­exposure therapy, unless essential for the treatment of
other conditions. Corticosteroids, other immunosuppressive agents, antimalarials, and immunosuppressive illnesses can
interfere with the development of active immunity after vaccination. When PEP is administered to an immunosuppressed
person, they should receive the current 4­dose vaccine schedule with an additional dose of vaccine on day 28. Furthermore, it
is especially important that a serum sample be tested for rabies antibody to ensure that an acceptable antibody response has
developed.

Pregnancy is not considered a contraindication to PEP because of the potential consequences of inadequately treated rabies
exposure. Several studies have shown no indication of increased incidence of abortion, premature births, or fetal
abnormalities associated with rabies vaccination. Rabies exposure in the mother is not an indication for pregnancy
termination.

http://2016.prepid.courses.aap.org/script/june?req=201612122128494337&status=submit 1/2
12/13/2016 June
People who have a history of serious hypersensitivity to components of rabies vaccine should be revaccinated with caution.
Optimally, the same vaccine (either human diploid cell rabies vaccine or purified chick embryo cell rabies vaccine) should be
used throughout the vaccination series. However, if severe allergic reactions occur, it may be advisable to switch to the
alternate vaccine to complete the series.

Human rabies immune globulin (HRIG) is administered only once, at the beginning of anti­rabies prophylaxis, to previously
unvaccinated persons. This will provide immediate antibodies until the body can respond to the vaccine by actively producing
antibodies of its own. If possible, the full dose of HRIG should be thoroughly infiltrated in the area around and into the
wounds. Any remaining volume should be injected intramuscularly at a site distant from vaccine administration. The HRIG
dose should never be administered in the same syringe or in the same anatomical site as the first vaccine dose. However,
subsequent doses of vaccine in the 4­dose series can be administered in the same anatomic location where the HRIG dose
was administered. If HRIG was not administered when vaccination was begun, it can be administered up to 7 days after the
administration of the first dose of vaccine. Beyond the seventh day, HRIG is not recommended because an antibody response
to the vaccine is presumed to have occurred. No more than the recommended dose should be administered because HRIG
can partially suppress active production of antibody. The recommended dose of HRIG is 20 IU/kg of body weight. This
formula is applicable to all age groups, including children.

* Required *
Take Survey  

PREP Pearls
All mammals are believed to be susceptible to rabies, but bats, raccoons, skunks, and foxes are more likely to be infected
than other animals. Coyotes, cattle, dogs, cats, and ferrets occasionally are infected. Bites of rodents (such as squirrels,
mice, and rats) or lagomorphs (rabbits, hares, and pikas) rarely require prophylaxis.

Post­exposure prophylaxis should be initiated as soon as possible after bites by known or suspected rabid animals.

Post­exposure prophylaxis consists of a regimen of 1 dose of rabies immune globulin and 4 doses of human rabies vaccine
over a 14­day period.

American Board of Pediatrics Content Specification(s)
Know the special products used for passive immunoprophylaxis for rabies

Know the indications and timing for passive immunoprophylaxis for rabies, in combination with active immunization

Suggested Readings
American Academy of Pediatrics. Rabies. In: Pickering LK, Baker CJ, Kimberlin DW, Long SS, eds. Red Book: 2012 Report of
the Committee on Infectious Diseases. 29th ed. Elk Grove Village, IL: American Academy of Pediatrics, 2012:600­607.

US Centers for Disease Control and Prevention. Rabies. US Centers for Disease Control and Prevention website.
http://www.cdc.gov/rabies/index.html. Updated September 24, 2013. http://www.cdc.gov/rabies/resources/contacts.html

Comment On This Question

Page 58 of 115

Contact Us
Links may open in separate window
Copyright © 2016   American Academy of Pediatrics. All rights reserved. Pop­up Blocker may need to be disabled

http://2016.prepid.courses.aap.org/script/june?req=201612122128494337&status=submit 2/2
12/13/2016 June

Welcome  mohammed alsaiary [ Logout ]

Home PREP Pearls FAQs My Bookmarks CME Information

Overview Home  > June

Editorial Board
June   Enter Keyword Search
January
Question View:   All (8) Jump to Question
February
Print this Page Add to my Bookmarks Page 59 of 115
March

April Assessment History Mode: Learner  Exam 

May ASSESSMENT PROGRESS:  Total Questions:  8  Questions Answered:  6  Correct Answers:  3


June

July You currently have 2 questions unanswered in this assessment

August

September
Question: 7
October
You are approached by a resident on the inpatient service who wants to discuss antibiotic therapy for a patient with impacted third
November molars (wisdom teeth). The area around one of these teeth is edematous, red, and tender. The patient reports intense pain radiating
to the throat. The cheek overlying the affected tooth is swollen, and the patient has dysphagia. The patient is febrile and reports she
December
has experienced chills. Cultures are obtained from around the affected tooth, and the microbiology lab reports that the anaerobic
culture has Gram­positive cocci growing. The resident expresses particular concern about potential antibiotic resistance.

Of the following, the MOST appropriate antibiotic to include in the patient’s treatment regimen is

A. amikacin

Claim Credit B. aztreonam

Evaluation C. ciprofloxacin
D. metronidazole
My Learning Plan
E. penicillin G

Submit   Reset  

Page 59 of 115

Contact Us
Links may open in separate window
Copyright © 2016   American Academy of Pediatrics. All rights reserved. Pop­up Blocker may need to be disabled

http://2016.prepid.courses.aap.org/script/june?q­page=7&req=201612122128501680 1/1
12/13/2016 June

Welcome  mohammed alsaiary [ Logout ]

Home PREP Pearls FAQs My Bookmarks CME Information

Overview Home  > June

Editorial Board
June   Enter Keyword Search
January
Question View:   All (8) Jump to Question
February
Print this Page Add to my Bookmarks Page 59 of 115
March

April Assessment History Mode: Learner  Exam 

May ASSESSMENT PROGRESS:  Total Questions:  8  Questions Answered:  7  Correct Answers:  3


June

July You currently have 1 questions unanswered in this assessment

August

September
Question: 7
October
You are approached by a resident on the inpatient service who wants to discuss antibiotic therapy for a patient with impacted third
November molars (wisdom teeth). The area around one of these teeth is edematous, red, and tender. The patient reports intense pain radiating
to the throat. The cheek overlying the affected tooth is swollen, and the patient has dysphagia. The patient is febrile and reports she
December
has experienced chills. Cultures are obtained from around the affected tooth, and the microbiology lab reports that the anaerobic
culture has Gram­positive cocci growing. The resident expresses particular concern about potential antibiotic resistance.

Of the following, the MOST appropriate antibiotic to include in the patient’s treatment regimen is

A. amikacin

Claim Credit B. aztreonam

Evaluation C. ciprofloxacin
D. metronidazole
My Learning Plan
E. penicillin G

Incorrect View Peer Results

Correct Answer: E Average Percent Correct:  80.00%

The isolated organism (a Gram­positive, anaerobic coccus) is a Peptostreptococcus species. Both
penicillin G and metronidazole are used to treat anaerobic infections, but anaerobic cocci are
sometimes highly resistant to metronidazole. Amikacin is used to treat Gram­negative infections, and
aminoglycosides as a class are not active against anaerobes. Aztreonam is an anti­pseudomonal β­
lactam antibiotic. Ciprofloxacin has good Gram­negative, but not substantial Gram­positive activity, and
its activity against anaerobes is unreliable.

The principles of management of patients with anaerobic infections include prevention of bacterial proliferation, through
changing the environment and antimicrobial therapy, and neutralization of toxins (eg, for gas gangrene). Important changes to
the environment of the anaerobic infection include increasing tissue oxygenation, eg, through improving blood circulation,
debriding necrotic tissue, draining pus, and alleviating obstruction. The topical application of oxygen­releasing compounds
and the use of hyperbaric oxygen therapy may improve the patient’s condition. Surgical intervention (drainage, debridement,
resection) is generally the most important and possibly the only required treatment.

Choosing antibiotics for anaerobic infections is often empiric and based primarily on the likely organisms and their expected in
vitro susceptibility pattern. The susceptibility of anaerobic bacteria to antimicrobial agents is summarized in the table (Table).

PREP Pearls
The principles of management of patients with anaerobic infections include prevention of bacterial proliferation, through
changing the environment and antimicrobial therapy, and neutralization of toxins.

Choosing antibiotics for anaerobic infections is often empiric and based primarily on the likely organisms and their expected
in vitro susceptibility pattern.

American Board of Pediatrics Content Specification(s)
Know that anaerobic cocci are generally susceptible to penicillins, cephalosporins, clindamycin and vancomycin, but are
sometimes highly resistant to metronidazole

http://2016.prepid.courses.aap.org/script/june?req=201612122130029182&status=submit 1/2
12/13/2016 June
Plan the treatment of a patient with suspected anaerobic infection according to clinical site, including the recognition of
antibiotic resistance (eg., metronidazole, clindamycin)

Suggested Readings
Brook I, Long SS. Anaerobic bacteria: classification, normal flora, and clinical concepts. In: Long SS, Pickering LK, Prober
CG, eds. Principles and Practice of Pediatric Infectious Diseases. 4th ed. New York, NY: Saunders Elsevier; 2012:958­966.

Brook I. Bacteroides and Prevotella species and other anaerobic Gram­negative bacilli. In: Long SS, Pickering LK, Prober
CG, eds. Principles and Practice of Pediatric Infectious Diseases. 4th ed. New York, NY: Saunders Elsevier; 2012:982­985.

Feingold AR, Meislich D. Anaerobic Gram­positive, nonsporulating bacilli (including Actinomycosis). In: Long SS, Pickering
LK, Prober CG, eds. Principles and Practice of Pediatric Infectious Diseases. 4th ed. New York, NY: Saunders Elsevier;
2012:990­992.

Comment On This Question

Page 59 of 115

Contact Us
Links may open in separate window
Copyright © 2016   American Academy of Pediatrics. All rights reserved. Pop­up Blocker may need to be disabled

http://2016.prepid.courses.aap.org/script/june?req=201612122130029182&status=submit 2/2
12/13/2016 June

Welcome  mohammed alsaiary [ Logout ]

Home PREP Pearls FAQs My Bookmarks CME Information

Overview Home  > June

Editorial Board
June   Enter Keyword Search
January
Question View:   All (8) Jump to Question
February
Print this Page Add to my Bookmarks Page 60 of 115
March

April Assessment History Mode: Learner  Exam 

May ASSESSMENT PROGRESS:  Total Questions:  8  Questions Answered:  7  Correct Answers:  3


June

July You currently have 1 questions unanswered in this assessment

August

September
Question: 8
October
You are asked to evaluate a 3 year­old­girl who is hospitalized with a diagnosis of meningitis. Her mother reports that she has had a
November 6­week history of fever associated with intermittent confusion, irritability, headache, vomiting, and decreased activity. She was
admitted after experiencing a 5­min generalized tonic­clonic seizure at home. According to her mother, she has been healthy prior to
December
the onset of this illness without significant medical problems or infections. Her mother states the family immigrated to the United
States from South America 3 months ago. On physical examination, her temperature is 38.7°C, pulse is 120 beats/min, respiratory
rate is 40 breaths/min, and blood pressure is 80/50 mm Hg. She is at the 25th percentile for weight. She is sleepy but easily
arousable, and without focal findings on examination. She has no adenopathy, hepatosplenomegaly, or rashes. Computed
tomography of the brain with and without contrast reveals mild hydrocephalus and basilar enhancement. Results of cerebrospinal
fluid examination reveal a white blood cell count of 300/µL (90% lymphocytes), protein of .35 g/dL (3.5 g/L), and glucose of 15 mg/dL
Claim Credit (.8 mmol/L).

Evaluation Of the following, the MOST likely causative agent of meningitis in this girl is
My Learning Plan
A. Cytomegalovirus
B. Enterovirus
C. Mycobacterium tuberculosis
D. Treponema pallidum

Submit   Reset  

Page 60 of 115

Contact Us
Links may open in separate window
Copyright © 2016   American Academy of Pediatrics. All rights reserved. Pop­up Blocker may need to be disabled

http://2016.prepid.courses.aap.org/script/june?q­page=8&req=201612122130034650 1/1
12/13/2016 June

Welcome  mohammed alsaiary [ Logout ]

Home PREP Pearls FAQs My Bookmarks CME Information

Overview Home  > June

Editorial Board
June   Enter Keyword Search
January
Question View:   All (8) Jump to Question
February
Print this Page Add to my Bookmarks Page 60 of 115
March

April Assessment History Mode: Learner  Exam 

May ASSESSMENT PROGRESS:  Total Questions:  8  Questions Answered:  8  Correct Answers:  4


June

July You currently have 2 required survey(s) that are incomplete. Please complete the survey(s) for the following
question(s) on this assessment:  Question 3, Question 6
August

September

October Question: 8
November You are asked to evaluate a 3 year­old­girl who is hospitalized with a diagnosis of meningitis. Her mother reports that she has had a
6­week history of fever associated with intermittent confusion, irritability, headache, vomiting, and decreased activity. She was
December
admitted after experiencing a 5­min generalized tonic­clonic seizure at home. According to her mother, she has been healthy prior to
the onset of this illness without significant medical problems or infections. Her mother states the family immigrated to the United
States from South America 3 months ago. On physical examination, her temperature is 38.7°C, pulse is 120 beats/min, respiratory
rate is 40 breaths/min, and blood pressure is 80/50 mm Hg. She is at the 25th percentile for weight. She is sleepy but easily
arousable, and without focal findings on examination. She has no adenopathy, hepatosplenomegaly, or rashes. Computed
tomography of the brain with and without contrast reveals mild hydrocephalus and basilar enhancement. Results of cerebrospinal
Claim Credit fluid examination reveal a white blood cell count of 300/µL (90% lymphocytes), protein of .35 g/dL (3.5 g/L), and glucose of 15 mg/dL
(.8 mmol/L).
Evaluation
Of the following, the MOST likely causative agent of meningitis in this girl is
My Learning Plan
A. Cytomegalovirus
B. Enterovirus
C. Mycobacterium tuberculosis
D. Treponema pallidum

Correct View Peer Results
Average Percent Correct:  95.71%

The girl in the vignette has chronic meningitis, arbitrarily defined by the presence of persistent signs
and symptoms of meningeal irritation and cerebrospinal fluid (CSF) pleocytosis for at least 4 weeks.
There are many infectious and noninfectious causes of chronic meningitis. Of the agents listed as
potential agents for the girl in the vignette, infection with Mycobacterium tuberculosis is the most likely
cause of meningitis. The history of possible exposure related to her immigrant status, the gradual onset
of symptoms, and the characteristic CSF abnormalities (lymphocytic pleocytosis, markedly elevated
protein, and marked hypoglycorrhachia) are all most consistent with tuberculous meningitis. In addition,
the computed tomography findings of hydrocephalus and basilar meningitis, while nonspecific, are consistent with tuberculous
meningitis. Although central nervous system (CNS) tuberculosis may present with intracranial tuberculoma and/or brain
abscess, chronic meningitis, which often is basilar, is the most common presentation of CNS tuberculosis in children, which
most commonly occurs between 6 months and 4 years of age. Infection with Treponema pallidum can result in chronic
meningitis, but the infection is usually acquired congenitally and CNS features are usually accompanied by other clinical
manifestations such as lymphadenopathy, hepatosplenomegaly, mucocutaneous lesions, and bony abnormalities. In addition,
the CSF profile is that of pleocytosis with elevated protein and normal glucose. Cytomegalovirus can cause chronic
meningitis, but almost exclusively in immunocompromised patients, including patients infected with HIV. The CSF in such
patients shows a mild pleocytosis with a mildly elevated protein and normal glucose. Chronic persistent enteroviral infection is
well documented in patients with agammaglobulinemia, which presents early in life with bacterial and opportunistic infections
and would be unlikely in the girl in the vignette.

Most patients with chronic meningitis have a gradual onset of symptoms, which often include fever, headache, altered mental
status, and vomiting. In about one­third of cases of chronic meningitis, a precise etiology is not found. The differential
diagnosis is very broad, but can be divided into entities that are more likely to be found in a geographic locale and thus
related to an increased risk of exposure (Table 1), or more likely to cause chronic meningitis in patients with underlying
predisposing conditions (Table 2). Parameningeal infections, such as brain abscess, viral encephalitis, subdural empyema,
and sinusitis, also can manifest as chronic meningitis. The most common noninfectious causes of chronic meningitis are listed
in Table 3.

http://2016.prepid.courses.aap.org/script/june?req=201612122131067933&status=submit 1/2
12/13/2016 June
In addition to the specific clinical features associated with chronic meningitis, clinical manifestations not related to the CNS
can help to differentiate between causes. These can include skin, pulmonary, and eye manifestations (Table 4).

Most causes of chronic meningitis result in a similar CSF profile, which includes a mild to moderate pleocytosis with a
lymphocytic predominance, low or normal glucose, and elevated protein concentrations. Causes of chronic meningitis with a
neutrophilic predominance in the CSF include infections with Candida, Aspergillus, and Nocardia species, and early in the
course of Mycobacterium tuberculosis infection. A neutrophilic predominance in the CSF can also occur in parameningeal
infections and with many of the noninfectious causes of chronic meningitis, such as systemic lupus erythematosus, chemical
meningitis, and drug­related meningitis (ibuprofen, isoniazid). The differential diagnosis of an eosinophilic chronic meningitis
is summarized in Table 5.

PREP Pearls
Chronic meningitis can be caused by a wide variety of infectious agents, parameningeal infections, and noninfectious
entities.

An exposure history and presence of underlying medical conditions are important considerations in the generation of the
differential diagnosis of chronic meningitis.

The clinical and cerebrospinal fluid findings in children with chronic meningitis are similar regardless of the specific cause.

American Board of Pediatrics Content Specification(s)
Formulate a differential diagnosis in a patient with subacute/chronic meningitis, including infectious and noninfectious
entities

Recognize the clinical manifestations of subacute/chronic meningitis

Suggested Readings
Swanson D, Harrison CJ. Chronic meningitis. In: Long SS, Pickering LK, Prober CG, eds. Principles and Practice of Pediatric
Infectious Diseases. 4th ed. Philadelphia, PA: Saunders Elsevier; 2012: 279­286.

van Well GTJ, Paes BF, Terwee CB, et al. Twenty years of pediatric tuberculous meningitis: a retrospective cohort study in the
western cape of South Africa. Pediatrics. 2009;123(1):e1­e8. DOI: http://dx.doi.org/10.1542/peds.2008­1353

Comment On This Question

Page 60 of 115

Contact Us
Links may open in separate window
Copyright © 2016   American Academy of Pediatrics. All rights reserved. Pop­up Blocker may need to be disabled

http://2016.prepid.courses.aap.org/script/june?req=201612122131067933&status=submit 2/2
12/13/2016 July

Welcome  mohammed alsaiary [ Logout ]

Home PREP Pearls FAQs My Bookmarks CME Information

Overview Home  > July

Editorial Board
July   Enter Keyword Search
January
Question View:   All (8) Jump to Question
February
Print this Page Add to my Bookmarks Page 62 of 115
March

April Assessment History Mode: Learner  Exam 

May ASSESSMENT PROGRESS:  Total Questions:  8  Questions Answered:  0  Correct Answers:  0


June

July You currently have 8 questions unanswered in this assessment

August

September
Question: 1
October
A 12­year­old boy has had fever, chills, and sore throat for 2 days. On further history, he states that 1 week ago, his dog brought
November home a dead rabbit, and he skinned the carcass before discarding it in the trash. On physical examination, his temperature is 39.2°C
and he appears tired but not toxic. He has bilateral shotty and tender cervical adenopathy, a diffusely red posterior pharynx, enlarged
December
red tonsils, and his spleen tip is palpable at the left costal margin. The remainder of the examination is unremarkable.

Of the following, the systemic treatment MOST likely to result in a permanent cure of this patient’s illness is

A. amoxicillin
B. ceftriaxone
Claim Credit C. doxycycline
Evaluation D. gentamicin

My Learning Plan E. meropenem

Submit   Reset  

Page 62 of 115

Contact Us
Links may open in separate window
Copyright © 2016   American Academy of Pediatrics. All rights reserved. Pop­up Blocker may need to be disabled

http://2016.prepid.courses.aap.org/script/july?q­page=1&req=201612122131231527 1/1
12/13/2016 July

Welcome  mohammed alsaiary [ Logout ]

Home PREP Pearls FAQs My Bookmarks CME Information

Overview Home  > July

Editorial Board
July   Enter Keyword Search
January
Question View:   All (8) Jump to Question
February
Print this Page Add to my Bookmarks Page 62 of 115
March

April Assessment History Mode: Learner  Exam 

May ASSESSMENT PROGRESS:  Total Questions:  8  Questions Answered:  1  Correct Answers:  0


June

July You currently have 7 questions unanswered in this assessment

August

September
Question: 1
October
A 12­year­old boy has had fever, chills, and sore throat for 2 days. On further history, he states that 1 week ago, his dog brought
November home a dead rabbit, and he skinned the carcass before discarding it in the trash. On physical examination, his temperature is 39.2°C
and he appears tired but not toxic. He has bilateral shotty and tender cervical adenopathy, a diffusely red posterior pharynx, enlarged
December
red tonsils, and his spleen tip is palpable at the left costal margin. The remainder of the examination is unremarkable.

Of the following, the systemic treatment MOST likely to result in a permanent cure of this patient’s illness is

A. amoxicillin
B. ceftriaxone
Claim Credit C. doxycycline
Evaluation D. gentamicin

My Learning Plan E. meropenem

Incorrect View Peer Results
Correct Answer: D Average Percent Correct:  67.69%

This child has oropharyngeal tularemia, and gentamicin is the most likely choice to result in complete
cure. It is customary to provide 7 to 10 days of therapy, based on clinical response. Streptomycin is a
reasonable alternative, but may be difficult to obtain. Amoxicillin and most β­lactam agents, as well as
carbapenems, have little activity against the causative agent, Francisella tularensis. Although
ceftriaxone has good in vitro activity against the organism, unusual for a β­lactam agent, a small
treatment series suggested lack of in vivo efficacy. Tetracyclines have been used for the treatment of
tularemia, but have an unacceptable relapse rate. In some parts of the world, fluoroquinolones are
used, but there is limited data for the use of fluoroquinolones in children.

In the United States in 2012, Missouri, Kansas, Arkansas, and Oklahoma accounted for 88 of the 149 tularemia cases
reported to the US Centers for Disease Control and Prevention. The infection is usually acquired by direct inoculation through
the skin or by inhalation, with the mode of entry often determining the clinical signs and symptoms. Rabbits and squirrels are
the most common animal source, although domestic cats and other animals have transmitted disease to humans via bites or
inoculation into areas of skin disruption. Tick bites are a prominent means of human disease, with Dermacentor andersoni
(wood tick), Dermacentor variabilis (dog tick), and Amblyomma americanum (lone star tick) primarily implicated in the United
States. Aerosolization of organisms can occur by manipulation of infected animals such as in the vignette or by other means
such as clearing brush or lawn mowing. Microbiology laboratory workers represent another risk group for aerosolized
exposure, and care should be taken to notify the microbiology laboratory if cultures are submitted from suspected tularemia
cases.

Most children with tularemia will present with fever, chills, malaise, mild abdominal pain, and loose stools after an incubation
period of 2 to 10 days, though incubation up to 3 weeks has been reported. Further clinical findings usually can be grouped
into 1 of 6 clinical syndromes, although almost every organ system can be affected in tularemia.

Ulceroglandular tularemia is characterized, in addition to the findings previously stated, by regional lymphadenitis related to a
papule at the inoculation site. This papule eventually will rupture and drain, resulting in a cutaneous ulcer. Glandular tularemia
lacks this ulcer marking the inoculation site. Oropharyngeal tularemia is the result of inhalation or ingestion of contaminated
material (eg, animal skin, food, or water), giving rise to a relatively nonspecific tonsillopharyngitis. Oral ulcers may be present.
Oculoglandular tularemia comes from inoculation onto the conjunctival surface, resulting in a nodular conjunctivitis and
preauricular adenitis. Typhoidal tularemia has few or no localizing findings, and individuals may appear septic. Pneumonic
tularemia, such as might be seen in a laboratory worker, is a nonspecific pneumonitis and likely has the highest mortality of
any of the clinical forms of tularemia. Of these, ulceroglandular or the glandular form is the most common presentation in

http://2016.prepid.courses.aap.org/script/july?req=201612122136217002&status=submit 1/2
12/13/2016 July
adults and children. Successful management of tularemia requires careful historical questioning, especially because the
preferred treatments seldom are given as empiric therapy for pharyngitis, sepsis, pneumonia, or musculoskeletal infections in
children.

PREP Pearls
Tularemia is strongly associated with animal exposure, especially to rabbits and squirrels, but also to other rodents and to
domestic animals such as cats.

The usual mode of transmission is via tick bite or contact with infected animal tissue.

In the United States, tularemia is most often seen in south central regions of the country.

American Board of Pediatrics Content Specification(s)
Understand the epidemiology and modes of transmission of tularemia (tick, respiratory droplet, animal contact)

Recognize clinical manifestations of tularemia

Suggested Readings
Rubin LG. Francisella tularensis (tularemia). In: Long SS, Pickering LK, Prober CG, eds. Principles and Practice of Pediatric
Infectious Diseases. 4th ed. Philadelphia, PA: Saunders Elsevier; 2012:897­899.

Schutze GE. Tularemia. In: Cherry JD, Harrison GJ, Kaplan SL, Steinbach WJ, eds. Feigin and Cherry’s Textbook of Pediatric
Infectious Diseases. 7th ed. Philadelphia, PA: Saunders Elsevier; 2014:1657­1665.

US Centers for Disease Control and Prevention. Summary of notifiable diseases – United States, 2012. MMWR Morbid Mortal
Wkly Rep. 2014;61(53):1­114.

Comment On This Question

Page 62 of 115

Contact Us
Links may open in separate window
Copyright © 2016   American Academy of Pediatrics. All rights reserved. Pop­up Blocker may need to be disabled

http://2016.prepid.courses.aap.org/script/july?req=201612122136217002&status=submit 2/2
12/13/2016 July

Welcome  mohammed alsaiary [ Logout ]

Home PREP Pearls FAQs My Bookmarks CME Information

Overview Home  > July

Editorial Board
July   Enter Keyword Search
January
Question View:   All (8) Jump to Question
February
Print this Page Add to my Bookmarks Page 63 of 115
March

April Assessment History Mode: Learner  Exam 

May ASSESSMENT PROGRESS:  Total Questions:  8  Questions Answered:  1  Correct Answers:  0


June

July You currently have 7 questions unanswered in this assessment

August

September
Question: 2
October
A 9­week­old female infant is admitted to the pediatric intensive care unit presenting with fever, grunting, and respiratory distress.
November She was born at term during a normal spontaneous vaginal delivery. At admission, a lumbar puncture was performed with the
following results:
December

White blood cell count, 1,200/µL (1.2 x 109/L) (differential: 83% segmented neutrophils, 9% monocytes, 8% eosinophils)
Protein, 272 mg/dL
Glucose, less than 10 mg/dL (0.6 mmol/L)

The Gram stain of the patient’s cerebrospinal fluid is shown in Figure 1a.
Claim Credit
Of the following, the reagent MOST likely giving the purple color to the organisms in the Gram stain is
Evaluation
A. alcohol
My Learning Plan
B. carbol­fuchsin
C. crystal violet
D. methylene blue
E. safranin

Submit   Reset  

Page 63 of 115

Contact Us
Links may open in separate window
Copyright © 2016   American Academy of Pediatrics. All rights reserved. Pop­up Blocker may need to be disabled

http://2016.prepid.courses.aap.org/script/july?q­page=2&req=201612122136221690 1/1
12/13/2016 July

Welcome  mohammed alsaiary [ Logout ]

Home PREP Pearls FAQs My Bookmarks CME Information

Overview Home  > July

Editorial Board
July   Enter Keyword Search
January
Question View:   All (8) Jump to Question
February
Print this Page Add to my Bookmarks Page 63 of 115
March

April Assessment History Mode: Learner  Exam 

May ASSESSMENT PROGRESS:  Total Questions:  8  Questions Answered:  2  Correct Answers:  0


June

July You currently have 6 questions unanswered in this assessment

August

September
Question: 2
October
A 9­week­old female infant is admitted to the pediatric intensive care unit presenting with fever, grunting, and respiratory distress.
November She was born at term during a normal spontaneous vaginal delivery. At admission, a lumbar puncture was performed with the
following results:
December

White blood cell count, 1,200/µL (1.2 x 109/L) (differential: 83% segmented neutrophils, 9% monocytes, 8% eosinophils)
Protein, 272 mg/dL
Glucose, less than 10 mg/dL (0.6 mmol/L)

The Gram stain of the patient’s cerebrospinal fluid is shown in Figure 1a.
Claim Credit
Of the following, the reagent MOST likely giving the purple color to the organisms in the Gram stain is
Evaluation
A. alcohol
My Learning Plan
B. carbol­fuchsin
C. crystal violet
D. methylene blue
E. safranin

Incorrect View Peer Results
Correct Answer: C Average Percent Correct:  83.08%

The preferred response is crystal violet. The Gram stain of the cerebrospinal fluid (CSF) from the infant
in the vignette (Figure 1b) reveals white blood cells and Gram­positive cocci in pairs, with some
intracellular organisms seen.

The bacterial culture grew group B streptococcus (Streptococcus agalactiae). The Gram stain is named
after the Danish microbiologist, Dr. Hans Christian Gram, who devised the method in 1884. This simple
staining system divides most bacteria into 2 groups. The staining procedure is central in microbiology,
as well as the clinical practice of pediatrics. The Gram staining depends upon the ability of certain bacteria (the Gram­
positives) to retain a complex purple dye, crystal violet; the stain then is modified by potassium iodine and decolorized with a
brief alcohol wash. Gram­negative bacteria do not retain the dye and can therefore be counterstained after the alcohol wash
with a red dye, safranin. This distinction turns out to be correlated with fundamental differences in the cell envelope of the 2
classes of bacteria. Carbol­fuchsin is the first dye used in an acid fast stain; this stain is then decolorized with acid alcohol.
The final step is a counterstain with methylene blue. Acid fast organisms remain red, while all others become blue.

A rapid and accurate diagnosis of bacterial meningitis is imperative because of the aggressive course, serious nature, and
high likelihood of morbidity and mortality associated with the condition. Currently, the gold standard for the diagnosis of
bacterial meningitis is CSF culture. Frequently, antibiotics are given before the CSF is collected, reducing the diagnostic value
of culture. The examination of CSF fluid, readily available within an hour, is utilized to make clinical decisions such as the
decision to commence antimicrobial agents. Parameters routinely examined in a CSF sample are listed in Table 1; normal
values of the CSF protein, glucose, white blood cell (WBC) count, and opening pressure are in Table 2.

Results of the CSF Gram stain are available rapidly after admission of a child suspected of meningitis. This simple test can be
viewed by a clinical caregiver in the microbiology laboratory. If bacteria are visualized on the Gram stain, the presumptive
diagnosis and etiology are known. Figure 2 is a Gram stain of Haemophilus influenzae type b; note the small Gram­negative
rods. Often, these small, Gram­negative rods may be faint pink and difficult to visualize on a stain. Figure 3 and Figure 4 are
Gram stains of the same organism, Neisseria meningitidis. In Figure 3, the bacteria appear purple and may be interpreted as
Gram­positive cocci. This stain was inadequately under­decolorized and the bacteria retained the purple dye. Figure 4
illustrates intracellular Gram­negative diplococci, a proper appearance of N meningitidis. These 2 stains illustrate the need for

http://2016.prepid.courses.aap.org/script/july?req=201612122137074503&status=submit 1/2
12/13/2016 July
caution in not overinterpreting the specimen. The culture is the gold standard for diagnosis of the meningitis. In Figure 5,
Gram­positive organisms are very abundant and in pairs. This figure is a stain from a culture of spinal fluid that isolated
Streptococcus pneumoniae.

PREP Pearls
The probability of seeing bacteria on a Gram stain in a patient with meningitis depends on number of organisms present:
25% of smears are positive with less than 103 colony­forming units (CFU)/mL, 60% of smears are positive with 103 to 105
CFU/mL; 97% of smears are positive with greater than 105 CFU/mL.

The examination of cerebrospinal fluid, readily available within an hour, is utilized to make clinical decisions such as
whether to commence antimicrobial agents.

The culture is the gold standard for diagnosis of the meningitis.

American Board of Pediatrics Content Specification(s)
Know which laboratory studies are helpful in defining etiologic agents causing meningitis

Interpret Gram stains of cerebrospinal fluid from patients with meningitis

Suggested Readings
Arcara KM. Blood chemistries and body fluids. In: Tschudy MM, Arcara KM, eds. The Harriet Lane Handbook. 19th ed.
Philadelphia, PA: Mosby Elsevier; 2012:639­650.

Chadwick SL, Wilson JW, Levin JE, Martin JM. Cerebrospinal fluid characteristics of infants who present to the emergency
department with fever: establishing normal values by week of age. Pediatr Infect Dis J. 2011;30(4):e63­e67. DOI:
http://dx.doi.org/10.1097/INF.0b013e31820ad2ba

Kim KS. Bacterial meningitis beyond the neonatal period. In: Cherry JD, Harrison GJ, Kaplan SL, Steinbach WJ, Hotez PJ,
eds. Feigin and Cherry’s Textbook of Pediatric Infectious Diseases. 7th ed. Philadelphia, PA: Saunders Elsevier; 2014:425­
461.

Smith PB, Garges HP, Cotton CM, Walsh TJ, Clark RH, Benjamin DK Jr. Meningitis in preterm neonates: importance of
cerebrospinal fluid parameters. Am J Perinatol. 2008;25(7):421­426. DOI: http://dx.doi.org/10.1055/s­0028­1083839

Comment On This Question

Page 63 of 115

Contact Us
Links may open in separate window
Copyright © 2016   American Academy of Pediatrics. All rights reserved. Pop­up Blocker may need to be disabled

http://2016.prepid.courses.aap.org/script/july?req=201612122137074503&status=submit 2/2
12/13/2016 July

Welcome  mohammed alsaiary [ Logout ]

Home PREP Pearls FAQs My Bookmarks CME Information

Overview Home  > July

Editorial Board
July   Enter Keyword Search
January
Question View:   All (8) Jump to Question
February
Print this Page Add to my Bookmarks Page 64 of 115
March

April Assessment History Mode: Learner  Exam 

May ASSESSMENT PROGRESS:  Total Questions:  8  Questions Answered:  2  Correct Answers:  0


June

July You currently have 6 questions unanswered in this assessment

August

September
Question: 3
October
You have decided to investigate whether or not the mean waiting time in the infectious diseases outpatient clinic exceeds 15
November minutes. In a random sample of 20 patients who were seen in the clinic on the first Monday of the month, the mean wait time from
arrival to being seen by the physician was 18 minutes.
December
Which of the following statistical tests should be used to determine if the mean wait time observed was significantly different from the
expected target wait time of 15 minutes?

A. chi­square test
B. Fisher's exact test
Claim Credit C. Student’s t­test
Evaluation D. Wilcoxon rank­sum test

My Learning Plan

Submit   Reset  

Page 64 of 115

Contact Us
Links may open in separate window
Copyright © 2016   American Academy of Pediatrics. All rights reserved. Pop­up Blocker may need to be disabled

http://2016.prepid.courses.aap.org/script/july?q­page=3&req=201612122137082472 1/1
12/13/2016 July

Welcome  mohammed alsaiary [ Logout ]

Home PREP Pearls FAQs My Bookmarks CME Information

Overview Home  > July

Editorial Board
July   Enter Keyword Search
January
Question View:   All (8) Jump to Question
February
Print this Page Add to my Bookmarks Page 64 of 115
March

April Assessment History Mode: Learner  Exam 

May ASSESSMENT PROGRESS:  Total Questions:  8  Questions Answered:  3  Correct Answers:  0


June

July You currently have 5 questions unanswered in this assessment

August

September
Question: 3
October
You have decided to investigate whether or not the mean waiting time in the infectious diseases outpatient clinic exceeds 15
November minutes. In a random sample of 20 patients who were seen in the clinic on the first Monday of the month, the mean wait time from
arrival to being seen by the physician was 18 minutes.
December
Which of the following statistical tests should be used to determine if the mean wait time observed was significantly different from the
expected target wait time of 15 minutes?

A. chi­square test
B. Fisher's exact test
Claim Credit C. Student’s t­test
Evaluation D. Wilcoxon rank­sum test

My Learning Plan

Incorrect View Peer Results
Correct Answer: C Average Percent Correct:  46.15%

The Student's t­test is used to compare continuous variables of normally distributed data. It is used to
determine whether the mean or average of a population significantly differs from a specific value or
hypothesized mean of another population. In this example, the hypothesized mean was the proposed
15 minute wait time, and the observed mean in the population was the 18 minute mean wait time. A
Student's t­test would therefore be used to determine if the numerical mean difference was significant.
The Wilcoxon rank­sum test is an appropriate alternative method to the t­test for comparing medians in
non­normally distributed instead of means.

Chi­square tests are used to determine differences in categorical variables or values between groups and non­normally
distributed data. It is not applicable to percentages or means, and therefore not appropriate to analyze the data in the
vignette. Typically, data appropriate for the chi­square test are displayed in a 2 x 2 contingency table so it can be ascertained
whether or not an association exists between an exposure and an outcome or disease. For the chi­square to be valid, at least
5 would be the expected frequency in each category or cell. In general, if less than 5 is expected in one of the cells in the 2 x
2 contingency table, then Fisher's exact test should be used to assess the differences in categorical values for a small
expected frequency.

* Required *
Take Survey  

PREP Pearls
The chi­square test is for categorical variables, where the expected values for all cells is over 5; use Fisher's exact test for
groups with smaller expected frequency.

Student's t­test is for continuous variables and analysis of means between groups.

The Wilcoxon rank­sum test can be used to analyze differences of medians between groups.

American Board of Pediatrics Content Specification(s)
Understand the appropriate use of the chi­square test versus a t­test

http://2016.prepid.courses.aap.org/script/july?req=201612122137437316&status=submit 1/2
12/13/2016 July
Interpret the results of chi­square tests

Suggested Readings
Dawson B, Trapp RG. Decisions about proportions in two independent groups. Basic and Clinical Biostatistics. 4th ed. New
York, NY: McGraw­Hill; 2004:146­153.

Shapiro ED, Baltimore RS. Epidemiology and biostatistics of infectious diseases. In: Cherry J, Demmler­Harrison GJ, Kaplan
SL, Steinbach WJ, Hotez P, eds. Feigin and Cherry's Textbook of Pediatric Infectious Diseases. 7th ed. Philadelphia, PA:
Saunders Elsevier; 2013:95­128.

Comment On This Question

Page 64 of 115

Contact Us
Links may open in separate window
Copyright © 2016   American Academy of Pediatrics. All rights reserved. Pop­up Blocker may need to be disabled

http://2016.prepid.courses.aap.org/script/july?req=201612122137437316&status=submit 2/2
12/13/2016 July

Welcome  mohammed alsaiary [ Logout ]

Home PREP Pearls FAQs My Bookmarks CME Information

Overview Home  > July

Editorial Board
July   Enter Keyword Search
January
Question View:   All (8) Jump to Question
February
Print this Page Add to my Bookmarks Page 65 of 115
March

April Assessment History Mode: Learner  Exam 

May ASSESSMENT PROGRESS:  Total Questions:  8  Questions Answered:  3  Correct Answers:  0


June

July You currently have 5 questions unanswered in this assessment

August

September
Question: 4
October
You are asked to evaluate a 17­year­old adolescent boy who is admitted to the hospital with a 1­month history of fever, headache,
November intermittent confusion, and blurred vision. According to the available medical records, he was diagnosed with acute HIV infection 6
months ago at an out­of­state hospital, but refused further evaluation or treatment at that time and was lost to follow­up. During your
December
evaluation, you note that he has had low­grade fevers since he was admitted 2 days ago and appears to be intermittently confused.
He has no focal neurologic findings except his disc margins are blurred. His cerebrospinal fluid (CSF) findings reveal increased
opening pressure, 20 lymphocytes/high power field, protein of 45 mg/dL, and glucose of 50 mg/dL (2.8 mmol/L). Latex agglutination
for Cryptococcus from the blood and CSF is positive.

Of the following, dissemination of this infection to the central nervous system is MOST likely the result of deficient
Claim Credit
A. CD4+ lymphocytes
Evaluation
B. immunoglobulin A
My Learning Plan
C. polymorphonuclear leukocytes
D. terminal complement component

Submit   Reset  

Page 65 of 115

Contact Us
Links may open in separate window
Copyright © 2016   American Academy of Pediatrics. All rights reserved. Pop­up Blocker may need to be disabled

http://2016.prepid.courses.aap.org/script/july?q­page=4&req=201612122137444504 1/1
12/13/2016 July

Welcome  mohammed alsaiary [ Logout ]

Home PREP Pearls FAQs My Bookmarks CME Information

Overview Home  > July

Editorial Board
July   Enter Keyword Search
January
Question View:   All (8) Jump to Question
February
Print this Page Add to my Bookmarks Page 65 of 115
March

April Assessment History Mode: Learner  Exam 

May ASSESSMENT PROGRESS:  Total Questions:  8  Questions Answered:  4  Correct Answers:  1


June

July You currently have 4 questions unanswered in this assessment

August

September
Question: 4
October
You are asked to evaluate a 17­year­old adolescent boy who is admitted to the hospital with a 1­month history of fever, headache,
November intermittent confusion, and blurred vision. According to the available medical records, he was diagnosed with acute HIV infection 6
months ago at an out­of­state hospital, but refused further evaluation or treatment at that time and was lost to follow­up. During your
December
evaluation, you note that he has had low­grade fevers since he was admitted 2 days ago and appears to be intermittently confused.
He has no focal neurologic findings except his disc margins are blurred. His cerebrospinal fluid (CSF) findings reveal increased
opening pressure, 20 lymphocytes/high power field, protein of 45 mg/dL, and glucose of 50 mg/dL (2.8 mmol/L). Latex agglutination
for Cryptococcus from the blood and CSF is positive.

Of the following, dissemination of this infection to the central nervous system is MOST likely the result of deficient
Claim Credit
A. CD4+ lymphocytes
Evaluation
B. immunoglobulin A
My Learning Plan
C. polymorphonuclear leukocytes
D. terminal complement component

Correct View Peer Results
Average Percent Correct:  96.92%

The patient in the vignette has cryptococcal meningoencephalitis, most commonly caused by the
fungus Cryptococcus neoformans, which accounts for 95% of cases of cryptococcosis in the United
States. This organism most often causes infection in the setting of HIV infection and AIDS and very
rarely affects immunocompetent individuals. Although the incidence of infection with this organism in
AIDS patients has decreased significantly since the widespread use of active antiretroviral therapy
(ART), it continues to be a formidable problem among those who are not on ART or where ART is not
available. Host factors are the most important determinants of cryptococcal infection, and
dissemination of the infection to the central nervous system (CNS) is the result of altered T­cell immunity and defective
phagocytosis. Thus, CD4+ lymphocytes are most essential in controlling infection, and individuals with CD4+ lymphocyte
counts less than 100/mm3 are most at risk of CNS dissemination. Dissemination to the CNS, soft tissue, and bone is thought
to represent altered or deficient T­cell immunity. The lung is the site of primary infection with this fungus. Neutrophils appear
to play a role in controlling the infection in the lung, where the infection is most often asymptomatic. Although
immunoglobulins and complement enhance opsonization of cryptococci, they do not appear to be vital to the host defense of
these organisms.

Cryptococcal infections related to altered T­cell immunity have also been documented in non­HIV infected patients. These
include patients with lymphoma, lymphoproliferative disorders, sarcoidosis, and patients receiving corticosteroid and other
immunosuppressive treatments. For reasons that are not entirely clear, cryptococcal infections are much less common in
children than adults, including children with HIV infection.

Central nervous system infection, most commonly in the form of meningoencephalitis, is the most common and most serious
clinical presentation of cryptococcosis. A subacute, indolent course is most typical and often has a waxing and waning
course. Headache, fever, and altered mental status are the most common presenting features of cryptococcal
meningoencephalitis. Specific meningeal signs are often absent. In patients with AIDS, increased intracranial pressure is
common, and often causes visual disturbances and hearing loss, in addition to headache and altered mental status. Focal
neurologic signs as a result of focal lesions in the brain (cryptococcomas) occur in about 10% of adults with AIDS who
develop cryptococcosis. However, these focal lesions have not been reported in pediatric patients with HIV infection. Thus,
focal lesions in the brain in children with HIV infection are more likely to represent infection with organisms other than
Cryptococcus, such as pyogenic brain abscess, tuberculosis, or Toxoplasma. Non­AIDS patients with CNS cryptococcal

http://2016.prepid.courses.aap.org/script/july?req=201612122138243567&status=submit 1/2
12/13/2016 July
infection typically have a lymphocytic pleocytosis with an elevated protein and low glucose on cerebrospinal fluid analysis.
The cerebrospinal fluid analysis in patients with AIDS can be normal or show mild pleocytosis.

PREP Pearls
Disseminated cryptococcal infection, such as meningoencephalitis, occurs mainly in individuals with altered T­cell immunity
and defective phagocytosis.

Cryptocococcal meningoencephalitis is indolent and presents with fever, headache, and altered mental status.

Increased intracranial pressure is common in HIV infected patients who have central nervous system cryptococcosis.

American Board of Pediatrics Content Specification(s)
Know the risk factors for cryptococcosis (phagocytic defects, cellular immune disorders)

Recognize characteristic clinical manifestations of cryptococcal meningoencephalitis (chronic, memory and judgment
defects, cranial neuropathies, mass lesion)

Suggested Readings
Chayakulkeeree M. Perfect JR. Cryptococcosis. Infect Dis Clin North Am. 2006;20(3):507­544. DOI:
http://dx.doi.org/10.1016/j.idc.2006.07.001

Perfect JR, Dismukes WE, Dromer F, et al. Clinical practice guidelines for the management of cryptococcal disease: 2010
update by the Infectious Diseases Society of America. Clin Infect Dis. 2010;50(3):291­322 .

Thompson GR, Patterson TF. Cryptococcus species. In: Long SS, Pickering LK, Prober CG, eds. Principles and Practice of
Pediatric Infectious Diseases. 4th ed. Philadelphia, PA: Saunders Elsevier; 2012:1220­1223.

Comment On This Question

Page 65 of 115

Contact Us
Links may open in separate window
Copyright © 2016   American Academy of Pediatrics. All rights reserved. Pop­up Blocker may need to be disabled

http://2016.prepid.courses.aap.org/script/july?req=201612122138243567&status=submit 2/2
12/13/2016 July

Welcome  mohammed alsaiary [ Logout ]

Home PREP Pearls FAQs My Bookmarks CME Information

Overview Home  > July

Editorial Board
July   Enter Keyword Search
January
Question View:   All (8) Jump to Question
February
Print this Page Add to my Bookmarks Page 66 of 115
March

April Assessment History Mode: Learner  Exam 

May ASSESSMENT PROGRESS:  Total Questions:  8  Questions Answered:  4  Correct Answers:  1


June

July You currently have 4 questions unanswered in this assessment

August

September
Question: 5
October
A 2­year­old boy was hospitalized with abrupt onset of an unsteady gait several hours ago. During the intake process, he had a 10­
November minute generalized tonic­clonic seizure that was controlled with the administration of intravenous lorazepam. His mother states that
prior to this hospitalization, his only medical problem was severe eczema. He has been treated in the past for bacterial infections of
December
his skin and was diagnosed with scabies 2 days ago. The mother applied the prescribed lindane 1% lotion to treat the scabies twice
since then, along with the petroleum jelly and tacrolimus 0.1% ointment that she uses when his eczema flares.

Of the following, the abrupt onset of ataxia and seizures is MOST likely caused by a

A. bacterial infection of the central nervous system spread from the skin

Claim Credit B. complication from the treatment for the eczema

Evaluation C. complication from the treatment for the scabies infestation
D. viral infection of the central nervous system spread from the skin
My Learning Plan

Submit   Reset  

Page 66 of 115

Contact Us
Links may open in separate window
Copyright © 2016   American Academy of Pediatrics. All rights reserved. Pop­up Blocker may need to be disabled

http://2016.prepid.courses.aap.org/script/july?q­page=5&req=201612122138249192 1/1
12/13/2016 July

Welcome  mohammed alsaiary [ Logout ]

Home PREP Pearls FAQs My Bookmarks CME Information

Overview Home  > July

Editorial Board
July   Enter Keyword Search
January
Question View:   All (8) Jump to Question
February
Print this Page Add to my Bookmarks Page 66 of 115
March

April Assessment History Mode: Learner  Exam 

May ASSESSMENT PROGRESS:  Total Questions:  8  Questions Answered:  5  Correct Answers:  1


June

July You currently have 3 questions unanswered in this assessment

August

September
Question: 5
October
A 2­year­old boy was hospitalized with abrupt onset of an unsteady gait several hours ago. During the intake process, he had a 10­
November minute generalized tonic­clonic seizure that was controlled with the administration of intravenous lorazepam. His mother states that
prior to this hospitalization, his only medical problem was severe eczema. He has been treated in the past for bacterial infections of
December
his skin and was diagnosed with scabies 2 days ago. The mother applied the prescribed lindane 1% lotion to treat the scabies twice
since then, along with the petroleum jelly and tacrolimus 0.1% ointment that she uses when his eczema flares.

Of the following, the abrupt onset of ataxia and seizures is MOST likely caused by a

A. bacterial infection of the central nervous system spread from the skin

Claim Credit B. complication from the treatment for the eczema

Evaluation C. complication from the treatment for the scabies infestation
D. viral infection of the central nervous system spread from the skin
My Learning Plan

Incorrect View Peer Results
Correct Answer: C Average Percent Correct:  89.23%

The patient in the vignette has multiple risk factors for developing neurotoxic effects from topical
lindane treatment, making this the most likely cause of his ataxia and seizure. His young age and
underlying skin disorder place him at risk for absorbing lindane through the skin, especially when the
medication is applied along with an oil­based product like petroleum jelly. When used, lindane should
only be applied once, as repeat exposure further increases the risks of neurologic adverse effects,
particularly seizures. While bacterial or viral infections of the central nervous system can cause abrupt
onset of ataxia and seizures, a prodromal illness or systemic symptoms would be expected at
presentation. Children with primary eczema herpeticum, for example, could develop central nervous system herpes infection,
a possibility worth considering in the stated scenario. Neither of the stated treatments used for the patient’s eczema have
been associated with neurotoxicity.

Lindane (γ­hexachlorocyclohexane) (Figure) is a pesticide that was registered for use in the US agricultural industry in the
1940s, and as a pharmaceutical in 1951 for the treatment of lice and scabies. The US Environmental Protection Agency
(EPA) began restricting lindane use for agricultural purposes during the 1970s because of its toxicity and ongoing
environmental contamination. By 2007, the EPA banned its use completely as an agricultural pesticide. In 2009, an
international ban on the use of lindane in the agricultural industry was implemented under the Stockholm Convention on
Persistent Organic Pollutants. A specific exemption allowed its use to continue as a second­line treatment for head lice and
scabies.

Lindane has been used for the treatment of lice and scabies since the 1950s. In 1995, the US Food and Drug Administration
(FDA) designated lindane as “second­line” treatment, intending that it only be used when other available treatments fail or
cannot be used. In 2003, after recognizing that better patient education was required to avoid inappropriate use of lindane,
and to alert the consumer about its potential toxicity, the FDA required that a medication guide be dispensed with every
prescription. Despite this change, reports of inappropriate use and episodes of toxicity (both from excessive topical use) and
from unintentional ingestion continued.

Lindane is a neurotoxin that acts through interference with the inhibitory neurotransmitter γ­aminobutyric acid. The
mechanism of action is not specific to arthropods, explaining why neurotoxicity from excessive absorption or accidental
ingestion accounts for the majority of moderate to serious adverse reactions in humans. Manifestations of neurotoxicity
include ataxia, disorientation, tremors, seizures, and death.

Lindane should be avoided for use in the treatment of scabies in children. When a child requires treatment for scabies,
permethrin 5% remains effective when used as an overnight application on the whole body. Off­label use of oral ivermectin,

http://2016.prepid.courses.aap.org/script/july?req=201612122138588569&status=submit 1/2
12/13/2016 July
particularly for heavily infested patients with scabies, is also known to be quite effective. Crotamiton 10% has a high failure
rate, but remains available.

All 6 of the other FDA­approved treatments for lice are safer and generally more effective than lindane (Table). Permethrin
and pyrethrin with piperonyl butoxide, the 2 over­the­counter treatment options for lice, are safer than lindane, but resistance
among lice is becoming more common. In addition to lindane, 4 other prescription medications are also approved for the
treatment of lice. Malathion has been shown to be nontoxic at low doses, and resistance has not been reported, but the
preparation requires an overnight application and has an unpleasant “skunky” odor. Benzyl alcohol lotion (5%) and spinosad
(0.9%) are other treatment options for lice and, even when compared to over­the­counter permethrin 1%, had better safety
profiles. Ivermectin lotion (0.5%) is another safe option. The 2015 American Academy of Pediatrics Clinical Report offering
guidance on the approach to head lice clearly states that lindane is not recommended for use in children.

The routine availability of safer and, in many cases, more effective treatments for lice and scabies have made lindane all but
obsolete. There are very few situations where its potential therapeutic benefit for use in a child would outweigh the risks.

PREP Pearls
Lindane interferes with γ­aminobutyric acid neurotransmission and is neurotoxic when absorbed or ingested.

Medications that are both safer and more effective than lindane for the treatment of lice and scabies are routinely available.

American Board of Pediatrics Content Specification(s)
Recognize the clinical manifestations of lindane toxicity

Suggested Readings
Devore CD, Schutze GE. Head lice. Pediatrics. 2015;135(5):e1355­e1365. DOI: http://dx.doi.org/10.1542/peds.2015­0746

Nolan K, Kamrath J, Levitt J. Lindane toxicity: a comprehensive review of the medical literature. Pediatr Derm.
2012;29(2):141­146. DOI: http://dx.doi.org/10.1111/j.1525­1470.2011.01519.x

Comment On This Question

Page 66 of 115

Contact Us
Links may open in separate window
Copyright © 2016   American Academy of Pediatrics. All rights reserved. Pop­up Blocker may need to be disabled

http://2016.prepid.courses.aap.org/script/july?req=201612122138588569&status=submit 2/2
12/13/2016 July

Welcome  mohammed alsaiary [ Logout ]

Home PREP Pearls FAQs My Bookmarks CME Information

Overview Home  > July

Editorial Board
July   Enter Keyword Search
January
Question View:   All (8) Jump to Question
February
Print this Page Add to my Bookmarks Page 67 of 115
March

April Assessment History Mode: Learner  Exam 

May ASSESSMENT PROGRESS:  Total Questions:  8  Questions Answered:  5  Correct Answers:  1


June

July You currently have 3 questions unanswered in this assessment

August

September
Question: 6
October
A 6­year­old boy presents with a 1­month history of low­grade fever, malaise, and back pain. His past medical history is significant
November for a diagnosis of a liver abscess caused by Staphylococcus aureus at 6 months of age, and suppurative cervical lymphadenitis
caused by Nocardia at 3 years of age. Laboratory evaluation shows leukocytosis and elevated sedimentation rate. Chest imaging by
December
computed tomography shows a left­sided pulmonary infiltrate with cavitation, with extension to the pleural space, chest wall, and two
of the contiguous ribs. Biopsy of the lung and ribs shows a granulomatous reaction with septate hyphae.

Of the following, the organism MOST likely to be isolated from this patient’s biopsy is

A. Aspergillus flavus

Claim Credit B. Aspergillus nidulans

Evaluation C. Aspergillus niger
D. Aspergillus terreus
My Learning Plan

Submit   Reset  

Page 67 of 115

Contact Us
Links may open in separate window
Copyright © 2016   American Academy of Pediatrics. All rights reserved. Pop­up Blocker may need to be disabled

http://2016.prepid.courses.aap.org/script/july?q­page=6&req=201612122138593568 1/1
12/13/2016 July

Welcome  mohammed alsaiary [ Logout ]

Home PREP Pearls FAQs My Bookmarks CME Information

Overview Home  > July

Editorial Board
July   Enter Keyword Search
January
Question View:   All (8) Jump to Question
February
Print this Page Add to my Bookmarks Page 67 of 115
March

April Assessment History Mode: Learner  Exam 

May ASSESSMENT PROGRESS:  Total Questions:  8  Questions Answered:  6  Correct Answers:  1


June

July You currently have 2 questions unanswered in this assessment

August

September
Question: 6
October
A 6­year­old boy presents with a 1­month history of low­grade fever, malaise, and back pain. His past medical history is significant
November for a diagnosis of a liver abscess caused by Staphylococcus aureus at 6 months of age, and suppurative cervical lymphadenitis
caused by Nocardia at 3 years of age. Laboratory evaluation shows leukocytosis and elevated sedimentation rate. Chest imaging by
December
computed tomography shows a left­sided pulmonary infiltrate with cavitation, with extension to the pleural space, chest wall, and two
of the contiguous ribs. Biopsy of the lung and ribs shows a granulomatous reaction with septate hyphae.

Of the following, the organism MOST likely to be isolated from this patient’s biopsy is

A. Aspergillus flavus

Claim Credit B. Aspergillus nidulans

Evaluation C. Aspergillus niger
D. Aspergillus terreus
My Learning Plan

Incorrect View Peer Results
Correct Answer: B Average Percent Correct:  24.62%

This patient has chronic granulomatous disease (CGD) and invasive aspergillosis, involving the lungs
and ribs, caused by Aspergillus nidulans. A nidulans is a more severe and aggressive form of invasive
aspergillosis, often poorly responsive to amphotericin B and other antifungal agents despite in vitro
susceptibility; it often requires resection of the involved area for therapeutic cure. The patient in the
vignette highlights the unique relationship between A nidulans and patients with CGD, and the severity
of this infection in these patients. A nidulans is almost exclusively invasive in patients with CGD.
Invasive aspergillosis is common in patients with CGD, and may be also caused by other Aspergillus
species, including Aspergillus fumigatus or Aspergillus flavus, and rarely, the other species of Aspergillus.

A fumigatus is the most common Aspergillus species isolated from immune­compromised hosts, including those with primary
immune disorders such as CGD. It is commonly associated with invasive pulmonary aspergillosis. A flavus and A niger are
most commonly seen in patients with invasive fungal sinusitis. A nidulans is almost exclusively seen in patients with CGD,
and its isolation in invasive aspergillosis should prompt an evaluation for CGD in the host. Aspergillus terreus rarely causes
invasive aspergillosis. However, when it does, it is important to note it is intrinsically resistant to amphotericin B, and
antifungal therapy should be adjusted accordingly.

Aspergillosis most often involves the sinuses and lungs, but it may spread to other sites, such as the bones or brain, by direct
extension. It may also disseminate to other organs, such as the liver and brain, through hematogenous dissemination. This
occurs in immunocompromised patients, often later in the course of their immune­compromising disorder. The predisposing
factors for invasive aspergillosis include neutropenia, prolonged corticosteroid treatment, deficient T­cell function, invasive
catheters or other breakdown of the skin barrier, and impaired neutrophil function associated with CGD. Patients who are
neutropenic from cancer chemotherapy or hematopoietic stem cell transplantation (HSCT) are particularly vulnerable to
invasive aspergillosis. Presentation of invasive aspergillosis in immune­compromised hosts may be subtle and nonspecific, so
a high index of suspicion is necessary to diagnose and initiate appropriate therapy. Blood cultures are almost always
negative, and imaging may provide a diagnostic clue as to whether invasive aspergillosis is present. A common noninvasive
method used to support the diagnosis of invasive aspergillosis is galactomannan antigen assay of the blood or
bronchoalveolar lavage fluid. However, the diagnosis is best established through culture and histopathologic examination of
tissue from a biopsy performed on the involved organ.

Invasive Aspergillus sinusitis occurs most often in neutropenic patients undergoing cancer chemotherapy or hematopoietic
stem cell transplantation, and usually presents with nasal congestion, facial pain or swelling, and headache. Physical
examination usually reveals a pale nasal septum and discolored or necrotic mucosa. Extension into bony structures and

http://2016.prepid.courses.aap.org/script/july?req=201612122139300600&status=submit 1/2
12/13/2016 July
spread of the disease to adjacent structures such as the orbit and brain may occur. Computed tomography of the sinuses
often shows invasion outside the sinus cavity; endoscopy with biopsy of the tissue establishes the diagnosis.

Invasive pulmonary aspergillosis is the most frequent manifestation of invasive aspergillosis in immune­compromised hosts,
especially patients who are neutropenic while undergoing cancer chemotherapy or HSCT, and those patients with CGD.
Aspergillus species are ubiquitous and the respiratory tract is an important portal of entry. Symptoms of invasive pulmonary
aspergillosis in the neutropenic patient include persistent fever despite broad spectrum antibiotics, dry cough, hemoptysis,
shortness of breath, and chest pain. Imaging in these neutropenic patients occasionally shows the classic “halo sign” of
angioinvasion surrounded by tissue ischemia, cavitary lesions, pulmonary nodules, and pulmonary infarcts. In patients with
CGD, invasive pulmonary aspergillosis may be the presenting opportunistic infection. The disease is insidious with prolonged
fever, fatigue, and elevated sedimentation rate, and slowly progressive in CGD because of the impaired neutrophil­mediated
killing of fungal hyphae. Contiguous spread from the lungs to the pleura, thoracic wall, ribs, and vertebrae often occurs in
patients with CGD. Imaging in patients with CGD and invasive pulmonary aspergillosis may show infiltrates, but not the other
characteristic imaging signs of the neutropenic patient.

Cerebral aspergillosis may result from direct extension from invasive fungal sinus disease or by hematogenous spread. It may
occur in both neutropenic patients undergoing treatment for cancers, as well as patients with primary immune disorders such
as CGD. Clinical presentation may be altered mental status, seizures, or hemiparesis. Single or multiple abscesses or
cerebral infarcts from angioinvasive hyphae producing thrombosed arteries create these symptoms. Imaging may show
cerebral infarcts, mycotic aneurysms, meningeal enhancement, or multiple, low enhancement lesions. The disease is often
fatal when advanced, so early diagnosis is necessary for the best outcome in this form of invasive aspergillosis.

Cutaneous aspergillosis may result from skin trauma in special hosts such as neonates and burn patients, who are immune­
compromised primarily from a poor skin barrier. Catheter insertion sites may also provide an entry for cutaneous invasive
aspergillosis. Cutaneous aspergillosis in immunocompromised and special hosts may disseminate if not recognized early and
treated appropriately. In neutropenic patients undergoing cancer chemotherapy or HSCT, skin nodules are the more common
manifestation of cutaneous aspergillosis, and result from hematogenous spread from other internal sites of infection.

PREP Pearls
Invasive aspergillosis caused by Aspergillus nidulans is almost exclusively seen in chronic granulomatous disease, and
may be more severe and refractory than other Aspergillus species.

Aspergillus fumigatus is the most common species isolated in invasive aspergillosis in all immune­compromised hosts.

Angioinvasion by Aspergillus species in neutropenic patients leads to localized infarction of involved organs, or
dissemination to liver, brain, and other organs.

American Board of Pediatrics Content Specification(s)
Recognize risk factors for disseminated aspergillosis (neutropenia, corticosteroids, T cell abnormalities, foreign bodies,
CGD)

Recognize the setting and clinical manifestations of disseminated aspergillosis (immunocompromised, persistent fever,
negative blood cultures, major embolic events, infarction of lungs/spleen/liver/ brain)

Suggested Readings
Henriet S, Verweij PE, Holland SM, Warris A. Invasive fungal infections in patients with chronic granulomatous disease. Adv
Exp Med Biol. 2013;764:27­55.

Henriet SS, Verweij PE, Warris A. Aspergillus nidulans and chronic granulomatous disease: a unique host­pathogen
interaction. J Infect Dis. 2012;206(7):1128­1137. DOI: http://dx.doi.org/10.1093/infdis/jis473

Steinbach WJ. Aspergillosis. In: Cherry J, Demmler­Harrison GJ, Kaplan SL, Steinbach WJ, Hotez P, eds. Feigin and Cherry’s
Textbook of Pediatric Infectious Diseases. 7th ed. Philadelphia, PA: Saunders Elsevier; 2013:2704­2722.

Comment On This Question

Page 67 of 115

Contact Us
Links may open in separate window
Copyright © 2016   American Academy of Pediatrics. All rights reserved. Pop­up Blocker may need to be disabled

http://2016.prepid.courses.aap.org/script/july?req=201612122139300600&status=submit 2/2
12/13/2016 July

Welcome  mohammed alsaiary [ Logout ]

Home PREP Pearls FAQs My Bookmarks CME Information

Overview Home  > July

Editorial Board
July   Enter Keyword Search
January
Question View:   All (8) Jump to Question
February
Print this Page Add to my Bookmarks Page 68 of 115
March

April Assessment History Mode: Learner  Exam 

May ASSESSMENT PROGRESS:  Total Questions:  8  Questions Answered:  6  Correct Answers:  1


June

July You currently have 2 questions unanswered in this assessment

August

September
Question: 7
October
You are asked to provide treatment advice for a 3­year­old boy who was hospitalized because he refused to stand or walk. He was
November well until 2 weeks ago when he developed fever to 38°C and limp. His mother states that after limping for several days, he refused to
stand or walk at all, noting that standing seemed to be painful for him. On physical examination, he appears well. His temperature is
December
37°C. His musculoskeletal examination is normal, but he cries when you palpate his lower back. His neurologic examination is also
normal, except that he cries and becomes irritable when you attempt to stand him up. He will not stand or walk. Plain radiographs of
the lumbar spine, pelvis, and lower extremities are normal. A magnetic resonance image of the spine and pelvis done with
gadolinium contrast shows narrowing of the L4­L5 disk space with surrounding edema. Blood work shows a normal complete blood
cell count and differential, an erythrocyte sedimentation rate of 65 mm/h, and a C­reactive protein of 140 mg/L (1,333.4 nmol/L). The
primary care team initiated treatment with intravenous clindamycin 2 days ago after blood cultures were obtained. You are called to
Claim Credit provide treatment advice because the blood cultures are positive for the growth of Kingella kingae.

Evaluation Of the following, the BEST treatment plan for the child’s infection is
My Learning Plan
A. continue therapy with clindamycin
B. discontinue clindamycin and start cefazolin
C. discontinue clindamycin and start oxacillin
D. discontinue clindamycin and start vancomycin

Submit   Reset  

Page 68 of 115

Contact Us
Links may open in separate window
Copyright © 2016   American Academy of Pediatrics. All rights reserved. Pop­up Blocker may need to be disabled

http://2016.prepid.courses.aap.org/script/july?q­page=7&req=201612122139310912# 1/1
12/13/2016 July

Welcome  mohammed alsaiary [ Logout ]

Home PREP Pearls FAQs My Bookmarks CME Information

Overview Home  > July

Editorial Board
July   Enter Keyword Search
January
Question View:   All (8) Jump to Question
February
Print this Page Add to my Bookmarks Page 68 of 115
March

April Assessment History Mode: Learner  Exam 

May ASSESSMENT PROGRESS:  Total Questions:  8  Questions Answered:  7  Correct Answers:  1


June

July You currently have 1 questions unanswered in this assessment

August

September
Question: 7
October
You are asked to provide treatment advice for a 3­year­old boy who was hospitalized because he refused to stand or walk. He was
November well until 2 weeks ago when he developed fever to 38°C and limp. His mother states that after limping for several days, he refused to
stand or walk at all, noting that standing seemed to be painful for him. On physical examination, he appears well. His temperature is
December
37°C. His musculoskeletal examination is normal, but he cries when you palpate his lower back. His neurologic examination is also
normal, except that he cries and becomes irritable when you attempt to stand him up. He will not stand or walk. Plain radiographs of
the lumbar spine, pelvis, and lower extremities are normal. A magnetic resonance image of the spine and pelvis done with
gadolinium contrast shows narrowing of the L4­L5 disk space with surrounding edema. Blood work shows a normal complete blood
cell count and differential, an erythrocyte sedimentation rate of 65 mm/h, and a C­reactive protein of 140 mg/L (1,333.4 nmol/L). The
primary care team initiated treatment with intravenous clindamycin 2 days ago after blood cultures were obtained. You are called to
Claim Credit provide treatment advice because the blood cultures are positive for the growth of Kingella kingae.

Evaluation Of the following, the BEST treatment plan for the child’s infection is
My Learning Plan
A. continue therapy with clindamycin
B. discontinue clindamycin and start cefazolin
C. discontinue clindamycin and start oxacillin
D. discontinue clindamycin and start vancomycin

Incorrect View Peer Results

Correct Answer: B Average Percent Correct:  84.62%

The child described in the vignette has a history, physical examination, imaging results, and laboratory
results consistent with a diagnosis of diskitis caused by Kingella kingae. Diskitis is uncommon in
children, so the approach to management is based on a small number of case series reports and
dictated by the individual patient’s response to treatment. In the case presented, management can be
directed specifically to the microbiologic cause because the blood cultures were positive. K kingae is a
fastidious Gram­negative rod that has increasingly been implicated as a cause of pediatric
osteoarticular infections, including diskitis. K kingae is typically highly susceptible to penicillins and
cephalosporins, although in vitro susceptibility to oxacillin is reduced, and low­level β­lactamase expression has been
described. The organism is also usually susceptible to macrolides, quinolones, and trimethoprim­sulfamethoxazole. In
contrast, between 40% and 100% of isolates of K kingae are resistant to clindamycin, and virtually all isolates are resistant to
vancomycin.

Infections of the spine may involve the vertebrae (osteomyelitis), the intervertebral disk (diskitis), or both (spondylodiskitis).
Spontaneous pyogenic diskitis occurs following hematogenous seeding during bacteremia. The incidence is quite low at 0.2
per 100,000, which is approximately 10 times less frequent than vertebral osteomyelitis. There is a paucity of case series and
case reports published on diskitis in children. Included in those series are reports of children with culture­negative diskitis who
are never treated with antibiotics and have a full recovery, lending question to whether this clinical diagnosis always
represents an active infection. Certainly, children with diskitis who have a positive blood or tissue culture require antibiotic
treatment.

A child with diskitis usually presents with pain localized to the region of the infection. Since nearly all pediatric cases involve
the disk spaces in the lumbar region, lower back pain is the primary complaint. Very young children may refuse to walk or
stand because weight­bearing worsens the discomfort. The diagnosis is especially difficult to make in infants because the
only complaint voiced by the parent may be that the infant is exceptionally irritable. When fever is present, it is
characteristically low grade. Localizing neurologic signs or symptoms such as weakness, reduced deep tendon reflexes, or
loss of bowel or bladder control are not typical, and if present in a febrile child with localized back pain, more strongly
suggests the possibility of a spinal epidural abscess rather than diskitis.

http://2016.prepid.courses.aap.org/script/july?req=201612122140075444&status=submit 1/3
12/13/2016 July
The diagnostic evaluation of a child with suspected diskitis includes laboratory tests and imaging studies. Laboratory
evaluation should include blood cultures, a complete blood cell count and differential, C­reactive protein (CRP), and
erythrocyte sedimentation rate (ESR). The presence of leukocytosis raises suspicion for infection, but its absence does not
rule it out. Biomarkers for inflammation (CRP, ESR) are almost always elevated in patients with diskitis, and while the finding
is nonspecific, the results provide a baseline marker for inflammation that can be followed during treatment. Blood cultures
are typically very low yield, but when positive, allow for definitive therapy with a targeted antibiotic.

Plain radiographs are neither sensitive nor specific for the diagnosis disk space infections, but might show paravertebral soft
tissue swelling or disc space narrowing, especially in a patient who has already had several weeks of symptoms. Computed
tomography (CT) of the spine with administration of intravenous contrast will usually reveal enhancement of an infected disk,
but magnetic resonance (MR) imaging is more reliable and avoids exposure to radiation. On noncontrasted T1­weighted MR
images, infected disks appear hypointense. In the presence of gadolinium contrast, the surrounding edema will enhance on
T2­weighted images. Associated findings include a narrowed disk space and loss of the intranuclear cleft. The Figure shows
a sagittal T2­weighted lumbar spine MR with narrowing of the L3­L4 intervertebral space and associated signal enhancement
characteristic of diskitis.

Since blood culture yields are low, CT­guided percutaneous biopsy with cultures can be considered. This approach may be
important for patients who are not improving or who have worsening symptoms on empiric therapy, as they may have an
unusual cause of their infection that is not being targeted by the current treatment regimen. On rare occasion, a patient with
worsening symptoms might benefit from an open biopsy with cultures. Even with aggressive attempts to obtain appropriate
cultures, a definitive microbiologic diagnosis frequently remains elusive. While culture­negative cases are most often
managed empirically with antibiotics, reports of case series indicate that some children managed with nonsteroidal anti­
inflammatory medications alone may have full recovery.

When blood or disk­space cultures are positive, Staphylococcus aureus is the most common organism identified. K kingae,
streptococcal, Salmonella, and Haemophilus species have also been implicated. In patients with underlying gastrointestinal or
genitourinary pathology, Gram­negative enteric and enterococcal infections have been reported, but are quite rare in children.
Mycobacterium tuberculosis and Brucella species have been reported to cause infection of the intervertebral disks, but in
such cases, bone involvement is almost always seen and is likely first to occur with subsequent adjacent spread to the disk
space.

The goal of treatment of diskitis is to eliminate any infection permanently while maintaining the stability of the spine. Acute
hematogenous (spontaneous) diskitis can usually be treated medically without surgical intervention, but because culture
results infrequently identify the causative agent, antibiotics are chosen empirically, if used at all. The patient is then monitored
by following clinical symptoms and inflammatory biomarker results to determine if any changes need to be made. Surgical
treatment may (rarely) be indicated for the diagnostic purpose of obtaining cultures, but is clearly indicated if the infection is
extensive and requires debridement.

Empiric antibiotics that target the usual microbiologic causes of diskitis are typically initiated after blood cultures are obtained.
Some experts prefer to start treatment with a first­generation cephalosporin such as cefazolin awaiting culture results. While
considered a narrow spectrum antibiotic, cefazolin will target methicillin susceptible S aureus, most streptococci, and K
kingae. The approach is comparatively inexpensive and usually very well tolerated. Patients who do not show signs of clinical
improvement during treatment with cefazolin can be switched to broader coverage, while those who show signs of clinical
improvement can have a longer term treatment plan developed using the assumption that the infection is caused by a
cefazolin­susceptible organism. Pyogenic diskitis is typically treated until the symptoms have resolved and the inflammatory
biomarker blood results have normalized. Treatment courses as brief as 2 weeks have been successful, but a 4­ to 6­week
course of antibiotics is usually prescribed.

* Required *
Take Survey  

PREP Pearls
In children, the diagnosis of diskitis is made clinically, and confirmed by magnetic resonance imaging.

Controversy remains as to whether culture­negative cases of pediatric diskitis need to be treated with antibiotics.

When used, empiric, rather than definitive antibiotic treatment of diskitis is usually necessary because cultures have low
yield.

The duration of antibiotic treatment for diskitis is usually 4 to 6 weeks, or at least until the symptoms have resolved and the
serum inflammatory biomarkers returned to normal levels.

American Board of Pediatrics Content Specification(s)
Plan the management of a patient with diskitis

Plan a diagnostic evaluation to establish the etiology of diskitis

Suggested Readings
Fernandez M, Carrol CL, Baker CJ. Discitis and vertebral osteomyelitis in children: an 18­year review. Pediatrics.
2000;105(6):1299­1304. DOI: http://dx.doi.org/10.1542/peds.105.6.1299

Spencer SJ, Wilson NI. Childhood discitis in a regional children's hospital. J Pediatr Orthop B. 2012;21(3):264­268. DOI:
http://dx.doi.org/10.1097/BPB.0b013e32834d3e94

Swanson KI, Resnick DK. Vertebral column infections. In: Hall WA, Kim PD, eds. Neurosurgical Infectious Disease. New York,
NY: Thieme Medical Publishers; 2014: 336 pp.

Comment On This Question

http://2016.prepid.courses.aap.org/script/july?req=201612122140075444&status=submit 2/3
12/13/2016 July
Page 68 of 115

Contact Us
Links may open in separate window
Copyright © 2016   American Academy of Pediatrics. All rights reserved. Pop­up Blocker may need to be disabled

http://2016.prepid.courses.aap.org/script/july?req=201612122140075444&status=submit 3/3
12/13/2016 July

Welcome  mohammed alsaiary [ Logout ]

Home PREP Pearls FAQs My Bookmarks CME Information

Overview Home  > July

Editorial Board
July   Enter Keyword Search
January
Question View:   All (8) Jump to Question
February
Print this Page Add to my Bookmarks Page 69 of 115
March

April Assessment History Mode: Learner  Exam 

May ASSESSMENT PROGRESS:  Total Questions:  8  Questions Answered:  7  Correct Answers:  1


June

July You currently have 1 questions unanswered in this assessment

August

September
Question: 8
October
After consulting on an adolescent with post­transplant, Epstein­Barr virus (EBV)­ associated B­cell lymphoma who developed a
November central catheter line infection, you have a discussion with your team about the oncogenic potential of EBV. During the discussion,
you are asked whether infections with herpesviruses other than EBV are associated with the development of cancer.
December
Of the following, the herpesvirus known for oncogenic potential is

A. cytomegalovirus
B. herpes B virus
C. human herpesvirus­7
Claim Credit
D. human herpesvirus­8
Evaluation

My Learning Plan
Submit   Reset  

Page 69 of 115

Contact Us
Links may open in separate window
Copyright © 2016   American Academy of Pediatrics. All rights reserved. Pop­up Blocker may need to be disabled

http://2016.prepid.courses.aap.org/script/july?q­page=8&req=201612122140079663 1/1
12/13/2016 July

Welcome  mohammed alsaiary [ Logout ]

Home PREP Pearls FAQs My Bookmarks CME Information

Overview Home  > July

Editorial Board
July   Enter Keyword Search
January
Question View:   All (8) Jump to Question
February
Print this Page Add to my Bookmarks Page 69 of 115
March

April Assessment History Mode: Learner  Exam 

May ASSESSMENT PROGRESS:  Total Questions:  8  Questions Answered:  8  Correct Answers:  1


June

July You currently have 2 required survey(s) that are incomplete. Please complete the survey(s) for the following
question(s) on this assessment:  Question 3, Question 7
August

September

October Question: 8
November After consulting on an adolescent with post­transplant, Epstein­Barr virus (EBV)­ associated B­cell lymphoma who developed a
central catheter line infection, you have a discussion with your team about the oncogenic potential of EBV. During the discussion,
December
you are asked whether infections with herpesviruses other than EBV are associated with the development of cancer.

Of the following, the herpesvirus known for oncogenic potential is

A. cytomegalovirus
B. herpes B virus
Claim Credit C. human herpesvirus­7
Evaluation D. human herpesvirus­8

My Learning Plan

Incorrect View Peer Results

Correct Answer: D Average Percent Correct:  89.23%

Human herpesvirus­8 (HHV­8) is known to cause Kaposi sarcoma (KS). While KS is best recognized
as a malignancy seen among HIV­infected patients with advanced AIDS, endemic KS also occurs in
adults not infected with HIV. Some primary effusion B­cell lymphomas are also caused by HHV­8.

In contrast, cytomegalovirus is not generally considered an oncogenic virus, although some work
suggests that its potential to contribute to malignant transformation should not be dismissed
completely. Herpes B virus is a simian virus that can be transmitted from macaque saliva to a human,
usually from a bite. Untreated infection leads to life­threatening hemorrhagic encephalitis. Infection with human herpesvirus­7
(HHV­7) is not usually associated with symptoms during acute infection or reactivation, but because it is a T­cell tropic virus,
some researchers have suggested a link to some types of lymphoma. A clear association between HHV­7 and cancer has yet
to be defined. Oncogenic links between latent infection with the other human herpesviruses, herpes simplex virus (HSV)­1,
HSV­2, varicella zoster virus, and human herpesvirus­6 have not been described.

Seven viruses, including Epstein­Barr virus (EBV) and HHV­8, are known to cause cancer in humans (Table). While each
known oncovirus has unique features that contribute to their cancer­causing potential, the viruses and the infections they
cause as a group share several important characteristics. First, cancer occurs in a very small number of those infected,
making it clear that the contribution of the virus is necessary but insufficient for malignant transformation. Second, the
malignancies develop in the context of a persistent infection, often many years to decades after the initial infection. Third, the
host immune system plays a key role either because of immune suppression and faulty immune surveillance, or via chronic
inflammatory changes that promote cell proliferation.

With each of the oncogenic viruses, persistent infection, virus latency, virus gene expression, and virus integration into the
host cell genome can all lead to alterations in cell signaling functions through genetic mutations. For example, mutations that
constitutively activate Ras, a potent cell­signaling protein, will lead to cellular proliferation, cell survival, and angiogenesis.
Likewise, mutations that inactivate tumor suppressor genes, such as p53, result in genetic instability, loss of inhibition to
signals of growth and apoptosis, and uncontrolled cellular proliferation. Persistent infection with an oncovirus can also lead to
a tissue microenvironment that promotes cell proliferation through excessive expression of proinflammatory mediators. Some
oncoviruses have also evolved to encode proteins that mimic host­signaling mechanisms that are able to subvert and
deregulate normal cell cycle activities.

The first virus to be directly linked to cancer in humans was EBV. Shortly after the 1964 discovery that EBV causes Burkitt
lymphoma, its role in development of nasopharyngeal carcinoma was appreciated. Both types of cancer have geographically
restricted distributions, with Burkitt lymphoma found most commonly in children living in central Africa within a few degrees of
the equator, and nasopharyngeal cancer found most commonly along the southern coast of China. Epstein­Barr virus is also

http://2016.prepid.courses.aap.org/script/july?req=201612122140379820&status=submit 1/2
12/13/2016 July
associated with post­transplant lymphoproliferative disorder, and individuals who are EBV­seronegative at the time of
transplant are at increased risk for developing EBV­associated lymphomas, predominantly of B­cell origin. Several EBV­
encoded proteins have been shown to immortalize B cells including the Epstein­Barr nuclear protein and latent membrane
proteins. Integration of EBV­specific DNA into the host genome is not thought to be a factor in tumorigenesis.

Hepatitis B virus (HBV) is a major cause of hepatocellular carcinoma. Hepatitis B virus­associated liver cancer has the highest
incidence in areas of the world where perinatal hepatitis B infection transmission rates are high. Unlike EBV, HBV does not
encode obvious oncoproteins. The viral genome of approximately 3,000 base pairs has only 4 open reading frames. The
largest encodes the viral polymerase. Two encode structural proteins, and the last encodes X protein, so named when its
function was unknown. X protein is now recognized as a transcriptional activator, but a direct role in tumorigenesis has never
been demonstrated. Integration of viral DNA into host cell chromosomal DNA does occur late in the chronic phase of hepatitis
B infection, but it remains unclear whether integration is required for malignant transformation of the hepatocyte. Hepatitis B
virus­associated hepatocellular carcinoma occurs after long­term persistence of infection, and may be secondary to
hepatocellular proliferation in response to the chronic insults and efforts to repair them.

Hepatitis C virus causes persistent infection more often than hepatitis B, and is increasingly recognized as a cause of
hepatocellular carcinoma in those with chronic infection. The proposed mechanism for malignant transformation is similar to
that described for HBV.

Human papillomaviruses (HPV) are divided into types that have high potential for causing malignancy, and types that have
low potential for causing malignancy. High­risk types (especially HPV type 16) have been linked to cervical, vaginal, vulvar,
anal, penile, and head and neck cancers. The HPV genome consists of 5 early genes and 3 late genes. Each of proteins
encoded by the 5 early genes (E1, 2, 5, 6, and 7) have been shown to be involved in altering signal transduction pathways
and/or cell cycle control. For example, E6 binds to 2 cellular proteins, E6­associated protein and p53. Once formed, this
heterotrimeric protein is rapidly degraded. Since p53 controls host cell replication, its premature degradation through the
formation of these heterotrimers can lead to uncontrolled cell cycle progression. E7 binds to host retinoblastoma family
proteins, stimulating the cell to enter the S­phase of the cell cycle in an uncontrolled manner. The majority of malignant cells
demonstrate integration of HPV DNA, and all malignant cells studied to date continue to express E6 and E7 proteins,
suggesting that these proteins are required for malignant transformation.

Human T­cell lymphotropic virus­1 infects T cells and is a known cause of adult T­cell leukemia. Like other retroviruses,
proviral DNA sequences are integrated into the host cell genome during virus replication. One of the best­characterized virus­
encoded oncoproteins is Tax, a potent dysregulator of host cell signaling.

Merkel cell polyomavirus, discovered in 2008, is the cause of a rare, but highly aggressive form of skin cancer. The virus is
found in the majority of Merkel cell cancers, integrated into the host cell genome. While several animal polyoma viruses have
been known to be associated with cancers for decades, this is the first polyoma virus with malignant potential described in
humans.

Human herpesvirus­8 is the cause of KS and some types of primary effusion lymphomas. The HHV­8 genome is complex,
encoding more than 80 open reading frames, but none are homologous to the known viral oncogenes encoded by the other
herpes family oncovirus, EBV. Human herpesvirus­8 does, however, encode several genes that are homologous to
mammalian genes with known functions related to cell cycle regulation, and it is this collection of genes that is implicated in
the oncogenic potential of this virus. One of these genes, the virus­encoded Bcl­2, is a homologue of host cellular gene
involved in rescuing cells from apoptosis.

PREP Pearls
The human oncoviruses, Epstein­Barr virus (EBV), human herpesvirus­8, hepatitis B and C viruses, human papillomavirus,
human T­cell lymphotropic virus­1, and Merkel cell polyoma virus contribute to the development of malignancies in a small
percentage of those infected.

Infection with the oncovirus is necessary, but not sufficient for the development of virus­associated cancer.

Years to decades are usually required for infection from an oncovirus to lead to cancer, although EBV­associated
malignancies are seen in the pediatric population.

American Board of Pediatrics Content Specification(s)
Know the viruses etiologically associated with cancer in humans

Suggested Readings
Mesri EA, Feitelson MA, Mungar K. Human oncogenesis: a cancer hallmark analysis. Cell Host Microbe. 2014;15(3):266­282.
DOI: http://dx.doi.org/10.1016/j.chom.2014.02.011

White MK, Pagano JS, Khalili K. Viruses and human cancers: a long road of discovery of molecular paradigms. Clin Micro
Rev. 2014;27(3):463­481. DOI: http://dx.doi.org/10.1128/CMR.00124­13

Comment On This Question

Page 69 of 115

Contact Us
Links may open in separate window
Copyright © 2016   American Academy of Pediatrics. All rights reserved. Pop­up Blocker may need to be disabled

http://2016.prepid.courses.aap.org/script/july?req=201612122140379820&status=submit 2/2
12/13/2016 August

Welcome  mohammed alsaiary [ Logout ]

Home PREP Pearls FAQs My Bookmarks CME Information

Overview Home  > August

Editorial Board
August   Enter Keyword Search
January
Question View:   All (8) Jump to Question
February
Print this Page Add to my Bookmarks Page 71 of 115
March

April Assessment History Mode: Learner  Exam 

May ASSESSMENT PROGRESS:  Total Questions:  8  Questions Answered:  0  Correct Answers:  0


June

July You currently have 8 questions unanswered in this assessment

August

September
Question: 1
October
A 16­year­old adolescent girl presents to you with a 3­day history of vaginal discharge and dysuria. She has 4 total sexual partners,
November both male and female, but she has not contacted any of them to ask if they have symptoms. Her male partners usually use
condoms. She had a normal menstrual period 3 weeks ago and last had sexual relations 7 days ago. She has no known drug
December
allergies. On examination, she is afebrile and looks well generally, but has mucopurulent discharge from her cervix. Gram stain of the
urethral discharge is shown in the Figure.

Of the following, the MOST important next step in treatment is to

A. administer intramuscular ceftriaxone

Claim Credit B. administer intramuscular ceftriaxone and oral azithromycin

Evaluation C. administer oral cefixime and oral azithromycin
D. await susceptibility testing before treating
My Learning Plan

Submit   Reset  

Page 71 of 115

Contact Us
Links may open in separate window
Copyright © 2016   American Academy of Pediatrics. All rights reserved. Pop­up Blocker may need to be disabled

http://2016.prepid.courses.aap.org/script/august?q­page=1&req=201612122140552476# 1/1
12/13/2016 August

Welcome  mohammed alsaiary [ Logout ]

Home PREP Pearls FAQs My Bookmarks CME Information

Overview Home  > August

Editorial Board
August   Enter Keyword Search
January
Question View:   All (8) Jump to Question
February
Print this Page Add to my Bookmarks Page 71 of 115
March

April Assessment History Mode: Learner  Exam 

May ASSESSMENT PROGRESS:  Total Questions:  8  Questions Answered:  1  Correct Answers:  0


June

July You currently have 7 questions unanswered in this assessment

August

September
Question: 1
October
A 16­year­old adolescent girl presents to you with a 3­day history of vaginal discharge and dysuria. She has 4 total sexual partners,
November both male and female, but she has not contacted any of them to ask if they have symptoms. Her male partners usually use
condoms. She had a normal menstrual period 3 weeks ago and last had sexual relations 7 days ago. She has no known drug
December
allergies. On examination, she is afebrile and looks well generally, but has mucopurulent discharge from her cervix. Gram stain of the
urethral discharge is shown in the Figure.

Of the following, the MOST important next step in treatment is to

A. administer intramuscular ceftriaxone

Claim Credit B. administer intramuscular ceftriaxone and oral azithromycin

Evaluation C. administer oral cefixime and oral azithromycin
D. await susceptibility testing before treating
My Learning Plan

Incorrect View Peer Results
Correct Answer: B Average Percent Correct:  98.44%

This adolescent girl has cervicitis and urethritis. Urethritis can be diagnosed with one of the following
criteria:

A visible abnormal urethral discharge, or  

A positive leukocyte esterase test from a male aged younger than 60 years who does not have a
history of kidney disease or bladder infection, prostate enlargement, urogenital anatomic
anomaly, or recent urinary tract instrumentation, or  

Microscopic evidence of urethritis (≥ 5 white blood cells per high­power field) on a Gram stain of a urethral smear

This Gram stain is strongly suggestive of gonococcal infection, and most authorities recommend treating for both gonococcal
and chlamydial infection in this situation, preferably with ceftriaxone and azithromycin in single doses at the time of the
outpatient encounter. Dual therapy with ceftriaxone and azithromycin are also indicated in the setting of known gonorrhea
infection without chlamydia to prevent the emergence of resistance. Ceftriaxone alone would not provide coverage for
chlamydial infection. Oral cefixime does not result in tissue levels as high as parenteral ceftriaxone, which is problematic in an
era of increasing cefixime minimal inhibitory concentrations. Awaiting susceptibility testing is incorrect for 2 reasons. First,
clinical laboratories do not routinely test gonococcal susceptibility, so a special request would be required for this assessment.
Secondly, delaying definitive treatment runs the risk of losing the patient in follow­up and engendering risk not only to that
patient, but also to her future sexual contacts.

Management of urethritis is complicated by several factors and is more complicated in the pediatric population. Pediatric
practitioners must be particularly aware of differing epidemiology and causes by age, in addition to recognizing which tests
are most suitable based on the clinical setting. The clinical presentations may be highly variable, and patients are often
asymptomatic, particularly adolescent boys. Also, changes in susceptibility patterns, particularly for gonococcal disease in the
United States, have led to recent changes in treatment recommendations.

In the prepubertal population, urethritis is less likely to be caused by traditional sexually transmitted infectious agents,
although such etiologies do occur particularly in the setting of sexual abuse. However, urethritis is more commonly caused by
irritants such as soaps or bubble bath, or trauma due to foreign body or masturbation in this age group. For sexually active
individuals, gonococcal and nongonococcal urethritis cause the majority of urethritis syndromes. The latter principally are
caused by Chlamydia trachomatis or Mycoplasma genitalium, with Ureaplasma urealyticum, Gardnerella vaginalis,
Mycoplasma hominis, Trichomonas vaginalis, herpes simplex virus, and other agents occasionally being implicated. Many

http://2016.prepid.courses.aap.org/script/august?req=201612122144000294&status=submit 1/2
12/13/2016 August
other agents unrelated to sexually transmitted infections, such as Shigella species, group A streptococcus, and Enterobius
vermicularis, can cause urethritis. Systemic diseases, such as Kawasaki disease, erythema multiforme, or as a component of
reactive arthritis with uveitis or conjunctivitis, can also cause urethritis. In 2013, gonococcal disease rates in the United States
as a whole were about 100 out of 100,000 population, with chlamydial disease exceeding 300 out of 100,000 population.

For gonococcal urethritis, the incubation period is approximately 2 to 7 days, but the diagnosis is often delayed in girls
compared to boys due to lack of appreciation of leucorrhea and absence of dysuria. Nongonococcal urethritis has a longer
incubation period of 2 to 3 weeks, and frequently may be asymptomatic or minimally symptomatic, particularly in girls.

Diagnostic testing for Neisseria gonorrhoeae and Chlamydia trachomatis is dictated largely by resource availability. Gram
stain of urethral discharge is potentially the quickest method to diagnose gonococcal urethritis, with findings of 5 white blood
cells or greater per high power field and demonstration of Gram­negative intracellular diplococci being highly sensitive and
specific (near 100% in males; approximately 66% sensitive in females). Although culture is the gold standard for diagnosis of
gonococcal and chlamydial urethritis and may be required for proof of infection in legal matters, these tests have largely been
replaced by nucleic acid amplification testing (NAAT) of urethral discharge or urine. The latter obviates the need for the more
uncomfortable procedure of obtaining a urethral swab specimen. The NAAT methods include polymerase chain reaction,
strand displacement amplification, or transcription­mediated amplification, among others, with transcription­mediated
amplification being the best studied. The NAAT methods are not recommended for testing rectal or pharyngeal specimens.

PREP Pearls
Urethritis causes differ by age group, and clinical signs, particularly in females, may be subtle.

Nucleic acid amplification testing has largely replaced culture and other methods for diagnosis of gonococcal and
chlamydial urethritis.

Changes in gonococcal susceptibility may have resulted in lessened efficacy of oral third­generation cephalosporins for
treating this condition.

American Board of Pediatrics Content Specification(s)
Plan appropriate diagnostic tests to define the infectious cause of urethritis in males and females by age

Know which of the agents causing urethritis in males and females can be shed asymptomatically and can be transmitted by
sexual contact

Suggested Readings
Braverman PK. Urethritis, vulvovaginitis, and cervicitis. In: Cherry JD, Harrison GJ, Kaplan SL, Steinbach WJ, Hotez PJ, eds.
Feigin and Cherry’s Textbook of Pediatric Infectious Diseases. 7th ed. Philadelphia, PA: Saunders Elsevier; 2013: 353­363.

US Centers for Disease Control and Prevention. 2015 sexually transmitted diseases treatment guidelines. US Centers for
Disease Control and Prevention website. http://www.cdc.gov/std/tg2015/default.htm. Updated June 23, 2015

Wald ER. Urethritis. In: Long SS, Pickering LK, Prober CG, eds. Principles and Practice of Pediatric Infectious Diseases. 4th
ed. Philadelphia, PA: Saunders Elsevier; 2012:530­535.

Workowski KA, Bolan GA. Sexually transmitted diseases treatment guidelines, 2015. MMWR Morbid Mortal Wkly Rep.
2015;64(3):1­137. http://www.cdc.gov/mmwr/preview/mmwrhtml/rr6403a1.htm

Comment On This Question

Page 71 of 115

Contact Us
Links may open in separate window
Copyright © 2016   American Academy of Pediatrics. All rights reserved. Pop­up Blocker may need to be disabled

http://2016.prepid.courses.aap.org/script/august?req=201612122144000294&status=submit 2/2
12/13/2016 August

Welcome  mohammed alsaiary [ Logout ]

Home PREP Pearls FAQs My Bookmarks CME Information

Overview Home  > August

Editorial Board
August   Enter Keyword Search
January
Question View:   All (8) Jump to Question
February
Print this Page Add to my Bookmarks Page 72 of 115
March

April Assessment History Mode: Learner  Exam 

May ASSESSMENT PROGRESS:  Total Questions:  8  Questions Answered:  1  Correct Answers:  0


June

July You currently have 7 questions unanswered in this assessment

August

September
Question: 2
October
A previously healthy, 12­year­old boy presents to the emergency department with a 3­day history of fatigue and a 1­day history of
November inability to move the left side of his body. A cardiovascular examination showed systolic and diastolic heart murmurs with an S3
gallop. Neurologic examination confirmed a left­sided hemiparesis. Magnetic resonance imaging of the brain revealed a high­
December
intensity area in the right frontal lobe, consistent with acute infarction and occlusion of the right middle cerebral artery.

Of the following, the MOST likely evaluation to reveal the cause of this child’s condition is an

A. echocardiogram
B. electrocardiogram
Claim Credit C. electroencephalogram
Evaluation D. erythrocyte sedimentation rate

My Learning Plan E. evaluation by a dentist

Submit   Reset  

Page 72 of 115

Contact Us
Links may open in separate window
Copyright © 2016   American Academy of Pediatrics. All rights reserved. Pop­up Blocker may need to be disabled

http://2016.prepid.courses.aap.org/script/august?q­page=2&req=201612122144007324# 1/1
12/13/2016 August

Welcome  mohammed alsaiary [ Logout ]

Home PREP Pearls FAQs My Bookmarks CME Information

Overview Home  > August

Editorial Board
August   Enter Keyword Search
January
Question View:   All (8) Jump to Question
February
Print this Page Add to my Bookmarks Page 72 of 115
March

April Assessment History Mode: Learner  Exam 

May ASSESSMENT PROGRESS:  Total Questions:  8  Questions Answered:  2  Correct Answers:  1


June

July You currently have 6 questions unanswered in this assessment

August

September
Question: 2
October
A previously healthy, 12­year­old boy presents to the emergency department with a 3­day history of fatigue and a 1­day history of
November inability to move the left side of his body. A cardiovascular examination showed systolic and diastolic heart murmurs with an S3
gallop. Neurologic examination confirmed a left­sided hemiparesis. Magnetic resonance imaging of the brain revealed a high­
December
intensity area in the right frontal lobe, consistent with acute infarction and occlusion of the right middle cerebral artery.

Of the following, the MOST likely evaluation to reveal the cause of this child’s condition is an

A. echocardiogram
B. electrocardiogram
Claim Credit C. electroencephalogram
Evaluation D. erythrocyte sedimentation rate

My Learning Plan E. evaluation by a dentist

Correct View Peer Results
Average Percent Correct:  98.44%

This child has an acute cerebral infarction complicating acute infective endocarditis (IE).
Echocardiogram revealed a large, echogenic vegetation on a previously unrecognized bicuspid aortic
valve, and his cerebral infarction most likely resulted from arterial emboli from the left­sided cardiac
vegetation to the middle cerebral artery in the brain.

The complications associated with IE include major arterial embolic events from cardiac vegetations
(Table 1). Embolic events to the lungs are associated with right­sided IE, resulting in septic pulmonary
infarcts, necrotizing pneumonia, or lung abscesses. Emboli associated with left­sided IE involve the major arteries of the
brain, visceral organs, or limbs. Emboli to the brain may cause occlusion of cerebral arteries and stroke, brain abscesses, or
mycotic aneurysms that may rupture and cause intracranial hemorrhages. Emboli to major visceral organs may cause
infarction or abscesses of the intestines, kidneys, liver, or spleen. Emboli to peripheral arteries may cause infections or
infarctions in limbs, conjunctival hemorrhages, or Janeway lesions. Late complications of these embolic events include limb
loss, delayed splenic rupture, and permanent hemiparesis.

Cardiac damage may also occur directly from IE and produce congestive heart failure, myocardial abscesses, infection of the
valves (may cause dysfunction of the valvular leaflets of native valves), valve ring abscesses, rupture of the chordae, or
dehiscence of prosthetic valves. Emboli causing occlusion of the coronary arteries may produce myocardial ischemia or
myocardial infarction. In addition, dysrhythmias and heart block may occur if the conduction system is involved in the IE and
damaged.

Persistent bacteremia or sepsis may also occur in IE, and death may result from direct cardiac involvement or from one of the
embolic complications of IE.

Complications of an immunopathogenic nature may also occur during IE, especially when it presents as a subacute condition.
These complications include glomerulonephritis with hematuria and proteinuria, splenomegaly, anemia, thrombocytopenia,
low serum complement levels, positive rheumatoid factor, Osler nodes in the extremities, and Roth spots in the eyes.

The diagnostic evaluation of a child suspected of having infective endocarditis includes serial sequential blood cultures to
detect persistent bacteremia with one of the more common bacteria associated with IE (Table 2). These organisms include
Staphylococcus aureus, Staphylococcus epidermidis (if indwelling catheter or other intravascular materials are present),
Streptococcus pneumoniae, Streptococcus agalactiae, viridans streptococci, Abiotrophia (nutritionally deficient streptococci),
and enterococci. In special hosts, Gram­negative bacteria and fungi may be detected. In addition, certain fastidious, oral–
pharyngeal Gram­negative coccobacilli, including Haemophilus aphrophilus, Aggregatibacter (formerly Actinobacillus)

http://2016.prepid.courses.aap.org/script/august?req=201612122144251700&status=submit 1/2
12/13/2016 August
actinomycetemcomitans, Cardiobacterium hominis, Eikenella corrodens, and Kingella kingae (the HACEK group) may be
identified in blood cultures.

Serologic tests may identify Coxiella burnetii, the causative organism of Q fever, or Bartonella species as causes of IE.

Molecular diagnosis using polymerase chain reaction nucleic acid detection tests may be used to detect organisms that are
difficult to grow in culture or detect serologically and may support the diagnosis of IE in special cases.

Other laboratory tests that support the diagnosis of IE include complete blood cell count, which may show anemia or
thrombocytopenia, and an elevated erythrocyte sedimentation rate (ESR). An elevated ESR is a nonspecific test for systemic
inflammation, and while supportive of the diagnosis of IE, ESR does not establish the diagnosis of IE. Immunologic
phenomena, including elevated rheumatoid factor, low serum complement, and hematuria with proteinuria, may also support
diagnosis of IE.

Echocardiography detects endocardial and valvular vegetations, valvular dehiscence, valvular insufficiency, and intracardiac
abscesses, as well as underlying structural anomalies that may have predisposed the host to IE, such as valvular disease or
septal detects.

If tachycardia, bradycardia, or an irregular heart rate is present—suggesting involvement of the conduction system by IE—
then an electrocardiogram should be performed to detect conduction defects, heart block, or ventricular arrhythmias.
However, the presence of an abnormal electrocardiogram does not establish the diagnosis of IE.

Clinical definitions of IE are important, since diagnosis can lead to clinical complications and overdiagnosis can lead to
unnecessary antimicrobial therapy. It can be difficult to distinguish between IE and an alternative source of infection in a
bacteremic young adult or adolescent with underlying heart disease. The Duke criteria or modifications of these criteria can
be used in establishing a diagnosis. The diagnostic evaluation using the Duke criteria can be found in Table 3.

An electroencephalogram should be performed in individuals with acute neurologic findings to detect the presence of
brainwave abnormalities suggesting destructive brain lesions or epileptiform activity. While supportive of the diagnosis of
stroke, which may be caused by IE, it does not establish the diagnosis of IE.

Examination by a dentist may be helpful to determine if a focal area of infection is the source of the infecting agent.

PREP Pearls
Stroke or other embolic phenomena may be a presenting symptom of infective endocarditis.

Presence of positive blood cultures and echocardiogram showing intracardiac vegetations are major criteria for diagnosis of
infective endocarditis.

Presence of glomerulonephritis, rheumatoid factor, and low serum complement suggest the presence of immunopathogenic
phenomena associated with infective endocarditis.

American Board of Pediatrics Content Specification(s)
Recognize complications of endocarditis (eg, septic emboli, valve dysfunction, nephritis, thrombocytopenia, stroke)

Plan the diagnostic evaluation of a patient with suspected endocarditis

Suggested Readings
Baddour LM, Wilson WR, Bayer AS, et al. Infective endocarditis: diagnosis, antimicrobial treatment, and management of
complications: a statement for the healthcare professional from the Committee on Rheumatic Fever, Endocarditis and
Kawasaki Disease, Council on Cardiovascular Disease in the Young, and the Council on Clinical Cardiology, Stroke, and
Cardiovascular Surgery and Anesthesia, American Heart Association: Endorsed by the Infectious Diseases Society of
America. Circulation. 2005;111:e394­e434. DOI: http://dx.doi.org/10.1161/CIRCULATIONAHA.105.165564

Saiman, L. Endocarditis and intravascular infections. In: Long SS, Pickering LK, Prober CG, eds. Principles and Practice of
Pediatric Infectious Disease. 3rd ed. Philadelphia, PA: Elsevier; 2008:269­280.

Starke, J. Infective endocarditis. In: Feigin R, Cherry J, Demmler­Harrison G, Kaplan S, eds. Feigin and Cherry’s Textbook of
Pediatric Infectious Diseases. 6th ed. New York, NY: Saunders Elsevier; 2009:365­389.

Comment On This Question

Page 72 of 115

Contact Us
Links may open in separate window
Copyright © 2016   American Academy of Pediatrics. All rights reserved. Pop­up Blocker may need to be disabled

http://2016.prepid.courses.aap.org/script/august?req=201612122144251700&status=submit 2/2
12/13/2016 August

Welcome  mohammed alsaiary [ Logout ]

Home PREP Pearls FAQs My Bookmarks CME Information

Overview Home  > August

Editorial Board
August   Enter Keyword Search
January
Question View:   All (8) Jump to Question
February
Print this Page Add to my Bookmarks Page 73 of 115
March

April Assessment History Mode: Learner  Exam 

May ASSESSMENT PROGRESS:  Total Questions:  8  Questions Answered:  2  Correct Answers:  1


June

July You currently have 6 questions unanswered in this assessment

August

September
Question: 3
October
You are asked to evaluate an asymptomatic neonate who was born earlier today. The mother of this infant had an untreated
November Neisseria gonorrhoeae infection during pregnancy. Neonatal ocular prophylaxis was administered to the newborn shortly after
delivery.
December
Of the following, the MOST appropriate next step with regards to the antimicrobial management of this patient is

A. ampicillin
B. azithromycin
C. ceftazidime
Claim Credit
D. ceftriaxone
Evaluation
E. no antimicrobial needed
My Learning Plan

Submit   Reset  

Page 73 of 115

Contact Us
Links may open in separate window
Copyright © 2016   American Academy of Pediatrics. All rights reserved. Pop­up Blocker may need to be disabled

http://2016.prepid.courses.aap.org/script/august?q­page=3&req=201612122144256543 1/1
12/13/2016 August

Welcome  mohammed alsaiary [ Logout ]

Home PREP Pearls FAQs My Bookmarks CME Information

Overview Home  > August

Editorial Board
August   Enter Keyword Search
January
Question View:   All (8) Jump to Question
February
Print this Page Add to my Bookmarks Page 73 of 115
March

April Assessment History Mode: Learner  Exam 

May ASSESSMENT PROGRESS:  Total Questions:  8  Questions Answered:  3  Correct Answers:  1


June

July You currently have 5 questions unanswered in this assessment

August

September
Question: 3
October
You are asked to evaluate an asymptomatic neonate who was born earlier today. The mother of this infant had an untreated
November Neisseria gonorrhoeae infection during pregnancy. Neonatal ocular prophylaxis was administered to the newborn shortly after
delivery.
December
Of the following, the MOST appropriate next step with regards to the antimicrobial management of this patient is

A. ampicillin
B. azithromycin
C. ceftazidime
Claim Credit
D. ceftriaxone
Evaluation
E. no antimicrobial needed
My Learning Plan

Incorrect View Peer Results
Correct Answer: D Average Percent Correct:  62.50%

Newborns such as the one in this vignette who are born to mothers with untreated Neisseria
gonorrhoeae infection are at high risk for infection themselves, and they should be given a single dose
of ceftriaxone, irrespective of signs or symptoms. Newborns with gonococcal ophthalmia neonatorum
should be treated with ceftriaxone, along with lavage of the infected eye frequently until discharge is
eliminated. Such neonates should be hospitalized and evaluated for disseminated gonococcal
infection. Neonates born to women with untreated cervical infection with N gonorrhoeae have a 30%
risk of acquiring ophthalmia neonatorum caused by N gonorrhoeae, with the risk decreasing to about
5% if neonatal ocular prophylaxis is administered. Prophylaxis against gonococcal ophthalmia neonatorum, usually with
erythromycin, is recommended for administration to all newborns as soon as possible after delivery.

Mother­to­child transmission of group B streptococcus (GBS) occurs in utero or intrapartum and approximately 50% of
newborns of mothers colonized with GBS become colonized by GBS themselves. The current approach for prevention of
GBS infections is to provide chemoprophylaxis to the colonized mother according to national guidelines from the US Centers
for Disease Control and Prevention that have been endorsed by the American Academy of Pediatrics (AAP) and the
American College of Obstetricians and Gynecologists. The Table shows the indications and nonindications for intrapartum
antibiotic prophylaxis for prevention of early­onset GBS disease.

Management of the neonate whose mother had vaginal­rectal cultures positive for GBS is as follows:

Well­appearing infants of any gestational age whose mother received adequate intrapartum GBS prophylaxis (≥4 hours
of penicillin, ampicillin, or cefazolin before delivery) should be observed for ≥48 hours, and no routine diagnostic testing
is recommended. Such infants can be discharged home as early as 24 hours after delivery, assuming that other
discharge criteria have been met, ready access to medical care exists, and that a person able to comply fully with
instructions for home observation will be present.  

For well­appearing infants born to mothers who had an indication for GBS prophylaxis but received no or inadequate
prophylaxis, if the infant is well­appearing at or older than 37 weeks and 0 days’ gestational age and the duration of
membrane rupture before delivery was less than 18 hours, then the infant should be observed for 48 hours or more, and
no routine diagnostic testing is recommended, although some experts would recommend a complete blood cell count
(CBC) with differential and platelets at 6 to 12 hours of life.  

If the infant is well­appearing and either younger than 37 weeks and 0 days’ gestational age or the duration of
membrane rupture before delivery was greater than or at 18 hours, then the infant should undergo a limited evaluation
(includes blood culture at birth) and a CBC with differential and platelets (at birth and/or at 6–12 hours of life) and

http://2016.prepid.courses.aap.org/script/august?req=201612122144500294&status=submit# 1/2
12/13/2016 August
observation for 48 hours or more. If signs of sepsis develop, a full diagnostic evaluation (includes a blood culture, a CBC
including white blood cell differential and platelet counts, chest radiograph [if respiratory abnormalities are present], and
lumbar puncture [if patient is stable enough to tolerate procedure and sepsis is suspected]) should be conducted and
antibiotic therapy initiated. Antibiotic therapy should be directed toward the most common causes of neonatal sepsis,
including intravenous ampicillin for GBS and coverage for other organisms (including Escherichia coli and other Gram­
negative pathogens) and should take into account local antibiotic resistance patterns.

In general, Chlamydia trachomatis infection of the neonate results from exposure during delivery. Newborns born to women
with C trachomatis infection of the cervix have a 50% to 75% risk of acquisition of C trachomatis infection. Among newborns
exposed to C trachomatis, the risk of conjunctivitis is 20% to 50% and the risk of pneumonia is 5% to 20%. A major clinical
manifestation of neonatal infection with C trachomatis is ophthalmia neonatorum. Other causes of ophthalmia neonatorum
must be considered, especially that caused by N gonorrhoeae. C trachomatis pneumonia in infants generally occurs at 3 to 12
weeks of age. Pregnant women should be screened for C trachomatis; if detected, pregnant women should be treated in
order to prevent adverse pregnancy outcomes, postpartum complications, and newborn infection. Prophylaxis to newborns of
women with untreated chlamydial infection has limited efficacy and is not recommended. Instead, such newborns should be
monitored to ascertain if infection does occur and, if so, to be sure that appropriate therapy is given.

Guidelines have been developed by the AAP for the management of a neonate who is delivered to a mother with active
herpetic genital lesions (Figure). The risk of transmitting herpes simplex virus (HSV) to an infant during delivery is determined
in part by the mother’s previous immunity to HSV. Women with primary genital HSV infections who are shedding HSV at
delivery are 10 to 30 times more likely to transmit the virus to their newborn infants than are women with recurrent HSV
infection who are shedding virus at delivery. With the availability of commercial serological tests that reliably can distinguish
type­specific HSV antibodies, it is now possible to determine the type of maternal infection. The management algorithm uses
both serological and virological studies to determine the risk of HSV transmission to the neonate and tailors management
accordingly. Before acyclovir treatment is initiated, specimens should be obtained from multiple sites (buffy coat,
cerebrospinal fluid, rectum, nose, eyes, mouth, and pharynx).

PREP Pearls
In addition to routine newborn ocular prophylaxis, neonates born to mothers with untreated Neisseria gonorrhoeae
infections should receive ceftriaxone.

Management of the neonate whose mother had vaginal­rectal cultures positive for group B streptococcus is determined by
the gestational age of the neonate, the duration of membrane rupture and the adequacy of maternal antibiotic prophylaxis.

Prophylaxis to newborns of women with untreated chlamydial infection has limited efficacy and is not recommended.

Guidelines have been developed by the American Academy of Pediatrics for the management of a neonate born to a
mother with active genital herpes simplex virus lesions.

American Board of Pediatrics Content Specification(s)
Plan the management of a neonate born to a mother with known genital tract culture positive for certain pathogens (eg,
C.trachomatis, N. gonorrhoeae, group B streptococci, herpes simplex virus)

Suggested Readings
Darville T, Rours GIJG. Chlamydia trachomatis. In: Long SS, Pickering LK, Prober CG, eds. Principles and Practice of
Pediatric Infectious Diseases. 4th ed. New York, NY: Saunders Elsevier; 2012:883­889.

Edwards MS, Baker CJ. Streptococcus agalactiae (Group B Streptococcus). In: Long SS, Pickering LK, Prober CG, eds.
Principles and Practice of Pediatric Infectious Diseases. 4th ed. New York, NY: Saunders Elsevier; 2012:707­712.

Hsu KK, Rice PA, Lieberman JM. Neisseria gonorrhoeae. In: Long SS, Pickering LK, Prober CG, eds. Principles and Practice
of Pediatric Infectious Diseases. 4th ed. New York, NY: Saunders Elsevier; 2012:741­748.

Kimberlin DW, Baley J, Committee on Infectious Diseases, Committee on Fetus and Newborn. Guidance on management of
asymptomatic neonates born to women with active genital herpes lesions. Pediatrics. 2013;131(2):e635­e646. DOI:
http://dx.doi.org/10.1542/peds.2012­3216

Prober CG. Herpes simplex virus. In Long SS, Pickering LK, Prober CG, eds. Principles and Practice of Pediatric Infectious
Diseases. 4th ed. New York, NY: Saunders Elsevier; 2012:1026­1035.

US Centers for Disease Control and Prevention. Prevention of perinatal group B streptococcal infection: revised guidelines
from CDC, 2010. MMWR Morb Mortal Wkly Rep. 2010;59(RR­10):1­31. http://www.cdc.gov/mmwr/pdf/rr/rr5910.pdf. Accessed
September 23, 2014

Comment On This Question

Page 73 of 115

Contact Us
Links may open in separate window
Copyright © 2016   American Academy of Pediatrics. All rights reserved. Pop­up Blocker may need to be disabled

http://2016.prepid.courses.aap.org/script/august?req=201612122144500294&status=submit# 2/2
12/13/2016 August

Welcome  mohammed alsaiary [ Logout ]

Home PREP Pearls FAQs My Bookmarks CME Information

Overview Home  > August

Editorial Board
August   Enter Keyword Search
January
Question View:   All (8) Jump to Question
February
Print this Page Add to my Bookmarks Page 74 of 115
March

April Assessment History Mode: Learner  Exam 

May ASSESSMENT PROGRESS:  Total Questions:  8  Questions Answered:  3  Correct Answers:  1


June

July You currently have 5 questions unanswered in this assessment

August

September
Question: 4
October
A 3­year­old girl is admitted with fever, mouth sores, and irritability. Her temperature is 38.3°C, heart rate is 130 beats/min,
November respiratory rate is 24 breaths/min, and blood pressure is 90/60 mm Hg. Physical examination is significant for an ill­appearing child
with mild photophobia and meningismus. You note several oral lesions (Figure 1), and a maculopapular rash on her hands and feet.
December
Laboratory evaluation reveals:

Peripheral white blood cell count (WBC), 8,000 cells/μL (8.0 x 109/L) (35% segmented neutrophils, 45% lymphocytes, and 20%
monocytes)  

Hemoglobin, 12 g/dL (120 g/L)  
Claim Credit
Platelet count, 220 x 103/μL (220 x 109/L)  
Evaluation
Cerebrospinal fluid (CSF) WBC, 300 cells/μL (35% segmented neutrophils, 45% lymphocytes, and 20% monocytes)  
My Learning Plan
CSF red blood cell count, 50 cells/μL 

CSF protein, 45 mg/dL  

CSF glucose, 50 mg/dL

Of the following, the MOST likely etiology of this girl’s infection is

A. coxsackievirus A
B. enterovirus 71
C. herpes simplex virus type 1
D. parechovirus 3

Submit   Reset  

Page 74 of 115

Contact Us
Links may open in separate window
Copyright © 2016   American Academy of Pediatrics. All rights reserved. Pop­up Blocker may need to be disabled

http://2016.prepid.courses.aap.org/script/august?q­page=4&req=201612122144504200# 1/1
12/13/2016 August

Welcome  mohammed alsaiary [ Logout ]

Home PREP Pearls FAQs My Bookmarks CME Information

Overview Home  > August

Editorial Board
August   Enter Keyword Search
January
Question View:   All (8) Jump to Question
February
Print this Page Add to my Bookmarks Page 74 of 115
March

April Assessment History Mode: Learner  Exam 

May ASSESSMENT PROGRESS:  Total Questions:  8  Questions Answered:  4  Correct Answers:  1


June

July You currently have 4 questions unanswered in this assessment

August

September
Question: 4
October
A 3­year­old girl is admitted with fever, mouth sores, and irritability. Her temperature is 38.3°C, heart rate is 130 beats/min,
November respiratory rate is 24 breaths/min, and blood pressure is 90/60 mm Hg. Physical examination is significant for an ill­appearing child
with mild photophobia and meningismus. You note several oral lesions (Figure 1), and a maculopapular rash on her hands and feet.
December
Laboratory evaluation reveals:

Peripheral white blood cell count (WBC), 8,000 cells/μL (8.0 x 109/L) (35% segmented neutrophils, 45% lymphocytes, and 20%
monocytes)  

Hemoglobin, 12 g/dL (120 g/L)  
Claim Credit
Platelet count, 220 x 103/μL (220 x 109/L)  
Evaluation
Cerebrospinal fluid (CSF) WBC, 300 cells/μL (35% segmented neutrophils, 45% lymphocytes, and 20% monocytes)  
My Learning Plan
CSF red blood cell count, 50 cells/μL 

CSF protein, 45 mg/dL  

CSF glucose, 50 mg/dL

Of the following, the MOST likely etiology of this girl’s infection is

A. coxsackievirus A
B. enterovirus 71
C. herpes simplex virus type 1
D. parechovirus 3

Incorrect View Peer Results

Correct Answer: B Average Percent Correct:  32.81%

The girl in the vignette has hand, foot, and mouth disease (HFMD) accompanied by meningitis. While
both coxsackievirus A serotypes and enterovirus 71 are associated with HFMD, enterovirus 71 has
been associated with more severe infection, including central nervous system (CNS) disease (eg,
aseptic meningitis, acute flaccid paralysis, rhombencephalitis). Children with uncomplicated HFMD
present with low­grade fever, an oral enanthem (lesions on the tongue and buccal mucosa), and an
exanthem. The exanthem can occur on the hands (Figure 2), feet (Figure 3), upper arms, thighs, and
buttocks, and can be vesicular, macular, and maculopapular. The enanthem can occur without the
exanthem and vice versa. The diagnosis of uncomplicated HFMD is made clinically. When etiologic confirmation is necessary
because of severe disease or complications (eg, CNS infection, heart failure, pulmonary edema, pulmonary hemorrhage),
viral culture and polymerase chain reaction assays are used. Treatment is supportive.

Coxsackievirus A serotypes also can cause herpangina. The onset of high fever and mouth pain is accompanied by an
enanthem involving the soft palate, tonsils, and uvula (Figure 4). Lesions typically do not occur on the tongue and buccal
mucosa. The CNS involvement is rare and occurs when herpangina is caused by enterovirus 71.

Herpes simplex virus (HSV) infection during childhood most commonly manifests as gingivostomatitis (Figure 5). Most
children present with fever and ulcerative lesions involving the gingiva and oral mucous membranes. Perioral vesicular
lesions are common. Herpes simplex virus meningitis is uncommon in young, healthy children and is not accompanied by oral
lesions. It usually is associated with genital HSV type 2 infection. Herpes simplex virus meningoencephalitis outside of the

http://2016.prepid.courses.aap.org/script/august?req=201612122145288732&status=submit 1/2
12/13/2016 August
neonatal period generally occurs in older children and adults. Human parechovirus infections usually occur in young infants
and can cause sepsis, meningitis, encephalitis, and hepatitis with coagulopathy, but do not usually occur in the mouth.

Noninfectious causes of stomatitis include cancer chemotherapy (ie, mucositis), neutropenia, Behcet syndrome, Stevens­
Johnson syndrome, periodic fever with aphthous stomatitis, pharyngitis and adenitis syndrome, and trauma. These conditions
are generally not accompanied by meningitis.

PREP Pearls
Coxsackievirus A serotypes and enterovirus 71 are associated with hand, foot, and mouth disease.

Enterovirus 71 has been associated with more severe infection, including central nervous system disease, heart failure,
and pulmonary edema and hemorrhage.

The most common manifestation of herpes simplex virus infection in children is ginginvostomatitits.

American Board of Pediatrics Content Specification(s)
Recognize the causes of noninfectious stomatitis (Behcet syndrome, Stevens‑Johnson syndrome, cancer
chemotherapeutic drugs)

Recognize the organism‑specific clinical manifestations of viral stomatitis

Suggested Readings
American Academy of Pediatrics. Enterovirus (Nonpoliovirus). In: Kimberlin DW, Brady MT, Jackson MA, Long SS, eds. Red
Book: 2015 Report of the Committee on Infectious Diseases. 30th ed. Elk Grove Village, IL: American Academy of Pediatrics;
2015: 333­336.

American Academy of Pediatrics. Herpes simplex. In: Kimberlin DW, Brady MT, Jackson MA, Long SS, eds. Red Book: 2015
Report of the Committee on Infectious Diseases. 30th ed. Elk Grove Village, IL: American Academy of Pediatrics; 2015: 434.

American Academy of Pediatrics. Human parechovirus infections. In: Kimberlin DW, Brady MT, Jackson MA, Long SS, eds.
Red Book: 2015 Report of the Committee on Infectious Diseases. 30th ed. Elk Grove Village, IL: American Academy of
Pediatrics; 2015: 592.

Comment On This Question

Page 74 of 115

Contact Us
Links may open in separate window
Copyright © 2016   American Academy of Pediatrics. All rights reserved. Pop­up Blocker may need to be disabled

http://2016.prepid.courses.aap.org/script/august?req=201612122145288732&status=submit 2/2
12/13/2016 August

Welcome  mohammed alsaiary [ Logout ]

Home PREP Pearls FAQs My Bookmarks CME Information

Overview Home  > August

Editorial Board
August   Enter Keyword Search
January
Question View:   All (8) Jump to Question
February
Print this Page Add to my Bookmarks Page 75 of 115
March

April Assessment History Mode: Learner  Exam 

May ASSESSMENT PROGRESS:  Total Questions:  8  Questions Answered:  4  Correct Answers:  1


June

July You currently have 4 questions unanswered in this assessment

August

September
Question: 5
October
You are consulted by the neonatologist on a 5­day­old baby who has just been transferred from an outside hospital to the intensive
November care unit with hypertonicity and possible seizures. He was born in a rural area and delivered at home by the mother’s relatives. The
mother had no prenatal care and is unimmunized. Ampicillin, gentamicin, and acyclovir have been started pending results of the
December
sepsis evaluation. The concern is that the baby has neonatal tetanus.

Of the following, the MOST appropriate management, in addition to metronidazole, for this neonate is

A. tetanus immune globulin
B. tetanus immune globulin and wide excision of the umbilical stump
Claim Credit C. tetanus toxoid and tetanus immune globulin
Evaluation D. tetanus toxoid and wide excision of the umbilical stump

My Learning Plan

Submit   Reset  

Page 75 of 115

Contact Us
Links may open in separate window
Copyright © 2016   American Academy of Pediatrics. All rights reserved. Pop­up Blocker may need to be disabled

http://2016.prepid.courses.aap.org/script/august?q­page=5&req=201612122145293107 1/1
12/13/2016 August

Welcome  mohammed alsaiary [ Logout ]

Home PREP Pearls FAQs My Bookmarks CME Information

Overview Home  > August

Editorial Board
August   Enter Keyword Search
January
Question View:   All (8) Jump to Question
February
Print this Page Add to my Bookmarks Page 75 of 115
March

April Assessment History Mode: Learner  Exam 

May ASSESSMENT PROGRESS:  Total Questions:  8  Questions Answered:  5  Correct Answers:  1


June

July You currently have 3 questions unanswered in this assessment

August

September
Question: 5
October
You are consulted by the neonatologist on a 5­day­old baby who has just been transferred from an outside hospital to the intensive
November care unit with hypertonicity and possible seizures. He was born in a rural area and delivered at home by the mother’s relatives. The
mother had no prenatal care and is unimmunized. Ampicillin, gentamicin, and acyclovir have been started pending results of the
December
sepsis evaluation. The concern is that the baby has neonatal tetanus.

Of the following, the MOST appropriate management, in addition to metronidazole, for this neonate is

A. tetanus immune globulin
B. tetanus immune globulin and wide excision of the umbilical stump
Claim Credit C. tetanus toxoid and tetanus immune globulin
Evaluation D. tetanus toxoid and wide excision of the umbilical stump

My Learning Plan

Incorrect View Peer Results
Correct Answer: A Average Percent Correct:  46.88%

As a result of widespread immunization, neonatal tetanus has become extremely rare in the United
States, with less than 40 cases reported annually since 1999. However, it is still prevalent in developing
countries where rates of immunization may be suboptimal. Pregnant women in particular may not be
adequately immunized and umbilical cord care practices may not be conducted in a sterile
environment. Therefore, given the available history, besides the routine bacterial and viral causes of
neonatal sepsis, it is important to consider tetanus in the differential diagnosis of this neonate.

The most appropriate management for suspected neonatal tetanus would be to give tetanus immune globulin in addition to
metronidazole, which is effective in decreasing the number of vegetative forms of Clostridium tetani. In cases of neonatal
tetanus, excision of the umbilical stump is no longer recommended. Immunization with diphtheria and tetanus toxoids as well
as acellular pertussis vaccine (DTaP) should be given on the standard schedule. Supportive care such as the management of
spasms with benzodiazepines, mechanical ventilation to protect the airway in case of severe spasms, and adequate nutrition
are also important. Generally, these neonates should be cared for in a very quiet environment because even the slightest
stimulation (such as from movement or bright light) may trigger the onset of spasms.

Tetanus is caused by a neurotoxin produced by C tetani, an anaerobic bacterium which has a worldwide distribution. It is
particularly common in warmer climates and during the summer months. The organism is a normal inhabitant of the soil, as
well as human and animal intestines. Spores of C tetani are ubiquitous in the environment, where they lie dormant until
anaerobic conditions favorable for transformation to the pathogenic vegetative form develop. Organisms then multiply rapidly
in wounds contaminated with C tetani and elaborate toxins. The vegetative forms of C tetani produce a potent plasmid­
encoded exotoxin (tetanospasmin), which binds to gangliosides at the myoneural junction of skeletal muscle and on neuronal
membranes in the spinal cord, thus blocking inhibitory responses to motor neurons by blocking γ­aminobutyric acid and
glycine. Therefore, severe muscular spasms develop, including trismus (spasm of the masseter muscle causing “lockjaw”)
and risus sardonicus (spasm of the facial muscles) (Figure 1 and Figure 2). Spasms are often exaggerated by any external
stimuli such as mentioned previously. Blockage of transmission of inhibitory responses at the neuromuscular junction can also
result in rigid paralysis. Small amounts of neurotoxin can also ascend the neural axis and cause localized tetanus. The timing
of onset of muscle involvement is proportional to the distance of the affected neuronal pathway from the site of injury.

The effect of tetanospasmin is permanent and recovery occurs by regeneration of new nerve terminals, a process that takes 6
to 8 weeks. The action of the toxin on the brain and sympathetic nervous system are not well documented and consciousness
is generally not impaired. Autonomic dysfunction, such as tachycardia, labile blood pressure, and arrhythmias, may often be
present. A second toxin with hemolytic and cytolytic properties, tetanolysin, is also elaborated. There are 4 clinical forms of
tetanus that reflect host factors and the site of inoculation: local, generalized, neonatal, and cephalic.

http://2016.prepid.courses.aap.org/script/august?req=201612122145585139&status=submit 1/2
12/13/2016 August
Risk factors for the acquisition of tetanus include wounds contaminated with dirt, excreta, and foreign bodies (nails, splinters).
Deep puncture wounds, as well as those with devitalized tissue such as during crush injuries, are at greatest risk because the
conditions are ideal for the organism to multiply and elaborate toxin. Other wounds at risk include surgical wounds, those
associated with parenteral drug abuse, and animal­related injuries. In 20% of cases, no known risk factor can be identified.
The incubation period is generally between 3 to 21 days; however, shorter incubation periods have been associated with
more heavily contaminated wounds and have a worse prognosis. Symptoms usually appear between 4 to 14 days in cases of
neonatal tetanus.

Management of a patient with tetanus includes both specific treatment, as well as supportive care. Human tetanus immune
globulin (TIG) is the mainstay of treatment and is given intramuscularly in a single dose (3,000­6,000 U), although the optimal
therapeutic dose has not been established, some experts recommend 500 U, which appears to be as effective and causes
less discomfort. Infiltration of part of the dose around the wound is recommended, however, the efficacy of this approach is
not proven. Use of the currently available preparation of TIG for intravenous or intrathecal administration has not been
approved in the United States.

While the use of immune globulin intravenous has not been licensed in cases of tetanus by the US Food and Drug
Administration, it can be considered for treatment in cases where TIG is not available. The dose used is 200 to 400 mg/kg. In
countries where TIG is not available, equine tetanus antitoxin in a dose of 1,500 to 3,000 U may be used after appropriate
desensitization if indicated. This product is no longer available in the United States.

In addition to the use of antitoxin, all wounds should be thoroughly cleaned and debrided, although extensive debridement of
puncture wounds is not needed and, as mentioned previously in the case of neonatal tetanus, neither is wide excision of the
umbilical stump. Oral metronidazole (30 mg/kg per day in 4 divided doses) should be used for reduction in the number of
vegetative spores. An alternate to this is parental penicillin G (100,000 U/kg per day in 4­6 divided doses). Tetracycline,
although not approved for children younger than 8 years of age, is another option. Treatment should be for 7 to 10 days.

As in the case with neonatal tetanus, supportive care with benzodiazepines to control tetanic spasms is important because
morbidity and mortality are related to the extent and severity of the spasms, which may prove to be fatal. In severe cases,
neuromuscular blockade may be indicated. Respiratory and nutritional support are also important. In addition, the use of
general antimicrobial therapy to treat secondary infections of wounds may be considered. If present, specific therapy directed
towards autonomic dysfunction may also be indicated. Finally, active immunization against tetanus should always be
undertaken during convalescence because tetanus disease may not result in immunity. Tetanus prophylaxis in routine wound
management is outlined in the Table.

PREP Pearls
Local wound care, tetanus immune globulin, and metronidazole are the mainstay of treatment for tetanus.

Supportive care with benzodiazepines or neuromuscular blockage (in severe cases) is also critical.

Wide excision of the umbilical stump is no longer recommended in cases of neonatal tetanus.

Immunization with diphtheria and tetanus toxoids, as well as acellular pertussis vaccine (DTaP) should be given on the
standard schedule because tetanus disease does not result in immunity.

American Board of Pediatrics Content Specification(s)
Plan the management of a patient with tetanus (effective antimicrobial, immune globulin, or antitoxin, benzodiazepines)

Know the risk factors for tetanus prone injuries (crush injury, soil contamination), and manage tetanus‑prone injuries
appropriately

Understand the pathogenesis of tetanus

Suggested Readings
American Academy of Pediatrics. Tetanus. In: Kimberlin DW, Brady MT, Jackson MA, Long SS, eds. Red Book: 2015 Report
of the Committee on Infectious Diseases. 30th ed. Elk Grove Village, IL: American Academy of Pediatrics; 2015: 773­778.

Brook I. Approach to the diagnosis and management of gastrointestinal tract infections. In: Long SS, Pickering LK, Prober
CG, eds. Principles and Practice of Pediatric Infectious Diseases. 4th ed. Philadelphia, PA: Saunders Elsevier; 2012: 966­
970.

Comment On This Question

Page 75 of 115

Contact Us
Links may open in separate window
Copyright © 2016   American Academy of Pediatrics. All rights reserved. Pop­up Blocker may need to be disabled

http://2016.prepid.courses.aap.org/script/august?req=201612122145585139&status=submit 2/2
12/13/2016 August

Welcome  mohammed alsaiary [ Logout ]

Home PREP Pearls FAQs My Bookmarks CME Information

Overview Home  > August

Editorial Board
August   Enter Keyword Search
January
Question View:   All (8) Jump to Question
February
Print this Page Add to my Bookmarks Page 76 of 115
March

April Assessment History Mode: Learner  Exam 

May ASSESSMENT PROGRESS:  Total Questions:  8  Questions Answered:  5  Correct Answers:  1


June

July You currently have 3 questions unanswered in this assessment

August

September
Question: 6
October
You are seeing an 18­year­old homosexual young man to review adherence to his antiretroviral medication regimen. His most recent
November CD4 T­cell count is 1,800 cells/µL, and his HIV RNA load is undetectable. The regimen has been very well tolerated, and he only
misses doses “once in a while.” He informs you that his sexual partner, who is also HIV­infected, takes different medications because
December
he “has hepatitis B.” Your patient asks you whether he should also be taking different medications so that he does not get infected
with hepatitis B. You explain that hepatitis B transmission is best prevented by both safe sexual practices and by vaccination.
Together, you review the patient’s immunization record and see that he received doses of hepatitis B vaccine at birth, at 2 months of
age, and at 9 months of age.

Of the following, the BEST advice to your patient regarding hepatitis B vaccination is that
Claim Credit
A. he is fully immunized and no additional vaccine doses are needed
Evaluation
B. he should receive a single booster dose of vaccine today
My Learning Plan C. he should receive another 3­dose series of vaccine starting today
D. he should undergo testing for hepatitis B surface antibody today

Submit   Reset  

Page 76 of 115

Contact Us
Links may open in separate window
Copyright © 2016   American Academy of Pediatrics. All rights reserved. Pop­up Blocker may need to be disabled

http://2016.prepid.courses.aap.org/script/august?q­page=6&req=201612122145589045 1/1
12/13/2016 August

Welcome  mohammed alsaiary [ Logout ]

Home PREP Pearls FAQs My Bookmarks CME Information

Overview Home  > August

Editorial Board
August   Enter Keyword Search
January
Question View:   All (8) Jump to Question
February
Print this Page Add to my Bookmarks Page 76 of 115
March

April Assessment History Mode: Learner  Exam 

May ASSESSMENT PROGRESS:  Total Questions:  8  Questions Answered:  6  Correct Answers:  1


June

July You currently have 2 questions unanswered in this assessment

August

September
Question: 6
October
You are seeing an 18­year­old homosexual young man to review adherence to his antiretroviral medication regimen. His most recent
November CD4 T­cell count is 1,800 cells/µL, and his HIV RNA load is undetectable. The regimen has been very well tolerated, and he only
misses doses “once in a while.” He informs you that his sexual partner, who is also HIV­infected, takes different medications because
December
he “has hepatitis B.” Your patient asks you whether he should also be taking different medications so that he does not get infected
with hepatitis B. You explain that hepatitis B transmission is best prevented by both safe sexual practices and by vaccination.
Together, you review the patient’s immunization record and see that he received doses of hepatitis B vaccine at birth, at 2 months of
age, and at 9 months of age.

Of the following, the BEST advice to your patient regarding hepatitis B vaccination is that
Claim Credit
A. he is fully immunized and no additional vaccine doses are needed
Evaluation
B. he should receive a single booster dose of vaccine today
My Learning Plan C. he should receive another 3­dose series of vaccine starting today
D. he should undergo testing for hepatitis B surface antibody today

Incorrect View Peer Results

Correct Answer: D Average Percent Correct:  71.88%

The patient described in the vignette is at increased risk for infection with hepatitis B because he is
HIV­infected and his sexual partner is known to be hepatitis B­infected. The patient is fully immunized
against hepatitis B, as there is clear documentation that he received 3 doses of vaccine at appropriate
intervals during infancy, but given his high­risk circumstances, he may need additional doses of
vaccine if serologic evidence of vaccine failure or waning immunity is demonstrated. He should
therefore undergo serologic testing for the presence of antibody against hepatitis B surface antigen.
The results of the testing will guide further management. In the high­risk circumstances described,
serologic testing to determine if the patient is already infected with hepatitis B should also be considered.

Antibody testing to document seroconversion among patients who are immunized with hepatitis B vaccine should not be
considered routine, but is recommended for individuals where future clinical management is influenced by knowing the
patient’s hepatitis B immune status (Table 1). Postvaccination antibody testing can be considered in individuals with
occupational exposure to blood and body fluids including healthcare students and professionals, emergency responders, and
public safety workers, and should be performed in such individuals if there is a known exposure to blood or body fluids.
Testing to document seroprotection from prior vaccine may also be required for employment or matriculation. Similarly,
postvaccination serologic testing should be performed on patients known to have potential or known exposure to hepatitis B
such as infants born to hepatitis B­infected mothers, and those who have sex with or share needles with a hepatitis B­infected
individual. Finally, postvaccination serologic testing should be performed in patients who are known to be at risk for vaccine
failure such as those requiring chronic hemodialysis, and those with underlying immune compromising conditions including
infection with HIV. Annual serologic testing is recommended for immune­compromised patients with known ongoing exposure.

The serologic test that is used as the surrogate for hepatitis B immunity is the hepatitis B surface antibody (HepBsAb) titer. A
HepBsAb concentration of 10 mIU/mL or higher, measured 4 weeks or longer after completion of a 3­dose hepatitis B vaccine
series, given at recommended intervals, is considered a reliable marker of protection against infection. If a patient is tested for
response to vaccine and found to be HepBsAb­negative, testing for the presence of hepatitis B surface antigen, a marker of
infection, is necessary. The detection of hepatitis B surface antigen without development of surface antibody indicates
infection, while absence of both antigen and antibody indicate either waning humoral immunity from prior doses of vaccine or
true primary vaccine failure.

Among patients who are immunized with a 3­dose series of hepatitis B vaccine and found to have HepBsAb titers under 10
mIU/mL, 25% to 50% show a robust response to a single booster dose, indicating an anamnestic response. Studies indicate

http://2016.prepid.courses.aap.org/script/august?req=201612122146238108&status=submit 1/3
12/13/2016 August
that 44% to 100% will develop a seroprotective response after completing a second 3­dose series at the recommended
intervals. For patients who are found to have low or absent postvaccination antibody titers, repeating the 3­dose schedule is
usually more practical than administering individual doses and rechecking titers after each dose, but if a titer is measured to
be 10 mIU/mL or greater 4 to 8 weeks after a single booster dose, the patient should be considered immune and no further
doses are needed. Immune­compromised individuals who have ongoing exposure to hepatitis B should have serologic testing
performed annually and should be given booster doses of vaccine to maintain HepBsAb concentrations of 10 mIU/mL or
higher.

Hepatitis B vaccine is included in the universal US recommended immunization schedule as a 3­dose series to be given at
birth, at 1 to 2 months of age, and at 6 to 18 months of age. Infants and children who are not vaccinated starting at birth
should receive 3 doses of hepatitis B vaccine at 0, 1, and 6 months. Increasing the interval between the first and second dose
beyond 4 weeks has little effect on immunogenicity or final antibody concentration. Intervals exceeding 5 months between
dose 2 and dose 3 result in higher antibody concentrations, but do not impact seroconversion rates. Other schedules have
also been used successfully, including doses of vaccine at 6, 10, and 14 weeks of age as one of the recommended schedules
of the World Health Organization’s Expanded Programme on Immunization.

The vaccine immunogen, hepatitis B surface antigen, is purified from recombinant yeast and used in both single antigen
vaccines and in several combination vaccines. All hepatitis B vaccines used in the United States include an aluminum
adjuvant. Single antigen hepatitis B vaccines that contain different concentrations of hepatitis B surface antigen are available.
Specific vaccine selection for individual patients depend on the age of the patient being immunized, the brand of vaccine
used, and, for adults, whether they are undergoing hemodialysis or are otherwise immune suppressed (Table 2).

Babies weighing less than 2,000 g at birth are recognized to have an attenuated response to hepatitis B vaccine in the first
month of life. In this circumstance, the mother’s hepatitis B status drives the vaccine recommendation:

If the baby’s mother is hepatitis B­infected, the baby should be given hepatitis B vaccine and hepatitis B immune
globulin (HBIG) at birth. The birth dose of the vaccine is not counted, and the infant should get 3 additional doses at 1,
2, and 6 months of age. Serologic testing to determine whether the infant has developed a protective antibody response
should be performed at 9 to 18 months of age.  

If the mother’s status is unknown, she should be tested. While awaiting those results, the infant should receive hepatitis
B vaccine and HBIG at birth. The birth dose of the vaccine is not counted and the vaccine series is administered at 1, 2,
and 6 months of age. Postvaccination testing is needed in this group only if the mother is found to be hepatitis B
infected.  

Babies born to mothers who are not hepatitis B­infected should have their hepatitis B vaccine series deferred until 1
month or at hospital discharge. Two subsequent doses are then administered 1 month and 6 months after the first dose.
Postvaccination serologic testing is not routinely recommended for these infants.

Several strategies have been explored in an effort to reduce primary vaccine failure among patients known to be at risk for
being nonresponders. Use of alternate vaccine schedules, use of vaccines with higher antigen concentrations, administration
of antigen intradermally instead of intramuscularly, and use of vaccine formulations that include newer adjuvants have all
shown some promise in small trials, but largely remain experimental. One exception is a specific high­antigen formulation of
hepatitis B vaccine that is specifically available for use in adults (20 years of age and older) who require chronic hemodialysis
(Table 2). Patients in this group may also receive a recommended 4­dose series of hepatitis B vaccine depending on the
specific formulation administered. Higher than standard doses of hepatitis B vaccine may also be more immunogenic in
children and adolescents with immune­compromising conditions, including those requiring dialysis, but specific
recommendations for this indication have not been made.

* Required *
Take Survey  

PREP Pearls
Postvaccination testing to document seroprotection against hepatitis B should be considered in patients with occupational
risks for exposure and in those with sexual or needle sharing partners known to be hepatitis B­infected.

Postvaccination testing should be performed annually in patients with compromised immune systems who are known to
have ongoing exposure risks

The surrogate of immunity to hepatitis B infection is a serum hepatitis B surface antibody concentration of 10 mIU/mL or
higher.

American Board of Pediatrics Content Specification(s)
Know the indications for revaccination for hepatitis B for patients who do not respond to the initial series

Know the approach to patients who have the potential not to respond to hepatitis B vaccine

Suggested Readings
Mast EE, Margolis HS, Fiore AE, et al. A comprehensive immunization strategy to eliminate transmission of hepatitis B virus
infection in the United States: recommendations of the Advisory Committee on Immunization Practices (ACIP) Part I:
immunization of infants, children, and adolescents. MMWR Morbid Mortal Wkly Rep. 2005;54(RR­16):1­32.
http://www.cdc.gov/mmwr/preview/mmwrhtml/rr5416a1.htm

Mast EE, Weinbaum CM, Fiore AE, et al. A comprehensive immunization strategy to eliminate transmission of hepatitis B
virus infection in the United States: recommendations of the Advisory Committee on Immunization Practices (ACIP) Part II:
immunization of adults. MMWR Morbid Mortal Wkly Rep. 2006;54(RR­16):1­25.
http://www.cdc.gov/mmwr/preview/mmwrhtml/rr5516a1.htm

Schillie SF, Murphy TV. Seroprotection after recombinant hepatitis B vaccination among newborn infants: a review. Vaccine.
2013;31(21):2506­2516. DOI: http://dx.doi.org/10.1016/j.vaccine.2012.12.012

http://2016.prepid.courses.aap.org/script/august?req=201612122146238108&status=submit 2/3
12/13/2016 August

Comment On This Question

Page 76 of 115

Contact Us
Links may open in separate window
Copyright © 2016   American Academy of Pediatrics. All rights reserved. Pop­up Blocker may need to be disabled

http://2016.prepid.courses.aap.org/script/august?req=201612122146238108&status=submit 3/3
12/13/2016 August

Welcome  mohammed alsaiary [ Logout ]

Home PREP Pearls FAQs My Bookmarks CME Information

Overview Home  > August

Editorial Board
August   Enter Keyword Search
January
Question View:   All (8) Jump to Question
February
Print this Page Add to my Bookmarks Page 77 of 115
March

April Assessment History Mode: Learner  Exam 

May ASSESSMENT PROGRESS:  Total Questions:  8  Questions Answered:  6  Correct Answers:  1


June

July You currently have 2 questions unanswered in this assessment

August

September
Question: 7
October
A healthy, well­appearing 2­year­old boy presents to the clinic with a chief complaint of nonpruritic, nonpainful rash on his face
November around his right eye of 7 days’ duration. Physical examination reveals an afebrile, well­appearing boy with 12 dome­shaped, waxy,
skin­colored papules, all with a central umbilication, clustered in the area on his face. The rest of his physical examination is
December
unremarkable (Figure).

Of the following, the cause of this child’s rash is MOST likely

A. Enterovirus exanthem
B. Gianotti­Crosti syndrome
Claim Credit C. herpes simplex virus
Evaluation D. molluscum contagiosum

My Learning Plan

Submit   Reset  

Page 77 of 115

Contact Us
Links may open in separate window
Copyright © 2016   American Academy of Pediatrics. All rights reserved. Pop­up Blocker may need to be disabled

http://2016.prepid.courses.aap.org/script/august?q­page=7&req=201612122146243265 1/1
12/13/2016 August

Welcome  mohammed alsaiary [ Logout ]

Home PREP Pearls FAQs My Bookmarks CME Information

Overview Home  > August

Editorial Board
August   Enter Keyword Search
January
Question View:   All (8) Jump to Question
February
Print this Page Add to my Bookmarks Page 77 of 115
March

April Assessment History Mode: Learner  Exam 

May ASSESSMENT PROGRESS:  Total Questions:  8  Questions Answered:  7  Correct Answers:  1


June

July You currently have 1 questions unanswered in this assessment

August

September
Question: 7
October
A healthy, well­appearing 2­year­old boy presents to the clinic with a chief complaint of nonpruritic, nonpainful rash on his face
November around his right eye of 7 days’ duration. Physical examination reveals an afebrile, well­appearing boy with 12 dome­shaped, waxy,
skin­colored papules, all with a central umbilication, clustered in the area on his face. The rest of his physical examination is
December
unremarkable (Figure).

Of the following, the cause of this child’s rash is MOST likely

A. Enterovirus exanthem
B. Gianotti­Crosti syndrome
Claim Credit C. herpes simplex virus
Evaluation D. molluscum contagiosum

My Learning Plan

Incorrect View Peer Results
Correct Answer: D Average Percent Correct:  100.00%

This child has molluscum contagiosum, a common viral infection of the skin in children and
adolescents, caused by a poxvirus. Gianotti­Crosti syndrome, also known as papular acrodermatitis of
childhood, is characterized by erythematous papules distributed symmetrically on the extremities,
buttocks, and face. It is a self­limited, immune­reactive process, most commonly associated with
infection with hepatitis A and B viruses, cytomegaloviruses, Epstein­Barr virus, adenoviruses,
enteroviruses, parainfluenza viruses, and respiratory syncytial virus. Enterovirus exanthems include
morbilliform rash, vesicular rash, urticarial rash, papulopustular rash, or even petechial rash.
Enteroviruses may also cause hand­foot­mouth disease with lesions on mucosa, palms, soles, and diaper area of infants and
young children. Herpes simplex virus causes painful, inflammatory vesicular lesions involving the mucosa of the oral and
genital areas, conjunctiva, and skin.

Molluscum contagiosum is most frequently seen in children younger than 10 years of age and transmission is through close
person­to­person contact or through contact with contaminated fomites. Overcrowding and warm climates may also facilitate
transmission of this skin infection. Skin lesions of molluscum contagiosum in healthy children characteristically are
noninflammatory, and consist of discreet, waxy, skin­colored papules, usually in a group or cluster, on the face, chest, or
extremities. Rarely, it may involve the mucous membranes, the mouth, or genital areas. In some cases, genital molluscum
contagiosum is sexually transmitted. Atypical molluscum contagiosum may occur in immune compromised patients, such as
those with HIV/AIDS or transplant recipients or patients with primary immune disorders. Atypical molluscum contagiosum is
characterized by large, deep lesions, often numbering in the hundreds, and spread diffusely over the entire body. Molluscum
dermatitis may superinfect patients with eczema or atopic dermatitis.

Most cases of molluscum contagiosum in healthy individuals are a benign, self­limited eruption that resolve in a few months or
sometimes in a few years, and therefore can be observed for spontaneous resolution. Treatment for persistent, extensive
lesions or where cosmetic reasons indicate treatment includes referral to a dermatologist for curettage to physically remove
the lesions, or for chemical local treatments with agents such as podophyllin to remove the lesions. In addition, in immune­
compromised hosts with extensive, persistent lesions, immune reconstitution or enhancement by reducing immune
suppression, when feasible, may help facilitate resolution of lesions.

PREP Pearls
Molluscum contagiosum is a common, self­limited skin eruption in healthy young children.

Molluscum contagiosum may cause severe skin eruption in immune­compromised patients.

http://2016.prepid.courses.aap.org/script/august?req=201612122147012484&status=submit 1/2
12/13/2016 August
Molluscum contagiosum lesions may be treated with local treatments to remove the lesions.

American Board of Pediatrics Content Specification(s)
Know means and source of spread of Molluscum contagiosum

Recognize the clinical manifestations of Molluscum contagiosum, including the usual course of the lesions in normal and
immunocompromised hosts

Suggested Readings
American Academy of Pediatrics. Molluscum contagiosum. In: Baker CJ, eds. Red Book Atlas of Pediatric Infectious
Diseases. 2nd ed. Elk Grove Village, IL: American Academy of Pediatrics; 2013:314­315.

Julapalli MR, Levy ML. Viral and fungal skin lesions. In: Cherry J, Demmler­Harrison GJ, Kaplan SL, Steinbach WJ, Hotez P,
eds. Feigin and Cherry’s Textbook of Pediatric Infectious Diseases. 7th ed. Philadelphia, PA: Saunders Elsevier; 2014:782­
792.

Comment On This Question

Page 77 of 115

Contact Us
Links may open in separate window
Copyright © 2016   American Academy of Pediatrics. All rights reserved. Pop­up Blocker may need to be disabled

http://2016.prepid.courses.aap.org/script/august?req=201612122147012484&status=submit 2/2
12/13/2016 August

Welcome  mohammed alsaiary [ Logout ]

Home PREP Pearls FAQs My Bookmarks CME Information

Overview Home  > August

Editorial Board
August   Enter Keyword Search
January
Question View:   All (8) Jump to Question
February
Print this Page Add to my Bookmarks Page 78 of 115
March

April Assessment History Mode: Learner  Exam 

May ASSESSMENT PROGRESS:  Total Questions:  8  Questions Answered:  7  Correct Answers:  1


June

July You currently have 1 questions unanswered in this assessment

August

September
Question: 8
October
You are seeing a 26 week gestation premature neonate in the neonatal intensive care unit for disseminated fungal disease. The
November infant has positive central and peripheral blood, urine and CSF cultures for a Candida spp that is not yet identified. His
ophthalmology examination is positive for retinal lesions consistent with fungal disease.
December
In addition to 5­flucytocine (5­FC), the MOST appropriate antifungal agent to use in the treatment of this neonate is

A. Amphotericin B deoxycholate
B. Caspofungin
C. Fluconazole
Claim Credit
D. Liposomal amphotericin B
Evaluation
E. Voriconazole
My Learning Plan

Submit   Reset  

Page 78 of 115

Contact Us
Links may open in separate window
Copyright © 2016   American Academy of Pediatrics. All rights reserved. Pop­up Blocker may need to be disabled

http://2016.prepid.courses.aap.org/script/august?q­page=8&req=201612122147016391 1/1
12/13/2016 August

Welcome  mohammed alsaiary [ Logout ]

Home PREP Pearls FAQs My Bookmarks CME Information

Overview Home  > August

Editorial Board
August   Enter Keyword Search
January
Question View:   All (8) Jump to Question
February
Print this Page Add to my Bookmarks Page 78 of 115
March

April Assessment History Mode: Learner  Exam 

May ASSESSMENT PROGRESS:  Total Questions:  8  Questions Answered:  8  Correct Answers:  2


June

July You currently have 1 required survey(s) that are incomplete. Please complete the survey(s) for the following
question(s) on this assessment:  Question 6
August

September

October Question: 8
November You are seeing a 26 week gestation premature neonate in the neonatal intensive care unit for disseminated fungal disease. The
infant has positive central and peripheral blood, urine and CSF cultures for a Candida spp that is not yet identified. His
December
ophthalmology examination is positive for retinal lesions consistent with fungal disease.

In addition to 5­flucytocine (5­FC), the MOST appropriate antifungal agent to use in the treatment of this neonate is

A. Amphotericin B deoxycholate
B. Caspofungin
Claim Credit C. Fluconazole
Evaluation D. Liposomal amphotericin B

My Learning Plan E. Voriconazole

Correct View Peer Results

Average Percent Correct:  65.62%

The most appropriate antifungal agent to use in the trTeatment of neonates with fungal meningitis is
amphotericin B deoxycholate. Neonates tolerate this formulation of amphotericin B well and there is
limited data on the use of liposomal amphotericin B in the treatment of fungal meningitis in this
population. Caspofungin and other echinocandins do not achieve adequate CSF concentrations to treat
Candida meningitis and should not be used for treatment. Fluconazole has excellent CSF penetration
and is active against most Candida species, however, treatment outcomes are varied when this agent
is used for first line therapy. Fluconazole may be used as step­down therapy after the patient has been
treated with several weeks of Amphotericin B with or without 5­FC and the Candida isolate has been proven to be susceptible
to fluconazole. Voriconazole has excellent CSF penetration and is active against most Candida species, however, there is
very limited experience on the use of voriconazole in the treatment of neonates with fungal meningitis. It may be used as
second­line therapy but is not recommended for use as first line therapy.

Fungi, in general, are rare causes of meningitis in children. Fungal meningitis is often chronic in nature, and patients may
have few meningeal signs or symptoms which often delays establishing the diagnosis. A large number of fungi and yeasts
may cause meningitis. Fungal meningitis ranges from the relatively common cryptococcal meningitis to the rare meningitis
due to large dimorphic or filamentous (septate and non­septate) fungi. The following agents are the most common causes of
meningitis in children and adults with normal immunologic status: Blastomyces dermatitidis, Coccidioides immitis,
Cryptococcus neoformans, Cladosporium species, Histoplasma capsulatum, and Paracocidioides brasiliensis. Inhalation is
the most common means by which many fungi infect the host. Other fungi may be inoculated directly into the skin; this
inoculation can occur in the outdoor environment or in the hospital setting.

The epidemiology of fungal meningitis is dependent upon a number of different factors including:

1. Geographic location of the person, various exposures or travel to an endemic area. The geographic distribution of
fungal meningitis varies in the United States and worldwide. For example, Histoplasmosis generally occurs in endemic
areas of the the Mississippi valley, while, coccidioidomycosis occurs in the San Joaquin Valley and the desert
Southwest of the United States and Mexico. Cryptococcosis is seen worldwide but is commonly associated with bird
droppings and nesting areas. Blastomycosis generally occurs in states bordering the Mississippi and Ohio River
basins, with occasional outbreaks occurring in the Great Lakes region and Canada.  

2. Immunocompetence of the person. With the exception of endemic and trauma­induced infections, invasive fungal
diseases mainly occurs in immunocompromised patients. Susceptibility patterns vary based on the degree and
category of immune dysfunction. Risk factors for brain infections include: HIV/AIDS, hematapoietic stem cell transplant

http://2016.prepid.courses.aap.org/script/august?req=201612122147262016&status=submit 1/2
12/13/2016 August
(HSCT), lymphoid malignancies, neutropenia, hereditary immune defects, diabetes mellitus, use of
immunosuppressive medications, intravenous drug abuse, and compromise of the blood brain barrier. Indwelling
catheters are a risk factor for candidemia and subsequent disseminated infection. Table 1 shows the fungi that
commonly cause meningitis in immunocompromised and immunocompetent persons.

The clinical manifestations in patients with CNS fungal or yeast meningitis are indistinguishable from chronic bacterial
meningitis. Persons with fungal meningitis generally present with headache, meningismus, nausea, vomiting, photophobia,
papilledema, visual impairment, personality changes, cranial nerve palsies, diminished consciousness, seizures,
hydrocephalus, or complications from increased intracranial pressure. Table 2 shows the most common clinical signs and
symptoms seen in CNS fungal and yeast infections and also some additional information on the organisms that cause fungal
meningitis. In general the CSF findings include a normal or only slightly depressed glucose value and an elevated protein
count. The CSF cell type and white blood cell count is variable making it difficult to distinguish between the different fungal
organisms that cause meningitis.

Other infectious and noninfectious conditions that may have clinical signs and symptoms and CSF indices similar to what is
seen in fungal meningitis include neurosyphilis, cerebral toxoplasmosis, neurocysticercosis, Lyme meningitis,
neurosarcoidosis, rheumatoid meningitis, systemic lupus erythematosus with CNS involvement, neuro­Behcet’s disease,
Sjogren’s syndrome, polyarteritis nodosa, granulomatous angiitis of the central nervous system, carcinomatous meningitis,
leukemia/lymphoma with CNS involvement and dermoid cyst.

PREP Pearls
Blastomyces dermatitidis, Coccidioides immitis, Cryptococcus neoformans, Cladosporium species, Histoplasma
capsulatum, and Paracocidioides brasiliensis are the most common causes of fungal meningitis in children with normal
immune systems.

Geographic location of the person, various exposures or travel to an endemic area and state of immunocompetence are
important epidemiology risk factors for the development of fungal meningitis.

In general, for fungal meningitis, the CSF cell type and count is variable, the glucose is often normal or only slightly
depressed and the protein content is usually elevated making it difficult to distinguish between the different fungal
organisms that cause meningitis and bacterial agents that cause chronic meningitis.

American Board of Pediatrics Content Specification(s)
Recognize the clinical and laboratory findings of fungal meningitis

Formulate a differential diagnosis in a patient with CSF findings characteristic of fungal meningitis to include infectious and
noninfectious causes

Suggested Readings
Honda H, Warren DK. Central nervous system infections: Meningitis and brain abscesses. Infect Dis Clin N Am 2009;23:609­
623 .

Lahoti S, Berger JR. Iatrogenic fungal infections of central nervous system. Curr Neurol Neurosci Rep 2013;13:399.

Pappas PG, Rex JH, Sobel JD, et al. Guidelines for the treatment of candidiasis. Clin Infect Dis 2004;38:161­189.

Romero JR, Jacobs RF. Fungal meningitis. In: Feigin & Cherry’s Textbook of Pediatric Infectious Diseases. Cherry JD, Kaplan
SL, Harrison GJ, Steinbach WJ and Hotez PJ, eds. 7th edition. Elsevier Saunders, Philadelphia, PA, 2014,pgs 473­481.

Scully EP, Baden LR, Katz JT. Fungal brain infections. Curr Opin Neurol 2008;21:347­352.

Sharma RR. Fungal infections of the nervous system: Current perspective and controversies in management. Int J Surg
2010;8:591­601.

Comment On This Question

Page 78 of 115

Contact Us
Links may open in separate window
Copyright © 2016   American Academy of Pediatrics. All rights reserved. Pop­up Blocker may need to be disabled

http://2016.prepid.courses.aap.org/script/august?req=201612122147262016&status=submit 2/2
12/13/2016 September

Welcome  mohammed alsaiary [ Logout ]

Home PREP Pearls FAQs My Bookmarks CME Information

Overview Home  > September

Editorial Board
September   Enter Keyword Search
January
Question View:   All (8) Jump to Question
February
Print this Page Add to my Bookmarks Page 80 of 115
March

April Assessment History Mode: Learner  Exam 

May ASSESSMENT PROGRESS:  Total Questions:  8  Questions Answered:  0  Correct Answers:  0


June

July You currently have 8 questions unanswered in this assessment

August

September
Question: 1
October
A 2­year­old girl presents to the pediatrician’s office with a 2­week history of right­sided otalgia and ear drainage. Her past medical
November history is significant for recurrent otitis media (5 episodes last year), at times with ear drainage, treated with oral antibiotics. One
previous episode of pneumonia was reported. The patient is below 5% for weight and height and the mother reports that she has
December
always been small, as she is a picky eater. The mother is also concerned that she is not hearing well. On examination, the patient is
febrile to 38.5°C. She is crying in pain and has exquisite tenderness on external manipulation of the tragus. She also has facial nerve
palsy. The right ear canal is tender and swollen with debris and purulent discharge. 

Of the following, the MOST appropriate therapy for this child would be

A. ciprofloxacin/dexamethasone otic drops 
Claim Credit
B. intravenous cefepime and vancomycin 
Evaluation
C. ofloxacin otic drops 
My Learning Plan
D. oral amoxicillin/clavulanic acid 

Submit   Reset  

Page 80 of 115

Contact Us
Links may open in separate window
Copyright © 2016   American Academy of Pediatrics. All rights reserved. Pop­up Blocker may need to be disabled

http://2016.prepid.courses.aap.org/script/september?q­page=1&req=201612122150175301 1/1
12/13/2016 September

Welcome  mohammed alsaiary [ Logout ]

Home PREP Pearls FAQs My Bookmarks CME Information

Overview Home  > September

Editorial Board
September   Enter Keyword Search
January
Question View:   All (8) Jump to Question
February
Print this Page Add to my Bookmarks Page 80 of 115
March

April Assessment History Mode: Learner  Exam 

May ASSESSMENT PROGRESS:  Total Questions:  8  Questions Answered:  1  Correct Answers:  0


June

July You currently have 7 questions unanswered in this assessment

August

September
Question: 1
October
A 2­year­old girl presents to the pediatrician’s office with a 2­week history of right­sided otalgia and ear drainage. Her past medical
November history is significant for recurrent otitis media (5 episodes last year), at times with ear drainage, treated with oral antibiotics. One
previous episode of pneumonia was reported. The patient is below 5% for weight and height and the mother reports that she has
December
always been small, as she is a picky eater. The mother is also concerned that she is not hearing well. On examination, the patient is
febrile to 38.5°C. She is crying in pain and has exquisite tenderness on external manipulation of the tragus. She also has facial nerve
palsy. The right ear canal is tender and swollen with debris and purulent discharge. 

Of the following, the MOST appropriate therapy for this child would be

A. ciprofloxacin/dexamethasone otic drops 
Claim Credit
B. intravenous cefepime and vancomycin 
Evaluation
C. ofloxacin otic drops 
My Learning Plan
D. oral amoxicillin/clavulanic acid 

Incorrect View Peer Results
Correct Answer: B Average Percent Correct:  81.97%

Otitis externa is an infection of the superficial skin of the external canal.  Exposure to water or humidity
predisposes the host to infection. On examination, erythema and narrowing of the canal lumen, along
with swelling of the auricle, may be seen. Debris in the canal reflects desquamation of the superficial
skin layer. It is important to differentiate otitis externa from malignant otitis externa, which is an invasive
infection of the cartilage and bone of the external ear canal.  Unlike otitis externa, malignant otitis
externa can lead to bony destruction and severe symptoms. The patient’s prolonged severe otalgia and
otorrhea and her past medical history of recurrent infections raises suspicion for an underlying
immunodeficiency and should make one think of malignant otitis externa.

The organisms involved are typically Staphylococcus aureus and Pseudomonas aeruginosa. Fungal etiologies such as
Candida or Aspergillus species (Aspergillus fumigatus or Aspergillus niger) are rare. Obtaining an adequate specimen for
culture is crucial to identify the microbiologic etiology. Treatment includes adequate surgical debridement to obtain cultures for
organism identification and targeted therapy and for local debridement of necrotic tissue. Antibiotic therapy should be
continued for 6 weeks’ duration, and provide antistaphylococcal and antipseudomonal coverage (eg, vancomycin and
cefepime). Fungal etiologies can be treated with liposomal amphotericin or voriconazole. Therapy may be transitioned from
parenteral to enteral depending on immune status of the patient and once significant clinical improvement has occurred. In
contrast, otitis externa has few complications and can be treated with topical otic quinolone drops with or without steroids
such as dexamethasone or hydrocortisone. Irrigation may be necessary. Topical clotrimazole otic solution can be used for
fungal infections.

Hosts with an underlying immunodeficiency including diabetes mellitus, AIDS, malignancy, and neutropenia are at greater risk
for malignant otitis externa. The diagnosis is clinical and thus one must have a high index of suspicion based on history and
examination. Physical examination findings may include a tender, swollen external auditory canal, with granulation tissue and
exposed bone at the floor of the canal. Facial nerve palsy is seen in 25% of patients. Laboratory evaluation may reveal a
normal white blood cell count and an elevated erythrocyte sedimentation rate. The infection spreads from the external
auditory canal to the skull base and then via venous channels and fascial planes until it leads to bony destruction of the skull
base and commonly the mastoid (Figure).  Complications arise from contiguous spread and may include other nerve palsies
such as glossopharyngeal, accessory and hypoglossal nerves, septic thrombophlebitis of the internal jugular vein and sigmoid
sinus, permanent hearing loss, intracranial abscess, and meningitis.

http://2016.prepid.courses.aap.org/script/september?req=201612122150582176&status=submit# 1/2
12/13/2016 September

PREP Pearls
A high index of suspicion is necessary to make the diagnosis of malignant otitis externa, which can be aggressive with
many complications.

American Board of Pediatrics Content Specification(s)
Know the clinical manifestations and the most likely predisposing factors and causes of otitis externa, including malignant
otitis externa

Recognize the complications of malignant otitis externa

Suggested Readings
Boyce TG. Otitis externa and necrotizing otitis externa. In: Long SS, Pickering LK, Prober CG, eds. Principles and Practice of
Pediatric Infectious Diseases: Expert Consult. 4th ed. Philadelphia, PA: Saunders Elsevier; 2012:220­221.

Handzel O, Halperin D. Necrotizing (malignant) external otitis. Am Fam Physician. 2003;68(2):309­312.
http://www.aafp.org/afp/2003/0715/p309.html

Klein JO. Otitis externa, otitis media, and mastoiditis. In: Bennett JE, Dolin R, Blaser MJ, eds. Mandell, Douglas, and
Bennett's Principles and Practice of Infectious Diseases. 8th ed. Philadelphia, PA: Saunders Elsevier; 2015:767­773.

Comment On This Question

Page 80 of 115

Contact Us
Links may open in separate window
Copyright © 2016   American Academy of Pediatrics. All rights reserved. Pop­up Blocker may need to be disabled

http://2016.prepid.courses.aap.org/script/september?req=201612122150582176&status=submit# 2/2
12/13/2016 September

Welcome  mohammed alsaiary [ Logout ]

Home PREP Pearls FAQs My Bookmarks CME Information

Overview Home  > September

Editorial Board
September   Enter Keyword Search
January
Question View:   All (8) Jump to Question
February
Print this Page Add to my Bookmarks Page 81 of 115
March

April Assessment History Mode: Learner  Exam 

May ASSESSMENT PROGRESS:  Total Questions:  8  Questions Answered:  1  Correct Answers:  0


June

July You currently have 7 questions unanswered in this assessment

August

September
Question: 2
October
A 16­year­old boy presents to the emergency department with a 2­day history of unilateral left­eye pain, a feeling of a foreign body
November sensation, a watery left eye, and photophobia. He also stated that his vision in his left eye was cloudy. This incident is his first
experience with eye disease. He does not wear glasses or contact lenses. He has no history of fever, allergies, travel, or trauma to
December
the eye. Examination of the eye reveals blepharospasm and conjunctival injection. No foreign body was identified. Examination
under slit lamp microscope after instillation of fluorescein dye shows a corneal ulcer.

Of the following, the MOST appropriate active agent to treat this condition is

A. benzalkonium chloride

Claim Credit B. carboxymethyl cellulose

Evaluation C. moxifloxacin
D. proparacaine
My Learning Plan
E. trifluorothymidine

Submit   Reset  

Page 81 of 115

Contact Us
Links may open in separate window
Copyright © 2016   American Academy of Pediatrics. All rights reserved. Pop­up Blocker may need to be disabled

http://2016.prepid.courses.aap.org/script/september?q­page=2&req=201612122150586239 1/1
12/13/2016 September

Welcome  mohammed alsaiary [ Logout ]

Home PREP Pearls FAQs My Bookmarks CME Information

Overview Home  > September

Editorial Board
September   Enter Keyword Search
January
Question View:   All (8) Jump to Question
February
Print this Page Add to my Bookmarks Page 81 of 115
March

April Assessment History Mode: Learner  Exam 

May ASSESSMENT PROGRESS:  Total Questions:  8  Questions Answered:  2  Correct Answers:  0


June

July You currently have 6 questions unanswered in this assessment

August

September
Question: 2
October
A 16­year­old boy presents to the emergency department with a 2­day history of unilateral left­eye pain, a feeling of a foreign body
November sensation, a watery left eye, and photophobia. He also stated that his vision in his left eye was cloudy. This incident is his first
experience with eye disease. He does not wear glasses or contact lenses. He has no history of fever, allergies, travel, or trauma to
December
the eye. Examination of the eye reveals blepharospasm and conjunctival injection. No foreign body was identified. Examination
under slit lamp microscope after instillation of fluorescein dye shows a corneal ulcer.

Of the following, the MOST appropriate active agent to treat this condition is

A. benzalkonium chloride

Claim Credit B. carboxymethyl cellulose

Evaluation C. moxifloxacin
D. proparacaine
My Learning Plan
E. trifluorothymidine

Incorrect View Peer Results

Correct Answer: E Average Percent Correct:  68.85%

This patient has a dendritic corneal ulcer due to a herpes simplex virus (HSV) infection of the epithelial
layer of the cornea. Treatment with a topical antiviral that has activity against HSV is indicated in this
patient.

Trifluorothymidine, also known as trifluridine or TFT, is a pyridine nucleoside antiviral compound that is
highly active in vitro against HSV types 1 and 2. The compound also has moderate activity against
vaccinia virus, varicella zoster virus, cytomegalovirus, and adenoviruses. Phosphorylation is required to
transform the compound into the active form, trifluridine triphosphate. The active triphosphate form inhibits DNA polymerase
activity and viral DNA synthesis. It is structurally related to idoxuridine but has less toxicity when applied topically.

Trifluridine­resistant strains of HSV with altered thymidine kinase activity have been produced in­laboratory and have also
occurred in patients. Trifluridine, supplied as a 1% ophthalmic solution, is only approved for topical use for the treatment of
primary keratoconjunctivitis and recurrent epithelial keratitis due to HSV. Trifluridine was also shown in randomized clinical
trials to be an effective treatment for herpetic keratitis in patients who were also intolerant or nonresponsive to topical
idoxuridine or vidarabine. Re­epithelialization of dendritic ulcers occurs at a mean of 6 days of treatment. Some experts also
recommend acute treatment and chronic suppression with oral acyclovir to shorten duration of primary keratitis and to prevent
sight­threatening, recurrent stromal keratitis caused by reactivation of HSV. Trifluridine has also been used to treat vaccinia
virus keratitis that complicates vaccination, ocular complications of natural smallpox, and keratitis associated with varicella
zoster virus.

Administration of trifluridine may cause local discomfort, irritation, or edema when applied topically to the eye. In addition,
hypersensitivity reactions, contact dermatitis, and keratopathy may occur in some patients.

Other antiviral topical agents that are effective to treat herpetic keratitis include idoxuridine ophthalmic, vidarabine ophthalmic,
acyclovir ophthalmic ointment 3%, ganciclovir ophthalmic gel 0.15%, and brivudine ophthalmic. In clinical trials, topical
trifluridine, acyclovir, and ganciclovir appear superior to topical idoxuridine or vidarabine for treatment of ocular disease due to
HSV. Systemic acyclovir has also been shown to be effective. If acyclovir­resistant herpes keratitis is suspected, systemic
foscarnet is indicated.

Along with topical antiviral therapy, treatment of herpetic keratitis may also include adjunctive therapies if the infection
involves the deeper layer of the cornea (endothelial or stromal keratitis). In this disease, systemic acyclovir or valacyclovir,
topical anti­inflammatory agents (such as dexamethasone or cyclosporine A), and keratoplasty (to debride necrotic corneal

http://2016.prepid.courses.aap.org/script/september?req=201612122153592025&status=submit 1/2
12/13/2016 September
tissue) may be indicated. Recurrences are also common. Patients who have herpetic keratitis, especially if stromal keratitis is
present, should also receive consultation by an ophthalmologist to determine if these therapeutic options are indicated.

Carboxymethyl cellulose, along with water, salts and polymers, is an ingredient in artificial tears and lubricant eye drops used
to treat keratoconjunctivitis sicca (dry eye syndrome). It does not have antiviral properties and would not be effective
treatment for ocular HSV infection.

Moxifloxacin is a fluoroquinolone antibiotic contained in ophthalmic drops used to treat bacterial keratitis but not viral keratitis.

Proparacaine is a topical anesthetic for the eye. It will relieve the pain associated with keratitis, but it will also delay re­
epithelialization and healing of the cornea. Therefore, use of proparacaine should also include consultation with an
ophthalmologist.

Benzalkonium chloride is a chemical with antibacterial properties that is used as a preservative in many different products,
including eye drops. It does not have any known antiviral effects. Eye drops containing benzalkonium chloride may cause
irritation of the eyes.

PREP Pearls
Trifluorothymidine, also known as trifluridine or TFT, is a pyridine nucleoside antiviral compound that is highly active in vitro
against HSV types 1 and 2.

The TFT compound also has moderate activity against vaccinia virus, varicella zoster virus, cytomegalovirus, and
adenoviruses.

Trifluridine was also shown in randomized clinical trials to be an effective treatment for herpetic keratitis in patients who
were also intolerant or nonresponsive to topical idoxuridine or vidarabine.

Trifluridine has also been used to treat vaccinia virus keratitis that complicates vaccination, ocular complications of natural
smallpox, and keratitis associated with varicella zoster virus

American Board of Pediatrics Content Specification(s)
Know the viruses against which trifluorothymidine is active and the indications for use

Suggested Readings
Fillmore GL, Ward TP, Bower KS, Dudenhoefer EJ, Grabenstein J, Berry G, Madigan W. Ocular complications in the
Department of Defense Smallpox Vaccination Program. Ophthalmology. 2004;111(11):2086­2093.
doi:10.1016/j.ophtha.2004.04.027. http://www.sciencedirect.com/science/article/pii/S0161642004008243. Accessed January
15, 2013

Guess S, Stone D, Chodosh J. Evidence­based treatment of herpes simplex virus keratitis: a systematic review. Ocular Surf.
2007;5(3):240­250. doi:10.1016/S1542­0124(12)70614­6.
 http://www.journals.elsevierhealth.com/periodicals/jtos/article/PIIS1542012412706146/fulltext. Accessed January 15, 2013

Wilhelmus KR. Antiviral treatment and other therapeutic interventions for herpes simplex epithelial keratitis. Cochrane
Database Sys Rev. 2010;(12):CD002898. doi:10.1002/14651858.CD002898.pub4.
http://onlinelibrary.wiley.com/doi/10.1002/14651858.CD002898.pub4/full. Accessed January 15, 2013

Comment On This Question

Page 81 of 115

Contact Us
Links may open in separate window
Copyright © 2016   American Academy of Pediatrics. All rights reserved. Pop­up Blocker may need to be disabled

http://2016.prepid.courses.aap.org/script/september?req=201612122153592025&status=submit 2/2
12/13/2016 September

Welcome  mohammed alsaiary [ Logout ]

Home PREP Pearls FAQs My Bookmarks CME Information

Overview Home  > September

Editorial Board
September   Enter Keyword Search
January
Question View:   All (8) Jump to Question
February
Print this Page Add to my Bookmarks Page 82 of 115
March

April Assessment History Mode: Learner  Exam 

May ASSESSMENT PROGRESS:  Total Questions:  8  Questions Answered:  2  Correct Answers:  0


June

July You currently have 6 questions unanswered in this assessment

August

September
Question: 3
October
An 8­year­old boy is referred to your office for evaluation 1 day after stepping on a nail while running through a construction site. He
November was wearing sneakers and the entry point was near his heel.  He was seen in the emergency department last night after the injury
where the puncture site was irrigated and a radiograph revealed no evidence of a foreign body.  White blood cell count at that time
December
was 9,800/µL with 48% neutrophils, 38% lymphocytes, 12% monocytes, 2% eosinophils, and erythrocyte sedimentation rate was 18
mm/hour.

He complains of swelling, pain, and redness at the site of the puncture wound on the base of his foot. He denies fever or chills.

His immunizations are documented as up to date.

Claim Credit Physical examination reveals a well­appearing boy who complains of pain with bearing weight. His vital signs show a temperature of
37.2°C, heart rate of 88 beats/min, and a blood pressure of 105/62 mm Hg.
Evaluation

My Learning Plan His examination is normal except for the heel of his left foot where there is an approximately 5 cm area of mild erythema surrounding
a puncture site. The area is red, warm, and tender to touch. No discharge is noted at the site of the puncture.

Of the following, the BEST next step in management of this child is to

A. administer a tetanus toxoid booster
B. begin treatment with ciprofloxacin 
C. reevaluation in 24 to 48 hours
D. refer to orthopedics for biopsy

Submit   Reset  

Page 82 of 115

Contact Us
Links may open in separate window
Copyright © 2016   American Academy of Pediatrics. All rights reserved. Pop­up Blocker may need to be disabled

http://2016.prepid.courses.aap.org/script/september?q­page=3&req=201612122153596555 1/1
12/13/2016 September

Welcome  mohammed alsaiary [ Logout ]

Home PREP Pearls FAQs My Bookmarks CME Information

Overview Home  > September

Editorial Board
September   Enter Keyword Search
January
Question View:   All (8) Jump to Question
February
Print this Page Add to my Bookmarks Page 82 of 115
March

April Assessment History Mode: Learner  Exam 

May ASSESSMENT PROGRESS:  Total Questions:  8  Questions Answered:  3  Correct Answers:  0


June

July You currently have 5 questions unanswered in this assessment

August

September
Question: 3
October
An 8­year­old boy is referred to your office for evaluation 1 day after stepping on a nail while running through a construction site. He
November was wearing sneakers and the entry point was near his heel.  He was seen in the emergency department last night after the injury
where the puncture site was irrigated and a radiograph revealed no evidence of a foreign body.  White blood cell count at that time
December
was 9,800/µL with 48% neutrophils, 38% lymphocytes, 12% monocytes, 2% eosinophils, and erythrocyte sedimentation rate was 18
mm/hour.

He complains of swelling, pain, and redness at the site of the puncture wound on the base of his foot. He denies fever or chills.

His immunizations are documented as up to date.

Claim Credit Physical examination reveals a well­appearing boy who complains of pain with bearing weight. His vital signs show a temperature of
37.2°C, heart rate of 88 beats/min, and a blood pressure of 105/62 mm Hg.
Evaluation

My Learning Plan His examination is normal except for the heel of his left foot where there is an approximately 5 cm area of mild erythema surrounding
a puncture site. The area is red, warm, and tender to touch. No discharge is noted at the site of the puncture.

Of the following, the BEST next step in management of this child is to

A. administer a tetanus toxoid booster
B. begin treatment with ciprofloxacin 
C. reevaluation in 24 to 48 hours
D. refer to orthopedics for biopsy

Incorrect View Peer Results

Correct Answer: C Average Percent Correct:  18.03%

Osteomyelitis in children may arise from hematogenous spread or direct inoculation. In this vignette,
the concern is related to traumatic injury potentially introducing infection directly into the calcaneus.
This scenario of a nail puncture wound through a sneaker has been associated with pseudomonas
infection. However, such infection is rare even in such a setting and typically presents 3 to 5 days or
more after the injury secondary to the growth of the opportunistic organism in necrotic tissue. Over the
initial 24 to 48 hours after such an injury, pain and swelling may be solely related to the inflammatory
response to the trauma. In the absence of purulence, lack of fever, or more severe symptoms,
reevaluation in 24 to 48 hours would be the most appropriate management of this patient. If there were concern for early
infection, Staphylococcus aureus or group A β­hemolytic Streptococcus would be the likely pathogens, not Pseudomonas.

This boy’s immunizations are up to date, as he is less than 5 years from his last tetanus vaccine, and a tetanus booster is not
needed. As noted previously, Pseudomonas infection is unlikely in this case, and therefore ciprofloxacin is not indicated.
Furthermore, in the setting where Pseudomonas infection is suspected after a nail puncture wound through a sneaker surgical
debridement in conjunction with short course (7 to 14 days), antipseudomonal therapy is important to resolve the infection.
Biopsy is not indicated at this point given the absence of concern for infection.

Other scenarios in which less common organisms may be the cause of osteomyelitis can be related to host factors (eg,
Salmonella in sickle cell disease, Burkholderia cepacia in chronic granulomatous disease) or specific clinical situations (eg,
Pseudomonas aeruginosa in intravenous drug abusers, polymicrobial bone infection after trauma with open fractures). 

Mycobacterium tuberculosis may seed bones or joins during the initial bacillemia after initial pulmonary infection.
Musculoskeletal infection then presents insidiously as a form of reactivation disease. Alternatively, direct extension from a
contiguous pulmonary site of infection can cause vertebral osteomyelitis (Pott disease).

http://2016.prepid.courses.aap.org/script/september?req=201612122154382337&status=submit 1/2
12/13/2016 September
A number of other examples of unusual organisms causing osteomyelitis include mandibular or vertebral actinomycosis;
Brucella infection of bone following consumption of unpasteurized milk products; and Bartonella henselae may also rarely
involve bone in disseminated cat scratch disease. Fungal infections with Coccidioides or Blastomyces may also rarely cause
osteomyelitis, but both of these infections typically present as pulmonary infection. Hematogenous spread can then lead to
boney involvement, however, the osteomyelitis related to either of these organisms is typically subacute or chronic and with
minimal or no pain at the site of bone infection.

PREP Pearls
Puncture wound osteomyelitis occurring after an injury from a nail going through a sneaker typically occurs 3 to 5 days
more after the injury.

Management of pseudomonal infection after a puncture wound injury generally requires surgical debridement of necrotic
bone or cartilage in conjunction with short­term antipseudomonal antibiotic therapy.

Other unusual etiologies of osteomyelitis may relate to underlying host factors (eg, sickle cell disease, chronic
granulomatous disease) and/or exposures to less common organisms (eg, blastomycosis, coccidiomycosis, actinomycosis,
Bartonella henselae).

American Board of Pediatrics Content Specification(s)
Appreciate the situations (eg, trauma, IV drug abuse, diabetes, sickle cell disease) in which less common organisms may
be the cause of osteomyelitis (eg, Mycobacterium tuberculosis, Pseudomonas aeruginosa)

Suggested Readings
Jacobs RF, McCarthy RE, Elser JM. Pseudomonas osteochondritis complicating nail puncture wounds of the foot in children:
a 10 year evaluation. J Infect Dis. 1989;160(4):657­661. doi: http://dx.doi.org/10.1093/infdis/160.4.657

Krogstad P. Osteomyelitis. In: Cherry JD, Demmler­Harrison GJ, Kaplan SL, Steinbach WJ, Hotez P, eds. Feigin and Cherry’s
Textbook of Pediatric Infectious Diseases. 7th ed. Philadelphia, PA: Saunders Elsevier; 2014:711­727.

Raz R, Miron D. Oral ciprofloxacxin for treatment of infection following nail puncture wounds of the foot. Clin Infect Dis.
1995;21(1):194­195. doi: http://dx.doi.org/10.1093/clinids/21.1.194

Comment On This Question

Page 82 of 115

Contact Us
Links may open in separate window
Copyright © 2016   American Academy of Pediatrics. All rights reserved. Pop­up Blocker may need to be disabled

http://2016.prepid.courses.aap.org/script/september?req=201612122154382337&status=submit 2/2
12/13/2016 September

Welcome  mohammed alsaiary [ Logout ]

Home PREP Pearls FAQs My Bookmarks CME Information

Overview Home  > September

Editorial Board
September   Enter Keyword Search
January
Question View:   All (8) Jump to Question
February
Print this Page Add to my Bookmarks Page 83 of 115
March

April Assessment History Mode: Learner  Exam 

May ASSESSMENT PROGRESS:  Total Questions:  8  Questions Answered:  3  Correct Answers:  0


June

July You currently have 5 questions unanswered in this assessment

August

September
Question: 4
October
A 10­year­old boy is admitted with endophthalmitis following penetrating trauma. Intraoperative cultures grow Escherichia coli with
November the following susceptibility pattern:

December Minimum inhibitory
Drug Interpretation
concentration

  Amikacin   ≤2   Susceptible

  Ampicillin   ≥32   Resistant

Claim Credit   Cefotaxime   ≤4   Susceptible

Evaluation   Gentamicin   ≤2   Susceptible


My Learning Plan
  Meropenem   ≤0.25   Susceptible

  Piperacillin   16   Susceptible

  Tobramycin   ≥16   Resistant

  Trimethoprim sulfa   ≥320   Resistant

Of the following, treatment of this organism with intravenous gentamicin is MOST likely to result in therapeutic failure because of

A. aminoglycoside cross resistance 
B. impaired ocular penetration 
C. once daily dosing
D. a pH at the site of infection greater than 7

Submit   Reset  

Page 83 of 115

Contact Us
Links may open in separate window
Copyright © 2016   American Academy of Pediatrics. All rights reserved. Pop­up Blocker may need to be disabled

http://2016.prepid.courses.aap.org/script/september?q­page=4&req=201612122154386244 1/1
12/13/2016 September

Welcome  mohammed alsaiary [ Logout ]

Home PREP Pearls FAQs My Bookmarks CME Information

Overview Home  > September

Editorial Board
September   Enter Keyword Search
January
Question View:   All (8) Jump to Question
February
Print this Page Add to my Bookmarks Page 83 of 115
March

April Assessment History Mode: Learner  Exam 

May ASSESSMENT PROGRESS:  Total Questions:  8  Questions Answered:  4  Correct Answers:  0


June

July You currently have 4 questions unanswered in this assessment

August

September
Question: 4
October
A 10­year­old boy is admitted with endophthalmitis following penetrating trauma. Intraoperative cultures grow Escherichia coli with
November the following susceptibility pattern:

December Minimum inhibitory
Drug Interpretation
concentration

  Amikacin   ≤2   Susceptible

  Ampicillin   ≥32   Resistant

Claim Credit   Cefotaxime   ≤4   Susceptible

Evaluation   Gentamicin   ≤2   Susceptible


My Learning Plan
  Meropenem   ≤0.25   Susceptible

  Piperacillin   16   Susceptible

  Tobramycin   ≥16   Resistant

  Trimethoprim sulfa   ≥320   Resistant

Of the following, treatment of this organism with intravenous gentamicin is MOST likely to result in therapeutic failure because of

A. aminoglycoside cross resistance 
B. impaired ocular penetration 
C. once daily dosing
D. a pH at the site of infection greater than 7

Incorrect View Peer Results
Correct Answer: B Average Percent Correct:  86.89%

Anaerobic conditions, acidic pH, and the presence of purulent material significantly impair the activity of
aminoglycosides. In addition, compared to β­lactam agents, intravenous aminoglycosides penetrate
very poorly into the eye. Therefore, use of an intravenous aminoglycoside for the treatment of
intraocular infection is not recommended. When aminoglycoside therapy is desired for treating the
causative organism in patients with endophthalmitis, it is administered by intravitreal injection and often
combined with an oral fluoroquinolone, which has good penetration into the posterior segment.

Compared to other antibiotic classes, aminoglycosides are less likely to develop resistance. Acquired aminoglycoside
resistance is primarily due to extrachromosomal elements that enzymatically modify aminoglycosides so that they are unable
to bind to their ribosomal targets. Cross resistance between specific aminoglycoside agents is incomplete, so individual
agents should be tested for in vitro susceptibility. Aminoglycoside cross­resistance is not a common cause of treatment
failure.

In once daily aminoglycoside dosing, high serum peak concentrations achieve efficient killing of susceptible organisms and,
even after serum aminoglycoside levels fall below the minimum inhibitory concentration (MIC) of the infecting organism, the
postantibiotic growth inhibition duration (ie, postantibiotic effect) correlates to the peak aminoglycoside concentration to MIC
ratio. In addition, the drug­free interval associated with the postantibiotic effect is thought to decrease the development of
adaptive resistance (reduced antimicrobial killing after initial aminoglycoside exposure) by exposed organisms. Studies have

http://2016.prepid.courses.aap.org/script/september?req=201612122155184057&status=submit 1/2
12/13/2016 September
shown that once daily intravenous aminoglycoside dosing is equally effective as other dosing regimens for selected
indications (eg, urinary tract infection caused by a multiply drug­resistant organism). 

PREP Pearls
Anaerobic conditions, acidic pH, and the presence of purulent material significantly impair the activity of aminoglycosides. 

Intravenous aminoglycosides penetrate very poorly into eye.

Cross resistance between specific aminoglycoside agents is incomplete, so individual agents should be tested for in vitro
susceptibility.

American Board of Pediatrics Content Specification(s)
Know the circumstances in which aminoglycoside activity is impaired (in purulent material, low pH, anaerobic conditions)

Suggested Readings
Bradley JS, Long SS. Principles of anti­infective therapy. In: Long SS, Pickering LK, Prober CG, eds. Principles and Practice
of Pediatric Infectious Diseases. 4th ed. Long SS, Pickering LK, Prober CG (eds). Philadelphia, PA: Saunders Elsevier;
2012:1412­1420.

Jorgenson JH, Pfaller MA, Carroll KC, et al. Manual of Clinical Microbiology, 11th ed. Washington, DC: ASM Press;
2015:2892 pp.

Comment On This Question

Page 83 of 115

Contact Us
Links may open in separate window
Copyright © 2016   American Academy of Pediatrics. All rights reserved. Pop­up Blocker may need to be disabled

http://2016.prepid.courses.aap.org/script/september?req=201612122155184057&status=submit 2/2
12/13/2016 September

Welcome  mohammed alsaiary [ Logout ]

Home PREP Pearls FAQs My Bookmarks CME Information

Overview Home  > September

Editorial Board
September   Enter Keyword Search
January
Question View:   All (8) Jump to Question
February
Print this Page Add to my Bookmarks Page 84 of 115
March

April Assessment History Mode: Learner  Exam 

May ASSESSMENT PROGRESS:  Total Questions:  8  Questions Answered:  4  Correct Answers:  0


June

July You currently have 4 questions unanswered in this assessment

August

September
Question: 5
October
A pediatrician is seeing a 30­month­old boy with sickle cell disease whose family just moved from another state. She would like
November advice on how to optimize his immunity to pneumococcal infections. The parents report that the boy completed his primary series of
pneumococcal vaccines, the last dose when he was about 18 months.
December
Of the following, the MOST important vaccine to administer at this time is

A. 13­valent pneumococcal conjugate vaccine
B. 23­valent pneumococcal polysaccharide vaccine
C. both 13­ and 23­valent pneumococcal conjugate vaccine 
Claim Credit
D. no pneumococcal vaccine 
Evaluation

My Learning Plan
Submit   Reset  

Page 84 of 115

Contact Us
Links may open in separate window
Copyright © 2016   American Academy of Pediatrics. All rights reserved. Pop­up Blocker may need to be disabled

http://2016.prepid.courses.aap.org/script/september?q­page=5&req=201612122155188589 1/1
12/13/2016 September

Welcome  mohammed alsaiary [ Logout ]

Home PREP Pearls FAQs My Bookmarks CME Information

Overview Home  > September

Editorial Board
September   Enter Keyword Search
January
Question View:   All (8) Jump to Question
February
Print this Page Add to my Bookmarks Page 84 of 115
March

April Assessment History Mode: Learner  Exam 

May ASSESSMENT PROGRESS:  Total Questions:  8  Questions Answered:  5  Correct Answers:  0


June

July You currently have 3 questions unanswered in this assessment

August

September
Question: 5
October
A pediatrician is seeing a 30­month­old boy with sickle cell disease whose family just moved from another state. She would like
November advice on how to optimize his immunity to pneumococcal infections. The parents report that the boy completed his primary series of
pneumococcal vaccines, the last dose when he was about 18 months.
December
Of the following, the MOST important vaccine to administer at this time is

A. 13­valent pneumococcal conjugate vaccine
B. 23­valent pneumococcal polysaccharide vaccine
C. both 13­ and 23­valent pneumococcal conjugate vaccine 
Claim Credit
D. no pneumococcal vaccine 
Evaluation

My Learning Plan
Incorrect View Peer Results
Correct Answer: B Average Percent Correct:  88.52%

The 2 pneumococcal vaccines in current use in children in the United States are the 13­valent
pneumococcal conjugate vaccine (PCV13) and the 23­valent pneumococcal polysaccharide vaccine
(PPSV23). Immunization with PCV13 is routine for all infants and children 2 through 59 months of age.
In addition, PCV13 is recommended for all children younger than 72 months who are at high risk or
presumed high risk of acquiring invasive pneumococcal disease (IPD), and if the child is 2 years of age
or older, he or she should also receive PPSV23. Children at high risk or presumed high risk are those
who are immunocompromised (anatomic or functional asplenia, HIV, or other immunocompromising
conditions), and those with an underlying medical condition that can increase  the risk (such as chronic heart disease, chronic
lung disease, diabetes mellitus, cerebrospinal fluid leaks, or cochlear implant). The Table lists the recommended doses of
vaccines, based upon the number and type of previous doses of pneumococcal vaccine received. The boy in the vignette
most likely has received an adequate series of pneumococcal conjugate vaccine, including a dose of PCV13 in his second
year of life. As can be seen from the table, he does not need another dose of PCV13. PPSV23 is licensed for use in children
2 years of age and older, and he should receive a single dose of PPSV23 at this time. The recommended interval between
PCV13 and PPSV23 is greater than or equal to 8 weeks, so he should not receive both 13­ and 23­valent pneumococcal
conjugate vaccine at this time.  

Immunization with PPSV23 does not induce immunologic memory or boosting with subsequent doses, but a second lifetime
dose (5 years after the first) is recommended for certain high­risk individuals, and those who are immunocompromised
(Table). Previously, children with sickle cell disease were recommended to receive the second dose 3 years after the first, but
the recommended interval has been changed to 5 years. Children 2 years or older with an underlying medical condition that
increases the risk of IPD should receive PPSV23 as soon as possible after a diagnosis is made. This includes children who
are candidates for solid organ transplantation and children in whom a splenectomy is planned. PPSV23 should be
administered at least 2 weeks before transplant or splenectomy. Optimally, doses of PCV13 should be completed before
PPSV23 is administered, with a minimum interval of 8 weeks between the last dose of PCV13 and the first dose of PPSV23. 

PREP Pearls
PCV13 is recommended for all children who are at high risk or presumed high risk of acquiring invasive pneumococcal
disease.

The use of a combination regimen of conjugate (PCV13) and polysaccharide (PPSV23) vaccines provides immunity to an
expanded number of pneumococcal serotypes in high­risk children

http://2016.prepid.courses.aap.org/script/september?req=201612122156055464&status=submit 1/2
12/13/2016 September

American Board of Pediatrics Content Specification(s)
Recommend the schedule for pneumococcal immunization in a child 24 to 59 months of age who is at high risk of invasive
pneumococcal disease, based upon the number of previous doses of pneumococcal vaccine received

Suggested Readings
American Academy of Pediatrics Committee on Infectious Diseases. Immunization for Streptococcus pneumoniae infections
in high­risk children. Pediatrics. 2014;134(6):1230­1233. doi: http://dx.doi.org/10.1542/peds.2014­2811

American Academy of Pediatrics Committee on Infectious Diseases. Recommendations for the prevention Streptococcus
pneumoniae infections in infants and children: use of 13­valent pneumococcal conjugate vaccine (PCV13) and pneumococcal
polysaccharide vaccine (PPSV23). Pediatrics. 2010;126(1):186­190. doi: http://dx.doi.org/10.1542/peds.2010­1280

Nuorti JP, Whitney CG. Prevention of pneumococcal disease among infants and children—use of 13­valent pneumococcal
conjugate vaccine and 23­valent pneumococcal polysaccharide vaccine. MMWR Recomm Rep. 2010;59(RR11):1­18.
http://www.cdc.gov/mmwr/preview/mmwrhtml/rr5911a1.htm

Rubin LG, Papsin B, Committee on Infectious Diseases, Section on Otolaryngology­Head and Neck Surgery. Policy
statement: cochlear implants in children: surgical site infections and prevention and treatment of acute otitis media and
meningitis. Pediatrics. 2010;126(2):381­391. doi: http://dx.doi.org/10.1542/peds.2010­1427

Comment On This Question

Page 84 of 115

Contact Us
Links may open in separate window
Copyright © 2016   American Academy of Pediatrics. All rights reserved. Pop­up Blocker may need to be disabled

http://2016.prepid.courses.aap.org/script/september?req=201612122156055464&status=submit 2/2
12/13/2016 September

Welcome  mohammed alsaiary [ Logout ]

Home PREP Pearls FAQs My Bookmarks CME Information

Overview Home  > September

Editorial Board
September   Enter Keyword Search
January
Question View:   All (8) Jump to Question
February
Print this Page Add to my Bookmarks Page 85 of 115
March

April Assessment History Mode: Learner  Exam 

May ASSESSMENT PROGRESS:  Total Questions:  8  Questions Answered:  5  Correct Answers:  0


June

July You currently have 3 questions unanswered in this assessment

August

September
Question: 6
October
Your next­door neighbor, a licensed daycare center operator, telephones you to state that a 4­year­old girl in her center has been
November diagnosed with head lice, began treatment, and returned to the center the next day. Her room supervisor is very upset because she
sees nits in the child’s hair and states she should be sent home until declared free of nits. The center’s nit policy is vague on this
December
topic and the neighbor asks for your opinion.

Of the following, the MOST appropriate factor that would allow this child to return to the center is

A. child and household contacts completing therapy with a standard pediculicide
B. child completing therapy with a standard pediculicide
Claim Credit C. child starting therapy with a standard pediculicide
Evaluation D. physician­verified cure of child

My Learning Plan

Submit   Reset  

Page 85 of 115

Contact Us
Links may open in separate window
Copyright © 2016   American Academy of Pediatrics. All rights reserved. Pop­up Blocker may need to be disabled

http://2016.prepid.courses.aap.org/script/september?q­page=6&req=201612122156063589 1/1
12/13/2016 September

Welcome  mohammed alsaiary [ Logout ]

Home PREP Pearls FAQs My Bookmarks CME Information

Overview Home  > September

Editorial Board
September   Enter Keyword Search
January
Question View:   All (8) Jump to Question
February
Print this Page Add to my Bookmarks Page 85 of 115
March

April Assessment History Mode: Learner  Exam 

May ASSESSMENT PROGRESS:  Total Questions:  8  Questions Answered:  6  Correct Answers:  0


June

July You currently have 2 questions unanswered in this assessment

August

September
Question: 6
October
Your next­door neighbor, a licensed daycare center operator, telephones you to state that a 4­year­old girl in her center has been
November diagnosed with head lice, began treatment, and returned to the center the next day. Her room supervisor is very upset because she
sees nits in the child’s hair and states she should be sent home until declared free of nits. The center’s nit policy is vague on this
December
topic and the neighbor asks for your opinion.

Of the following, the MOST appropriate factor that would allow this child to return to the center is

A. child and household contacts completing therapy with a standard pediculicide
B. child completing therapy with a standard pediculicide
Claim Credit C. child starting therapy with a standard pediculicide
Evaluation D. physician­verified cure of child

My Learning Plan

Incorrect View Peer Results
Correct Answer: C Average Percent Correct:  36.07%

A diagnosis of head lice is not an indication to exclude a healthy child from daycare or school. If a
diagnosis is made, the parent or guardian should be notified, and the child can remain in class until the
end of the day. Treatment can be started later that day, and the child can return to the facility the
following day.

Household contacts should be checked for lice, and treatment is indicated for those with nits or live lice
within 1 cm of the scalp and for those who share a bed with the affected individual. A head lice
diagnosis does not necessarily require a physician visit, and over­the­counter treatments are available. Ideally, however, the
diagnosis should be made by an individual (eg, school nurse) experienced in managing head lice infestation.

Head lice, caused by infestation with the insect Pediculus humanus capitis, is a condition of relatively minor medical
consequence that nonetheless generates great concern in the lay public. It affects primarily preschool and elementary school
children of all socioeconomic strata, but is less common in black children compared to children of other races. Affected
individuals can be asymptomatic or present with head itching. Head lice do not transmit any disease and thus are of minor
public health concern. Diagnosis is made by visualizing nits, nymphs, or lice in the scalp; dandruff, benign hair casts, and
external debris, among other entities, are conditions commonly misdiagnosed as lice infestations by inexperienced
practitioners. Topical treatment with 1% permethrin lotion or pyrethrins with piperonyl butoxide is both safe and effective in
most instances. Other topical and off­label oral therapies have been utilized, fueled in part by concerns for increasing
resistance of head lice to the commonly used pediculicides.

The incubation period from egg­laying to hatching is 7 to 12 days, with maturation to adult stage 12 days later; by the time the
diagnosis is made in a child, the infestation may have been present for weeks. Transmission is mainly by direct head­to­head
contact, although transmission from combs, brushes, or hats is possible.

Given that head lice pose minimal public health concerns, restricting infested children from school or daycare raises
significant legal concerns that have led to the rather minimal restriction policy recommended by the American Academy of
Pediatrics and most other medical authorities. Some authorities consider school restriction for head lice or notification of
parents of classmates as a violation of the infested child’s civil liberties. Furthermore, more restrictive policies, such as “no­nit”
rules requiring children to be free of nits before return to daycare or school, have not been effective in controlling
transmission. Similarly, screening for nits or live lice in large groups of children in classrooms or schools has been shown to
be ineffective. However, it may be prudent to check other children in the room or class who may be most likely to have had
head­to­head contact with the infested child. 

http://2016.prepid.courses.aap.org/script/september?req=201612122156502809&status=submit# 1/2
12/13/2016 September

PREP Pearls
Head lice infestation poses only minimal public health concerns because they do not spread disease.

Infested children can complete the school day, be started on therapy, and then return to class or daycare the following day.
No test of cure is required.

A “no­nit” requirement to restrict children from school or daycare has been shown to be ineffective in controlling head lice
transmission.

American Board of Pediatrics Content Specification(s)
Make recommendations for a child­care center for control of lice infestation

Suggested Readings
American Academy of Pediatrics. Infections spread by direct contact. In: Kimberlin DW, Brady MT, Jackson MA, Long SS,
eds. Red Book: 2015 Report of the Committee on Infectious Diseases. 30th ed. Elk Grove Village, IL: American Academy of
Pediatrics; 2015:156­157.

Devore CD, Schutze GE, Okamoto J, et al. Head lice. Pediatrics. 2015;135(5):e1355­e1365. doi:
http://dx.doi.org/10.1542/peds.2015­0746

Elston DM. Ectoparasites (lice and scabies). In: Long SS, Pickering LK, Prober CG, eds. Principles and Practice of Pediatric
Infectious Diseases. 4th ed. Philadelphia, PA: Saunders Elsevier; 2012:1257­1261.

Comment On This Question

Page 85 of 115

Contact Us
Links may open in separate window
Copyright © 2016   American Academy of Pediatrics. All rights reserved. Pop­up Blocker may need to be disabled

http://2016.prepid.courses.aap.org/script/september?req=201612122156502809&status=submit# 2/2
12/13/2016 September

Welcome  mohammed alsaiary [ Logout ]

Home PREP Pearls FAQs My Bookmarks CME Information

Overview Home  > September

Editorial Board
September   Enter Keyword Search
January
Question View:   All (8) Jump to Question
February
Print this Page Add to my Bookmarks Page 86 of 115
March

April Assessment History Mode: Learner  Exam 

May ASSESSMENT PROGRESS:  Total Questions:  8  Questions Answered:  6  Correct Answers:  0


June

July You currently have 2 questions unanswered in this assessment

August

September
Question: 7
October
You are visiting rural Central America with a medical mission team. Each year, the mission group visits the same community to
November provide some basic healthcare and to implement community­wide public health projects, including the establishment of several clean
water wells. One of the annual efforts is to administer community­based mass treatment with antihelminthic medication (so­called
December
“deworming”) to reduce disease caused by chronic parasitic infections. During clinic hours, a mother explains that she sees worms in
the stool of her 8­year­old child, even though the child takes the deworming medicine every year. The child is otherwise healthy, but
“doesn't have a very good appetite.” The child’s mother holds up a specimen cup containing a fresh stool sample, stating that the
worms appear to be the same type that she has seen previously.

You confirm your suspicion of intestinal ascariasis microscopically and verify from the medical record that the child has been
Claim Credit diagnosed and treated for ascariasis 3 years in a row. You consider whether additional laboratory testing should be performed.

Evaluation Of the following, the MOST likely laboratory finding in this child with recurrent intestinal ascariasis is the

My Learning Plan A. absence of eosinophil­specific peroxidase
B. absence of secretory immunoglobulin A
C. presence of anti­ascaris immunoglobulin G
D. presence of anti­human immune deficiency virus immunoglobulin G

Submit   Reset  

Page 86 of 115

Contact Us
Links may open in separate window
Copyright © 2016   American Academy of Pediatrics. All rights reserved. Pop­up Blocker may need to be disabled

http://2016.prepid.courses.aap.org/script/september?q­page=7&req=201612122156508591 1/1
12/13/2016 September

Welcome  mohammed alsaiary [ Logout ]

Home PREP Pearls FAQs My Bookmarks CME Information

Overview Home  > September

Editorial Board
September   Enter Keyword Search
January
Question View:   All (8) Jump to Question
February
Print this Page Add to my Bookmarks Page 86 of 115
March

April Assessment History Mode: Learner  Exam 

May ASSESSMENT PROGRESS:  Total Questions:  8  Questions Answered:  7  Correct Answers:  0


June

July You currently have 1 questions unanswered in this assessment

August

September
Question: 7
October
You are visiting rural Central America with a medical mission team. Each year, the mission group visits the same community to
November provide some basic healthcare and to implement community­wide public health projects, including the establishment of several clean
water wells. One of the annual efforts is to administer community­based mass treatment with antihelminthic medication (so­called
December
“deworming”) to reduce disease caused by chronic parasitic infections. During clinic hours, a mother explains that she sees worms in
the stool of her 8­year­old child, even though the child takes the deworming medicine every year. The child is otherwise healthy, but
“doesn't have a very good appetite.” The child’s mother holds up a specimen cup containing a fresh stool sample, stating that the
worms appear to be the same type that she has seen previously.

You confirm your suspicion of intestinal ascariasis microscopically and verify from the medical record that the child has been
Claim Credit diagnosed and treated for ascariasis 3 years in a row. You consider whether additional laboratory testing should be performed.

Evaluation Of the following, the MOST likely laboratory finding in this child with recurrent intestinal ascariasis is the

My Learning Plan A. absence of eosinophil­specific peroxidase
B. absence of secretory immunoglobulin A
C. presence of anti­ascaris immunoglobulin G
D. presence of anti­human immune deficiency virus immunoglobulin G

Incorrect View Peer Results

Correct Answer: C Average Percent Correct:  24.59%

The child in the vignette has recurrent intestinal ascariasis. Since the child is otherwise healthy,
additional laboratory testing is unnecessary. If testing were performed, it would very likely show the
presence of anti­ascaris immunoglobulin G (IgG). While the specific humoral and cellular responses to
this infection are robust, they offer little protection against reinfection when children are re­exposed to
ascarid ova through the ingestion of contaminated water or food. Reinfection is to be expected unless
environmental sources are reduced or eliminated. Reinfection does not herald the presence of a
humoral or cellular immune disorder such as secretory immunoglobulin A (IgA) deficiency or HIV
infection.

Ascariasis is also associated with peripheral blood eosinophilia, and these effector cells are thought to have antihelminthic
properties, but eosinophil­specific peroxidase deficiency is a very rare abnormality that is not known to be associated with
increased risk for recurrent parasitic infection or any other health problem.

Understanding the mechanisms of host defense that are active in protecting against helminth infection proves challenging. As
a group, helminths are parasitic multicellular worms covered by an integument that protects them from environmental
stresses. The general host immune responses to trematodes (flukes), cestodes (tapeworms), and nematodes (roundworms)
are similar, including the observation that even robust pathogen­specific responses fail to protect the host from reinfection.
This observation highlights the major obstacle to vaccine development; immunologic surrogates of protective immunity do not
exist. Vaccines to protect against helminthic infections are not currently available.

Most helminths cannot complete their life cycle in human hosts because their eggs or larvae will only develop in water, soil, a
specific plant, arthropod, or nonhuman animal. The geographic distribution of worm infestations therefore reflects the
environmental conditions that support their replication. Since most helminths cannot actively replicate in the human host, the
infection is limited to the life span of the parasites in the original inoculum unless or until the individual is reinfected. One
notable exception is the nematode Strongyloides stercoralis, which is able to replicate in humans. An individual’s infection can
persist, and during periods of immunosuppression, accelerated replication can lead to “hyperinfection,” even after a light
inoculum many years in the past.

http://2016.prepid.courses.aap.org/script/september?req=201612122157323122&status=submit# 1/2
12/13/2016 September
Helminthic infections stimulate robust immune reactions, resulting in parasite­specific cellular and humoral responses.
Although these responses are sometimes used for diagnosing a specific infection, they do not appear to offer sustained
protection against the pathogen.

Innate immune responses are responsible for the recruitment of macrophages and eosinophils to the proximity of the invading
worm, and through the release of toxic mediators, contribute to host cell efforts to eradicate it. Parasite­specific IgG and
immunoglobulin E antibodies that bind to surface antigens fix complement and direct additional cellular effectors to the site of
infection. Despite the abundance of IgA in the intestinal lumen, there is no conclusive evidence that secretory IgA is involved
in protective immunity against intestinal worms. Studies have failed to clearly identify a universal protective effector
mechanism. In most cases, the nature of the response that ultimately clears the worm is unknown.

A pervasive theme of resistance to helminthic infections is that of “concomitant immunity,” a state where the host is protected
from further infection with a given species by ongoing persistent infection with the same pathogen. Parasites from the initial
infection induce an immune response that, while incapable of killing the initial pathogen, is able to prevent additional incoming
parasites from successful invasion. This hypothesis stipulates that adult worms, but not invasive larval stages, express
immune evasion mechanisms rending them resistant to clearance by the immune response. In addition, there must be shared
common antigens between the adult and larval forms. When new larval forms attempt to invade, the now mature host immune
response is capable of recognizing and killing them through recognition of the antigens shared by the 2 forms of the worm, as
the larval forms lack the immune evasive properties of the adults. While the immune evasive mechanisms of the resident
adult worm allow it to persist unperturbed, any larval forms attempting to invade are recognized by pathogen­specific host
immunity and cleared efficiently. In some instances, immunity becomes fully apparent only after pharmacologic eradication of
the primary infection. Following treatment with anti­parasitic medication older individuals tend to be resistant to reinfection,
although children routinely become reinfected when they are re­exposed.

Peripheral blood eosinophilia is a characteristic laboratory finding in children infected with many helminths, suggesting a role
for these cells in host defense and/or disease pathogenesis. Eosinophilia is specifically seen when there is migration of
worms and/or ongoing presence of worms in tissues. High numbers of eosinophils, and in some studies, high concentrations
of eosinophil degranulation products, are able to kill worms in vitro; in mouse models, depletion of eosinophils results in a
greater worm burden, supporting a protective role for these cells in worm host defense. Histologically, in humans,
degranulated eosinophils are sometimes found on and around degenerating worms further supporting a host defense role.
The antihelminthic activities of eosinophil­derived neurotoxin (EDN), eosinophil cationic protein (ECP), and major basic
protein (MBP) are significantly more potent in vitro than activity derived from the cell’s peroxidase­hydrogen peroxide­halide
oxidation complex. Killing of helminths by eosinophils via antibody­dependent cellular cytotoxicity (ADCC) is another attractive
and widely cited host defense mechanism. Recent reports have established a role for the T helper 2 cytokine interleukin­5 in
resistance to Strongyloides stercoralis. In most other settings, however, despite clear evidence that eosinophils can kill
helminthes in vitro, their importance for resistance to infection or role in pathogen eradication remains to be proven.

None of the classic inherited immune deficiency diseases are specifically associated with recurrent, unusually severe or
elevated risk for severe worm infestations, although classic descriptions of Strongyloides hyperinfection syndrome is most
common among patients with severe secondary cellular immune deficiency, such as those with AIDS and those treated with
high­dose glucocorticoids. This clinical observation supports an important role for cellular immunity in protecting the host,
perhaps through controlling, but not eliminating, the worm burden. 

* Required *
Take Survey  

PREP Pearls
Protective immunity following helminth infection in childen is incomplete, making reinfection common.

Eosinophilia is a common laboratory finding in patients infected with trematodes and nematodes, but the role of eosinophils
in protective host defense remains controversial

American Board of Pediatrics Content Specification(s)
Understand the mechanisms of host defense that are active in protecting against multicellular parasites, eg., worms

Suggested Readings
Grencis RK. Immunity to helminthes: resistance, regulation, and susceptibility to gastrointestinal nematodes. Annu Rev
Immunol. 2015;33:201­225. doi: http://dx.doi.org/10.1146/annurev­immunol­032713­120218

King CH. Defining the necessary steps for effective control of helminthic infections. Clin Infect Dis. 2016;62(2):208­209. doi:
http://dx.doi.org/10.1093/cid/civ833

Comment On This Question

Page 86 of 115

Contact Us
Links may open in separate window
Copyright © 2016   American Academy of Pediatrics. All rights reserved. Pop­up Blocker may need to be disabled

http://2016.prepid.courses.aap.org/script/september?req=201612122157323122&status=submit# 2/2
12/13/2016 September

Welcome  mohammed alsaiary [ Logout ]

Home PREP Pearls FAQs My Bookmarks CME Information

Overview Home  > September

Editorial Board
September   Enter Keyword Search
January
Question View:   All (8) Jump to Question
February
Print this Page Add to my Bookmarks Page 87 of 115
March

April Assessment History Mode: Learner  Exam 

May ASSESSMENT PROGRESS:  Total Questions:  8  Questions Answered:  7  Correct Answers:  0


June

July You currently have 1 questions unanswered in this assessment

August

September
Question: 8
October
You are seeing a previously healthy 20­month­old male child for a 3­day history of worsening bilateral upper eyelid erythema and
November swelling. By history, the patient has had an upper respiratory tract infection with nasal congestion, rhinorrhea, cough, and fever to
38.3°C for the last 2 weeks. Over the last 3 days, his mother has noted that his eyes are almost swollen shut when he wakes up in
December
the morning. His upper eyelids are very puffy and mildly erythematous, but don’t seem to cause him any pain, and the swelling
improves after he has been up for several hours. His immunizations are up­to­date for age, he does not attend day care, and he has
had no new exposures.

His physical examination is significant for a temperature of 37.8°C, bilateral upper eyelid puffiness with mild overlying erythema, full
extraocular eye movements with no pain with eye movement, there is no displacement of either globe, and there is no tenderness to
Claim Credit palpation of the eyelids. Moderate nasal congestion is present with a yellowish rhinorrhea. Lungs have transmitted upper airway
sounds with no wheezes or rales. The remainder of the physical examination is within normal limits.
Evaluation
A complete blood cell count shows a white blood cell count of 12,000/µL (12 x 109/L) , hemoglobin of 11.8 g/dL (118 g/L), platelets
My Learning Plan
200 x 103/µL (200 x 109/L) with segmented neutrophils of 60%, lymphocytes of 38%, and monocytes of 2%. Serum albumin is 3 g/dL
(30 g/L) and urinalysis is normal.

Of the following, the MOST likely cause of this patient’s eye swelling is

A. allergic inflammation
B. bacterial periorbital cellulitis
C. hypoproteinemia
D. inflammatory edema
E. orbital cellulitis

Submit   Reset  

Page 87 of 115

Contact Us
Links may open in separate window
Copyright © 2016   American Academy of Pediatrics. All rights reserved. Pop­up Blocker may need to be disabled

http://2016.prepid.courses.aap.org/script/september?q­page=8&req=201612122157328122 1/1
12/13/2016 September

Welcome  mohammed alsaiary [ Logout ]

Home PREP Pearls FAQs My Bookmarks CME Information

Overview Home  > September

Editorial Board
September   Enter Keyword Search
January
Question View:   All (8) Jump to Question
February
Print this Page Add to my Bookmarks Page 87 of 115
March

April Assessment History Mode: Learner  Exam 

May ASSESSMENT PROGRESS:  Total Questions:  8  Questions Answered:  8  Correct Answers:  0


June

July You currently have 1 required survey(s) that are incomplete. Please complete the survey(s) for the following
question(s) on this assessment:  Question 7
August

September

October Question: 8
November You are seeing a previously healthy 20­month­old male child for a 3­day history of worsening bilateral upper eyelid erythema and
swelling. By history, the patient has had an upper respiratory tract infection with nasal congestion, rhinorrhea, cough, and fever to
December
38.3°C for the last 2 weeks. Over the last 3 days, his mother has noted that his eyes are almost swollen shut when he wakes up in
the morning. His upper eyelids are very puffy and mildly erythematous, but don’t seem to cause him any pain, and the swelling
improves after he has been up for several hours. His immunizations are up­to­date for age, he does not attend day care, and he has
had no new exposures.

His physical examination is significant for a temperature of 37.8°C, bilateral upper eyelid puffiness with mild overlying erythema, full
extraocular eye movements with no pain with eye movement, there is no displacement of either globe, and there is no tenderness to
Claim Credit
palpation of the eyelids. Moderate nasal congestion is present with a yellowish rhinorrhea. Lungs have transmitted upper airway
Evaluation sounds with no wheezes or rales. The remainder of the physical examination is within normal limits.

My Learning Plan
A complete blood cell count shows a white blood cell count of 12,000/µL (12 x 109/L) , hemoglobin of 11.8 g/dL (118 g/L), platelets
200 x 103/µL (200 x 109/L) with segmented neutrophils of 60%, lymphocytes of 38%, and monocytes of 2%. Serum albumin is 3 g/dL
(30 g/L) and urinalysis is normal.

Of the following, the MOST likely cause of this patient’s eye swelling is

A. allergic inflammation
B. bacterial periorbital cellulitis
C. hypoproteinemia
D. inflammatory edema
E. orbital cellulitis

Incorrect View Peer Results
Correct Answer: D Average Percent Correct:  45.90%

The child in the vignette has inflammatory edema, most likely caused by a sinusitis. Allergic
inflammation is less likely given the lack of exposure, and hypoproteinemia causing periorbital edema
is unlikely with an albumin level of 3 g/dL (30 g/L) and a normal urinalysis. Orbital cellulitis is unlikely,
given the improvement in his symptoms as the day goes on, the lack of eye pain or pain with eye
movement, and proptosis. Likewise, bacterial periorbital cellulitis is not likely given the gradual onset of
his symptoms, improvement in his symptoms, lack of fever, bilateral nature of his condition, and lack of
significant periorbital erythema.

Swelling of the tissue around the eye or swelling of the eye itself may be caused by infectious and noninfectious causes.
Noninfectious causes of eye swelling include:

1. Allergic inflammation that includes contact hypersensitivity or angioneurotic edema.
2. Blunt trauma in that history provides the key to the diagnosis. In these cases, bruising of the area and eyelid swelling
increase over 48 hours and then resolve over a several day period.
3. Localized edema in which there is bilateral, boggy, nontender, nondiscolored, soft tissue swelling that may be caused
by hypoproteinemia or congestive heart failure.
4. Tumors of the eye such as hemangiomas of the eyelid, ocular tumors (eg, retinoblastoma, choroidal melanoma), and
orbital neoplasms (eg, neuroblastoma, rhabdomyosarcoma).

These usually cause a gradual onset of proptosis in the absence of inflammation.

http://2016.prepid.courses.aap.org/script/september?req=201612122158120623&status=submit 1/3
12/13/2016 September
Infectious causes of eye swelling may be periorbital or orbital. Periorbital or preseptal cellulitis is defined as an infection that is
anterior to the orbital septum, while orbital or post­septal cellulitis is defined as an infection that is posterior to the orbital
septum (Figure 1). The orbital septum is a thin connective tissue extension from the periosteum of the orbital rim that is
reflected into the upper and lower tarsal plates of the eyelids. It serves as a barrier between the superficial lids and the orbit
and prevents spread of infection from a preseptal site into the orbital space. Veins that drain the orbit, the ethmoid and
maxillary sinuses, and the skin of the eye and periorbital tissues represent an anastomosing and valveless network. This
venous system provides opportunities for spread of infection from one anatomic site to another and predisposes to
involvement of the cavernous sinus (Figure 2). The eye and the paranasal sinuses are closely contiguous structures that
share common elements. The roof of the orbit makes up the floor of the frontal sinus, and the floor of the orbit is the roof of
the maxillary sinus. The medial wall of the orbit is formed by the frontal maxillary process, the lacrimal bone, the lamina
papyracea of the ethmoid bone, and a small part of the sphenoid bone (Figure 3). Therefore, an infection of the mucosa of
the paranasal sinuses can spread to involve the bone and the intraorbital contents. Involvement of the intraorbital contents
occurs through bony dehiscences in the lamina papyracea of the ethmoid or frontal bones, or via foramina through which the
ethmoidal arteries pass.

Infectious causes of periorbital or preseptal cellulitis occur in several different settings:

1. It may occur secondary to a localized infection or inflammation of the conjunctiva, eyelids, or adjacent structures (eg,
conjunctivitis, hordeolum, acute chalazion, dacryocystitis, dacroadenitis, and impetigo). Preseptal cellulitis may also
result from traumatic bacterial cellulitis, which is the development of a secondary bacterial infection at sites of local
skin trauma (including insect bites) or spread of infection from a focus of impetigo. The overlying periorbital skin can
be intensely erythematous and swollen, leading to shininess of the skin. Most patients are afebrile and are rarely
bacteremic. The most common causative organisms are Staphylococcus aureus and group A streptococcus
(Streptococcus pyogenes).
2. It may be secondary to hematogenous dissemination of nasopharyngeal pathogens to the periorbital tissue.
Bacteremic periorbital cellulitis is most often seen in children younger than 18 months of age who have had a
preceding viral upper respiratory infection (URI). Patients develop a sudden increase in temperature, accompanied by
the acute onset and rapid progression of eyelid swelling. Swelling usually begins in the inner canthus of the upper and
lower eyelid and can obscure the eyeball within 12 hours. Periorbital tissues are usually erythematous; however, if the
swelling has been rapidly progressive, the tissues may have a violaceous discoloration. Tenderness is usually mild
and the globe is normal, and extraocular eye movements are intact. The pathogenesis of these infections is
hematogenous dissemination from a portal of entry in the nasopharynx. Prior to universal immunization against
Haemophilus influenzae type b, this organism caused about 80% of cases of bacteremic periorbital cellulitis, with
Streptococcus pneumoniae accounting for the remaining 20%. With the use of universal vaccination for H influenzae
type b and S pneumoniae, there has been a substantial decline in the number of cases of bacteremic periorbital
cellulitis. A precise bacteriologic diagnosis usually is made by recovery of an organism from blood culture; however, S
pneumoniae still remains the most likely pathogen in a child who has completed their H influenzae type b vaccine
series. Less common pathogens include: nontypeable H influenzae and group A streptococcus (S pyogenes).
3. It may be a manifestation of inflammatory edema or a sympathetic effusion in patients with acute sinusitis. The
pathogenesis is attributed to the venous drainage of the eyelid and surrounding structures. The inferior and superior
ophthalmic veins, which drain the lower and upper lid, pass through or next to the ethmoid sinus. If the sinuses are
completely congested, venous drainage is physically impeded, resulting in soft tissue swelling of the eyelids that is
most prominent at the medial aspect of the lids. Often, there is a history of intermittent early morning periorbital
swelling that resolves after a few hours, however, the swelling can be persistent and progressive. Eye swelling can be
impressive with significant degrees of erythema and tenderness is variable. There is no displacement of the globe or
impairment of extraocular eye movements. The peripheral white blood cell count is unremarkable, blood cultures are
always negative, and sinus radiographic studies show ipsilateral ethmoiditis or pansinusitis. In cases of inflammatory
edema, the infecting organisms are the same as those that cause uncomplicated acute sinusitis, namely S
pneumoniae, nontypeable H influenzae, and Moraxella catarrhalis.

Orbital or post­septal cellulitis may develop in several different ways, with the most common being secondary to acute or
chronic sinusitis, accounting for 80% to 90% of the cases. The infection may extend and results in the development of a
subperiosteal abscess, orbital abscess and cellulitis, or cavernous sinus thrombosis. Patients with orbital disease caused by
sinusitis usually develop the sudden onset of erythema and swelling of the eye after several days of a viral URI. Eye pain can
precede the swelling and is often significant. Infection of the orbit is suggested by the presence of proptosis (usually the globe
is displaced anteriorly and downward), impairment of and pain with extraocular eye movements (most commonly, upward or
lateral gaze), or loss of visual acuity or chemosis (edema of the bulbar conjunctiva). Fever and other systemic signs are
variable. Most orbital infections involve the formation of a subperiosteal abscess. Among young children, the subperiosteal
abscess results from ethmoiditis and ethmoid osteitis, while in adolescents, the subperiosteal abscess may be a complication
of frontal sinusitis and osteitis (Pott puffy tumor). Most commonly, the abscesses in all ages are located along the medial wall
of the orbit. Other complications of orbital cellulitis may include:

1. Compressive optic neuropathy with optic nerve edema and ischemia, which is indicated by the development of an
afferent pupillary defect, dilation of the retinal venules, and optic disk edema on examination. Afferent pupillary defect
is indicative of compromise of the optic nerve and, in severe cases, central retinal artery and vein occlusion may be
seen.
2. Meningitis and brain abscess are the 2 most common intracranial complications and the development or worsening of
headache, changes in mental status, increase in height of fever, vomiting, drowsiness, motor deficits, or seizures
should prompt further evaluation.

PREP Pearls
In patients with orbital cellulitis, an afferent pupillary defect is an indication of optic nerve compromise.

Inflammatory bilateral periorbital edema is a common presenting finding in acute sinusitis in infants and children.

The most common location of the subperiosteal abscesses in orbital cellulitis is along the medial wall of the orbit.

American Board of Pediatrics Content Specification(s)
Distinguish bacterial periorbital cellulitis from sinusitis­related eye swelling based upon clinical manifestations and results of
laboratory and diagnostic imaging studies

Recognize complications of orbital cellulitis (cavernous sinus thrombosis, optic nerve ischemia)

http://2016.prepid.courses.aap.org/script/september?req=201612122158120623&status=submit 2/3
12/13/2016 September

Suggested Readings
Bedwell J, Bauman NM. Management of pediatric orbital cellulitis and abscess. Curr Opin Otolaryngol Head Neck Surg.
2011;19(6):467­473. doi: http://dx.doi.org/10.1097/MOO.0b013e32834cd54a

Brooks I. Microbiology and antimicrobial treatment of orbital and intracranial complications of sinusitis in children and their
management. Inter J Pediatr Otolaryngol. 2009;73(9):1183­1186. doi: http://dx.doi.org/10.1016/j.ijporl.2009.01.020

DeMuri GP, Wald ER. Complications of acute bacterial sinusitis in children. Pediatr Infect Dis J. 2011;30(8):701­702. doi:
http://dx.doi.org/10.1097/INF.0b013e31822855a0

Hussein M, Miller AM, Steinkuller PG. Ocular infections. In: Cherry JD, Harrison GJ, Kaplan SL, Steinbach WJ and Hotez PJ,
eds. Feigin and Cherry’s Textbook of Pediatric Infectious Diseases. 7th ed. Philadelphia, PA: Saunders Elsevier; 2014:794­
822.

Tovilla­Canales JL, Nava A, Tovilla y Pomar JL. Orbital and periorbital infections. Curr Opin Ophthal. 2001;12(5):335­341.

Wald ER. Periorbital and orbital infections. Pediatrics In Review. 2004;25(9):312­320. doi: http://dx.doi.org/10.1542/pir.25­9­
312

Comment On This Question

Page 87 of 115

Contact Us
Links may open in separate window
Copyright © 2016   American Academy of Pediatrics. All rights reserved. Pop­up Blocker may need to be disabled

http://2016.prepid.courses.aap.org/script/september?req=201612122158120623&status=submit 3/3
12/13/2016 October

Welcome  mohammed alsaiary [ Logout ]

Home PREP Pearls FAQs My Bookmarks CME Information

Overview Home  > October

Editorial Board
October   Enter Keyword Search
January
Question View:   All (8) Jump to Question
February
Print this Page Add to my Bookmarks Page 89 of 115
March

April Assessment History Mode: Learner  Exam 

May ASSESSMENT PROGRESS:  Total Questions:  8  Questions Answered:  0  Correct Answers:  0


June

July You currently have 8 questions unanswered in this assessment

August

September
Question: 1
October
A 7­year­old girl had onset of fever 4 days ago on January 2, accompanied by headache and severe pain in her extremities and
November joints. She resides in the Midwest, but on further questioning, she had returned from a family vacation to the Caribbean the day prior
to illness onset. On physical examination, her temperature is 39.2°C, pulse is 126 beats/min, respiratory rate is 26 breaths/min, and
December
blood pressure is 100/70 mm Hg. She appears acutely ill, but not toxic. She has mild conjunctival suffusion, a faint, diffuse
erythematous papular rash, and tenderness to palpation diffusely in all extremities with swelling of both wrists and knees.

Of the following, the MOST likely route of acquisition of this infectious illness is

A. airborne

Claim Credit B. contact

Evaluation C. foodborne
D. respiratory droplet
My Learning Plan
E. vectorborne

Submit   Reset  

Page 89 of 115

Contact Us
Links may open in separate window
Copyright © 2016   American Academy of Pediatrics. All rights reserved. Pop­up Blocker may need to be disabled

http://2016.prepid.courses.aap.org/script/october?q­page=1&req=201612122158309373 1/1
12/13/2016 October

Welcome  mohammed alsaiary [ Logout ]

Home PREP Pearls FAQs My Bookmarks CME Information

Overview Home  > October

Editorial Board
October   Enter Keyword Search
January
Question View:   All (8) Jump to Question
February
Print this Page Add to my Bookmarks Page 89 of 115
March

April Assessment History Mode: Learner  Exam 

May ASSESSMENT PROGRESS:  Total Questions:  8  Questions Answered:  1  Correct Answers:  0


June

July You currently have 7 questions unanswered in this assessment

August

September
Question: 1
October
A 7­year­old girl had onset of fever 4 days ago on January 2, accompanied by headache and severe pain in her extremities and
November joints. She resides in the Midwest, but on further questioning, she had returned from a family vacation to the Caribbean the day prior
to illness onset. On physical examination, her temperature is 39.2°C, pulse is 126 beats/min, respiratory rate is 26 breaths/min, and
December
blood pressure is 100/70 mm Hg. She appears acutely ill, but not toxic. She has mild conjunctival suffusion, a faint, diffuse
erythematous papular rash, and tenderness to palpation diffusely in all extremities with swelling of both wrists and knees.

Of the following, the MOST likely route of acquisition of this infectious illness is

A. airborne

Claim Credit B. contact

Evaluation C. foodborne
D. respiratory droplet
My Learning Plan
E. vectorborne

Incorrect View Peer Results

Correct Answer: E Average Percent Correct:  92.00%

This child most likely acquired chikungunya virus, a mosquito­borne illness, on her family vacation.
Although dengue fever is another possibility, it is less likely to present with rash, conjunctivitis, and
frank arthritis than chikungunya infection. Knowledge of major routes of transmission of infection is
helpful not only in infection control in the healthcare setting, but also in discerning patterns of disease
to identify and control outbreaks of infection. In this case, knowledge of the characteristics of the
disease, plus knowledge of geographic epidemiology, allows for focused diagnostic testing and also
assessing risk to other family members and contacts.

All of the choices listed are known modes of transmission of infectious diseases. Diseases transmitted via the airborne (small
particle aerosol) route are uncommon, limited mainly (under ordinary exposure circumstances) to measles, severe acute
respiratory syndrome, smallpox, pulmonary tuberculosis, and varicella. Contact transmission can either be direct, such as by
touching an infected wound, or indirect via contaminated equipment such as a poorly disinfected endoscope. Foodborne
transmission, such as with salmonellosis, is essentially a form of contact transmission that occurs via the gastrointestinal
route. Respiratory droplet transmission requires exposure to larger droplets than for airborne transmission and includes many
respiratory viruses. Vectorborne transmission is a form of pathogen transmission where an intermediate host, in this case an
insect and often a mosquito, transmits a pathogen between humans or between animals and humans via a bite. In the case
presented, the mosquito vector is likely present in her home community, but transmission of disease in this setting is unlikely
given the winter season with few circulating mosquitoes, as well as few humans with active disease to act as a source of
infection. Therefore, it would be important to query family members for any signs of illness, but likelihood of spread from the
affected child to her home community is exceedingly low.

Examples of vectorborne infections are listed in the Table 

PREP Pearls
Although hospital isolation precautions usually are characterized as standard, contact, respiratory, or airborne, a more
detailed knowledge of major routes of infection transmission is necessary for patient management

Vectorborne infections have implications for spread of disease and appropriate infection control measures

http://2016.prepid.courses.aap.org/script/october?req=201612122159541094&status=submit# 1/2
12/13/2016 October

American Board of Pediatrics Content Specification(s)
Know the major routes of transmission/acquisition of micro‑organism (eg,type of contact, common vehicle, airborne,
vectorborne)

Suggested Readings
Halstead SB. Chikungunya. In: Cherry JD, Harrison GJ, Kaplan SL, Steinbach WJ, eds. Feigin and Cherry’s Textbook of
Pediatric Infectious Diseases. 7th ed. Philadelphia, PA: Saunders Elsevier; 2014:2241­2248 .

Siegel JD. Pediatric infection prevention and control.In: Long SS, Pickering LK, Prober CG, eds. Principles and Practice of
Pediatric Infectious Diseases. 4th ed. Philadelphia, PA: Saunders Elsevier; 2012:9­24 .

World Health Organization. Vector­borne diseases fact sheet 387, March 2014. World Health Organization website.
http://www.who.int/mediacentre/factsheets/fs387/en/

Comment On This Question

Page 89 of 115

Contact Us
Links may open in separate window
Copyright © 2016   American Academy of Pediatrics. All rights reserved. Pop­up Blocker may need to be disabled

http://2016.prepid.courses.aap.org/script/october?req=201612122159541094&status=submit# 2/2
12/13/2016 October

Welcome  mohammed alsaiary [ Logout ]

Home PREP Pearls FAQs My Bookmarks CME Information

Overview Home  > October

Editorial Board
October   Enter Keyword Search
January
Question View:   All (8) Jump to Question
February
Print this Page Add to my Bookmarks Page 90 of 115
March

April Assessment History Mode: Learner  Exam 

May ASSESSMENT PROGRESS:  Total Questions:  8  Questions Answered:  1  Correct Answers:  0


June

July You currently have 7 questions unanswered in this assessment

August

September
Question: 2
October
One of your adolescent patients is planning a trip to a polio­endemic region to visit her grandmother who has been receiving
November palliative chemotherapy and is not expected to survive much longer. The patient just celebrated her 20th birthday and the family
reports that they are planning to travel within the next 5 weeks and plan to stay for approximately 2 to 4 weeks. She completed her
December
immunization series with poliomyelitis virus vaccine, including receipt of the booster dose as a child.

Of the following, the BEST recommendation for this patient is

A. a single dose of inactivated poliovirus vaccine should be administered
B. a single dose of oral poliovirus vaccine should be administered 
Claim Credit C. 2 doses of inactivated poliovirus vaccine should be given 4 weeks apart 
Evaluation D. no further doses of vaccine are needed because the patient has been completely immunized

My Learning Plan

Submit   Reset  

Page 90 of 115

Contact Us
Links may open in separate window
Copyright © 2016   American Academy of Pediatrics. All rights reserved. Pop­up Blocker may need to be disabled

http://2016.prepid.courses.aap.org/script/october?q­page=2&req=201612122159545313# 1/1
12/13/2016 October

Welcome  mohammed alsaiary [ Logout ]

Home PREP Pearls FAQs My Bookmarks CME Information

Overview Home  > October

Editorial Board
October   Enter Keyword Search
January
Question View:   All (8) Jump to Question
February
Print this Page Add to my Bookmarks Page 90 of 115
March

April Assessment History Mode: Learner  Exam 

May ASSESSMENT PROGRESS:  Total Questions:  8  Questions Answered:  2  Correct Answers:  1


June

July You currently have 6 questions unanswered in this assessment

August

September
Question: 2
October
One of your adolescent patients is planning a trip to a polio­endemic region to visit her grandmother who has been receiving
November palliative chemotherapy and is not expected to survive much longer. The patient just celebrated her 20th birthday and the family
reports that they are planning to travel within the next 5 weeks and plan to stay for approximately 2 to 4 weeks. She completed her
December
immunization series with poliomyelitis virus vaccine, including receipt of the booster dose as a child.

Of the following, the BEST recommendation for this patient is

A. a single dose of inactivated poliovirus vaccine should be administered
B. a single dose of oral poliovirus vaccine should be administered 
Claim Credit C. 2 doses of inactivated poliovirus vaccine should be given 4 weeks apart 
Evaluation D. no further doses of vaccine are needed because the patient has been completely immunized

My Learning Plan

Correct View Peer Results
Average Percent Correct:  72.00%

Adults who are at increased risk of exposure to wild or vaccine­derived polioviruses and who previously
completed their primary immunization series with either inactivated poliovirus vaccine (IPV) or oral
poliovirus vaccine (OPV) should receive a single dose of IPV. Currently available data do not support
the need for additional lifetime booster doses. Since the patient in the vignette has previously been
completely immunized, response choice A is the best response. Oral poliovirus vaccine can be
excreted in the stool for several weeks and rarely up to 2 months, therefore this would not be a suitable
option for her given that her grandmother is immunocompromised. Further, OPV is no longer available
in the United States.

Unimmunized adults who are planning travel to a polio endemic area and those with increased risk of exposure, ie, during an
outbreak, working in healthcare settings, refugee camps, or during any humanitarian crisis, should complete their primary
immunization series with IPV given at intervals of 4 to 8 weeks or 1 to 2 months with a third dose 6 to 12 months after the
second dose. If there is not enough time to complete the 3 recommended doses, the following alternatives may be used:

If protection is not needed for ≥ 8 weeks, 3 doses of IPV should be given at least 4 weeks apart
If protection is not needed for 4 to 8 weeks, 2 doses of IPV should be given at least 4 weeks apart 
If protection is needed in less than 4 weeks, then a single dose of IPV should be given

Children who have not completed their primary immunization series (regardless of whether or not they are planning travel to
an endemic area) should receive the complete vaccine series, which consists of 4 doses. These schedules can be found at
http://redbook.solutions.aap.org/SS/Immunization_Schedules.aspx .

In addition to these standard recommendations, travelers may be affected by the new World Health Organization and US
Centers for Disease Control and Prevention temporary poliovirus vaccination recommendations that apply to individuals
residing for 4 consecutive weeks or more in countries with ongoing poliovirus transmission, and who leave to go to polio­free
countries. In such cases, the following would apply:

All residents and long term visitors (> 4 weeks), should receive an additional dose of OPV or IPV between 4 weeks and
12 months before international travel and have the dose documented

Residents and long term visitors who are currently in those countries and must travel with fewer than 4 weeks’ notice
and have not been vaccinated with OPV or IPV within the previous 4 week to 12 month period, should receive a dose at
least by the time of departure

The list of countries affected by these recommendations can be found at www.polioeradication.org.

http://2016.prepid.courses.aap.org/script/october?req=201612122200283907&status=submit# 1/2
12/13/2016 October
Individuals who are immunocompromised, including those with HIV, antibody deficiency syndromes (eg, common variable
immunodeficiency), malignancies such as leukemia, lymphomas, or other malignant neoplasms, or those on
immunosuppressive therapy with pharmacologic agents or those receiving radiation therapy should receive IPV. Protective
immune responses to IPV, however, cannot be ensured in such individuals.

Immunization during pregnancy is generally not recommended. If immunization is required, then IPV should be used.
Premature infants should also receive IPV based on their chronologic age, as recommended with all other vaccines.

Household contacts of immunocompromised individuals, including those with HIV infection, should receive IPV. In the event
that OPV is inadvertently administered, close contact should be minimized for at least 4 to 6 weeks. Such household
members should be counseled on practices that would minimize exposure of susceptible individuals to excreted poliovirus.
These practices include good handwashing and avoiding diaper changing of the immunized child by the immunocompromised
individual. In recipients of OPV, besides excretion in the stool for several weeks, virus persists in the throat for approximately
1 to 2 weeks. In the United States, receipt of OPV would not occur because it is no longer available. Individuals vaccinated
overseas may have received OPV.

PREP Pearls
A single booster dose of inactivated poliovirus vaccine (IPV) is recommended for adults who are at increased risk of
exposure to wild or vaccine­derived polioviruses and who previously completed their primary immunization series with
either IPV or oral poliovirus vaccine. 

Immunocompromised individuals and their household contacts should also receive IPV.

Travelers should check the World Health Organization and US Centers for Disease Control and Prevention websites prior
to travel to countries where poliovirus transmission may be a concern for any updated vaccination recommendations.

American Board of Pediatrics Content Specification(s)
Plan poliovirus immunization schedule for a patient preparing to travel to an endemic area

Manage a patient who requires poliovirus immunization because of special circumstances (eg, exposure to disease,
outbreak control, immunocompromised patient, HIV‑infected patient and their siblings, pregnancy, prematurity,
unimmunized adult contacts)

Suggested Readings
American Academy of Pediatrics. Poliovirus infections. In: Kimberlin DW, Brady MT, Jackson MA, Long SS, eds. Red Book:
2015 Report of the Committee on Infectious Diseases. 30th ed. Elk Grove Village, IL. American Academy of Pediatrics:
2015:644­650 .

Global Polio Eradication Initiative. Home page. Global Polio Eradication Initiative website. http://www.polioeradication.org

Comment On This Question

Page 90 of 115

Contact Us
Links may open in separate window
Copyright © 2016   American Academy of Pediatrics. All rights reserved. Pop­up Blocker may need to be disabled

http://2016.prepid.courses.aap.org/script/october?req=201612122200283907&status=submit# 2/2
12/13/2016 October

Welcome  mohammed alsaiary [ Logout ]

Home PREP Pearls FAQs My Bookmarks CME Information

Overview Home  > October

Editorial Board
October   Enter Keyword Search
January
Question View:   All (8) Jump to Question
February
Print this Page Add to my Bookmarks Page 91 of 115
March

April Assessment History Mode: Learner  Exam 

May ASSESSMENT PROGRESS:  Total Questions:  8  Questions Answered:  2  Correct Answers:  1


June

July You currently have 6 questions unanswered in this assessment

August

September
Question: 3
October
The parents of an 11­year­old boy request a consultation to discuss the indications for human papillomavirus (HPV) vaccine for their
November son.  

December Of the following, the indication for HPV vaccine in boys is to prevent

A. genital warts
B. infection of female partners
C. oropharyngeal cancer
Claim Credit D. penile cancer

Evaluation

My Learning Plan Submit   Reset  

Page 91 of 115

Contact Us
Links may open in separate window
Copyright © 2016   American Academy of Pediatrics. All rights reserved. Pop­up Blocker may need to be disabled

http://2016.prepid.courses.aap.org/script/october?q­page=3&req=201612122200288907 1/1
12/13/2016 October

Welcome  mohammed alsaiary [ Logout ]

Home PREP Pearls FAQs My Bookmarks CME Information

Overview Home  > October

Editorial Board
October   Enter Keyword Search
January
Question View:   All (8) Jump to Question
February
Print this Page Add to my Bookmarks Page 91 of 115
March

April Assessment History Mode: Learner  Exam 

May ASSESSMENT PROGRESS:  Total Questions:  8  Questions Answered:  3  Correct Answers:  2


June

July You currently have 5 questions unanswered in this assessment

August

September
Question: 3
October
The parents of an 11­year­old boy request a consultation to discuss the indications for human papillomavirus (HPV) vaccine for their
November son.  

December Of the following, the indication for HPV vaccine in boys is to prevent

A. genital warts
B. infection of female partners
C. oropharyngeal cancer
Claim Credit D. penile cancer

Evaluation

My Learning Plan Correct View Peer Results

Average Percent Correct:  34.00%

The primary indication for human papillomavirus (HPV) vaccine in boys is for prevention of genital
warts caused by serotypes 6 and 11. Until higher percentages of women are adequately vaccinated, it
is feasible that male HPV vaccination could provide some degree of herd immunity for female partners,
however, this has not been conclusively demonstrated and is not an indication for male HPV
vaccination. Similarly, although penile cancers are most often caused by the same serotypes as female
cervical, vulvar, and vaginal cancers, the incidence of this form of cancer is low enough that statistical
benefit from HPV vaccine in males was not demonstrated in the vaccine trials. Similarly, although
oropharyngeal cancers have been associated with HPV16 included in the vaccine, vaccination has not been shown to prevent
these lesions and is not an approved indication for vaccination.

Human papillomavirus is associated with a spectrum of lesions from common cutaneous warts to genital and oropharyngeal
cancers. Genital infections with HPV range from genital warts or condylomata acuminata (90% of which are caused by
serotypes 6 and 11) to genital cancers. In women, it has been demonstrated that cervical dysplasia and genital cancers
including cervical, vulvar, and vaginal lesions are associated with specific oncogenic strains of HPV. High­risk types in this
regard include types 16 and 18. These strains can cause a spectrum of lesions ranging from low­grade cervical cell
abnormalities to high­grade abnormalities that are precursors to genital and anogenital cancers; however, less common penile
cancers in males are associated with these same strains.  More recently, a significant proportion (up to two­thirds) of
oropharyngeal cancers have also been associated with HPV type 16. Another rare manifestation of genital HPV infection is
vertical transmission of the virus to the infant, resulting in juvenile recurrent respiratory papillomatosis.

However, most genital HPV infections are subclinical and resolve spontaneously within 1 to 2 years. If the virus persists
beyond that time frame and progresses to high­grade squamous intraepithelial lesions, which are considered precancerous,
therapy is required.

Genital HPV infections are transmitted through sexual intercourse and direct skin­to­skin contact, with the highest prevalence
of infection occurring in 20­ to 24­year­old individuals.  The cancers are detected 10 to 20 years or more after the initial HPV
infection. Currently, it is estimated that 27,000 HPV­related cancers, 12,000 new cervical cancers, and 4,000 deaths per year
are attributable to these infections.

The HPV vaccine (9­valent or 4­valent) is indicated for males aged 9 to 21 years (and through 26 years of age in men who
have sex with men and immunocompromised or HIV­infected males). These vaccines both contain types 6 and 11 for genital
wart protection and types 16 and 18. The newer 9­valent vaccine adds the additional oncogenic serotypes 31, 33, 45, 52, and
58. A bivalent HPV vaccine containing types 16 and 18 is also available for women.

PREP Pearls
http://2016.prepid.courses.aap.org/script/october?req=201612122200583283&status=submit 1/2
12/13/2016 October
Genital human papillomavirus (HPV) infections have specific strains associated with genital warts or condylomata
acuminata (types 6 and 11) and genital cancers (types 16, 18).

The majority of genital HPV infections resolve spontaneously, but persistent infection with an oncogenic strain can lead to
high­grade squamous intraepithelial lesions and a spectrum of genital and orophayrngeal cancers.

Human papillomavirus vaccination is indicated routinely for females and males beginning at ages 11 to 12 years, although
vaccine can be given as early as 9 years of age.

American Board of Pediatrics Content Specification(s)
Recognize the clinical manifestations of papillomavirus infection according to source (eg, common warts, anogenital warts,
laryngeal papillomas) and mode of transmission (eg, sexual, maternal, newborn)

Understand the usual course of HPV disease and subclinical infection and disease

Suggested Readings
American Academy of Pediatrics. Human papillomaviruses. In: Kimberlin DW, Brady MT, Jackson MA, Long SS, eds. Red
Book: 2015 Report of the Committee on Infectious Diseases. 30th ed. Elk Grove Village, IL: American Academy of Pediatrics;
2015:576­583. .

Markowitz LE, Dunne EF, Saraiya M, et al. Papilloma virus vaccination: recommendations of the Advisory Committee on
immunization Practices. MMWR Recomm Rep. 2014;63(RR­05):1­30.
http://www.cdc.gov/mmwr/preview/mmwrhtml/rr6305a1.htm

Petrosky E, Bocchini JA Jr, Hariri S, et al. Use of 9­valent human papillomavirus (HPV) vaccine: Updated HPV vaccination
recommendations of the Advisory Committee on Immunization Practices. MMWR Morb Mortal Wkly Rep. 2015;64(11):300­
304. http://www.cdc.gov/mmwr/preview/mmwrhtml/mm6411a3.htm

Comment On This Question

Page 91 of 115

Contact Us
Links may open in separate window
Copyright © 2016   American Academy of Pediatrics. All rights reserved. Pop­up Blocker may need to be disabled

http://2016.prepid.courses.aap.org/script/october?req=201612122200583283&status=submit 2/2
12/13/2016 October

Welcome  mohammed alsaiary [ Logout ]

Home PREP Pearls FAQs My Bookmarks CME Information

Overview Home  > October

Editorial Board
October   Enter Keyword Search
January
Question View:   All (8) Jump to Question
February
Print this Page Add to my Bookmarks Page 92 of 115
March

April Assessment History Mode: Learner  Exam 

May ASSESSMENT PROGRESS:  Total Questions:  8  Questions Answered:  3  Correct Answers:  2


June

July You currently have 5 questions unanswered in this assessment

August

September
Question: 4
October
The hospitalist service calls you for a consult on a 14­year­old boy who was admitted with complaints of fever for the past week
November along with swollen, tender, right­sided axillary and cervical lymphadenopathy. The patient also has an ulcer with raised edges on the
right shoulder (Figure). He is complaining of chills and body aches. He is otherwise previously healthy. The family does report that
December
they have been spending a lot of time outside since the children have been home for the summer holidays. They report going
camping, hunting and fishing. The family owns 2 dogs, 1 snake, and a turtle. The hospital team would like your input prior to sending
any laboratory tests or initiating antibiotic therapy.

Of the following, the MOST likely infectious agent responsible for the patient’s condition is

A. Bartonella henselae
Claim Credit
B. Francisella tularensis
Evaluation
C. Mycobacterium tuberculosis
My Learning Plan
D. Pasteurella multocida
E. Staphylococcus aureus

Submit   Reset  

Page 92 of 115

Contact Us
Links may open in separate window
Copyright © 2016   American Academy of Pediatrics. All rights reserved. Pop­up Blocker may need to be disabled

http://2016.prepid.courses.aap.org/script/october?q­page=4&req=201612122200588908 1/1
12/13/2016 October

Welcome  mohammed alsaiary [ Logout ]

Home PREP Pearls FAQs My Bookmarks CME Information

Overview Home  > October

Editorial Board
October   Enter Keyword Search
January
Question View:   All (8) Jump to Question
February
Print this Page Add to my Bookmarks Page 92 of 115
March

April Assessment History Mode: Learner  Exam 

May ASSESSMENT PROGRESS:  Total Questions:  8  Questions Answered:  4  Correct Answers:  2


June

July You currently have 4 questions unanswered in this assessment

August

September
Question: 4
October
The hospitalist service calls you for a consult on a 14­year­old boy who was admitted with complaints of fever for the past week
November along with swollen, tender, right­sided axillary and cervical lymphadenopathy. The patient also has an ulcer with raised edges on the
right shoulder (Figure). He is complaining of chills and body aches. He is otherwise previously healthy. The family does report that
December
they have been spending a lot of time outside since the children have been home for the summer holidays. They report going
camping, hunting and fishing. The family owns 2 dogs, 1 snake, and a turtle. The hospital team would like your input prior to sending
any laboratory tests or initiating antibiotic therapy.

Of the following, the MOST likely infectious agent responsible for the patient’s condition is

A. Bartonella henselae
Claim Credit
B. Francisella tularensis
Evaluation
C. Mycobacterium tuberculosis
My Learning Plan
D. Pasteurella multocida
E. Staphylococcus aureus

Incorrect View Peer Results

Correct Answer: B Average Percent Correct:  86.00%

The boy in the vignette most likely has ulceroglandular tularemia. Given the history of significant
outdoor activity, it would seem likely that he was bitten by either a tick or a biting fly with the
subsequent development of a vesicle at the site of the bite. Following the development of either a
papule or vesicle, the area may ulcerate as in this patient. This is usually accompanied by regional
lymphadenopathy as well as systemic symptoms.

Ulcerated lesions generally are not seen in patients with staphylococcal associated lymphadenitis or
Bartonella infections. The latter may present initially in a similar manner with a papule at the site of the scratch, but these
typically do not ulcerate. The history is too acute for the patient to have Mycobacterium tuberculosis, which usually presents
with unilateral lymphadenopathy that is generally not acutely inflamed or tender. Patients with Pasteurella multocida infections
usually have a clear history of an animal bite, often from a cat or a dog. They typically develop wound infections at the site of
the bite that are accompanied by regional lymphadenopathy and systemic symptoms, such as fever and chills.

Tularemia is the third most common tick­borne disease in the United States and can infect more than 100 animal species. It is
a zoonotic disease in which humans are accidental hosts. Infection is most commonly acquired from the bite of a flea or tick
that then transfers the organism. Other biting vectors, such as mosquitos or mites, can also transmit the disease. Infections
can also be transmitted via aerosols generated during lawn mowing, brush cutting or by direct contact with wild rabbits or
other infected animals (eg, ground squirrels, hares, beavers, voles, musk rats) while skinning, dressing, or eating these
animals. Infections can also be transmitted through contaminated food or water (likely facilitated by the organism’s ability to
survive in amebae). Other animals that have occasionally been implicated in disease transmission include cats, sheep,
hamsters, and prairie dogs. Person­to­person transmission through contact with infected ulcers generally is not a concern. At­
risk individuals have occupational or recreational exposure to infected animals or their habitats (eg, rabbit hunters and
trappers), people exposed to certain ticks or biting insects, and laboratory technicians working with Francisella tularensis,
which aerosolizes easily when grown in culture.

In areas where the highest numbers of cases have been reported (predominantly Missouri, Kansas, South Dakota, and
Arkansas), approximately one­third of the cases occurred in children, with those between the ages of 1 year through 14 years
having the highest attack rate. Most cases occur during the months of June to October.

Four species are known; however, human disease is due primarily to 2 subspecies: F tularensis subspecies tularensis (type
A), and F tularensis subspecies holarctica (type B). Subspecies B has a worldwide distribution and is less virulent, but can

http://2016.prepid.courses.aap.org/script/october?req=201612122201295158&status=submit# 1/2
12/13/2016 October
cause typhoidal infection in immunocompromised children. Subspecies B undergoes a waterborne cycle mainly, with
muskrats, beavers, and ground voles as reservoirs that also transmit the infection along with mosquitos.

Histopathology of lymph nodes in patients with tularemia usually shows granuloma formation with an area of central necrosis
that may contain bacteria. This must be differentiated from other infectious granulomatous disorders such as Bartonella,
Yersinia, and lymphogranuloma venereum. Suppurative changes distinguish this group from nonsuppurative granulomatous
disorders such as tuberculosis, leprosy, toxoplasmosis, syphilis, and brucellosis.

* Required *
Take Survey  

PREP Pearls
Multiple animal species (>100) have been implicated in the transmission of Francisella tularensis.

The most common mode of transmission of Francisella tularensis is through the bite of ticks and biting flies that carry the
organism.

Children 1 year of age through 14 years of age have the highest attack rate.

American Board of Pediatrics Content Specification(s)
Understand the epidemiology and modes of transmission of tularemia (tick, respiratory droplet, animal contact)

Formulate a differential diagnosis of granulomatous lymphadenitis with central necrosis, including cat scratch, tularemia,
chlamydia

Suggested Readings
American Academy of Pediatrics. Tularemia. In:  Kimberlin DW, Brady MT, Jackson MA, Long SS, eds. Red Book: 2015
Report of the Committee on Infectious Disease. 30th ed. Elk Grove Village, IL: American Academy of Pediatrics; 2015:839­
841. http://redbook.solutions.aap.org/chapter.aspx?sectionid=88187264&bookid=1484

Asano S. Granulomatous lymphadenitis. J Clin Exp Hematop. 2012;52(1):1­16. doi: http://dx.doi.org/10.3960/jslrt.52.1

Rubin LR. Francisella tularensis (Tularemia) In: Long SS, Pickering LK, Prober CG, eds. Principles and Practice of Pediatric
Infectious Diseases. 4th ed. Philadelphia, PA: Churchill Livingstone Elsevier; 2012:897­899 .

Comment On This Question

Page 92 of 115

Contact Us
Links may open in separate window
Copyright © 2016   American Academy of Pediatrics. All rights reserved. Pop­up Blocker may need to be disabled

http://2016.prepid.courses.aap.org/script/october?req=201612122201295158&status=submit# 2/2
12/13/2016 October

Welcome  mohammed alsaiary [ Logout ]

Home PREP Pearls FAQs My Bookmarks CME Information

Overview Home  > October

Editorial Board
October   Enter Keyword Search
January
Question View:   All (8) Jump to Question
February
Print this Page Add to my Bookmarks Page 93 of 115
March

April Assessment History Mode: Learner  Exam 

May ASSESSMENT PROGRESS:  Total Questions:  8  Questions Answered:  4  Correct Answers:  2


June

July You currently have 4 questions unanswered in this assessment

August

September
Question: 5
October
Researchers at Whatsamatta University wanted to determine if different colors had different rates of attracting mosquitoes. They
November tested mosquitoes in a closed environment with mosquito traps of monochromic colors of black, blue, red, green, and white,
recording numbers of mosquitoes trapped by color. By analysis of variance of repeated experiments, they determined with a p­value
December
of 0.05 that there was a difference in mean numbers of mosquitoes trapped by color. They then sought to determine which color(s)
were most likely to attract mosquitoes, utilizing the Bonferroni test. 

Of the following, the BEST description of this latter statistical analysis is

A. construct validity

Claim Credit B. imputation

Evaluation C. multiple regression analysis
D. post hoc analysis
My Learning Plan

Submit   Reset  

Page 93 of 115

Contact Us
Links may open in separate window
Copyright © 2016   American Academy of Pediatrics. All rights reserved. Pop­up Blocker may need to be disabled

http://2016.prepid.courses.aap.org/script/october?q­page=5&req=201612122201299377 1/1
12/13/2016 October

Welcome  mohammed alsaiary [ Logout ]

Home PREP Pearls FAQs My Bookmarks CME Information

Overview Home  > October

Editorial Board
October   Enter Keyword Search
January
Question View:   All (8) Jump to Question
February
Print this Page Add to my Bookmarks Page 93 of 115
March

April Assessment History Mode: Learner  Exam 

May ASSESSMENT PROGRESS:  Total Questions:  8  Questions Answered:  5  Correct Answers:  2


June

July You currently have 3 questions unanswered in this assessment

August

September
Question: 5
October
Researchers at Whatsamatta University wanted to determine if different colors had different rates of attracting mosquitoes. They
November tested mosquitoes in a closed environment with mosquito traps of monochromic colors of black, blue, red, green, and white,
recording numbers of mosquitoes trapped by color. By analysis of variance of repeated experiments, they determined with a p­value
December
of 0.05 that there was a difference in mean numbers of mosquitoes trapped by color. They then sought to determine which color(s)
were most likely to attract mosquitoes, utilizing the Bonferroni test. 

Of the following, the BEST description of this latter statistical analysis is

A. construct validity

Claim Credit B. imputation

Evaluation C. multiple regression analysis
D. post hoc analysis
My Learning Plan

Incorrect View Peer Results
Correct Answer: D Average Percent Correct:  60.00%

These investigators performed a post hoc analysis, meaning that the testing was done “after the fact”
of initial testing to determine the true source of difference among 3 or more groups. Construct validity
indicates how well a test (in this case, attraction of mosquitoes to different colored traps) reflects an
underlying construct (eg, whether trap color would impact mosquito trapping rates in outdoor
environments or whether clothing color would alter mosquito biting habits). Imputation refers to a group
of techniques to account for missing data elements, commonly in retrospective studies where not all
patients have all data elements of interest (eg, laboratory tests) obtained. It is a manipulation of the
data after the fact, but does not apply to the mosquito study. Multiple regression analysis is a means of predicting one variable
by analyzing several variables. It does not apply to the study described, but often prompts researchers to perform post hoc
analyses such as unplanned subgroup analyses in a “fishing expedition” to find any correlation with a low p­value.

Ideally, all statistical analyses in research would be planned prior to conducting the study. However, sometimes post hoc
analyses are performed when the initial, planned analysis shows a significant difference, but does not identify the source of
this difference from among 3 or more variables. The main problem with improperly performed post hoc analyses arises with
repeated statistical measurements, which increases the risk of type 1 error, concluding that a difference exists when it does
not.

In the mosquito example in the vignette, we have 5 different colors tested and we know that a difference exists somewhere in
that mix. To find out where, we could perform multiple t tests, each comparing one color against another. However, doing this
violates rules underlying these statistical tests, increasing the chance that one will find a p­value less than 0.05 in at least one
of the testing pairs, when in fact the p­value is much higher. This is a type 1 error. In fact, the real type 1 error rate with
repeated testing is 1 – (1 – α)κ, where α is the type 1 error rate (eg, 0.05) and κ is the number of comparisons (eg, 10). So,
running the math for our example if the investigators simply performed multiple pairwise comparisons of the colors and found
and reported a p­value of 0.05, in fact the true error rate would be 0.40, a markedly different answer. Thus, investigators need
to use different methods when comparing multiple groups.

Many statistical methods have been developed to minimize this type 1 error rate in multiple comparison situations. Examples
of such tests include the Bonferroni procedure, Tukey’s procedure, Dunnet’s correction, and Fisher’s least significant
difference, among others. All have the disadvantage of being conservative, ie, overcorrecting the p­value to avoid type 1 error,
and therefore contribute to many discussions among researchers and statisticians on how to put the data in the best light
possible while avoiding misleading conclusions. 

http://2016.prepid.courses.aap.org/script/october?req=201612122201586097&status=submit# 1/2
12/13/2016 October

PREP Pearls
Post hoc analysis is statistical testing performed after data collection in a research study.

Improperly performed post hoc analyses have great risk for type 1 error.

Post hoc analyses are commonly performed in studies with 3 or more groups, or subgroups.

Many statistical methods, such as the Bonferroni procedure, are available to minimize type 1 errors in these situations.

American Board of Pediatrics Content Specification(s)
Recognize a post­hoc analysis

Understand the strengths and limitations of post­hoc analyses

Suggested Readings
Salkind NJ. Statistics For People Who (Think They) Hate Statistics. 2nd ed. Thousand Oaks, CA: SAGE Publications;
2004:193­211 .

Sedgwick P. One­way analysis of variance: post hoc testing. BMJ. 2014;349:g7067. doi: http://dx.doi.org/10.1136/bmj.g7067

Sedgwick P. Pitfalls of statistical hypothesis testing: multiple testing. BMJ. 2014;349:g5310. doi:
http://dx.doi.org/10.1136/bmj.g5310

Wang R, Lagakos SW, Ware JH, Hunter DJ, Drazen JM. Statistics in medicine: reporting of subgroup analyses in clinical
trials. N Engl J Med. 2007;357:2189­2194. doi: http://dx.doi.org/10.1056/NEJMsr077003

Comment On This Question

Page 93 of 115

Contact Us
Links may open in separate window
Copyright © 2016   American Academy of Pediatrics. All rights reserved. Pop­up Blocker may need to be disabled

http://2016.prepid.courses.aap.org/script/october?req=201612122201586097&status=submit# 2/2
12/13/2016 October

Welcome  mohammed alsaiary [ Logout ]

Home PREP Pearls FAQs My Bookmarks CME Information

Overview Home  > October

Editorial Board
October   Enter Keyword Search
January
Question View:   All (8) Jump to Question
February
Print this Page Add to my Bookmarks Page 94 of 115
March

April Assessment History Mode: Learner  Exam 

May ASSESSMENT PROGRESS:  Total Questions:  8  Questions Answered:  5  Correct Answers:  2


June

July You currently have 3 questions unanswered in this assessment

August

September
Question: 6
October
A family that is moving to Europe for a year is being seen in your travel clinic. The parents request advice on immunizing their
November healthy 2­year­old prior to their move. She has received all age­appropriate vaccinations, as shown:

December Vaccine Number of Doses Age at Last Dose

   Hepatitis B 3 9 months

   Hib 4 15 months

   DTaP 4 15 months
Claim Credit
   IPV 3 15 months
Evaluation
   MMR 1 12 months
My Learning Plan
   Varicella 1 12 months

   Hepatitis A 2 22 months

   Pneumococcal conjugate 13­valent 4 15 months

Of the following, the BEST advice for additional immunization at this time is that the girl should receive

A. Diphtheria tetanus and pertussis vaccines (DTaP)
B. Inactivated polio vaccine (IPV)
C. Measles, mumps, rubella vaccines (MMR)
D. Meningococcal conjugate vaccine quadrivalent (MCV4)

Submit   Reset  

Page 94 of 115

Contact Us
Links may open in separate window
Copyright © 2016   American Academy of Pediatrics. All rights reserved. Pop­up Blocker may need to be disabled

http://2016.prepid.courses.aap.org/script/october?q­page=6&req=201612122201589847 1/1
12/13/2016 October

Welcome  mohammed alsaiary [ Logout ]

Home PREP Pearls FAQs My Bookmarks CME Information

Overview Home  > October

Editorial Board
October   Enter Keyword Search
January
Question View:   All (8) Jump to Question
February
Print this Page Add to my Bookmarks Page 94 of 115
March

April Assessment History Mode: Learner  Exam 

May ASSESSMENT PROGRESS:  Total Questions:  8  Questions Answered:  6  Correct Answers:  2


June

July You currently have 2 questions unanswered in this assessment

August

September
Question: 6
October
A family that is moving to Europe for a year is being seen in your travel clinic. The parents request advice on immunizing their
November healthy 2­year­old prior to their move. She has received all age­appropriate vaccinations, as shown:

December Vaccine Number of Doses Age at Last Dose

   Hepatitis B 3 9 months

   Hib 4 15 months

   DTaP 4 15 months
Claim Credit
   IPV 3 15 months
Evaluation
   MMR 1 12 months
My Learning Plan
   Varicella 1 12 months

   Hepatitis A 2 22 months

   Pneumococcal conjugate 13­valent 4 15 months

Of the following, the BEST advice for additional immunization at this time is that the girl should receive

A. Diphtheria tetanus and pertussis vaccines (DTaP)
B. Inactivated polio vaccine (IPV)
C. Measles, mumps, rubella vaccines (MMR)
D. Meningococcal conjugate vaccine quadrivalent (MCV4)

Incorrect View Peer Results
Correct Answer: C Average Percent Correct:  64.00%

Children who are traveling abroad should be brought up to date on their routine vaccines. These
include influenza, hepatitis B, tetanus, diphtheria, pertussis, measles, mumps, rubella, and poliomyelitis
vaccines. There are special travel­related considerations for measles and poliomyelitis vaccines.
Routinely, the first dose of MMR is given at 12 months of age, and the second at 4 to 6 years of age.
However, because mumps and measles are endemic in most of the world, there are 2 modifications to
the recommended schedule for children who are traveling. Infants who are 6 to 12 months of age
should receive a dose of MMR vaccine before beginning travel and then receive the 2 routine doses,
after they turn 12 months of age. Children who are older than 12 months should receive the second dose of MMR before
beginning travel, as long as 28 days have elapsed since the first dose. Although the girl in the vignette has received all age­
appropriate vaccines, it is important that she receive the second dose of MMR at this time. Travel to Europe is no exception
because of the current high incidence of measles in several European countries. MCV4 is not recommended, except for
children with high­risk conditions such as anatomic or functional asplenia or persistent complement component deficiency.
However, if a child will travel to or reside in a country in which meningococcal disease epidemics are likely, he or she should
receive age­appropriate doses of MCV4. Diphtheria, tetanus and pertussis vaccines (DTaP) are not needed as the girl is up to
date on these vaccines. A further dose of IPV is not indicated at this time. At or after 18 years of age, a future booster dose of
IPV is recommended if she travels to a polio­endemic area. In addition to the 1­time booster dose of IPV, travelers may be
affected by World Health Organization and US Centers for Disease Control and Prevention temporary poliovirus vaccination
recommendations for individuals residing for 4 consecutive weeks or more in countries with ongoing poliovirus transmission.

http://2016.prepid.courses.aap.org/script/october?req=201612122202332035&status=submit 1/3
12/13/2016 October
The list of countries that are either “exporting,”  “infected” or “ vulnerable” to international spread of wild poliovirus or cVDPV,
are constantly updated at http://www.polioeradication.org/Keycountries.aspx. These persons should receive an additional
dose of OPV or IPV between 4 weeks and 12 months before international travel and have the dose documented. Residents
and long­term visitors who are currently in those countries and must travel with fewer than 4 weeks’ notice, and have not
been vaccinated with OPV or IPV within the previous 4 week­12 month period, should receive a dose at least by the time of
departure.

Hepatitis A vaccine (HAV) is routine for all children in the United States as part of the immunization schedule, but catch­up
immunization may not have been completed. The HAV vaccine status should always be reviewed before foreign travel
because the risk of acquisition is high in resource­limited nations, and intermediate­to­low risk exists in most of the rest of the
world. Two monovalent hepatitis A vaccines (GlaxoSmithKline and Merck & Co.) are available in the United States and one
bivalent (hepatitis A and B, Twinrix, GlaxoSmithKline) vaccine. Both are formalin­inactivated whole virus vaccines. A
monovalent vaccine is given as a single dose before travel, with administration of the second dose 6 to 12 months later.
Optimally, the bivalent vaccine requires 2 doses before travel, with completion of the third dose after 6 months. Immune
globulin intramuscular is recommended for babies younger than 12 months of age.

A meningococcal vaccine may be recommended or required prior to foreign travel. For example, outbreaks of serogroup A
meningococcal disease regularly occur in the sub­Saharan African region known as the meningitis belt during the dry season
of December to June, and proof of receipt of a quadrivalent meningococcal vaccine is required for pilgrims entering Saudi
Arabia. Meningococcal group B vaccines are not recommended for travel. The Table summarizes recommendations for use
of meningococcal vaccines licensed in the United States.

Salmonella Typhi and Salmonella Paratyphi infection are acquired by ingestion of contaminated food or water or by contact
with carriers. The highest current incidence of enteric fever is in south and southeast Asia, but exists throughout Asia, eastern
Europe, Africa, and Latin America. Immunization is recommended even for short travel periods to high­risk areas; about 1 in 7
US cases occur after trips of less than 2 weeks duration. Two vaccines are currently available in the United States, an oral
live­attenuated Salmonella Typhi Ty21a strain vaccine formulated as enteric­coated capsules containing lyophilized bacteria
(Vivotif Crucell/PaxVax), and an intramuscular Salmonella Typhi Vi polysaccharide vaccine (Typhim Vi, Sanofi Pasteur). The
live­attenuated vaccine provides some cross­protection against S paratyphi and is approved for children 6 years of age and
older. The intramuscular polysaccharide vaccine is approved for children 2 years of age and older. Booster doses are
recommended for travelers every 5 years for the oral Ty21a vaccine and every 2 to 3 years for the capsular polysaccharide
vaccine.

Both United States licensed rabies vaccines are cell culture­derived, inactivated virus vaccines. Human diploid cell vaccine
(HDCV, RabAvert, Sanofi Pasteur) and purified chick embryo cell culture vaccine (PCECV, Imovax Rabies, GSK) are
administered as intramuscular injection, in the deltoid for optimal immunogenicity. Advance planning is essential because both
vaccines are administered in a 3­dose regimen (days 0, 7, and 21–28). Very few countries are considered rabies­free
(http://www.who.int/rabies/Global_distribution_risk_humans_contracting_rabies_2013.png?ua=1A). Rabies prophylaxis is a
strong consideration for children who have even a small risk of being in contact with rabid animals. This includes travel to
rural areas or farms, and those who are camping. The most common exposure is from stray dogs, encountered throughout
the developing world. Moreover, cell culture­derived vaccines are not always available abroad, and there is a higher risk of
adverse reactions from older technology vaccines.

Yellow fever is restricted to the South American and African continents, specifically within tropical sub­Saharan Africa and in
the rainforests of tropical South America, (see http://www.who.int/ith/YFrisk.pdf?ua=1 and http://www.who.int/ith/en/ for up­to­
date yellow fever maps).

The vaccine is injectable, live attenuated virus 17D­204 strain (YF­VAX, Sanofi Pasteur) administered as a single dose
subcutaneously. Certain countries require proof of vaccination before entry under the International Health Regulations (WHO).
Some countries require proof of vaccination from all travelers; others may require proof of vaccination for persons traveling
from or through a country with yellow fever transmission risk. Requirements and recommendations are periodically updated
by the WHO (http://www.who.int/ith/2016­ith­annex1.pdf?ua=1). Although the duration of vaccine­induced immunity is now
established to be lifetime, some countries have not explicitly endorsed this and may enforce a 10­year validity. In addition,
vaccination is recommended for all persons 9 months of age and older who are traveling to or living in areas where there is
risk of yellow fever transmission. Since this is a live attenuated virus vaccine, travelers with a contraindication to vaccination
should be advised not to travel to areas with significant ongoing transmission of yellow fever. Vaccine should be administered
no less than 10 days prior to the planned date of entry. Immunization must be entered and validated in the specific section of
the International Certificate of Vaccination or Prophylaxis (ICVP).

Japanese encephalitis (JE), caused by a flavivirus transmitted by Culex mosquitoes, is endemic to Asia, chiefly in rural
agricultural areas where flooding irrigation is practiced. Although the reported incidence of JE among visitors to Asia is less
than 1 case per 1 million travelers, the vaccine is recommended if the person plans to stay 1 month or longer in endemic
areas during the transmission season. This includes those who will be based in urban areas but will visit endemic rural areas.
The currently licensed vaccine in the United States is inactivated Vero Cell Vaccine SA14­14­2 (IXIARO®, Valneva), for use in
persons 2 months and older. It is administered as 2 doses intramuscularly, 28 days apart, to be completed at least 1 week
prior to potential exposure.

The need for malaria chemoprophylaxis depends upon destinations within a country, accommodations, activities, duration of
travel, and malaria parasite resistance patterns. The CDC provides specific recommendations for the indications and use of
malaria chemoprophylaxis by destination (http://www.cdc.gov/malaria/travelers/country_table/p.html). The goal of
chemoprophylaxis is primarily the prevention of Plasmodium falciparum infection, which causes the most severe morbidity
and can result in death. Among US travelers, more than 80% of malaria acquired in Africa or the Americas are P falciparum
infections. The epidemiology of chloroquine resistance in P falciparum is the most important factor in the choice of drug. The
drugs in use also prevent Plasmodium vivax infection, with the exception of chloroquine­resistant P vivax in Papua New
Guinea and Indonesia. A general guide to the chemoprophylaxis drugs of choice is as follows:

If the destination has chloroquine­susceptible P falciparum malaria, use chloroquine. Chloroquine­resistant P falciparum
are widespread, and the only areas with reliably susceptible organisms are in Latin America west of the Panama Canal
and in a few parts of the Caribbean.
If the destination has chloroquine­resistant P falciparum, use mefloquine, doxycycline, or atovaquone/proguanil. This
includes the vast majority of malaria­endemic destinations around the world.
If the destination has mefloquine­resistant P falciparum, use atovaquone/proguanil or doxycycline. For example,
mefloquine resistance is present in Southeast Asia in parts of Cambodia, Thailand, and Vietnam.
Atovaquone/proguanil may be preferable to doxycycline or mefloquine for short­term travel (< 2­3 weeks) because of the
convenience of being able to stop the drug 7 days after departure from a malarious area.

http://2016.prepid.courses.aap.org/script/october?req=201612122202332035&status=submit 2/3
12/13/2016 October

PREP Pearls
Children who are travelling abroad should be brought up to date on their routine vaccines, including special considerations
for measles and polio vaccines.

Destination­specific epidemiology dictates the administration of travel­specific vaccines.

The drug­resistance profile of Plasmodium species varies considerably by geography.

American Board of Pediatrics Content Specification(s)
Recognize when malaria prophylaxis is necessary for foreign travel

Recommend specific vaccine administration prior to foreign travel (eg, MMR, Salmonella, cholera, Japanese encephalitis,
poliovirus vaccines)

Suggested Readings
Global Polio Eradication Initiative. Key countries. Global Polio Eradication Initiative website.
http://www.polioeradication.org/Keycountries.aspx

Hahn WO, Pottinger PS. Malaria in the traveler: how to manage before departure and evaluate upon return. Med Clin North
Am. 2016;100(2):289­302. doi: http://dx.doi.org/10.1016/j.mcna.2015.09.008

US Centers for Disease Control and Prevention. The Yellow Book: CDC Health Information for International Travel 2016.
Oxford, England: Oxford University Press; 2016:688 pp.

World Health Organization. International travel and health 2012. World Health Organization website.
http://who.int/ith/ITH_EN_2012_WEB_1.2.pdf?ua=1

Comment On This Question

Page 94 of 115

Contact Us
Links may open in separate window
Copyright © 2016   American Academy of Pediatrics. All rights reserved. Pop­up Blocker may need to be disabled

http://2016.prepid.courses.aap.org/script/october?req=201612122202332035&status=submit 3/3
12/13/2016 October

Welcome  mohammed alsaiary [ Logout ]

Home PREP Pearls FAQs My Bookmarks CME Information

Overview Home  > October

Editorial Board
October   Enter Keyword Search
January
Question View:   All (8) Jump to Question
February
Print this Page Add to my Bookmarks Page 95 of 115
March

April Assessment History Mode: Learner  Exam 

May ASSESSMENT PROGRESS:  Total Questions:  8  Questions Answered:  6  Correct Answers:  2


June

July You currently have 2 questions unanswered in this assessment

August

September
Question: 7
October
A 16­year­old previously healthy adolescent presents with yellow­white vaginal discharge for 3 days. The patient is sexually active
November with several male partners and uses condoms occasionally. On examination, she has mucopurulent exudate and bleeding at the
cervical os. She cannot recall her last menstrual period and her urine pregnancy test is positive.
December
Of the following, the MOST appropriate next step is

A. fetal ultrasonography to date the pregnancy
B. nucleic acid amplification test for Chlamydia trachomatis and Neisseria gonorrhoeae
C. nucleic acid amplification test for Chlamydia trachomatis and Neisseria gonorrhoeae plus wet prep for Trichomonas
Claim Credit vaginalis
Evaluation D. nucleic acid amplification test for Chlamydia trachomatis, Neisseria gonorrhoeae, and Trichomonas vaginalis

My Learning Plan

Submit   Reset  

Page 95 of 115

Contact Us
Links may open in separate window
Copyright © 2016   American Academy of Pediatrics. All rights reserved. Pop­up Blocker may need to be disabled

http://2016.prepid.courses.aap.org/script/october?q­page=7&req=201612122202336097 1/1
12/13/2016 October

Welcome  mohammed alsaiary [ Logout ]

Home PREP Pearls FAQs My Bookmarks CME Information

Overview Home  > October

Editorial Board
October   Enter Keyword Search
January
Question View:   All (8) Jump to Question
February
Print this Page Add to my Bookmarks Page 95 of 115
March

April Assessment History Mode: Learner  Exam 

May ASSESSMENT PROGRESS:  Total Questions:  8  Questions Answered:  7  Correct Answers:  2


June

July You currently have 1 questions unanswered in this assessment

August

September
Question: 7
October
A 16­year­old previously healthy adolescent presents with yellow­white vaginal discharge for 3 days. The patient is sexually active
November with several male partners and uses condoms occasionally. On examination, she has mucopurulent exudate and bleeding at the
cervical os. She cannot recall her last menstrual period and her urine pregnancy test is positive.
December
Of the following, the MOST appropriate next step is

A. fetal ultrasonography to date the pregnancy
B. nucleic acid amplification test for Chlamydia trachomatis and Neisseria gonorrhoeae
C. nucleic acid amplification test for Chlamydia trachomatis and Neisseria gonorrhoeae plus wet prep for Trichomonas
Claim Credit vaginalis
Evaluation D. nucleic acid amplification test for Chlamydia trachomatis, Neisseria gonorrhoeae, and Trichomonas vaginalis

My Learning Plan

Incorrect View Peer Results

Correct Answer: D Average Percent Correct:  28.00%

Per the 2015 US Centers for Disease Control and Prevention (CDC) sexually transmitted diseases
treatment guidelines, the 2 major diagnostic signs of cervicitis are purulent endocervical exudate and
sustained endocervical bleeding easily induced by gentle passage of a cotton swab into the cervical os.
The evaluation and treatment of pregnant adolescents with cervicitis is the same as it is for
nonpregnant women. Dating the pregnancy is unnecessary, as women in all trimesters should be
tested and treated.  Testing should target the organisms most commonly implicated in cervicitis,
including. Chlamydia trachomatis, Neisseria gonorrhoeae, and Trichomonas vaginalis. The most
sensitive and specific test for all 3 organisms is a nucleic acid amplification test (NAAT) of the urine and/or endocervical
sample. Microscopy or the wet prep test for T vaginalis shows 50% sensitivity and is no longer recommended. Other
organisms that may play a role are Gardnerella vaginalis, which causes bacterial vaginosis (BV), Mycoplasma genitalium, and
herpes simplex virus 2 (HSV2). Bacterial vaginosis is diagnosed through the visualization of clue cells, epithelial cells with
bacteria adherent to the border, and through a positive whiff test on a KOH slide. There are currently no US Food and Drug
Administration­approved tests for Mycoplasma genitalium. The utility of testing for HSV2 is unknown. Presumptive treatment
for cervicitis is one dose of azithromycin 1 g orally or doxycycline 100 mg twice daily for 7 days. Treatment for N gonorrhoeae
and T vaginalis should be offered if testing is positive or if the patient is high risk. Azithromycin is the recommended choice in
pregnancy as doxycyline is contraindicated. Metronidazole to treat BV is recommended in pregnancy. M genitalium has been
linked to pelvic inflammatory disease, preterm delivery, and infertility; treatment with moxifloxacin should be considered in
cases of persistent or recurrent cervicitis that has been adequately treated for typical pathogens. All sexual partners of
patients with cervicitis in the past 60 days should be tested and treated. Patients should be instructed to abstain from sexual
activity for 7 days. 

Screening pregnant women for sexually transmitted diseases serves to prevent pregnancy complications and neonatal
infections, thus routine testing of all pregnant women is recommended by the CDC. Additionally, pregnant women younger
than 25 years of age and those at high risk for chlamydia and gonorrhea should be retested during the third trimester. If a
pregnant women is found to be infected with chlamydia, a test of cure should be obtained 3 to 4 weeks post­therapy, 3
months following treatment, and in the third trimester, as reinfection is common. Testing for HIV and syphilis is recommended
routinely for pregnant women and for all those that test positive for chlamydia or gonorrhea.  Routine screening for
trichomonas and HSV is not recommended. 

PREP Pearls
Pregnant women should tested and treated accordingly for sexually transmitted infections. 

http://2016.prepid.courses.aap.org/script/october?req=201612122203116723&status=submit# 1/2
12/13/2016 October
Nucleic acid amplification test is the test of choice for Trichomonas.

Treatment for Mycoplasma genitalium should be considered for recurrent and persistent cervicitis.

American Board of Pediatrics Content Specification(s)
Identify the microbiologic etiologies of cervicitis in a sexually active adolescent female (eg, C.trachomatis, N.gonorrhoeae,
herpes simplex virus, Trichomonas)

Know which cervical infections (based on microbiologic etiology) warrant screening and antimicrobial therapy in at‑risk
pregnant women

Plan the initial empiric antimicrobial therapy of cervicitis

Suggested Readings
American Academy of Pediatrics. Sexually transmitted infections in adolescents and children. In: Kimberlin DW, Brady MT,
Jackson MA, Long SS, eds. Red Book: 2015 Report of the Committee on Infectious Disease. 30th ed. Elk Grove Village, IL:
American Academy of Pediatrics; 2015:177­188.

US Centers for Disease Control and Prevention. 2015 sexually transmitted diseases  treatment guidelines. MMWR Morbid
Mortal Wkly Rep. 2015;64(3):1­135. http://www.cdc.gov/std/tg2015/tg­2015­print.pdf

Comment On This Question

Page 95 of 115

Contact Us
Links may open in separate window
Copyright © 2016   American Academy of Pediatrics. All rights reserved. Pop­up Blocker may need to be disabled

http://2016.prepid.courses.aap.org/script/october?req=201612122203116723&status=submit# 2/2
12/13/2016 October

Welcome  mohammed alsaiary [ Logout ]

Home PREP Pearls FAQs My Bookmarks CME Information

Overview Home  > October

Editorial Board
October   Enter Keyword Search
January
Question View:   All (8) Jump to Question
February
Print this Page Add to my Bookmarks Page 96 of 115
March

April Assessment History Mode: Learner  Exam 

May ASSESSMENT PROGRESS:  Total Questions:  8  Questions Answered:  7  Correct Answers:  2


June

July You currently have 1 questions unanswered in this assessment

August

September
Question: 8
October
A 4­month­old former 36­week premature infant son of a chicken farmer from the central United States presents with a 2­month
November history of increasing abdominal distention and 2 weeks of vomiting, fussiness, and intermittent fever to 39.5°C. He is referred to the
emergency center after massive hepatosplenomegaly is noted on physical examination by his pediatrician. On admission, his
December
temperature is 37°C, heart rate is 134 beats/minute, respiratory rate is 39 breaths/minute, and blood pressure 92/64 is mm Hg.
Weight and length are 75th percentile for age. Physical examination is notable for a distended abdomen with hepatomegaly to the
pelvis and a spleen that is palpable 6 cm below the costal margin.

Laboratory studies reveal a hemoglobin of 9.9 g/dL (99 g/L), platelet count of 55 x 103/µL (55 x 109/L) and lactate dehydrogenase of
1,542 U/L (25.8 µkat/L). Computed tomography of the chest, abdomen, and pelvis show a confluent airspace opacity in the right
Claim Credit upper lobe; bulky and confluent lymphadenopathy in the mediastinum, paratracheal, prevascular, hilar and retroperitoneal regions;
hepatosplenomegaly; and a retrocrural soft tissue mass measuring 2.3 x 2.4 x 1.1 cm (Figure 1). Biopsies of a retroperitoneal lymph
Evaluation
node and the soft tissue mass demonstrate multiple noncaseating granulomata (Figure 2) with yeast­like organisms (Figure 3).
My Learning Plan
A. amphotericin B
B. fluconazole
C. itraconazole
D. micafungin

Submit   Reset  

Page 96 of 115

Contact Us
Links may open in separate window
Copyright © 2016   American Academy of Pediatrics. All rights reserved. Pop­up Blocker may need to be disabled

http://2016.prepid.courses.aap.org/script/october?q­page=8&req=201612122203121098 1/1
12/13/2016 October

Welcome  mohammed alsaiary [ Logout ]

Home PREP Pearls FAQs My Bookmarks CME Information

Overview Home  > October

Editorial Board
October   Enter Keyword Search
January
Question View:   All (8) Jump to Question
February
Print this Page Add to my Bookmarks Page 96 of 115
March

April Assessment History Mode: Learner  Exam 

May ASSESSMENT PROGRESS:  Total Questions:  8  Questions Answered:  8  Correct Answers:  3


June

July You currently have 1 required survey(s) that are incomplete. Please complete the survey(s) for the following
question(s) on this assessment:  Question 4
August

September

October Question: 8
November A 4­month­old former 36­week premature infant son of a chicken farmer from the central United States presents with a 2­month
history of increasing abdominal distention and 2 weeks of vomiting, fussiness, and intermittent fever to 39.5°C. He is referred to the
December
emergency center after massive hepatosplenomegaly is noted on physical examination by his pediatrician. On admission, his
temperature is 37°C, heart rate is 134 beats/minute, respiratory rate is 39 breaths/minute, and blood pressure 92/64 is mm Hg.
Weight and length are 75th percentile for age. Physical examination is notable for a distended abdomen with hepatomegaly to the
pelvis and a spleen that is palpable 6 cm below the costal margin.

Laboratory studies reveal a hemoglobin of 9.9 g/dL (99 g/L), platelet count of 55 x 103/µL (55 x 109/L) and lactate dehydrogenase of
Claim Credit 1,542 U/L (25.8 µkat/L). Computed tomography of the chest, abdomen, and pelvis show a confluent airspace opacity in the right
upper lobe; bulky and confluent lymphadenopathy in the mediastinum, paratracheal, prevascular, hilar and retroperitoneal regions;
Evaluation hepatosplenomegaly; and a retrocrural soft tissue mass measuring 2.3 x 2.4 x 1.1 cm (Figure 1). Biopsies of a retroperitoneal lymph
My Learning Plan node and the soft tissue mass demonstrate multiple noncaseating granulomata (Figure 2) with yeast­like organisms (Figure 3).

A. amphotericin B
B. fluconazole
C. itraconazole
D. micafungin

Correct View Peer Results

Average Percent Correct:  76.00%

The infant described in the vignette has a disseminated fungal infection due to Histoplasma
capsulatum and amphotericin B is the preferred antifungal therapy. Amphotericin B is also the drug of
choice for the treatment of systemic candidiasis in neonates, severe coccidioidomycosis, severe
cryptococcosis, moderate­to­severe blastomycosis, and mucormycosis. In addition, prior to the
availability of voriconazole, amphotericin B was the preferred agent for the treatment of Aspergillus.

Histoplasma capsulatum var capsulatum are fungi that live in the soil as spore­bearing mycelia, but
convert to yeast at 37°C. The organism’s growth in the environment is promoted by moist, nitrogen­rich soil and the nutrients
found in bird (including chickens) and bat droppings. Although the organism is found in most parts of the world (eg, Central
and South America, Africa, Asia, and Australia), it is highly endemic in the central United States (eg, Mississippi, Ohio, and
Missouri River Valleys).

The majority of patients infected with H capsulatum are asymptomatic or have a self­limited febrile respiratory illness without
sequelae. The development of symptomatic disease in immunocompetent individuals is thought to be inoculum­dependent. In
general, development of disease and disease severity is more likely in individuals with primary or acquired cellular immunity
disorders and infants younger than 1 year of age, such as the patient described in the vignette. Progressive dissemination of
H capsulatum infection acquired during infancy is common.

The definitive diagnosis of H capsulatum infection is by growth of the organism from tissue, sputum, blood, or bone marrow
on standard mycologic media. Histopathology demonstrating 2­ to 4­µm yeast forms and granulomatous inflammation is
highly suggestive of infection. Serologic assays or detection of antigen in tissues or fluids (eg, urine, blood) are widely
available, but cross­reactivity due to infection with other endemic mycoses or molds can occur.

For immunocompetent children with mild­to­moderate infection that has not improved after 4 to 6 weeks, itraconazole for 6 to
12 weeks is recommended. Itraconazole is preferred over fluconazole because it is less likely to induce resistance and is
more effective for the treatment of histoplasmosis. A lipid formulation of amphotericin B is preferred as the initial therapy for

http://2016.prepid.courses.aap.org/script/october?req=201612122203386255&status=submit# 1/2
12/13/2016 October
severe pulmonary infections, meningitis, disseminated infections, and all immunocompromised hosts. The duration of
amphotericin B treatment depends on the site of infection, immune status of the host, and clinical and laboratory response.

In cases of severe pulmonary infection, amphotericin B usually is administered for 1 to 2 weeks followed by itraconazole for
12 weeks. For meningitis, amphotericin B is given for 3 months followed by itraconazole for 12 months. Patients with
disseminated infection, such as the infant in the vignette, generally receive 2 weeks of amphotericin B followed by 6 months
of itraconazole. Antifungal treatment in immunocompromised patients may be prolonged or indefinite. Echinocandins (eg,
micafungin) are ineffective for the treatment of histoplasmosis.

PREP Pearls
Amphotericin B is the antifungal drug of choice for disseminated histoplasmosis.

Amphotericin B is the drug of choice for the treatment of systemic candidiasis in neonates, severe coccidioidomycosis,
severe cryptococcosis, moderate­to­severe blastomycosis, and mucormycosis.

Prior to the availability of voriconazole, amphotericin B was the preferred agent for the treatment of Aspergillus.

The majority of patients infected with Histoplasma capsulatum are asymptomatic or have a self­limited febrile respiratory
illness without sequelae.

Symptomatic and severe disease is more likely in individuals with primary or acquired cellular immunity disorders and
infants younger than 1 year of age.

American Board of Pediatrics Content Specification(s)
Know the clinical situations/pathogens when amphotericin is the drug of choice (Aspergillus, systemic candidiasis, severe
coccidioidomycosis, cryptococcosis, disseminated histoplasmosis, blastomycosis mucormycosis) and how this,has
changed due to new azoles

Suggested Readings
American Academy of Pediatrics. Histoplasmosis. In: Kimberlin DW, Brady MT, Jackson MA, Long SS, eds. Red Book: 2015
Report of the Committee on Infectious Diseases. 30th ed. Elk Grove Village, IL: American Academy of Pediatrics; 2015:445­
448..

Jorgenson JH, Pfaller MA, Carroll KC, et al. Manual of Clinical Microbiology, 11th ed. Washington, DC: ASM Press;
2015:2892 pp.

Kleiman MB. Histoplasmosis. In: Cherry JD, Demmler­Harrison GJ, Kaplan SL, Steinbach WJ, Hotez PJ, eds. Feigin and
Cherry’s Textbook of Pediatric Infectious Diseases. 7th ed. Philadelphia, PA: Saunders Elsevier; 2013:2807­2831.

US Centers for Disease Control and Prevention. Information for healthcare professionals about histoplasmosis. US Centers
for Disease Control and Prevention website. http://www.cdc.gov/fungal/diseases/histoplasmosis/health­professionals.html.
Updated February 8, 2016

Comment On This Question

Page 96 of 115

Contact Us
Links may open in separate window
Copyright © 2016   American Academy of Pediatrics. All rights reserved. Pop­up Blocker may need to be disabled

http://2016.prepid.courses.aap.org/script/october?req=201612122203386255&status=submit# 2/2
12/13/2016 November

Welcome  mohammed alsaiary [ Logout ]

Home PREP Pearls FAQs My Bookmarks CME Information

Overview Home  > November

Editorial Board
November   Enter Keyword Search
January
Question View:   All (8) Jump to Question
February
Print this Page Add to my Bookmarks Page 98 of 115
March

April Assessment History Mode: Learner  Exam 

May ASSESSMENT PROGRESS:  Total Questions:  8  Questions Answered:  0  Correct Answers:  0


June

July You currently have 8 questions unanswered in this assessment

August

September
Question: 1
October
You are asked to evaluate a 9­year­old previously healthy girl, who is admitted to the pediatric intensive care unit for acute onset of
November hypotension, fever, and multiorgan system dysfunction. She had been well until 24 hours ago, when she developed fever, sore
throat, and a generalized erythematous rash on her trunk. Over the next 12 hours, she became increasingly confused and was taken
December
to the emergency department, where she was found to be hypotensive, requiring intense fluid resuscitation and pressor support. You
note that her serum creatinine is twice the upper limit of normal, her platelet count is 75 x 103/µL (75 x 109/L), and that her blood
culture is growing Streptococcus pyogenes. 

Of the following, the MOST likely additional clinical manifestation to be found in this child is

Claim Credit A. arthritis

Evaluation B. erythema marginatum
C. soft tissue necrosis
My Learning Plan
D. subcutaneous nodules

Submit   Reset  

Page 98 of 115

Contact Us
Links may open in separate window
Copyright © 2016   American Academy of Pediatrics. All rights reserved. Pop­up Blocker may need to be disabled

http://2016.prepid.courses.aap.org/script/november?q­page=1&req=201612122203540631 1/1
12/13/2016 November

Welcome  mohammed alsaiary [ Logout ]

Home PREP Pearls FAQs My Bookmarks CME Information

Overview Home  > November

Editorial Board
November   Enter Keyword Search
January
Question View:   All (8) Jump to Question
February
Print this Page Add to my Bookmarks Page 98 of 115
March

April Assessment History Mode: Learner  Exam 

May ASSESSMENT PROGRESS:  Total Questions:  8  Questions Answered:  1  Correct Answers:  0


June

July You currently have 7 questions unanswered in this assessment

August

September
Question: 1
October
You are asked to evaluate a 9­year­old previously healthy girl, who is admitted to the pediatric intensive care unit for acute onset of
November hypotension, fever, and multiorgan system dysfunction. She had been well until 24 hours ago, when she developed fever, sore
throat, and a generalized erythematous rash on her trunk. Over the next 12 hours, she became increasingly confused and was taken
December
to the emergency department, where she was found to be hypotensive, requiring intense fluid resuscitation and pressor support. You
note that her serum creatinine is twice the upper limit of normal, her platelet count is 75 x 103/µL (75 x 109/L), and that her blood
culture is growing Streptococcus pyogenes. 

Of the following, the MOST likely additional clinical manifestation to be found in this child is

Claim Credit A. arthritis

Evaluation B. erythema marginatum
C. soft tissue necrosis
My Learning Plan
D. subcutaneous nodules

Incorrect View Peer Results
Correct Answer: C Average Percent Correct:  84.62%

The patient in the vignette has streptococcal toxic shock­like syndrome (STSS), an acute, life­
threatening infection that is characterized by rapid onset shock and accelerated multiorgan system
involvement. The patient in the vignette meets the clinical case criteria, which have been established
for the diagnosis of STSS (Table). One important and unique feature of STSS is a rapidly progressing
and often destructive soft­tissue infection, which is characterized by localized swelling and erythema
followed by ecchymoses, sloughing of the skin and progression to necrotizing fasciitis, myositis, or
gangrene. Arthritis, erythema marginatum, and subcutaneous nodules are manifestations of acute
rheumatic fever, an immune­mediated complication of group A streptococcal (GAS) infection, and are not typical findings in
STSS. The most common ports of entry of Streptococcus pyogenes in patients who develop STSS include the skin and
pharynx, although half of patients who develop STSS do not have an identified portal of entry.  Varicella infection in
unimmunized children and blunt trauma are predisposing factors for pediatric invasive group A streptococcal infections and
STSS. Rapid­onset hypotension is one of the hallmarks of STSS. Acute renal failure occurs in nearly all patients within 72
hours of presentation and is the result of a combination of factors including hypotension, myoglobinuria, and hemoglobinuria.
Fever and altered mental status are common initial manifestations of infection, and an influenza­like syndrome occurs in
about 20% of patients. Common laboratory features include an initial leukocytosis with a left shift, hypoalbuminemia,
hypocalcemia, thrombocytopenia, and evidence of disseminated intravascular coagulation. A positive blood culture for S
pyogenes occurs in approximately 60% of patients.

The GAS cell wall is composed of a peptidoglycan polymer backbone and lipoteichoic acid components, which provide
thickness, rigidity, and structural stability. These components of the cell wall may also play a role in pathogenesis by activation
of the alternative complement pathway (peptidoglycan) and facilitating adherence of the bacterium to pharyngeal epithelial
cells (lipoteichoic acid). Over 100 serotypes of group A streptococci have been identified based on a unique M protein
antigen, which is covalently linked to the cell wall. Certain M types have been more strongly associated with invasive disease.
The M1 serotype has been the leading cause of severe invasive GAS infection, including STSS and necrotizing fasciitis. The
group A carbohydrate, which is a rhamnose polymer with side chains of N­acetyl­glucosamine, is responsible for group
specificity of streptococci (group A). Some strains of S pyogenes possess a thick capsule, composed of hyaluronic acid,
which helps the organism resist phagocytosis. 

Group A streptococcus is responsible for extracellular products that are important in pathogenesis. Streptococcal pyrogenic
exotoxins (Spe) type A, B, and C act as superantigens and trigger T­cell proliferation and cytokine release, resulting in
endotoxin­induced shock, fever, and suppression of antibody synthesis. Spe A and Spe B have been associated with severe

http://2016.prepid.courses.aap.org/script/november?req=201612122204444381&status=submit# 1/2
12/13/2016 November
cases of STSS and scarlet fever. Mild cases of scarlet fever have been associated with spe C. Group A streptococcus also
produces cytolysins, such as streptolysin S, streptolysin O, hyaluronidase, streptokinase, and nicotinamide­adenine
dinucleotidase. These products have the capability of injuring cell membranes, lysing red blood cells, and damaging
subcellular organelles. Other extracellular products produced by GAS include bacteriocins, which promote infection and
persistence of colonization, peptidases, which cleave host chemokines and complement factors, and erythrogenic toxins,
which play a role in the scarlatiniform rash.

PREP Pearls
Streptococcal toxic shock­like syndrome is an acute, life­threatening infection caused by Streptococcus pyogenes and
characterized by rapid onset shock and multiorgan system involvement.  

Soft tissue necrosis is a common feature of streptococcal toxic shock­like syndrome.

S pyogenes possesses cell wall components and extracellular products that are important in pathogenesis. The presence
of streptococcal pyrogenic exotoxins has been associated with streptococcal toxic shock­like syndrome and necrotizing
fasciitis. 

American Board of Pediatrics Content Specification(s)
Recognize the major cell wall constituents of group A streptococcus and exotoxins associated with disease‑producing
strains

Recognize the clinical manifestations of streptococcal toxic shock­like syndrome

Suggested Readings
Lappin E, Ferguson AJ. Gram­positive toxic shock syndromes. Lancet Infect Dis. 2009;9(5):281­290. doi:
http://dx.doi.org/10.1016/S1473­3099(09)70066­0

Laupland KB, Davies IID, Low DE, et al. Invasive group A streptococcal disease in children and association with varicella­
zoster infection: Ontario group A streptococcal study group. Pediatrics. 2000;105(5):E60.
http://pediatrics.aappublications.org/content/105/5/e60.long

Nizet V, Arnold JC. Streptococcus pyogenes (group A streptococcus). In: Long SS, Pickering LK, Prober CG, et al, eds.
Principles and Practice of Pediatric Infectious Diseases. 4th ed. Philadelphia, PA: Saunders Elsevier; 2012:698­706..

Comment On This Question

Page 98 of 115

Contact Us
Links may open in separate window
Copyright © 2016   American Academy of Pediatrics. All rights reserved. Pop­up Blocker may need to be disabled

http://2016.prepid.courses.aap.org/script/november?req=201612122204444381&status=submit# 2/2
12/13/2016 November

Welcome  mohammed alsaiary [ Logout ]

Home PREP Pearls FAQs My Bookmarks CME Information

Overview Home  > November

Editorial Board
November   Enter Keyword Search
January
Question View:   All (8) Jump to Question
February
Print this Page Add to my Bookmarks Page 99 of 115
March

April Assessment History Mode: Learner  Exam 

May ASSESSMENT PROGRESS:  Total Questions:  8  Questions Answered:  1  Correct Answers:  0


June

July You currently have 7 questions unanswered in this assessment

August

September
Question: 2
October
You are consulted for antibiotic recommendations on a 6­year­old girl who was admitted overnight with fever, cough, and respiratory
November difficulty. Her parents report that she has not been feeling well for the last couple of weeks and has been complaining of tiredness.
She then developed low­grade fever (37.8­38.3°C) for the past 7 days. She was seen by her primary care physician and treated with
December
a 5­day course of azithromycin and showed some improvement for a short time. However, she then developed a cough along with
the continuing fever, both of which seem to have become progressively worse. Upon admission, the primary team started ceftriaxone
and vancomycin. She has remained febrile overnight with a maximum temperature of 40.2°C. Initial blood cultures are negative. A
chest radiograph was obtained on admission (Figure ).

Of the following, the BEST initial empiric antibiotic therapy for this child would be
Claim Credit
A. ampicillin/sulbactam
Evaluation
B. ceftazidime and gentamicin
My Learning Plan C. meropenem
D. vancomycin and gentamicin

Submit   Reset  

Page 99 of 115

Contact Us
Links may open in separate window
Copyright © 2016   American Academy of Pediatrics. All rights reserved. Pop­up Blocker may need to be disabled

http://2016.prepid.courses.aap.org/script/november?q­page=2&req=201612122204455162 1/1
12/13/2016 November

Welcome  mohammed alsaiary [ Logout ]

Home PREP Pearls FAQs My Bookmarks CME Information

Overview Home  > November

Editorial Board
November   Enter Keyword Search
January
Question View:   All (8) Jump to Question
February
Print this Page Add to my Bookmarks Page 99 of 115
March

April Assessment History Mode: Learner  Exam 

May ASSESSMENT PROGRESS:  Total Questions:  8  Questions Answered:  2  Correct Answers:  1


June

July You currently have 6 questions unanswered in this assessment

August

September
Question: 2
October
You are consulted for antibiotic recommendations on a 6­year­old girl who was admitted overnight with fever, cough, and respiratory
November difficulty. Her parents report that she has not been feeling well for the last couple of weeks and has been complaining of tiredness.
She then developed low­grade fever (37.8­38.3°C) for the past 7 days. She was seen by her primary care physician and treated with
December
a 5­day course of azithromycin and showed some improvement for a short time. However, she then developed a cough along with
the continuing fever, both of which seem to have become progressively worse. Upon admission, the primary team started ceftriaxone
and vancomycin. She has remained febrile overnight with a maximum temperature of 40.2°C. Initial blood cultures are negative. A
chest radiograph was obtained on admission (Figure ).

Of the following, the BEST initial empiric antibiotic therapy for this child would be
Claim Credit
A. ampicillin/sulbactam
Evaluation
B. ceftazidime and gentamicin
My Learning Plan C. meropenem
D. vancomycin and gentamicin

Correct View Peer Results

Average Percent Correct:  61.54%

The child in the vignette has a lung abscess. In this situation, the best initial choice of antibiotics should
consist of a regimen that provides coverage for Staphylococcus aureus, Streptococcus pneumoniae,
as well as anaerobic organisms found in the upper respiratory tract. Therefore, ampicillin/sulbactam is
the best response.

Alternate antibiotic combinations that may be used include other β­lactam combinations such as
ticarcillin­clavulanate or piperacillin/tazobactam, or third­generation cephalosporins with clindamycin.
Depending on local epidemiology and concern for Streptococcus pneumoniae isolates with reduced susceptibility to penicillin,
methicillin­resistant Staphylococcus aureus or in critically ill patients, use of vancomycin or linezolid should also be
considered. Multiple other antibiotic combinations depending on individual patient risk factors may also be used.

Once culture results are available, antibiotics can be tailored to the specific organisms identified, although obtaining cultures
may often be a challenge and depends on the size and location of the abscess. Direct aspiration under ultrasonography (US)
or computed tomography (CT) guidance is difficult where the abscess is not large or peripheral. Bronchoscopy may be helpful
particularly if a foreign body is also suspected.

In cases of parenchymal lung abscess, conservative management with antibiotics alone is sufficient. Intravenous antibiotic
therapy is generally continued until the patient is stable and afebrile at which point oral antibiotics may be instituted. Total
duration of therapy is generally 2 to 3 weeks, but may be prolonged in complicated cases. When the lesion is peripheral with
no communication with the airway, CT­ or US­guided drainage or catheter placement is feasible. Drainage may also be
necessary when the lesion is greater than 4 cm in diameter, causes mediastinal shift, or compromises respiratory status,
requiring intubation and mechanical ventilation. Operative therapy such as surgical wedge resection is only considered in
specific situations such as failure of antibiotic therapy, associated pyopneumothorax, tension pneumatocele, large size of the
abscess, proximity to the mediastinum, or complications resulting due to rupture of the abscess into adjacent tissue. Rarely,
lobar resection, such as in cases of gangrenous lung, may be required.

In most cases, the diagnosis of lung abscess is made on plain radiographs taken both in the erect, as well as the lateral
decubitus position. An abscess cavity with an air fluid level (in cases where the abscess ruptures into the bronchus allowing
air to enter) and atelectasis of the surrounding lung tissue is generally seen. Pleural thickening may be observed in the case
of a sub­pleural abscess.

http://2016.prepid.courses.aap.org/script/november?req=201612122205208288&status=submit# 1/2
12/13/2016 November
The optimal imaging modality used to define a lung abscess is CT of the chest, which enables accurate estimation of the size
and number of abscesses, involvement of  surrounding lung tissue, as well as distinction of an abscess from empyema and
necrotizing pneumonia, which lacks enhancement on contrast media and a distinct air fluid level.

Ultrasonography, while it has the advantage that it is portable, relatively inexpensive, and involves no radiation exposure, is
not as precise as CT in differentiating a lung abscess from an empyema. Magnetic resonance imaging and nuclear imaging
add little to the information obtained by CT. Concern for radiation exposure with CT is mitigated by the fact that modern
scanners that use automatic dose modulation can limit the amount of radiation dose a person is exposed to.

While multiple risk factors have been associated with the development of a lung abscess, the 2 main mechanisms that explain
its formation are aspiration and hematogenous spread. Children with altered mental status, swallowing dysfunction, or
neurodegenerative or myopathic disorders are at risk for developing lung abscess. Immunocompromised children on
chemotherapy represent another high­risk group. In children with congenital heart disease, tricuspid or pulmonary valve
endocarditis represents risk factors for the development of a lung abscess. Children with hypogammaglobulinemia,
hyperimmunoglobulin E, chronic granulomatous disease are also at risk although bronchiectasis is more common in those
with hypogammaglobulinemia. Other high risk groups include patients with immotile cilia syndrome and cystic fibrosis
although large abscesses are uncommon in the latter. Other situations in which a lung abscess may develop include chronic
aspiration in association with a foreign body, hospital­associated infection (particularly in an individual on prolonged
mechanical ventilation), septic emboli, or in individuals with anatomical abnormalities such as congenital cystic adenomatoid
malformation or pulmonary sequestration.

Further work­up to identify any underlying predisposition such as genetic analysis to identify the STAT3 genetic mutation,
along with measurement of immunoglobulin levels for suspected hyper immunoglobulin E syndrome and dihydrorhodamine
reduction test in cases of suspected chronic granulomatous disease, may also be indicated.

Necrotizing pneumonia usually occurs as a result of a localized lung infection by pyogenic bacteria. Viral infection may be
another common predisposing factor. In an otherwise healthy child, it can generally be treated with appropriate antibiotics and
usually resolves without sequelae. In some cases, it may progress to develop into a localized lung abscess, pneumatocele, or
bronchopleural fistula.

Staphylococcus aureus is one of the major causative organisms in necrotizing pneumonia although multiple other organisms
such as Streptococcus pneumoniae, group A streptococcus, Clostridium perfringens, Mycoplasma pneumoniae, and
adenovirus have been described. In an otherwise normal healthy child, initiation of a third­generation cephalosporin with
nafcillin with or without vancomycin (depending on local methicillin­resistant S aureus epidemiology) is appropriate. Where
anaerobes are a consideration, clindamycin, in addition to a third­generation cephalosporin, can be used. Alternatively,
combination therapy with a β­lactamase inhibitor and piperacillin or ticarcillin would also be appropriate. Once the causative
organism has been identified, therapy can be modified according to culture results. Every effort should be made to obtain
appropriate cultures either using fiberoptic bronchoscopy, or in the case of intubated patients, by bronchoalveolar lavage or
sputum cultures in patients where adequate samples can be obtained, so that antibiotic treatment can be tailored to the
appropriate pathogen.

PREP Pearls
In cases of parenchymal lung abscess, conservative management with antibiotics alone is sufficient and surgical drainage
is limited to select cases.

Initial diagnosis of lung abscess is usually made by plain radiographs, although computed tomography is considered the
optimal diagnostic modality to obtain more specific information.

Necrotizing pneumonia resulting from localized lung infection by pyogenic bacteria is also treated with antimicrobial therapy
alone in the majority of cases and often resolves without sequelae.

Multiple antibiotic choices are available for the treatment of both lung abscess and necrotizing pneumonia. Individual
patient risk factors and local epidemiology should be taken into consideration for initial empiric therapy.

American Board of Pediatrics Content Specification(s)
Plan the evaluation of a patient with pulmonary abscess for predisposing factors and underlying abnormalities

Plan the management of a patient with pulmonary abscess or necrotizing pneumonia (drugs, procedures)

Suggested Readings
Mani CS, Murray DL. Acute pneumonia and its complications. In: Long SS, Pickering LK, Prober CG, eds. Principles and
Practice of Pediatric Infectious Diseases. 4th ed. Philadelphia, PA: Saunders Elsevier; 2012:235­245.

Wheeler JG, Jacobs RF. Complications of pneumonia. In: Cherry JD, Harrison GJ, Kaplan SL, Steinbach WJ, Hotez PJ, eds.
Feigen and Cherry’s Textbook of Pediatric Infectious Diseases. 7th ed. Philadelphia, PA: Saunders Elsevier, 2014:306­322.

Comment On This Question

Page 99 of 115

Contact Us
Links may open in separate window
Copyright © 2016   American Academy of Pediatrics. All rights reserved. Pop­up Blocker may need to be disabled

http://2016.prepid.courses.aap.org/script/november?req=201612122205208288&status=submit# 2/2
12/13/2016 November

Welcome  mohammed alsaiary [ Logout ]

Home PREP Pearls FAQs My Bookmarks CME Information

Overview Home  > November

Editorial Board
November   Enter Keyword Search
January
Question View:   All (8) Jump to Question
February
Print this Page Add to my Bookmarks Page 100 of 115
March

April Assessment History Mode: Learner  Exam 

May ASSESSMENT PROGRESS:  Total Questions:  8  Questions Answered:  2  Correct Answers:  1


June

July You currently have 6 questions unanswered in this assessment

August

September
Question: 3
October
A 10­year­old boy with moderate asthma presents to your office in January for evaluation of a 1­day history of temperature to 39.7°C,
November sore throat, cough with symptoms of substernal burning pain, and malaise. He has not required therapy with steroids for the past 3
months.
December
He received a dose of seasonal inactivated influenza vaccine 8 days before this visit.  He had not had an influenza vaccine for the 2
years prior because, according to his history, he was “always sick” when the vaccination was due.

Influenza activity has been reported in the community by the department of health but not at excessive numbers compared with
previous influenza seasons.

Claim Credit On physical examination, he is uncomfortable but has a nontoxic appearance. His vital signs are as follows: temperature: 39.3°C
respiratory rate, 26 breaths/min; heart rate, 103 beats/min; blood pressure, 102/60 mm Hg; and oxygen saturation, 95%.
Evaluation

My Learning Plan His head, ears, eyes, nose, and throat examination findings are notable for nasal congestion and mild erythema of the pharynx. His
neck is supple, without significant lymphadenopathy. His lungs have a few scattered rales and wheezes, without focality; no
retractions are noted. S1 and S2 heart sounds are normal. Pulse rate is 2+/2+ with a capillary refill of less than 2 seconds. His
abdomen is benign, without organomegaly. His extremities are without clubbing, cyanosis, or edema. Neurologic examination
findings are grossly normal. The results of a rapid influenza test in your office are positive.

The parents ask why he developed influenza despite receiving vaccine.

Of the following, the BEST explanation for why this child developed influenza despite vaccination is because of

A. decreased effectiveness of influenza vaccine in children
B. inadequate time from receipt of vaccine to virus exposure
C. mismatch of the circulating strain of influenza to the vaccine strain
D. receipt of only 1 dose of vaccine this season

Submit   Reset  

Page 100 of 115

Contact Us
Links may open in separate window
Copyright © 2016   American Academy of Pediatrics. All rights reserved. Pop­up Blocker may need to be disabled

http://2016.prepid.courses.aap.org/script/november?q­page=3&req=201612122205214382 1/1
12/13/2016 November

Welcome  mohammed alsaiary [ Logout ]

Home PREP Pearls FAQs My Bookmarks CME Information

Overview Home  > November

Editorial Board
November   Enter Keyword Search
January
Question View:   All (8) Jump to Question
February
Print this Page Add to my Bookmarks Page 100 of 115
March

April Assessment History Mode: Learner  Exam 

May ASSESSMENT PROGRESS:  Total Questions:  8  Questions Answered:  3  Correct Answers:  1


June

July You currently have 5 questions unanswered in this assessment

August

September
Question: 3
October
A 10­year­old boy with moderate asthma presents to your office in January for evaluation of a 1­day history of temperature to 39.7°C,
November sore throat, cough with symptoms of substernal burning pain, and malaise. He has not required therapy with steroids for the past 3
months.
December
He received a dose of seasonal inactivated influenza vaccine 8 days before this visit.  He had not had an influenza vaccine for the 2
years prior because, according to his history, he was “always sick” when the vaccination was due.

Influenza activity has been reported in the community by the department of health but not at excessive numbers compared with
previous influenza seasons.

Claim Credit On physical examination, he is uncomfortable but has a nontoxic appearance. His vital signs are as follows: temperature: 39.3°C
respiratory rate, 26 breaths/min; heart rate, 103 beats/min; blood pressure, 102/60 mm Hg; and oxygen saturation, 95%.
Evaluation

My Learning Plan His head, ears, eyes, nose, and throat examination findings are notable for nasal congestion and mild erythema of the pharynx. His
neck is supple, without significant lymphadenopathy. His lungs have a few scattered rales and wheezes, without focality; no
retractions are noted. S1 and S2 heart sounds are normal. Pulse rate is 2+/2+ with a capillary refill of less than 2 seconds. His
abdomen is benign, without organomegaly. His extremities are without clubbing, cyanosis, or edema. Neurologic examination
findings are grossly normal. The results of a rapid influenza test in your office are positive.

The parents ask why he developed influenza despite receiving vaccine.

Of the following, the BEST explanation for why this child developed influenza despite vaccination is because of

A. decreased effectiveness of influenza vaccine in children
B. inadequate time from receipt of vaccine to virus exposure
C. mismatch of the circulating strain of influenza to the vaccine strain
D. receipt of only 1 dose of vaccine this season

Incorrect View Peer Results
Correct Answer: B Average Percent Correct:  79.49%

The effectiveness of influenza vaccination depends on host factors (eg, age, immunocompetence), the
degree of match between circulating viruses, and the outcome of effectiveness measured. In addition,
it requires 1 to 2 weeks from influenza  vaccination to develop protective immunity. Therefore, the best
explanation for why the child described in the vignette developed influenza despite vaccination is
because of inadequate time from receipt of vaccine to virus exposure.

Although decreased effectiveness of influenza vaccination is seen in 6­month to 2­year­old children, by
10 years of age, effectiveness ranging from 50% to 95% has been reported, depending on the closeness of the match
between circulating and vaccine strains of the virus. Although not definitive, the absence of increased cases of influenza as
reported by the local department of health argues against a mismatch between vaccine and circulating strains.  Although he
has not received influenza vaccine regularly, children 9 years or older require only 1 dose of vaccine per year.

The US Centers for Disease Control and Prevention (CDC) conducts annual studies to assess influenza vaccine
effectiveness comparing laboratory­confirmed influenza infection rates in vaccinated vs unvaccinated populations. In years in
which the strains in the vaccine match closely to circulating strains, efficacy rates of 50% to 60% are typically seen. When a
drifted or shifted strain appears, effectiveness rates as low as 20% may be observed. Even in the absence of effectiveness
against influenza infection, vaccinated individuals may develop milder illness (eg, lower rates of hospitalization, intensive care
admission) compared with unvaccinated people.

http://2016.prepid.courses.aap.org/script/november?req=201612122206189852&status=submit 1/2
12/13/2016 November
During the last several years, quadrivalent influenza vaccines with 2 B and 2 A strains have been introduced, providing the
potential for improved vaccine effectiveness when strains from the 2 different B lineages circulate. Currently, however, the
CDC does not recommend the quadrivalent over the trivalent vaccine, and both are available. Current data have indicated
decreased effectiveness of the quadrivalent live attenuated vaccine against the A (H1N1) strain during the past 2 seasons;
thus, the live attenuated influenza vaccine is not recommended for use in the 2016­2017 influenza season.

PREP Pearls
Influenza vaccine effectiveness depends on host (eg, age, immune status) and virus (eg, match between circulating and
vaccine strains) factors. When circulating and vaccine strains are well matched, vaccine effectiveness is typically 50% to
60% for prevention of laboratory­confirmed influenza infection in vaccinated versus unvaccinated individuals.

It takes up to 2 weeks after influenza immunization to develop protective immunity.

American Board of Pediatrics Content Specification(s)
Know the effectiveness of influenza vaccine

Suggested Readings
Chung JR, Flannery B, Thompson MG, et al. Seasonal effectiveness of live attenuated and inactivated influenza vaccine.
Pediatrics. 2016;137(2):E201552603. doi: http://dx.doi.org/10.1542/PEDS.2015­2603

Committee on Infectious Diseases, American Academy of Pediatrics. Influenza. In: Red Book: 2015 Report of the Committee
on Infectious Diseases. Elk Grove Village, IL: American Academy of Pediatrics; 2015.

US Centers for Disease Control and Prevention. Seasonal Influenza Vaccine Effectiveness, 2005­2016.
http://www.cdc.gov/flu/professionals/vaccination/effectiveness­studies.htm. Accessed July 30, 2016

Comment On This Question

Page 100 of 115

Contact Us
Links may open in separate window
Copyright © 2016   American Academy of Pediatrics. All rights reserved. Pop­up Blocker may need to be disabled

http://2016.prepid.courses.aap.org/script/november?req=201612122206189852&status=submit 2/2
12/13/2016 November

Welcome  mohammed alsaiary [ Logout ]

Home PREP Pearls FAQs My Bookmarks CME Information

Overview Home  > November

Editorial Board
November   Enter Keyword Search
January
Question View:   All (8) Jump to Question
February
Print this Page Add to my Bookmarks Page 101 of 115
March

April Assessment History Mode: Learner  Exam 

May ASSESSMENT PROGRESS:  Total Questions:  8  Questions Answered:  3  Correct Answers:  1


June

July You currently have 5 questions unanswered in this assessment

August

September
Question: 4
October
Your hospital recently made a change in the guidelines for empiric choice of antibiotics for community­acquired pneumonia,
November substituting ampicillin for ceftriaxone as the drug of choice if a bacterial cause is suspected. A medical student who just returned
from a global health rotation points out that her textbook still lists β­lactamase producing Haemophilus influenzae strains, primarily
December
capsular type b, as a cause of bacterial pneumonia in many countries. She asks you for the evidence that supports the lack of need
to cover H influenzae type b.

Of the following, the BEST reason that empiric use of a cephalosporin to treat pneumonia in the United State is unnecessary is
because H influenzae type b nowadays

A. is rarely β­lactamase resistant 
Claim Credit
B. only causes disease in American Indian/Alaska Native children
Evaluation
C. only causes disease in immune­compromised children
My Learning Plan
D. rarely colonizes children

Submit   Reset  

Page 101 of 115

Contact Us
Links may open in separate window
Copyright © 2016   American Academy of Pediatrics. All rights reserved. Pop­up Blocker may need to be disabled

http://2016.prepid.courses.aap.org/script/november?q­page=4&req=201612122206193914 1/1
12/13/2016 November

Welcome  mohammed alsaiary [ Logout ]

Home PREP Pearls FAQs My Bookmarks CME Information

Overview Home  > November

Editorial Board
November   Enter Keyword Search
January
Question View:   All (8) Jump to Question
February
Print this Page Add to my Bookmarks Page 101 of 115
March

April Assessment History Mode: Learner  Exam 

May ASSESSMENT PROGRESS:  Total Questions:  8  Questions Answered:  4  Correct Answers:  1


June

July You currently have 4 questions unanswered in this assessment

August

September
Question: 4
October
Your hospital recently made a change in the guidelines for empiric choice of antibiotics for community­acquired pneumonia,
November substituting ampicillin for ceftriaxone as the drug of choice if a bacterial cause is suspected. A medical student who just returned
from a global health rotation points out that her textbook still lists β­lactamase producing Haemophilus influenzae strains, primarily
December
capsular type b, as a cause of bacterial pneumonia in many countries. She asks you for the evidence that supports the lack of need
to cover H influenzae type b.

Of the following, the BEST reason that empiric use of a cephalosporin to treat pneumonia in the United State is unnecessary is
because H influenzae type b nowadays

A. is rarely β­lactamase resistant 
Claim Credit
B. only causes disease in American Indian/Alaska Native children
Evaluation
C. only causes disease in immune­compromised children
My Learning Plan
D. rarely colonizes children

Incorrect View Peer Results
Correct Answer: D Average Percent Correct:  69.23%

Haemophilus influenzae is a Gram­negative coccobacillus that commonly colonizes the upper
respiratory tract. It exists as encapsulated (typeable) or unencapsulated (nontypeable) strains. The
encapsulated strains express 1 of 6 antigenically distinct capsular polysaccharides (types a­f). The vast
majority of invasive H influenzae disease worldwide is caused by H influenzae type b (Hib). Prior to
introduction of polysaccharide­protein conjugate Hib vaccines, colonization with Hib was common
among children younger than 6 years of age. The point prevalence was between 1% and 5% and up to
10% in daycare centers, where studies documented that up to three­quarters of children became
colonized at some time. Carriage rates were as high as 90% in households or daycare centers where a case of invasive
disease had occurred. Because of this high prevalence, invasive Hib diseases were common, with a peak incidence between
6 and 18 months of age, and Hib caused the majority of bacterial meningitis in infants and young children in the United
States. Epiglottitis peaked between 2 and 4 years of age. Less than 1 case of invasive type b disease now occurs per
100,000 children younger than 5 years of age, a decrease of 99% from the pre­Hib conjugate vaccine era (Figure).

The correct rationale for the current antibiotic guideline in the vignette is that colonization (carriage) of Hib in children is now
rare, though carriage is still high in resource­limited countries where Hib vaccination is not widespread. β­lactamase
resistance rates in Hib are still high. Invasive Hib disease is reported primarily in unimmunized and underimmunized children.
American Indian/Alaska Natives and immune­compromised children are at increased risk for invasive Hib disease, but not
exclusively so. Rates of Hib disease among American Indian/Alaska Native (AI/AN) children remain 8 to 10 times higher than
among non­AI/AN children. H influenzae type a has emerged as a significant cause of invasive disease in some North
American indigenous populations. Immune­compromised children at increased risk are those with functional or anatomic
asplenia, HIV infection, immunoglobulin deficiency including immunoglobulin G2 subclass deficiency, early component
complement deficiency, receipt of a hematopoietic stem cell transplant, or receipt of chemotherapy or radiation therapy. A
child who develops Hib disease despite appropriate immunization should be evaluated for the possible presence of one of
these conditions.

The dramatic reduction of nasopharyngeal carriage of Hib is a consequence of widespread immunization with polysaccharide­
protein conjugate Hib vaccines. It is a herd effect, in that even infants who are too young to be immunized have decreased
carriage and invasive disease. Immunized children are less likely to acquire H influenzae type b colonization, owing to
secretion of type­specific anticapsular antibody into the nasopharyngeal mucosa. The lower prevalence of carriage interrupts
transmission. There is also an indirect effect of decrease in invasive disease in adults.

http://2016.prepid.courses.aap.org/script/november?req=201612122207278916&status=submit# 1/2
12/13/2016 November
The majority of strains of H influenzae that colonize the upper respiratory tract are nontypable H influenzae (NTHi), and up to
80% of humans are colonized at some point in their life. Nontypable H influenzae cause mucosal infections such as otitis
media, conjunctivitis, and sinusitis, but can also cause invasive disease and currently cause the majority of invasive
Haemophilus disease in children (Table). Most cases occur in newborn infants and immune­compromised hosts. There is a
high incidence of invasive NTHi disease during pregnancy and in the perinatal period, which includes neonatal early­onset
sepsis, invasive infection in the mother, and fetal death. Neonates acquire the organism by contact with amniotic fluid or
genital tract secretions. Two percent to 8% of early­onset sepsis is caused by H influenzae, mostly NTHi, with most cases
occurring in the first 24 hours of life. Disease manifestations are septicemia, pneumonia and meningitis, and the morbidity and
mortality are high. Prematurity, low birthweight, premature rupture of membranes, and chorioamnionitis are risk factors. Late­
onset NTHi sepsis is occasionally observed.

The current estimate of invasive disease due to all Haemophilus influenzae in the United States is 5,200 cases (1.63/100,000)
with 675 deaths (0.21/100,000). The Table summarizes data on invasive H influenzae disease from the CDC Emerging
Infections Program Network Active Bacterial Core Surveillance (ABCs) Report for 2014, which captures isolation of H
influenzae from a normally sterile site in a resident of one of the defined surveillance areas. About one­quarter of cases were
bacteremia without focus and about 7% were meningitis.

PREP Pearls
There is a decrease of 99% in Haemophilus influenzae type b (Hib) cases, largely due to the effect of active immunization
with conjugate vaccines on carriage of the related microorganisms.

Certain groups are still at increased risk for invasive Hib disease: under­immunized, American Indian/Alaska Native, and
immune­compromised.

Nontypable H influenzae currently cause the majority of invasive H disease in children, with most cases occurring in
neonates.

American Board of Pediatrics Content Specification(s)
Understand the epidemiology of Haemophilus influenzae colonization and disease to include type b disease in immunized
infants and children

Understand that nontypeable Haemophilus influenzae is a significant pathogen in newborn infants and immunodeficient
hosts and can cause invasive infection

Know the effect of active immunization with available childhood vaccines on carriage of the related microorganisms

Suggested Readings
Barenkamp SJ. Haemophilus influenzae. In: Cherry J, Demmler­Harrison GJ, Kaplan SL, Steinbach WJ, Hotez P, eds. Feigin
and Cherry’s Textbook Of Pediatric Infectious Diseases. 7th ed. Philadelphia, PA: Saunders Elsevier; 2013:1665­1684..

Briere EC, Rubin L, Moro PL, Cohn A, Clark T, Messonnier N. Prevention and control of Haemophilus influenzae type b
disease: recommendations of the advisory committee on immunization practices (ACIP). MMWR Recomm Rep.
2014;63(RR01):1­14. http://www.cdc.gov/mmwr/preview/mmwrhtml/rr6301a1.htm

Collins S, Litt DJ, Flynn S, Ramsay ME, Slack MP, Ladhani SN. Neonatal invasive Haemophilus influenzae disease in
England and Wales: epidemiology, clinical characteristics, and outcome. Clin Infect Dis. 2015;60(12):1786­1792. doi:
http://dx.doi.org/10.1093/cid/civ194

Schuchat A, Rosenstein Messonnier N. From pandemic suspect to the postvaccine era: the Haemophilus influenzae story.
Clin Infect Dis. 2007;44(6):817­819. doi: http://dx.doi.org/10.1086/511886

US Centers for Disease Control and Prevention. Active Bacterial Core Surveillance (ABCs) report emerging infections
program network Haemophilus influenzae, provisional­2014. US Centers for Disease Control and Prevention website.
http://www.cdc.gov/abcs/reports­findings/survreports/hib14.pdf

US Centers for Disease Control and Prevention. Prevention of pneumococcal disease among infants and children—use of 13­
valent pneumococcal conjugate vaccine and 23­valent pneumococcal polysaccharide vaccine. MMWR Recomm Rep.
2010;59(RR11):1­18. http://www.cdc.gov/mmwr/preview/mmwrhtml/rr5911a1.htm

Comment On This Question

Page 101 of 115

Contact Us
Links may open in separate window
Copyright © 2016   American Academy of Pediatrics. All rights reserved. Pop­up Blocker may need to be disabled

http://2016.prepid.courses.aap.org/script/november?req=201612122207278916&status=submit# 2/2
12/13/2016 November

Welcome  mohammed alsaiary [ Logout ]

Home PREP Pearls FAQs My Bookmarks CME Information

Overview Home  > November

Editorial Board
November   Enter Keyword Search
January
Question View:   All (8) Jump to Question
February
Print this Page Add to my Bookmarks Page 102 of 115
March

April Assessment History Mode: Learner  Exam 

May ASSESSMENT PROGRESS:  Total Questions:  8  Questions Answered:  4  Correct Answers:  1


June

July You currently have 4 questions unanswered in this assessment

August

September
Question: 5
October
A 9­year­old boy with acute lymphoblastic leukemia on maintenance chemotherapy presents to the emergency department (ED) with
November fever and a papulovesicular rash. Polymerase chain reaction testing of a specimen obtained from the base of a vesicle confirms a
diagnosis of varicella. The child is placed in airborne and contact isolation in a negative pressure room.
December
The nurse who cared for the patient in ED for the hour prior to the diagnosis reports to employee health after the exposure. She
admits to being in the room for almost an hour and did not wear any personal protective equipment. You are consulted to advise
regarding management of the nurse after this episode. Review of her employee health records reveals that she has been working at
your institution for 5 years and that, at the time of her hire, she provided documentation of prior receipt of 2 doses of varicella
vaccine.
Claim Credit
Of the following, the BEST recommendation for the nurse after this exposure is to
Evaluation
A. allow her to work unless she develops clinical signs of fever or rash during the 8 to 21 days after the exposure
My Learning Plan
B. furlough her from patient care activities during the incubation period of 8 to 21 days after exposure
C. obtain a serology to assess her immunity status to determine her clearance to work during the incubation period
D. revaccinate her immediately to boost her immunity and allow her to work without restriction during the incubation period

Submit   Reset  

Page 102 of 115

Contact Us
Links may open in separate window
Copyright © 2016   American Academy of Pediatrics. All rights reserved. Pop­up Blocker may need to be disabled

http://2016.prepid.courses.aap.org/script/november?q­page=5&req=201612122207282822# 1/1
12/13/2016 November

Welcome  mohammed alsaiary [ Logout ]

Home PREP Pearls FAQs My Bookmarks CME Information

Overview Home  > November

Editorial Board
November   Enter Keyword Search
January
Question View:   All (8) Jump to Question
February
Print this Page Add to my Bookmarks Page 102 of 115
March

April Assessment History Mode: Learner  Exam 

May ASSESSMENT PROGRESS:  Total Questions:  8  Questions Answered:  5  Correct Answers:  2


June

July You currently have 3 questions unanswered in this assessment

August

September
Question: 5
October
A 9­year­old boy with acute lymphoblastic leukemia on maintenance chemotherapy presents to the emergency department (ED) with
November fever and a papulovesicular rash. Polymerase chain reaction testing of a specimen obtained from the base of a vesicle confirms a
diagnosis of varicella. The child is placed in airborne and contact isolation in a negative pressure room.
December
The nurse who cared for the patient in ED for the hour prior to the diagnosis reports to employee health after the exposure. She
admits to being in the room for almost an hour and did not wear any personal protective equipment. You are consulted to advise
regarding management of the nurse after this episode. Review of her employee health records reveals that she has been working at
your institution for 5 years and that, at the time of her hire, she provided documentation of prior receipt of 2 doses of varicella
vaccine.
Claim Credit
Of the following, the BEST recommendation for the nurse after this exposure is to
Evaluation
A. allow her to work unless she develops clinical signs of fever or rash during the 8 to 21 days after the exposure
My Learning Plan
B. furlough her from patient care activities during the incubation period of 8 to 21 days after exposure
C. obtain a serology to assess her immunity status to determine her clearance to work during the incubation period
D. revaccinate her immediately to boost her immunity and allow her to work without restriction during the incubation period

Correct View Peer Results

Average Percent Correct:  48.72%

Given that the healthcare worker in this vignette was previously vaccinated with 2 doses of vaccine,
she would be considered immune to the virus. The Advisory Committee on Immunization Practices
recommendation in such a setting would be to allow the individual to work while monitoring for signs or
symptoms of varicella (fever, vesicular skin lesions) during the 8­ to 21­day incubation after exposure.

Nosocomial transmission of varicella to and from healthcare providers is a major concern, especially
for those caring for oncology patients or other immunosuppressed hosts. The US Centers for Disease
Control and Prevention (CDC) recommends proof of immunity to varicella for all healthcare providers upon employment. Proof
of immunity includes any of the following:

1. A diagnosis or verified history of varicella or herpes­zoster virus by a healthcare provider.
2. Written documentation of receipt of 2 doses of vaccine
3. Laboratory evidence of immunity or acute varicella infection

For a susceptible healthcare provider exposed to acute varicella infection, furlough from work during the potential incubation
period of 8 to 21 days after exposure is indicated.  Susceptible healthcare providers include those without a history of
varicella, without vaccination, or with only 1 dose of vaccine. Additionally, if the second dose was received more than 5 days
after the exposure to the virus, the individual would still be considered susceptible.

Serology is not recommended to assess immunity after varicella vaccination. The serology is not necessarily sensitive enough
to detect response to vaccine and a negative serology after varicella vaccination does not imply susceptibility to infection or
vaccine failure.  There are no data to suggest benefit from an additional dose of vaccine in a fully immunized person.

The CDC recommends that all hospitals and other healthcare settings maintain an immunization record for all providers. All
new employees should have their status assessed prior to employment and undergo an annual review to ensure
immunizations are up to date. All providers should be immune to measles, mumps, rubella, and varicella.  Those with
potential exposure to blood or body fluids should be immune to hepatitis B. Unfortunately, up to 20% of adults may not have
detectable hepatitis B surface antibody after 3 doses of vaccine. Such seronegative individuals should receive up to 3
additional doses of hepatitis B vaccine, but if still seronegative after an additional vaccine series, no further doses are
indicated.

http://2016.prepid.courses.aap.org/script/november?req=201612122207574386&status=submit# 1/2
12/13/2016 November
Annual influenza vaccine should also be administered and all healthcare providers should be given a single dose of Tdap
vaccine if they did not previously receive it. Additional routine vaccine recommendations for adults (eg, pneumococcal
vaccine, zoster vaccine, Td boosters) should be advised as well.

PREP Pearls
Varicella immunity is defined as healthcare provider documented clinical disease (varicella or herpes zoster), documented
receipt of 2 doses of varicella vaccine, or laboratory evidence of immunity or confirmed infection.

Healthcare providers should have their immunization status verified on employment and updated annually to ensure
optimal protection for the provider and patients.

American Board of Pediatrics Content Specification(s)
Develop a policy for immunizing health‑care professionals in a hospital as well as a plan for how to implement the policy
(eg, to assure all persons who have contact with patients are immune to rubella and/or varicella)

Make recommendations for control of an epidemic/outbreak of hospital­acquired infection

Suggested Readings
American Academy of Pediatrics. Hepatitis B. In: Kimberlin DW, Brady MT, Jackson MA, Long SS, eds. Red Book: 2015
Report of the Committee on Infectious Diseases. 30th ed. Elk Grove Village, IL: American Academy of Pediatrics; 2015: 400­
423.

American Academy of Pediatrics. Varicella­zoster virus infections. In: Kimberlin DW, Brady MT, Jackson MA, Long SS, eds.
Red Book: 2015 Report of the Committee on Infectious Diseases. 30th ed. Elk Grove Village, IL: American Academy of
Pediatrics; 2015: 846­860.

Shefer A, Atkinson W, Friedman C, et al. Immunization of health­care personnel: recommendations of the Advisory
Committee on Immunization Practices (ACIP). MMWR Recomm Rep. 2011;60(RR07):1­45.
http://www.cdc.gov/mmwr/preview/mmwrhtml/rr6007a1.htm

Comment On This Question

Page 102 of 115

Contact Us
Links may open in separate window
Copyright © 2016   American Academy of Pediatrics. All rights reserved. Pop­up Blocker may need to be disabled

http://2016.prepid.courses.aap.org/script/november?req=201612122207574386&status=submit# 2/2
12/13/2016 November

Welcome  mohammed alsaiary [ Logout ]

Home PREP Pearls FAQs My Bookmarks CME Information

Overview Home  > November

Editorial Board
November   Enter Keyword Search
January
Question View:   All (8) Jump to Question
February
Print this Page Add to my Bookmarks Page 103 of 115
March

April Assessment History Mode: Learner  Exam 

May ASSESSMENT PROGRESS:  Total Questions:  8  Questions Answered:  5  Correct Answers:  2


June

July You currently have 3 questions unanswered in this assessment

August

September
Question: 6
October
A previously healthy 11­year­old girl is afebrile and asymptomatic on day 5 of ceftriaxone therapy for meningococcal meningitis; that
November evening, she develops a temperature of 38.6°C. On physical examination, she is alert with a normal neurologic examination. The
only abnormality on examination is erythema, mild edema, and mild tenderness of the interphalangeal joint of her left third toe. The
December
on­call resident obtains a contrast­enhanced computed tomographic scan of the head, which is normal, except for a small,
nonenhancing right subdural effusion.

Of the following, the MOST appropriate next step in management is to

A. add intravenous vancomycin

Claim Credit B. administer oral ibuprofen

Evaluation C. aspirate and culture interphalangeal joint fluid
D. obtain neurosurgical consultation
My Learning Plan

Submit   Reset  

Page 103 of 115

Contact Us
Links may open in separate window
Copyright © 2016   American Academy of Pediatrics. All rights reserved. Pop­up Blocker may need to be disabled

http://2016.prepid.courses.aap.org/script/november?q­page=6&req=201612122207580479 1/1
12/13/2016 November

Welcome  mohammed alsaiary [ Logout ]

Home PREP Pearls FAQs My Bookmarks CME Information

Overview Home  > November

Editorial Board
November   Enter Keyword Search
January
Question View:   All (8) Jump to Question
February
Print this Page Add to my Bookmarks Page 103 of 115
March

April Assessment History Mode: Learner  Exam 

May ASSESSMENT PROGRESS:  Total Questions:  8  Questions Answered:  6  Correct Answers:  2


June

July You currently have 2 questions unanswered in this assessment

August

September
Question: 6
October
A previously healthy 11­year­old girl is afebrile and asymptomatic on day 5 of ceftriaxone therapy for meningococcal meningitis; that
November evening, she develops a temperature of 38.6°C. On physical examination, she is alert with a normal neurologic examination. The
only abnormality on examination is erythema, mild edema, and mild tenderness of the interphalangeal joint of her left third toe. The
December
on­call resident obtains a contrast­enhanced computed tomographic scan of the head, which is normal, except for a small,
nonenhancing right subdural effusion.

Of the following, the MOST appropriate next step in management is to

A. add intravenous vancomycin

Claim Credit B. administer oral ibuprofen

Evaluation C. aspirate and culture interphalangeal joint fluid
D. obtain neurosurgical consultation
My Learning Plan

Incorrect View Peer Results
Correct Answer: B Average Percent Correct:  61.54%

This child experienced secondary fever with bacterial meningitis, indicating a return of fever following
an initial resolution of fever. A careful physical examination revealed the most likely cause of immune­
mediated arthritis associated with meningococcal disease. Although meningococcal infectious arthritis
can occur with meningitis, the timing of the presentation of arthritis, relatively late in treatment, is more
suggestive of reactive arthritis, which is treated with anti­inflammatory agents. Although aspiration of
the involved joint could be considered, this is unlikely to alter management.

Meningococci are uniformly susceptible to ceftriaxone and vancomycin is not effective against this pathogen. Subdural
effusions are relatively common in bacterial meningitis and, as in this case, usually asymptomatic. Obtaining cranial imaging
in this otherwise well­appearing child likely was not indicated, nor would be neurosurgical consultation because draining an
uninfected subdural effusion would provide no benefit to the child. On the other hand, if this child had presented with
irritability, toxic appearance, or severe headache, then subdural empyema (infected subdural effusion) is a possibility and
cranial imaging may be helpful. Suspected subdural empyemas may require operative intervention.

Secondary fevers in bacterial meningitis usually are not directly related to problems with treating the meningitis, as in the
uncommon scenario presented in the vignette. Nosocomial viral infections and phlebitis at an intravascular catheter site are
common causes of secondary fevers. Patients receiving initial dexamethasone therapy for meningitis may appear to have
secondary fevers because steroids are potent antipyretics, and discontinuation of dexamethasone may be followed by return
of fever. In one case series, without the use of steroids, 66% of children with meningococcal meningitis are afebrile by day 3
of therapy, compared with 51% of pneumococcal meningitis patients.

Prolonged fevers, as well as secondary fevers, can have more serious causes as well. Other foci of infection (eg, epidural
abscess, pneumonia, osteomyelitis) may be present, and intracranial complications such as venous thrombosis or stroke from
cerebral vasculitis frequently are accompanied by fever. Usually, these more severe complications are accompanied by
clearly abnormal findings on physical examination, such as irritability or altered mental status, seizures, or focal neurologic
signs.

PREP Pearls
Common causes of secondary fevers in bacterial meningitis include nosocomial viral infection and phlebitis.

Subdural effusions are common in bacterial meningitis and do not by themselves require intervention.

http://2016.prepid.courses.aap.org/script/november?req=201612122208353136&status=submit 1/2
12/13/2016 November
Serious complications of bacterial meningitis, such as cerebral infarction or vasculitis, may be accompanied by fever and
significant neurologic signs. 

American Board of Pediatrics Content Specification(s)
Develop a differential diagnosis in a patient with meningitis whose fever is persistent or recurrent

Recognize the complications of meningitis (eg, subdural effusions, seizures, empyema, cerebral venous
thrombosis/infarction, fever)

Suggested Readings
Esterle TM, Edwards KM. Concerns of secondary fever in Streptococcus pneumoniae meningitis in an era of increasing
antibiotic resistance. Arch Pediatr Adol Med. 1996;150(5):552­554. doi:
http://dx.doi.org10.1001/archpedi.1996.02170300106023

Goedvolk C, von Rosenstiel IA, Bos A. Immune complex associated complications in the subacute phase of meningococcal
disease: incidence and literature review. Arch Dis Child. 2003;88(10):927­930. doi: http://dx.doi.org10.1136/adc.88.10.927

Lin TY, Nelson JD, McCracken GH. Fever during treatment for bacterial meningitis. Pediatr Infect Dis. 1984;3(4):319­322.

Snedecker JD, Kaplan SL, Dodge PR, Holmes SJ, Feigin RD. Subdural effusion and its relationship with neurologic sequelae
of bacterial meningitis in infancy: a prospective study. Pediatrics. 1990;86(2):163­170.
http://pediatrics.aappublications.org/content/86/2/163

Tunkel AR, Hartman BJ, Kaplan SL, et al. Practice guidelines for the management of bacterial meningitis. Clin Infect Dis.
2004;39(9):1267­1284. doi: http://dx.doi.org/10.1086/425368

Comment On This Question

Page 103 of 115

Contact Us
Links may open in separate window
Copyright © 2016   American Academy of Pediatrics. All rights reserved. Pop­up Blocker may need to be disabled

http://2016.prepid.courses.aap.org/script/november?req=201612122208353136&status=submit 2/2
12/13/2016 November

Welcome  mohammed alsaiary [ Logout ]

Home PREP Pearls FAQs My Bookmarks CME Information

Overview Home  > November

Editorial Board
November   Enter Keyword Search
January
Question View:   All (8) Jump to Question
February
Print this Page Add to my Bookmarks Page 104 of 115
March

April Assessment History Mode: Learner  Exam 

May ASSESSMENT PROGRESS:  Total Questions:  8  Questions Answered:  6  Correct Answers:  2


June

July You currently have 2 questions unanswered in this assessment

August

September
Question: 7
October
You are evaluating a 13­year­old international adoptee from Africa for chronic abdominal pain and indigestion. As you are discussing
November the differential diagnosis, one of your trainees asks you about the common reservoirs of Helicobacter pylori, given the high
prevalence and incidence of colonization particularly among certain populations.
December
Of the following, the major reservoir for H pylori now appears to be

A. food sources
B. humans
C. soil
Claim Credit
D. water
Evaluation

My Learning Plan
Submit   Reset  

Page 104 of 115

Contact Us
Links may open in separate window
Copyright © 2016   American Academy of Pediatrics. All rights reserved. Pop­up Blocker may need to be disabled

http://2016.prepid.courses.aap.org/script/november?q­page=7&req=201612122208360324 1/1
12/13/2016 November

Welcome  mohammed alsaiary [ Logout ]

Home PREP Pearls FAQs My Bookmarks CME Information

Overview Home  > November

Editorial Board
November   Enter Keyword Search
January
Question View:   All (8) Jump to Question
February
Print this Page Add to my Bookmarks Page 104 of 115
March

April Assessment History Mode: Learner  Exam 

May ASSESSMENT PROGRESS:  Total Questions:  8  Questions Answered:  7  Correct Answers:  2


June

July You currently have 1 questions unanswered in this assessment

August

September
Question: 7
October
You are evaluating a 13­year­old international adoptee from Africa for chronic abdominal pain and indigestion. As you are discussing
November the differential diagnosis, one of your trainees asks you about the common reservoirs of Helicobacter pylori, given the high
prevalence and incidence of colonization particularly among certain populations.
December
Of the following, the major reservoir for H pylori now appears to be

A. food sources
B. humans
C. soil
Claim Credit
D. water
Evaluation

My Learning Plan
Incorrect View Peer Results
Correct Answer: B Average Percent Correct:  84.62%

Helicobacter pylori has been found in different populations all over the globe. Neither food, soil, nor
water sources have been proven to be reservoirs of H pylori. In fact, humans appear to be the major, if
not the sole reservoir of this organism.

In contrast to adults, H pylori infection in children is characterized by reduction in polymorphonuclear
and mononuclear cell infiltration, and decreased incidence of gastroduodenal ulceration with intact
epithelium. Given the decreased levels of mucosal atrophy and metaplasia, gastric adenocarcinoma
has not been reported in young children. In addition, several H pylori genes have been identified as being virulence
associated. Among them, the cytotoxin­associated cagA­positive H pylori strain has been associated with increased risk of
development of gastroduodenal ulcer disease.

Populations at high risk of H pylori infection include those with poor socioeconomic status (SES). However, populations of
high SES but with reported high prevalence of atrophic gastritis, intestinal metaplasia, and gastric cancer risk have been
reported from certain Southeast Asian countries such as South Korea and Japan. Conversely, certain populations of low SES
represented by many African and some coastal and low altitude regions of Latin America (despite a high prevalence of
infection in early childhood) have predominantly nonatrophic gastritis with low proportion of metaplastic changes.

These observed differences in epidemiology of H pylori infection illustrate its complex epidemiology which involves an
interplay between environmental and host factors, as well as those related to the bacterium itself. H pylori infection affects
approximately one­half of the world’s population, and has been associated with chronic gastritis, duodenal and gastric ulcer
disease, as well as gastric mucosa­associated lymphoid tissue (MALT) lymphoma and adenocarcinoma.

Associated risk factors include poor home sanitation, lack of adequate and clean water supply, household crowding,
increased family size, and poor dietary habits. As noted previously, while this is an infection that has a worldwide prevalence,
colonization and infection are more prevalent and occur earlier in resource­poor settings where over 80% to 90% of children
may be colonized as early as the first decade of life, with up to 60% becoming so within the first 1 to 3 months of life.
Colonization in children is always accompanied by gastric mucosal inflammation, however, the degree of inflammation is less
than that seen in adults.

H pylori­infected children in industrialized countries such as the United States, western Europe, Australia, Canada, and
certain Latin American countries such as Chile have been reported to have markedly reduced to decreased levels of atrophy,
metaplasia, and dysplasia, hence the decreased risk of cancer. This may also be related to the H pylori genotype present. For
example, the cagA gene associated with increased virulence is present in only 50% to 60% of isolates from Western
countries, but in greater than 90% of isolates from east Asian countries. Another extensively studied H pylori gene is the
vacuolating cytotoxin (vacA) gene, which has also been associated with higher degrees of inflammation, atrophy, and

http://2016.prepid.courses.aap.org/script/november?req=201612122209094231&status=submit# 1/3
12/13/2016 November
intestinal metaplasia. Phylogenetic analysis of global strains reveals there are 7 populations of H pylori, including hpAfrica1,
hpAfrica2, hpEastAsia, hpEurope, hpNEAfrica, hpAsia2, and hpSahul. Recent studies have shown that these different
populations all vary in their carcinogenic potential and contribute to the variation in incidence of gastric cancer among different
geographic regions.

Dietary influences are also important and may contribute to infection and risk of developing cancer later in life. Populations
with low SES tend to have diets that are high in salt and low in animal protein, as well as fruits and vegetables. Populations of
low SES who have low cancer rates have diets that consume foods rich in antioxidants such as fish and seafood, as well as
tropical fruits and vegetables. In addition, these populations, particularly in western Africa, also have high parasite burdens,
which tend to produce an anti­inflammatory Th­2 type immune response against H pylori that may lead to decrease in gastric
cancer risk. In an animal model, concurrent helminthic infection considerably reduced H pylori­associated gastric
inflammatory cytokines and chemokines. Similar responses have been seen with another helminthic infection, ie,
Schistosoma japonicum. This may in part also explain the lower cancer rates in populations with low SES in western Africa
and some parts of coastal Latin America.

Patients who are colonized with H pylori are for the most part asymptomatic. Those who present with symptoms often
complain of heartburn, dyspepsia, nausea, vomiting, or halitosis. A definitive causal relationship between recurrent abdominal
pain of childhood and H pylori infection has not yet been established. It has been established, however, that H pylori does not
play any major causative role in gastroesophageal reflux disease in children or adults. Patients with erosive gastritis or peptic
ulcer disease may present with hematemesis or melena. Severe infection usually results in ulcer formation and/or loss of
glandular tissue. Patients with duodenal ulcers are not at increased risk of gastric cancer, while the reverse is true of those
with gastric ulcers because the latter often develop atrophic gastritis. Some of these patients will develop intestinal
metaplasia, which may progress further to dysplasia and invasive adenocarcinoma. Of those infected, 15% to 20% will
develop gastroduodenal ulcer disease, however, the true incidence of gastroduodenal ulcer disease in children is unknown. 

Less than 1% of infected patients will ever develop gastric cancer. Intestinal type adenocarcinoma is more common in
patients with gastric cancer, whereas the diffuse type adenocarcinoma is more frequent in populations at low risk than those
at high risk. Anatomically, stomach adenocarcinomas are classified as noncardia and cardia cancers. H pylori is a recognized
risk factor for noncardia cancer. Given that these cancers take decades to develop following initial infection, gastric
adenocarcinoma has not been reported in children.

There are 2 well­established phenotypes of gastric inflammation associated with H pylori infection (Figure): a nonatrophic,
antral predominant gastritis associated with duodenal ulcer disease and a progressive, multifocal, atrophic gastritis associated
with gastric cancer. Acute infection is acquired in early childhood and may present as a mild illness with epigastric pain and
nausea. This is usually transient and symptoms resolve within 2 weeks. Histologically neutrophilic infiltration is seen, followed
by gradual infiltration of all classes of inflammatory cells, predominantly lymphocytes. Transient hypochlorhydria follows and
may be mediated by cytokines such as interleukin 1 or by the bacterium itself. At this stage, an acute H pylori infection can be
diagnosed by a positive urea breath test and negative immunoglobulin G anti­H pylori antibodies.

Following a long latent period, patients may go onto develop chronic gastritis. Histologically, an inflammatory infiltrate of
lymphocytes and plasma cells is seen, although there maybe varying degrees of both acute and chronic inflammation noted.
Chronic gastritis then follows 2 distinct patterns. An antral predominant gastritis is characteristic of individuals who develop
duodenal ulcers. Acid hypersecretion is the hallmark of duodenal ulcer disease. Gastric metaplasia, commonly in the first part
of the duodenum, develops as a protective mechanism from acid injury. This metaplasia appears to be essential for H pylori
colonization of the duodenal mucosa. The second pattern of chronic gastritis is one of progressive pangastritis or multifocal
atrophic gastritis characterized by infection of both the gastric corpus and antrum with progressive development of gastric
atrophy and intestinal metaplasia. Eleven percent of these patients may go on to develop gastric cancer. Low acid secretion is
the hallmark of this form of gastritis. H pylori is more tolerant of a low pH than most bacteria; it cannot proliferate in areas of
higher acid production such as the gastric corpus. H pylori eradication after treatment reverses gastric atrophy from both the
antrum and the corpus, but intestinal metaplasia is not reversible.

Several other bacterial factors besides the vacA and cagA genes have been associated with colonization, persistence, and
virulence of H pylori. Bacterial flagella and urease play a role in colonization, while factors involved in persistence besides
flagella are adhesins, lipopolysaccharide, γ glutamyl transpeptidase, and other regulatory T cells (Tregs).

Given how common colonization and infection with H pylori is, routine testing is not recommended. The goal of testing in
select populations or individuals should be to determine the cause of the gastrointestinal symptoms and not the presence of H
pylori infection. Evidence­based management guidelines for the management and testing of H pylori have been published.

* Required *
Take Survey  

PREP Pearls
Most Helicobacter pylori gastrointestinal infections in children are asymptomatic or characterized by nonspecific signs and
symptoms.

H pylori disease in children is different from adults in that there is a reduction in polymorphonuclear infiltration and
gastrointestinal ulceration, as well as gastric atrophy, thus gastric carcinoma is not seen in young children

In general, the goal of testing in select populations or individuals should be to determine the cause of the gastrointestinal
symptoms and not the presence of H pylori infection.

While in general H pylori infections are seen in populations of low socioeconomic status, there are distinct geographic
variations that depend on genetic, individual, and environmental factors.

American Board of Pediatrics Content Specification(s)
Know the epidemiology and pathophysiology characteristic of Helicobacter pylori infection

Distinguish the clinical situations in which Helicobacter pylori infection is a likely etiologic agent (eg, duodenal ulcer, peptic
ulcer) or is associated (adenocarcinoma of the stomach, MALTOMA)

Suggested Readings
http://2016.prepid.courses.aap.org/script/november?req=201612122209094231&status=submit# 2/3
12/13/2016 November
Correa P, Piazuelo MB. Natural history of Helicobacter pylori infection. Dig Liver Dis. 2008;40(7):490­496. doi:
http://dx.doi.org/10.1016/j.dld.2008.02.035

Gold BD. Helicobacter pylori. In: Long SS, Pickering LK, Prober CG, eds. Principles and Practice of Pediatric Infectious
Diseases. 4th ed. Philadelphia, PA: Saunders Elsevier; 2012:908­915. .

Harris PR, Smythies LE, Smith PD, et al. Role of childhood infection in the sequelae of H.pylori disease. Gut Microbes.
2013;4(6):426­438. doi: http://dx.doi.org/10.4161/gmic.26943

Kodaman N, Pazos A, Schneider BG, et al. Human and Helicobacter pylori coevolution shapes the risk of gastric disease.
Proc Natl Acad Sci USA. 2014;111(4):1455­1460. doi:  http://dx.doi.org/10.1073/pnas.1318093111

Koletzko S, Jones NL, Goodman KJ, et al. Evidence based guidelines from ESPGHAN and NASPGHAN for Helicobacter
pylori infection in children. J Pediatr Gastroenterol Nutr. 2011;53(2):230­243. doi:
http://dx.doi.org/10.1097/MPG.0b013e3182227e90

Comment On This Question

Page 104 of 115

Contact Us
Links may open in separate window
Copyright © 2016   American Academy of Pediatrics. All rights reserved. Pop­up Blocker may need to be disabled

http://2016.prepid.courses.aap.org/script/november?req=201612122209094231&status=submit# 3/3
12/13/2016 November

Welcome  mohammed alsaiary [ Logout ]

Home PREP Pearls FAQs My Bookmarks CME Information

Overview Home  > November

Editorial Board
November   Enter Keyword Search
January
Question View:   All (8) Jump to Question
February
Print this Page Add to my Bookmarks Page 105 of 115
March

April Assessment History Mode: Learner  Exam 

May ASSESSMENT PROGRESS:  Total Questions:  8  Questions Answered:  7  Correct Answers:  2


June

July You currently have 1 questions unanswered in this assessment

August

September
Question: 8
October
You are consulting on a previously healthy 9­year­old boy who presented in May with fever (up to 40°C [104°F]) for 4 days prior to
November his admission to the hospital. His symptoms include severe headaches, myalgias and lethargy. The boy has a generalized,
nonpruritic, maculopapular, and petechial rash (Figure). His hemodynamic status has been normal throughout the illness, and his
December
laboratory parameters reveal mild leucopenia, thrombocytopenia, hyponatremia, and elevated hepatic transaminases. A blood
culture has been negative and his symptoms rapidly improved after doxycycline was initiated. Your working diagnosis is a spotted
fever group rickettsiosis. There is no history of a tick bite, but the boy lives in a rural wooded area, and spends a significant amount
of time playing in the woods, where he is in contact with rodents and other small wild mammals. The family owns several free­
roaming dogs. You conclude that the most likely cause of his illness is Rickettsia rickettsii, and you are asked to explain to the
primary team how various vectors and animals are involved in maintenance of the causative organism in nature.
Claim Credit
Of the following, the main reservoir hosts for R rickettsii are
Evaluation
A. dogs 
My Learning Plan
B. humans
C. ticks
D. wild mammals

Submit   Reset  

Page 105 of 115

Contact Us
Links may open in separate window
Copyright © 2016   American Academy of Pediatrics. All rights reserved. Pop­up Blocker may need to be disabled

http://2016.prepid.courses.aap.org/script/november?q­page=8&req=201612122209099387 1/1
12/13/2016 November

Welcome  mohammed alsaiary [ Logout ]

Home PREP Pearls FAQs My Bookmarks CME Information

Overview Home  > November

Editorial Board
November   Enter Keyword Search
January
Question View:   All (8) Jump to Question
February
Print this Page Add to my Bookmarks Page 105 of 115
March

April Assessment History Mode: Learner  Exam 

May ASSESSMENT PROGRESS:  Total Questions:  8  Questions Answered:  8  Correct Answers:  2


June

July You currently have 1 required survey(s) that are incomplete. Please complete the survey(s) for the following
question(s) on this assessment:  Question 7
August

September

October Question: 8
November You are consulting on a previously healthy 9­year­old boy who presented in May with fever (up to 40°C [104°F]) for 4 days prior to
his admission to the hospital. His symptoms include severe headaches, myalgias and lethargy. The boy has a generalized,
December
nonpruritic, maculopapular, and petechial rash (Figure). His hemodynamic status has been normal throughout the illness, and his
laboratory parameters reveal mild leucopenia, thrombocytopenia, hyponatremia, and elevated hepatic transaminases. A blood
culture has been negative and his symptoms rapidly improved after doxycycline was initiated. Your working diagnosis is a spotted
fever group rickettsiosis. There is no history of a tick bite, but the boy lives in a rural wooded area, and spends a significant amount
of time playing in the woods, where he is in contact with rodents and other small wild mammals. The family owns several free­
roaming dogs. You conclude that the most likely cause of his illness is Rickettsia rickettsii, and you are asked to explain to the
Claim Credit primary team how various vectors and animals are involved in maintenance of the causative organism in nature.

Evaluation Of the following, the main reservoir hosts for R rickettsii are

My Learning Plan
A. dogs 
B. humans
C. ticks
D. wild mammals

Incorrect View Peer Results

Correct Answer: C Average Percent Correct:  46.15%

Most species of the genus Rickettsia are maintained as vertically transmitted symbionts (by
transovarial transmission) in arthropods. This is in contrast to many other arthropod­borne pathogens
of humans, which require another vertebrate species to maintain the life cycle. For rickettsiae, the main
reservoir hosts are the arthropod vectors themselves (tick, mite, louse, flea, or other insect). Some
rickettsiae do spend a part of their life cycle in vertebrate hosts, which act as the amplifying host and
appear to be essential to the organism's survival in nature. However, definitive data are lacking on the
role of specific vertebrates, including dogs and wild mammals. Rickettsia rickettsii has been isolated
from various small mammals. Some, including ground squirrels, meadow voles, and chipmunks, develop rickettsemia of
magnitude enough to infect laboratory­reared ticks, but others such as dogs and wild rats, do not sustain rickettsemia that
would routinely infect ticks. Some vertebrates, including humans, are dead­end hosts. Therefore, for the scenario of R
rickettsii infectionin the vignette, the main reservoir host is a tick, most commonly a Dermacentor species tick.

The Table lists main examples of human Rickettsia infections in which the critical arthropod vectors and animal hosts are
known. Several other candidate rickettsiae identified in nature have the potential to cause human infections, but are not to
date linked with confirmed cases.

PREP Pearls
Most rickettsiae are maintained by transovarian transmission in arthropods and may not require an animal reservoir in their
life cycle.

The arthropod reservoirs of rickettsiae are diverse and geographically widespread, but not all species are proven vectors of
infection to humans.

Several other rickettsiae identified in nature are candidates to cause human infections, but confirmed cases are lacking.

American Board of Pediatrics Content Specification(s)
http://2016.prepid.courses.aap.org/script/november?req=201612122209413919&status=submit 1/2
12/13/2016 November
Know the arthropod vectors and animal hosts that are critical factors in human Rickettsia infection

Suggested Readings
Parola P, Paddock CD, Socolovschi C, et al. Update on tick­borne rickettsioses around the world: a geographic approach. Clin
Microbiol Rev. 2013;26(4):657­702. doi: http://dx.doi.org/10.1128/CMR.00032­13

US Centers for Disease Control and Prevention. Other tick­borne spotted fever rickettsial infections. US Centers for Disease
Control and Prevention website. http://www.cdc.gov/otherspottedfever/. Updated February 15, 2012.

Walker DH, Bouyer DH. Rickettsia and orientia. In: Versalovic J, Carroll KC, Funke G, Jorgensen JH, Landry ML, Warnock
DW, eds. Manual of Clinical Microbiology. 10th ed. Washington, DC: ASM Press; 2011:2552 pp.

Comment On This Question

Page 105 of 115

Contact Us
Links may open in separate window
Copyright © 2016   American Academy of Pediatrics. All rights reserved. Pop­up Blocker may need to be disabled

http://2016.prepid.courses.aap.org/script/november?req=201612122209413919&status=submit 2/2
12/13/2016 December

Welcome  mohammed alsaiary [ Logout ]

Home PREP Pearls FAQs My Bookmarks CME Information

Overview Home  > December

Editorial Board
December   Enter Keyword Search
January
Question View:   All (8) Jump to Question
February
Print this Page Add to my Bookmarks Page 107 of 115
March

April Assessment History

May ASSESSMENT PROGRESS:  Total Questions:  8  Questions Answered:  0  Correct Answers:  0

June

July You currently have 8 questions unanswered in this assessment

August

September Question: 1
October An otherwise healthy 9­year­old girl underwent complete splenectomy 1 month ago following abdominal injury in a motor vehicle
November accident. Her family has relocated recently and she presents to you for advice on infection risk. She received all recommended
vaccines, including 7­valent pneumococcal conjugate vaccines, as an infant, but none since her splenectomy.
December
Of the following actions, the one that is the MOST appropriate at the current visit to prevent pneumococcal disease is

A. daily penicillin orally and medical alert bracelet
B. one dose 13­valent pneumococcal vaccine and daily penicillin orally
C. one dose 13­valent pneumococcal vaccine and one dose 23­valent pneumococcal polysaccharide vaccine
Claim Credit
D. one dose 23­valent pneumococcal polysaccharide vaccine and daily penicillin orally 
Evaluation

My Learning Plan
Submit   Reset  

Page 107 of 115

Contact Us
Links may open in separate window
Copyright © 2016   American Academy of Pediatrics. All rights reserved. Pop­up Blocker may need to be disabled

http://2016.prepid.courses.aap.org/script/december?q­page=1&req=201612122209590013 1/1
12/13/2016 December

Welcome  mohammed alsaiary [ Logout ]

Home PREP Pearls FAQs My Bookmarks CME Information

Overview Home  > December

Editorial Board
December   Enter Keyword Search
January
Question View:   All (8) Jump to Question
February
Print this Page Add to my Bookmarks Page 107 of 115
March

April Assessment History

May ASSESSMENT PROGRESS:  Total Questions:  8  Questions Answered:  1  Correct Answers:  0

June

July You currently have 7 questions unanswered in this assessment

August

September Question: 1
October An otherwise healthy 9­year­old girl underwent complete splenectomy 1 month ago following abdominal injury in a motor vehicle
November accident. Her family has relocated recently and she presents to you for advice on infection risk. She received all recommended
vaccines, including 7­valent pneumococcal conjugate vaccines, as an infant, but none since her splenectomy.
December
Of the following actions, the one that is the MOST appropriate at the current visit to prevent pneumococcal disease is

A. daily penicillin orally and medical alert bracelet
B. one dose 13­valent pneumococcal vaccine and daily penicillin orally
C. one dose 13­valent pneumococcal vaccine and one dose 23­valent pneumococcal polysaccharide vaccine
Claim Credit
D. one dose 23­valent pneumococcal polysaccharide vaccine and daily penicillin orally 
Evaluation

My Learning Plan
Incorrect View Peer Results

Correct Answer: B Average Percent Correct:  35.00%

Asplenia/hyposplenia of any etiology increases risk for pneumococcal infection and warrants
prophylactic measures. In this child’s instance, she should receive daily chemoprophylaxis plus
pneumococcal immunization as recommended for groups at high risk for pneumococcal disease, which
include anatomic or functional asplenia, HIV infection, cochlear implant, cerebrospinal fluid leak, and
other immunocompromising conditions. The latter group includes congenital immunodeficiencies,
chronic renal failure, and nephrotic syndrome, and diseases requiring use of immunosuppressive
drugs.

Immunization varies depending on quantity and types of prior pneumococcal immunizations received. This child has had no
prior doses of either the 13­valent conjugate vaccine (PCV13) or the 23­valent polysaccharide vaccine (PPSV23). Therefore,
she should receive PCV­13 at this visit, followed by 1 dose of PPSV23 at least 8 weeks later, plus a second dose 5 years after
the first.

A daily penicillin orally and medical alert bracelet is appropriate advice, but is incomplete in that no pneumococcal vaccination
is given. A medical alert bracelet is a good decision, but does not directly impact preventive measures. PCV13 administration
should precede PPSV23, and giving 1 dose of PCV13 and 1 dose of PPSV23 is incorrect both for this reason as well as
omitting chemoprophylaxis.

Children and adults with anatomic or functional asplenia have increased risk for pneumococcal infection, including fulminant
pneumococcal sepsis, as high as 350­fold compared to immunocompetent children. This risk decreases with age, but persists
in adults at least 25 years following splenectomy. For children with sickle cell disease, prophylactic penicillin can be
discontinued at 5 years of age if they have regular medical care and no episodes of severe pneumococcal disease or surgical
splenectomy. However, the duration of chemoprophylaxis for asplenic children with other conditions is uncertain, with many
experts electing to continue prophylaxis into adulthood. Duration of chemoprophylaxis following surgical splenectomy in
otherwise healthy individuals is controversial. Most experts recommend prophylaxis 1 year following splenectomy.

Penicillin is the drug of choice for pneumococcal chemoprophylaxis, typically oral penicillin V 125 mg twice daily for children
younger than 3 years of age and 250 mg twice daily for children older than 3 years of age. Oral amoxicillin 20 mg/kg per day
also is acceptable. Children with anaphylactic reactions to penicillins in the past can receive erythromycin 250 mg twice daily,
though efficacy of this regimen in an era of increasing macrolide­resistant pneumococci is unknown. Such patients may
benefit from penicillin hypersensitivity testing to verify their allergy. Regardless of chemoprophylactic regimen, patients and
families should be counseled to seek medical attention for febrile illnesses.

PREP Pearls

http://2016.prepid.courses.aap.org/script/december?req=201612122210565170&status=submit 1/2
12/13/2016 December
Underlying conditions conferring an increased risk of pneumococcal infections include anatomic or functional asplenia, HIV
infection, cochlear implant, cerebrospinal fluid leak, and other immunocompromising conditions.

Chemoprophylaxis, in addition to immunoprophylaxis, is important to prevent pneumococcal infections in at risk individuals.

Duration of chemoprophylaxis in asplenic individuals, other than for sickle cell disease patients, has not been well studied.

American Board of Pediatrics Content Specification(s)
Know the indications for penicillin chemoprophylaxis for pneumococcal infection in children with repeated or severe
pneumococcal infections that may be indicative of certain types of immune dysfunction (eg, splenic dysfunction, HIV)

Suggested Readings
American Academy of Pediatrics. Immunization in immunocompromised children. In: Kimberlin DW, Brady MT, Jackson MA,
Long SS, eds. Red Book: 2015 Report of the Committee on Infectious Diseases. 30th ed. Elk Grove Village, IL: American
Academy of Pediatrics; 2015:74­89..

American Academy of Pediatrics. Pneumococcal infections. In: Kimberlin DW, Brady MT, Jackson MA, Long SS, eds. Red
Book: 2015 Report of the Committee on Infectious Diseases. 30th ed. Elk Grove Village, IL: American Academy of Pediatrics;
2015:626­638.

Bradley JS. Chemoprophylaxis. In: Long SS, Pickering LK, Prober CG, eds. Principles and Practice of Pediatric Infectious
Diseases. 4th ed. Philadelphia, PA: Saunders Elsevier; 2012:68­76.

Comment On This Question

Page 107 of 115

Contact Us
Links may open in separate window
Copyright © 2016   American Academy of Pediatrics. All rights reserved. Pop­up Blocker may need to be disabled

http://2016.prepid.courses.aap.org/script/december?req=201612122210565170&status=submit 2/2
12/13/2016 December

Welcome  mohammed alsaiary [ Logout ]

Home PREP Pearls FAQs My Bookmarks CME Information

Overview Home  > December

Editorial Board
December   Enter Keyword Search
January
Question View:   All (8) Jump to Question
February
Print this Page Add to my Bookmarks Page 108 of 115
March

April Assessment History

May ASSESSMENT PROGRESS:  Total Questions:  8  Questions Answered:  1  Correct Answers:  0

June

July You currently have 7 questions unanswered in this assessment

August

September Question: 2
October You are seeing an 18­month­old patient who was referred to your clinic for recurrent upper respiratory tract infections. He has also
November had 2 episodes of gastroenteritis, one of which required admission for intravenous fluids. The child has otherwise been growing and
developing normally. His past history is significant for the fact that he was born at 32 weeks of gestation secondary to maternal
December preeclampsia. He stayed in the hospital for 2 weeks due to issues with feeding and neonatal hyperbilirubinemia. Once discharged
from the hospital, he did well until 4 months of age when his mother returned to work and he began to attend childcare. Since then,
he seems to get a runny nose every 2 weeks and has been diagnosed with 4 episodes of otitis media. He was also diagnosed with
respiratory syncytial virus bronchiolitis at 5 months of age and since then has had recurrent episodes of wheezing with colds for
which he is usually given a 5­day course of oral steroids. His mother is frustrated that he is constantly sick because he has to stay
home and she has to miss work. He has otherwise been growing and developing normally. She states that he has a good appetite
Claim Credit and is thriving. His immunizations are up­to­date for his age.

Evaluation The primary care physician sent some of his laboratory tests after talking with you earlier on the phone and the results are available
for review:
My Learning Plan
Complete blood cell count: within normal limits for age
Immunoglobulin A: 35 mg/dL (350 mg/L) (normal, 14­123 mg/dL)
Immunoglobulin G: 299 mg/dL (3 g/L) (normal, 424­1051 mg/dL)
Immunoglobulin M: 97 mg/dL (970 mg/L) (normal, 48­168 mg/dL)
Immunoglobulin E: 5 mg/dL (normal, 0.3­29 mg/dL)
Lymphocyte subsets: normal numbers and distribution of T cells and B cells
Titers to diphtheria, tetanus toxoid, and pneumococcal vaccine serotypes: within normal limits

Of the following, the MOST likely diagnosis of this patient is

A. selective immunoglobulin A deficiency
B. steroid induced hypogammaglobulinemia
C. transient hypogammaglobulinemia of infancy
D. X­linked agammaglobulinemia

Submit   Reset  

Page 108 of 115

Contact Us
Links may open in separate window
Copyright © 2016   American Academy of Pediatrics. All rights reserved. Pop­up Blocker may need to be disabled

http://2016.prepid.courses.aap.org/script/december?q­page=2&req=201612122210570014 1/1
12/13/2016 December

Welcome  mohammed alsaiary [ Logout ]

Home PREP Pearls FAQs My Bookmarks CME Information

Overview Home  > December

Editorial Board
December   Enter Keyword Search
January
Question View:   All (8) Jump to Question
February
Print this Page Add to my Bookmarks Page 108 of 115
March

April Assessment History

May ASSESSMENT PROGRESS:  Total Questions:  8  Questions Answered:  2  Correct Answers:  0

June

July You currently have 6 questions unanswered in this assessment

August

September Question: 2
October You are seeing an 18­month­old patient who was referred to your clinic for recurrent upper respiratory tract infections. He has also
November had 2 episodes of gastroenteritis, one of which required admission for intravenous fluids. The child has otherwise been growing and
developing normally. His past history is significant for the fact that he was born at 32 weeks of gestation secondary to maternal
December preeclampsia. He stayed in the hospital for 2 weeks due to issues with feeding and neonatal hyperbilirubinemia. Once discharged
from the hospital, he did well until 4 months of age when his mother returned to work and he began to attend childcare. Since then,
he seems to get a runny nose every 2 weeks and has been diagnosed with 4 episodes of otitis media. He was also diagnosed with
respiratory syncytial virus bronchiolitis at 5 months of age and since then has had recurrent episodes of wheezing with colds for
which he is usually given a 5­day course of oral steroids. His mother is frustrated that he is constantly sick because he has to stay
home and she has to miss work. He has otherwise been growing and developing normally. She states that he has a good appetite
Claim Credit and is thriving. His immunizations are up­to­date for his age.

Evaluation The primary care physician sent some of his laboratory tests after talking with you earlier on the phone and the results are available
for review:
My Learning Plan
Complete blood cell count: within normal limits for age
Immunoglobulin A: 35 mg/dL (350 mg/L) (normal, 14­123 mg/dL)
Immunoglobulin G: 299 mg/dL (3 g/L) (normal, 424­1051 mg/dL)
Immunoglobulin M: 97 mg/dL (970 mg/L) (normal, 48­168 mg/dL)
Immunoglobulin E: 5 mg/dL (normal, 0.3­29 mg/dL)
Lymphocyte subsets: normal numbers and distribution of T cells and B cells
Titers to diphtheria, tetanus toxoid, and pneumococcal vaccine serotypes: within normal limits

Of the following, the MOST likely diagnosis of this patient is

A. selective immunoglobulin A deficiency
B. steroid induced hypogammaglobulinemia
C. transient hypogammaglobulinemia of infancy
D. X­linked agammaglobulinemia

Incorrect View Peer Results
Correct Answer: C Average Percent Correct:  85.00%

The patient in the vignette has transient hypogammaglobulinemia of infancy (THI). The Table explains
the expected immunoglobulin levels in other humoral immunodeficiencies (congenital or drug­induced).
Isolated low immunoglobulin G (IgG) levels with a low normal immunoglobulin A (IgA) makes the
diagnosis of THI most likely.

Transient hypogammglobulinemia of infancy is usually seen in children between the ages of 6 months
to 3 years. The incidence is estimated to be between 2 to 60 out of 100,000 live births and is slightly
more common in boys. Transfer of maternal antibody to the fetus usually begins during the third trimester of gestation and
decreases with a half­life of 25 to 30 days after birth, reaching a nadir at 3 to 6 months of age. After birth a healthy term infant
usually starts to make immunoglobulin M (IgM), followed by IgG and then IgA. Sometimes, this physiologic nadir is prolonged.
This may be particularly pronounced in preterm infants, where IgG levels can reach very low concentrations and reach a nadir
earlier than in infants born at term.

The majority of patients with THI have normal levels of IgM and detectable IgA by 6 to 12 months of age. This supports the
diagnosis of THI in these patients because absence of IgA is the most common finding in permanent antibody deficiency
syndromes.

http://2016.prepid.courses.aap.org/script/december?req=201612122211245640&status=submit# 1/2
12/13/2016 December
Children with THI present with recurrent respiratory tract infections (sinusitis, otitis media, pneumonia) and viral infections,
although invasive bacterial infections are rare. These patients have low levels of IgG (< 2 standard deviations below the
mean), and up to 80% may also have low serum IgA concentrations. A very small percentage may have low IgM levels.

While THI occurs with greater frequency in families with other immunodeficiency syndromes, the exact cause of the delay in
IgG production in THI is unknown. The condition is, however, self­limited; most children have normal antibody production by
2­4 years of age and often no treatment is necessary. However, in children with recurrent or severe infections, immune
globulin intravenous (IGIV) may be considered for a period of 3 to 6 months, after which the need should be reassessed.

In contrast to THI, X­linked hypogammaglobulinemia (XLA) is far less common, with the incidence being 1 in 200,000 to
250,000 individuals. However, it accounts for 85% of cases of agammaglobulinemia and occurs as a result of mutations in the
gene that encodes Bruton tyrosine kinase on chromosome X. These children do well in the first few months of life when
transferred maternal antibody protects them from infections, although half of these patients become symptomatic by 1 year of
age and almost all by 5 years of age. All the major immunoglobulin classes (IgG, IgM, and IgA) are very low (< 2 standard
deviations below the mean for age). They also respond poorly to immunizations and do not develop specific antibody
responses to infection. Circulating CD19+ B lymphocytes are absent or may be present in very low numbers and this is often
what helps differentiate this from THI. T lymphocytes remain normal in number and function in these patients. Acute infections
may be associated with neutropenia.  

Children with XLA are prone to recurrent bacterial infections in infancy and childhood. These include otitis media, upper and
lower respiratory tract infections such as sinusitis and pneumonia, and osteoarticular infections. Invasive bacterial infections
such as bacteremia and meningitis are also common. These infections are usually caused by encapsulated bacteria,
particularly Haemophilus influenzae type b, Streptococcus pneumoniae, Staphylococcus aureus, and Pseudomonas species.
Chronic diarrhea and malabsorption may also be seen in up to 50% of affected adults, as well as in a small proportion of
children. Salmonella species and Campylobacter jejuni are also common causes of enteritis in these patients. Persistent
Mycoplasma and Ureaplasma infections of the respiratory and urogenital tract are also seen.

In addition to bacterial infections, chronic rotavirus and Giardia lamblia infections of the gastrointestinal tract and enteroviral
meningoencephalitis, which is often quite severe, are also seen in these patients. The latter can follow a chronic course with
insidious onset of loss of cognitive skills, ataxia, and paresthesias, or else can manifest as an acute illness with fever,
headache, and seizures. The incidence of autoimmune diseases (thrombocytopenia, hemolytic anemia, rheumatoid arthritis)
is also increased in these patients. Some individuals with chronic echovirus and coxsackievirus infections may develop
multisystem involvement and go on to develop chronic meningoencephalitis, dermatomyositis, and/or hepatitis.

Treatment of XLA is with lifelong intravenous immune globulin infusions along with treatment for any specific co­infection or
condition that accompanies XLA.

PREP Pearls
Transient hypogammaglobulinemia of infancy is usually seen in children between the ages of 6 months to 3 years and
occurs as a result of prolonged physiologic nadir in immunoglobulin production, which is more pronounced in premature
infants.

Transient hypogammaglobulinemia of infancy viral and bacterial infections, particularly of the respiratory tract, are common,
although invasive bacterial infections are rare.

X­linked hypogammaglobulinemia is an X­linked disorder of antibody production as a result of agammaglobulinemia, and
while bacterial infections predominate, these patients also have a susceptibility to develop chronic enteroviral infections, in
particular enteroviral meningoencephalitis.

American Board of Pediatrics Content Specification(s)
Recognize age­related occurrence and laboratory abnormalities of transient hypogammaglobulinemia

Plan a diagnostic evaluation for a patient with suspected X‑linked agammaglobulinemia

Suggested Readings
Addison EE. Infectious complications of antibody deficiency. In: Long SS, Pickering LK, Prober CG, eds. Principles and
Practice of Pediatric Infectious Diseases. 4th ed. Philadelphia, PA: Saunders Elsevier; 2012:609­615.

Primary Antibody Deficiencies. In: Rich RR, Fleisher TA, Shearer WT, Schroeder Jr HW, Frew AJ, Weyand CM, eds. Clinical
Immunology Principles and Practice. 3rd ed. Philadelphia, PA: Mosby Elsevier; 2008:527­528.

Comment On This Question

Page 108 of 115

Contact Us
Links may open in separate window
Copyright © 2016   American Academy of Pediatrics. All rights reserved. Pop­up Blocker may need to be disabled

http://2016.prepid.courses.aap.org/script/december?req=201612122211245640&status=submit# 2/2
12/13/2016 December

Welcome  mohammed alsaiary [ Logout ]

Home PREP Pearls FAQs My Bookmarks CME Information

Overview Home  > December

Editorial Board
December   Enter Keyword Search
January
Question View:   All (8) Jump to Question
February
Print this Page Add to my Bookmarks Page 109 of 115
March

April Assessment History

May ASSESSMENT PROGRESS:  Total Questions:  8  Questions Answered:  2  Correct Answers:  0

June

July You currently have 6 questions unanswered in this assessment

August

September Question: 3
October You are asked to see a 12­year­old­boy who is admitted to the pediatric intensive care unit for sepsis caused by methicillin­resistant
November Staphylococcus aureus infection (MRSA). The boy has a history of pre­B cell acute lymphocytic leukemia that is currently in
remission and is currently receiving maintenance chemotherapy. He was admitted with fever and hypotension and was found to have
December MRSA bacteremia. He has been managed for ongoing sepsis with fluid resuscitation and pressor support but has had one week of
persistently positive blood cultures with MRSA despite treatment with vancomycin. Vancomycin trough levels have been between 15
to 20 µg/mL (10.4­13.8 µmol/L) and the minimum inhibitory concentration of the organism to vancomycin is 1 µg/mL (0.7 µmol/L). An
indwelling central venous catheter was removed at the time of admission, but the boy’s blood cultures continue to be positive for
MRSA. Echocardiogram does not reveal any vegetations and ultrasonography of the vessels around the explanted central venous
catheter is negative for thrombus. There is no focus of infection on physical examination, and a chest radiograph and computed
Claim Credit tomography of the chest and abdomen does not reveal a focus of infection. The team is asking you for advice regarding additional
antibiotics or alternatives to vancomycin for the treatment of this infection. 
Evaluation
Of the following, the MOST appropriate antimicrobial agent to treat this infection is
My Learning Plan
A. cefepime
B. colistin
C. daptomycin
D. meropenem

Submit   Reset  

Page 109 of 115

Contact Us
Links may open in separate window
Copyright © 2016   American Academy of Pediatrics. All rights reserved. Pop­up Blocker may need to be disabled

http://2016.prepid.courses.aap.org/script/december?q­page=3&req=201612122211251265# 1/1
12/13/2016 December

Welcome  mohammed alsaiary [ Logout ]

Home PREP Pearls FAQs My Bookmarks CME Information

Overview Home  > December

Editorial Board
December   Enter Keyword Search
January
Question View:   All (8) Jump to Question
February
Print this Page Add to my Bookmarks Page 109 of 115
March

April Assessment History

May ASSESSMENT PROGRESS:  Total Questions:  8  Questions Answered:  3  Correct Answers:  0

June

July You currently have 5 questions unanswered in this assessment

August

September Question: 3
October You are asked to see a 12­year­old­boy who is admitted to the pediatric intensive care unit for sepsis caused by methicillin­resistant
November Staphylococcus aureus infection (MRSA). The boy has a history of pre­B cell acute lymphocytic leukemia that is currently in
remission and is currently receiving maintenance chemotherapy. He was admitted with fever and hypotension and was found to have
December MRSA bacteremia. He has been managed for ongoing sepsis with fluid resuscitation and pressor support but has had one week of
persistently positive blood cultures with MRSA despite treatment with vancomycin. Vancomycin trough levels have been between 15
to 20 µg/mL (10.4­13.8 µmol/L) and the minimum inhibitory concentration of the organism to vancomycin is 1 µg/mL (0.7 µmol/L). An
indwelling central venous catheter was removed at the time of admission, but the boy’s blood cultures continue to be positive for
MRSA. Echocardiogram does not reveal any vegetations and ultrasonography of the vessels around the explanted central venous
catheter is negative for thrombus. There is no focus of infection on physical examination, and a chest radiograph and computed
Claim Credit tomography of the chest and abdomen does not reveal a focus of infection. The team is asking you for advice regarding additional
antibiotics or alternatives to vancomycin for the treatment of this infection. 
Evaluation
Of the following, the MOST appropriate antimicrobial agent to treat this infection is
My Learning Plan
A. cefepime
B. colistin
C. daptomycin
D. meropenem

Incorrect View Peer Results
Correct Answer: C Average Percent Correct:  92.50%

The boy in the vignette has prolonged bacteremia caused by methicillin­resistant Staphylococcus
aureus (MRSA). Of the options listed, only daptomycin has activity against MRSA and would be an
appropriate alternative agent to vancomycin in the treatment of this infection. Colistin is an agent that
has activity against multidrug­resistant Gram­negative infections, but does not have activity against
Gram­positive organisms. Cefepime and meropenem do not have activity against MRSA.

Daptomycin is rapidly bactericidal against S aureus and is approved for use in adults with complicated
skin and soft tissue infection caused by susceptible gram positive bacteria and for the treatment of S aureus bacteremia
including right sided endocarditis. Although daptomycin has not been approved for use in pediatric patients and there are
limited data for its use in the pediatric population, it has been used successfully in clinical situations similar to that in the
vignette. In 1 study of pediatric patients (mean age 6.5 years) with persistent S aureus bacteremia despite treatment with
more traditional antibiotics, the addition or substitution of daptomycin therapy resulted in bacteriologic cure in 6 of 7 evaluable
patients and clinical improvement in 14 of 16 patients. A recent study from Italy reported that daptomycin therapy was well
tolerated and successful in 12 patients (mean age 6 months), most of whom had coagulase­negative S aureus (CONS) blood
stream infection (BSI) complicating cardiac surgery. The use of daptomycin in these patients allowed the management of
CONS BSI without removal of a central venous catheter. A recent report from Europe documented the clinical experience with
the drug in 81 children and adolescent patients with Gram­positive bacteremia, skin and soft tissue infection, osteomyelitis,
endocarditis, and foreign body and prosthetic infection. This report concluded that daptomycin alone or in combination with
other antibiotics and surgery achieved high clinical success and was well tolerated.

Daptomycin is a 13­member amino acid cyclic lipopeptide antibiotic derived from the fermentation of Streptomyces
roseosporus. This natural product contains a hydrophilic core and hydrophobic tail. Although the mechanism of action is not
completely understood, the hydrophobic tail binds irreversibly into the cell membrane of Gram­positive bacteria; this calcium­
dependent binding process leads to membrane depolarization through a potassium ion efflux. The end result is the inhibition
of DNA, RNA, and protein synthesis and rapid cell death. Daptomycin has a broad spectrum of activity against gram­positive
bacteria including staphylococci, streptococci (including S pneumoniae), and enterococci.  The drug is active against all
strains of staphylococci, including methicillin­resistant strains (MRSA), vancomycin­resistant strains (VRSA), and enterococci,

http://2016.prepid.courses.aap.org/script/december?req=201612122211524859&status=submit# 1/2
12/13/2016 December
with the subset of vancomycin­resistant enterococci (VRE). Daptomycin is also active against Lactobacillus, Pediococcus,
Leuconostoc, Bacillus, and Listeria species, as well as some anaerobes including peptostreptococci and Clostridia. 
Daptomycin does not have reliable activity against Actinomyces and has no activity against Gram­negative bacteria. 

Daptomycin is inactivated by pulmonary surfactant, which inhibits antibacterial activity, results in poor penetration into the lung
epithelial lining and parenchyma, and reduced efficacy in the treatment of pneumonia. Daptomycin has been shown to be
inferior to ceftriaxone in the treatment of pneumonia in adults. Thus, this agent has no role in the treatment of pneumonia. 

PREP Pearls
Daptomycin has been used in the pediatric population to treat Gram­positive infections including skin and soft tissue
infections and staphylococcal bacteremia.

Daptomycin works by a calcium­dependent cell binding process leading to inhibition of DNA, RNA, protein synthesis, and
rapid cell death.

Daptomycin is inactivated by pulmonary surfactant and thus not useful for the treatment of pneumonia. 

American Board of Pediatrics Content Specification(s)
Understand the current clinical situations (excluding pneumonia) in which daptomycin can be used in pediatrics

Understand the mechanism of action of daptomycin

Understand the antibacterial spectrum of activity of daptomycin

Suggested Readings
Ardura MI, Mejias A, Katz KS, et al. Daptomycin therapy for invasive Gram­positive bacterial infections in children. Pediatr
Infect Dis J. 2007;26(12):1128­1132. doi: http://dx.doi.org/10.1097/INF.0b013e31814523f8

Principi N, Caironi M, Venturini F, Pani L, Esposito S. Daptomycin in paediatrics: current knowledge and the need for future
research. J Antimicrob Chemother 2015;70(3):643­648. doi: http://dx.doi.org/10.1093/jac/dku453

Syriopoulou V, Dailiana Z, Dmitriy N, et al. Clinical experience with daptomycin for the treatment of gram­positive infections in
children and adolescents. Pediatr Infect Dis J. 2016;35(5):511­516. doi: http://dx.doi.org/10.1097/INF.0000000000001076

Tedeschi S, Tumietto F, Conti M, et al. Use of daptomycin in critically ill children with bloodstream infections and complicated
skin and soft tissue infections. Pediatr Infect Dis J. 2016;35(2):180­182. doi: http://dx.doi.org/10.1097/INF.0000000000000962

Comment On This Question

Page 109 of 115

Contact Us
Links may open in separate window
Copyright © 2016   American Academy of Pediatrics. All rights reserved. Pop­up Blocker may need to be disabled

http://2016.prepid.courses.aap.org/script/december?req=201612122211524859&status=submit# 2/2
12/13/2016 December

Welcome  mohammed alsaiary [ Logout ]

Home PREP Pearls FAQs My Bookmarks CME Information

Overview Home  > December

Editorial Board
December   Enter Keyword Search
January
Question View:   All (8) Jump to Question
February
Print this Page Add to my Bookmarks Page 110 of 115
March

April Assessment History

May ASSESSMENT PROGRESS:  Total Questions:  8  Questions Answered:  3  Correct Answers:  0

June

July You currently have 5 questions unanswered in this assessment

August

September Question: 4
October You are seeing a 30­day­old, 30­week former premature infant who is hospitalized for late­onset group B streptococcal (GBS)
November bacteremia and meningitis. 

December In your discussions with the team regarding the pathogenesis and risk factors for GBS infection, you would be sure to discuss that,
of the following, invasive infection in this infant correlates BEST with low concentrations of maternal and infant serum antibodies
directed against

A. B­hemolysin
B. C5a­ase
Claim Credit C. capsular polysaccharide

Evaluation D. lipoteichoic acid

My Learning Plan

Submit   Reset  

Page 110 of 115

Contact Us
Links may open in separate window
Copyright © 2016   American Academy of Pediatrics. All rights reserved. Pop­up Blocker may need to be disabled

http://2016.prepid.courses.aap.org/script/december?q­page=4&req=201612122211529546# 1/1
12/13/2016 December

Welcome  mohammed alsaiary [ Logout ]

Home PREP Pearls FAQs My Bookmarks CME Information

Overview Home  > December

Editorial Board
December   Enter Keyword Search
January
Question View:   All (8) Jump to Question
February
Print this Page Add to my Bookmarks Page 110 of 115
March

April Assessment History

May ASSESSMENT PROGRESS:  Total Questions:  8  Questions Answered:  4  Correct Answers:  0

June

July You currently have 4 questions unanswered in this assessment

August

September Question: 4
October You are seeing a 30­day­old, 30­week former premature infant who is hospitalized for late­onset group B streptococcal (GBS)
November bacteremia and meningitis. 

December In your discussions with the team regarding the pathogenesis and risk factors for GBS infection, you would be sure to discuss that,
of the following, invasive infection in this infant correlates BEST with low concentrations of maternal and infant serum antibodies
directed against

A. B­hemolysin
B. C5a­ase
Claim Credit C. capsular polysaccharide

Evaluation D. lipoteichoic acid

My Learning Plan

Incorrect View Peer Results

Correct Answer: C Average Percent Correct:  82.50%

Group B streptococci (GBS) are classified into serotypes based on type­specific capsular
polysaccharides, which consist of repeating units of monosaccharides and a sialic acid residue. The
capsule is thought to confer virulence by interfering with opsonization and phagocytosis, while the sialic
acid residue is thought to be important in the pathogenesis of infection with some capsular serotypes
(type III and possibly others). Currently, serotypes Ia and III are the most common causes of early­
onset GBS infection, while serotype III is the most common cause of late­onset infection.  

Vertical transmission is the major route of transmission of GBS in infants who develop invasive early onset GBS infection.
Although 50% of infants born to GBS­colonized mothers become colonized, disease develops in 1% to 2% of these infants. A
combination of maternal, bacterial, and host factors are important in determining which infants develop invasive infection.
These include the inoculum of GBS, the presence of maternal bacteriuria, strain virulence, prematurity, premature rupture of
membranes, and prolonged rupture of membranes. In addition, low serum concentrations of immunoglobulin G (IgG) to
serotype­specific polysaccharide capsule of the colonizing strain at delivery has been shown to be an important risk factor for
the development of invasive GBS infection. Several investigators have shown that invasive GBS disease caused by a specific
capsular polysaccharide strain occurs primarily in infants of women who have low serum concentrations of antibodies directed
against that specific polysaccharide capsular strain. This correlation has been shown to be important in the pathogenesis of
early­ and late­onset GBS disease caused by serotypes Ia, Ib, II, and III. Since the transplacental transfer of serotype­specific
antibody increases dramatically after 32 weeks of gestation, premature infants may not receive protective serotype­specific
IgG levels and this lack of protective antibody is an important factor in their susceptibility to GBS invasive infection. Antibody
directed against the capsular polysaccharide serotype of one type does not provide cross protection against other capsular
polysaccharide types. B­hemolysin is an extracellular product elaborated by GBS and is thought to play a role in virulence.
C5a­ase is a serine esterase enzyme that activates complement component 5a, which helps the bacteria evade host
defenses by hindering neutrophil accumulation at the site of infection. Lipoteichoic acid also is an extracellular product of
GBS, whose pathogenic role is not clear. There is no evidence that the lack of antibody to these extracellular substances
correlate with increased risk of infection. 

The rationale for immunoprophylaxis in the prevention of neonatal GBS infection is based on the observation that antibody
against the type­specific capsular polysaccharide is protective against infection with type­specific serotypes. Active
immunization of mothers with serotype­specific capsular polysaccharide vaccine can offer protection of the infant against the
vaccine strain serotype. An investigational type III polysaccharide­tetanus conjugate vaccine given to pregnant women was
shown to be highly immunogenic and shown to offer persistence of type­specific capsular antibodies in their neonates at 2
months of age.  

http://2016.prepid.courses.aap.org/script/december?req=201612122212166734&status=submit# 1/2
12/13/2016 December

PREP Pearls
Invasive group B streptococcus disease caused by a specific capsular polysaccharide strain occurs primarily in infants of
women who have low serum concentrations of antibodies directed against that specific polysaccharide capsular strain.

Active immunization of mothers with serotype specific capsular polysaccharide vaccine can offer protection of the infant
against the vaccine strain serotype.  

American Board of Pediatrics Content Specification(s)
Know the role of serotype­specific antibody in susceptibility to group B streptococcal infection

Suggested Readings
Baker CJ, Rench MA, McInnes P. Immunization of pregnant women with group B streptococcal type III polysaccharide­
tetanus toxoid conjugate vaccine. Vaccine. 2003;21(24):3468­3472. doi: http://dx.doi.org/10.1016/S0264­410X(03)00353­0

Pannaraj PS, Baker CJ, Group B streptococcal infections. In: Cherry J, Demmler­Harrison GJ, Kaplan SL, Steinbach WJ,
Hotez P, eds. Feigin and Cherry’s Textbook of Pediatric Infectious Diseases. Philadelphia, PA: Saunders Elsevier; 2013:1153­
1169.

Phares CR, Lynfield R, Farley MM, et al. Epidemiology of invasive group B streptococcal disease in the US, 1999­2005.
JAMA. 2008;299(17):2056­2065. doi: http://dx.doi.org/10.1001/jama.299.17.2056

Comment On This Question

Page 110 of 115

Contact Us
Links may open in separate window
Copyright © 2016   American Academy of Pediatrics. All rights reserved. Pop­up Blocker may need to be disabled

http://2016.prepid.courses.aap.org/script/december?req=201612122212166734&status=submit# 2/2
12/13/2016 December

Welcome  mohammed alsaiary [ Logout ]

Home PREP Pearls FAQs My Bookmarks CME Information

Overview Home  > December

Editorial Board
December   Enter Keyword Search
January
Question View:   All (8) Jump to Question
February
Print this Page Add to my Bookmarks Page 111 of 115
March

April Assessment History

May ASSESSMENT PROGRESS:  Total Questions:  8  Questions Answered:  4  Correct Answers:  0

June

July You currently have 4 questions unanswered in this assessment

August

September Question: 5
October A previously healthy 16­year­old adolescent developed a mildly tender swelling in his right inguinal area that ultimately became
November fluctuant. He was seen by a general surgeon who performed drainage and biopsy, reported as showing granulomatous
lymphadenitis with central necrosis. Additional testing revealed normal complete blood cell count and negative HIV serology. On
December further questioning, he recalls being scratched by his girlfriend’s cat about 6 weeks ago, which then developed a small “bump” about
a week later and ultimately resolved. He is sexually active with 1 lifetime female partner who is thought to be healthy. His physical
examination is normal, with the exception of a healing surgical scar in the right inguinal area and a 2­mm circular scar on his right
ankle, said to be the site of the previous bump.

You request further staining of the biopsy tissue, which is shown in the Figure.

Claim Credit Of the following, the MOST likely cause of this patient’s lymphadenitis is
Evaluation
A. Bartonella henselae
My Learning Plan B. Chlamydia trachomatis
C. Francisella tularensis
D. Mycobacterium fortuitum

Submit   Reset  

Page 111 of 115

Contact Us
Links may open in separate window
Copyright © 2016   American Academy of Pediatrics. All rights reserved. Pop­up Blocker may need to be disabled

http://2016.prepid.courses.aap.org/script/december?q­page=5&req=201612122212170797 1/1
12/13/2016 December

Welcome  mohammed alsaiary [ Logout ]

Home PREP Pearls FAQs My Bookmarks CME Information

Overview Home  > December

Editorial Board
December   Enter Keyword Search
January
Question View:   All (8) Jump to Question
February
Print this Page Add to my Bookmarks Page 111 of 115
March

April Assessment History

May ASSESSMENT PROGRESS:  Total Questions:  8  Questions Answered:  5  Correct Answers:  1

June

July You currently have 3 questions unanswered in this assessment

August

September Question: 5
October A previously healthy 16­year­old adolescent developed a mildly tender swelling in his right inguinal area that ultimately became
November fluctuant. He was seen by a general surgeon who performed drainage and biopsy, reported as showing granulomatous
lymphadenitis with central necrosis. Additional testing revealed normal complete blood cell count and negative HIV serology. On
December further questioning, he recalls being scratched by his girlfriend’s cat about 6 weeks ago, which then developed a small “bump” about
a week later and ultimately resolved. He is sexually active with 1 lifetime female partner who is thought to be healthy. His physical
examination is normal, with the exception of a healing surgical scar in the right inguinal area and a 2­mm circular scar on his right
ankle, said to be the site of the previous bump.

You request further staining of the biopsy tissue, which is shown in the Figure.

Claim Credit Of the following, the MOST likely cause of this patient’s lymphadenitis is
Evaluation
A. Bartonella henselae
My Learning Plan B. Chlamydia trachomatis
C. Francisella tularensis
D. Mycobacterium fortuitum

Correct View Peer Results
Average Percent Correct:  95.00%

The photomicrograph is a Warthin­Starry stain showing microabscesses and black­staining bacteria.
The clinical presentation is most consistent with cat scratch disease, caused by Bartonella henselae.
However, the remaining responses are all plausible causes of inguinal granulomatous adenitis, given
the appropriate clinical scenario. Lymphogranuloma venereum caused by Chlamydia trachomatis is a
sexually transmitted disease causing (usually) tender suppurative inguinal adenitis. Tularemia can be
transmitted by cats, although usually from bites rather than scratches and more commonly
accompanied by fever. Mycobacterium fortuitum, a rapidly growing nontuberculous mycobacterium,
causes panniculitis in cats, but rarely is transmitted to humans from cats. Thus, of the responses provided, B henselae is by
far the most likely.

It should be noted that the Warthin­Starry stain is not specific for B henselae, and also will stain mycobacteria, spirochetes,
microsporidia, and Helicobacter pylori, among other organisms. Sensitivity may be low as well; molecular testing of tissue
may reveal B henselae in stain­negative specimens.

Overall, the differential diagnosis of granulomatous lymphadenitis is broad. B henselae infection typically shows epithelioid
granulomas early in the course of lymphadenitis, later progressing to a more necrotic process. In addition to tularemia,
lymphogranuloma venereum, and mycobacterial infections, similar histology may be seen in a variety of infections, including
brucellosis, yersiniosis, endemic fungal infections, and toxoplasmosis.

PREP Pearls
A variety of infectious agents are associated with granulomatous lymphadenitis.

Warthin­Starry staining of tissue can aid in the diagnosis of cat scratch disease, but it is not specific for Bartonella henselae
organisms.

American Board of Pediatrics Content Specification(s)

http://2016.prepid.courses.aap.org/script/december?req=201612122212435485&status=submit 1/2
12/13/2016 December
Formulate a differential diagnosis of granulomatous lymphadenitis with central necrosis, including cat scratch, tularemia,
chlamydia

Recognize typical histologic appearance and staining characteristics of Bartonella species in tissue specimens

Suggested Readings
American Academy of Pediatrics. Cat scratch disease (Bartonella henselae). In: Kimberlin DW, Brady MT, Jackson MA, Long
SS, eds. Red Book: 2015 Report of the Committee on Infectious Diseases. 30th ed. Elk Grove Village, IL: American Academy
of Pediatrics; 2015:280­283.

James DG. A clinicopathological classification of granulomatous disorders. Postgrad Med J. 2000;76(898):457­465. doi:
http://dx.doi.org/10.1136/pmj.76.898.457

Schutze GE, Jacobs, RF. Bartonella species (cat­scratch disease). In: Long SS, Pickering LK, Prober CG, eds. Principles and
Practice of Pediatric Infectious Diseases. 4th ed. Philadelphia, PA: Saunders Elsevier; 2012:856­861.

Comment On This Question

Page 111 of 115

Contact Us
Links may open in separate window
Copyright © 2016   American Academy of Pediatrics. All rights reserved. Pop­up Blocker may need to be disabled

http://2016.prepid.courses.aap.org/script/december?req=201612122212435485&status=submit 2/2
12/13/2016 December

Welcome  mohammed alsaiary [ Logout ]

Home PREP Pearls FAQs My Bookmarks CME Information

Overview Home  > December

Editorial Board
December   Enter Keyword Search
January
Question View:   All (8) Jump to Question
February
Print this Page Add to my Bookmarks Page 112 of 115
March

April Assessment History

May ASSESSMENT PROGRESS:  Total Questions:  8  Questions Answered:  5  Correct Answers:  1

June

July You currently have 3 questions unanswered in this assessment

August

September Question: 6
October You are seeing a 15­year­old adolescent for fever, chills, severe fatigue, intermittent chest pain, and headache for the last 4 days.
November Past history is significant for a “heart murmur” as an infant that per his parents resolved at the age of 3 years. Physical examination
is significant for a tired­appearing adolescent who is lying on the examination table. He has a temperature of 39.2°C, heart rate of 80
December beats/min, and respiratory rate of 20 breaths/min. His lungs are clear to auscultation. His heart has a grade III/VI harsh systolic
murmur heard best at the left sternal border. His spleen is palpable 3 cm below the left costal margin. His extremities are warm and
well perfused.  He has some scattered petechiae on the palms and soles of his feet and on his conjunctivae bilaterally.

Of the following, the MOST important diagnostic test to confirm the diagnosis in this patient is

A. blood culture
Claim Credit
B. C­reactive protein
Evaluation
C. echocardiogram
My Learning Plan D. rheumatoid factor

Submit   Reset  

Page 112 of 115

Contact Us
Links may open in separate window
Copyright © 2016   American Academy of Pediatrics. All rights reserved. Pop­up Blocker may need to be disabled

http://2016.prepid.courses.aap.org/script/december?q­page=6&req=201612122212439704 1/1
12/13/2016 December

Welcome  mohammed alsaiary [ Logout ]

Home PREP Pearls FAQs My Bookmarks CME Information

Overview Home  > December

Editorial Board
December   Enter Keyword Search
January
Question View:   All (8) Jump to Question
February
Print this Page Add to my Bookmarks Page 112 of 115
March

April Assessment History

May ASSESSMENT PROGRESS:  Total Questions:  8  Questions Answered:  6  Correct Answers:  2

June

July You currently have 2 questions unanswered in this assessment

August

September Question: 6
October You are seeing a 15­year­old adolescent for fever, chills, severe fatigue, intermittent chest pain, and headache for the last 4 days.
November Past history is significant for a “heart murmur” as an infant that per his parents resolved at the age of 3 years. Physical examination
is significant for a tired­appearing adolescent who is lying on the examination table. He has a temperature of 39.2°C, heart rate of 80
December beats/min, and respiratory rate of 20 breaths/min. His lungs are clear to auscultation. His heart has a grade III/VI harsh systolic
murmur heard best at the left sternal border. His spleen is palpable 3 cm below the left costal margin. His extremities are warm and
well perfused.  He has some scattered petechiae on the palms and soles of his feet and on his conjunctivae bilaterally.

Of the following, the MOST important diagnostic test to confirm the diagnosis in this patient is

A. blood culture
Claim Credit
B. C­reactive protein
Evaluation
C. echocardiogram
My Learning Plan D. rheumatoid factor

Correct View Peer Results

Average Percent Correct:  67.50%

Infective endocarditis (IE) is a clinical diagnosis that requires a high index of suspicion because
patients present in many cases with nonspecific symptoms such as fever, chills, malaise, fatigue, and
weight loss. Some patients may present with stroke­like symptoms due to embolic events or with
congestive heart failure. Patients with renal failure or patients previously treated with antibiotics may
not present with fevers creating a challenge diagnostically. The interval between an event that
produces high­grade bacteremia and the onset of symptoms of IE is usually less than 2 weeks,
however, in many cases, a delay in diagnosis averages 5 weeks. The patient in the vignette has signs
and symptoms that are suspicious for endocarditis. Blood culture is the most important test to confirm the diagnosis of
endocarditis. C­reactive protein, rheumatoid factor, and echocardiogram are all adjunctive tests to the diagnosis of
endocarditis.

Almost any type of structural heart disease may predispose to IE, especially if the defect results in turbulence of blood flow.
Congenital heart disease, especially patent ductus arteriosus, ventricular septal defect, coarctation of the aorta, bicuspid
aortic valve, tetralogy of Fallot, and pulmonic stenosis, is responsible for 6% to 24% of IE cases. Rheumatic heart disease
with mitral and/or aortic valve involvement accounts for 5% or less of IE cases in developed countries; however, in developing
countries, it is the most common predisposing risk factor for IE. Degenerative cardiac lesions (eg, calcified mitral annulus,
calcific nodular lesions secondary to arteriosclerotic cardiovascular disease, post­myocardial infarction thrombus) are
becoming a greater risk factor for IE in patients without underlying valvular disease. Other conditions, such as luetic heart
disease, arterio­arterial fistulas, hemodialysis shunts or fistulas, intracardiac pacemaker wires, intracardiac prostheses, and
injection drug use are also risk factors for IE. However, over 50% of patients with IE have no recognized underlying cardiac
disease or predisposing risk factor.

The signs and symptoms of IE are determined by the extent of local cardiac disease, continuous bacteremia, and the degree
of involvement of distant organs as a result of embolization, metastatic infection, and circulating immune complexes. The
clinical findings are highly variable and mimic those of many other diseases. However, unexplained embolic phenomena in
any organ should suggest the diagnosis of endocarditis, especially in children with known heart disease. Table 1 shows the
most common clinical manifestations of IE in children.

Table 2 lists the diagnostic tests that may be helpful in the diagnosis of IE in children.

Blood culture is the most important diagnostic procedure in a patient suspected of having endocarditis. The bacteremia
usually is of low grade and continuous. The first 2 cultures yield the organism 90% of the time; in two­thirds of cases, all the

http://2016.prepid.courses.aap.org/script/december?req=201612122213076892&status=submit# 1/3
12/13/2016 December
blood cultures are positive. Three to 5 samples of blood for culture should be obtained from different sites within the first 24
hours in children with suspected endocarditis. Adequate blood volume per kg of body weight for culture is desirable for
optimal yield, however, this may be difficult in small children. The samples should be injected into thioglycolate and trypticase
soy (or brain­heart infusion) broth and held for at least 3 weeks to detect slow­growing organisms. If Gram­positive cocci grow
in the broth but fail to grow on subculture, nutritionally variant streptococci should be suspected and subculture should be
performed on media with either L­cysteine or pyridoxal phosphate. Many nonspecific laboratory tests are abnormal in patients
with IE. The erythrocyte sedimentation rate is elevated in 80% to 90% of cases. However, it may be normal or low when
congestive heart failure or renal failure is present. Serum C­reactive protein levels are usually elevated initially and return to
normal during the course of effective therapy. An increase during therapy may result from treatment failure, but can also be
caused by drug allergy or intercurrent infection. Rheumatoid factor is positive in 25% to 50% of children with IE. A positive test
may be a diagnostic aid in cases of culture negative endocarditis when other causes are excluded. Anemia is present in
approximately 40% of patients, especially those with long­standing disease. Since many patients with cyanotic heart disease
normally have a compensatory polycythemia, a serial drop in hematocrit is of more significance than a single measurement.
Hematuria and proteinuria is present in 25% to 50% of cases and is usually secondary to microemboli in the kidneys and may
be accompanied by pyuria, casts, and bacteriuria. Electrocardiogram is useful in the evaluation of patients with endocarditis
because it detects arrhythmias and conduction disturbances that may complicate disease. Ventricular ectopy may be related
to myocardial ischemia, myocarditis, or myocardial abscess. New conduction defects imply extension of infection beyond the
valve ring into the myocardium. Any degree of atrioventricular block, new left bundle branch block, or a new right bundle
branch block with a left anterior hemiblock may represent extension of infection from the aortic valve into the ventricular
septum. Junctional tachycardia, Wenckebach atrioventricular block, or complete heart block may be produced by extension of
the infection from the mitral valve annulus into the atrioventricular node or proximal His bundle. Echocardiography is a
valuable adjunct to the diagnosis and treatment of IE in children. Echocardiography can be performed by the traditional
transthoracic approach or the transesophageal approach. The sensitivity and specificity of transthoracic echocardiography
has been shown to range from 36% to 100% in children with IE in various series of pediatric patients. Transesophageal
echocardiography usually is more sensitive than transthoracic echocardiography in the detection of intracardiac vegetations
and is positive in 70% to 95% of adults with suspected endocarditis. It is significantly more sensitive in the detection of
vegetations and complications in infected prosthetic valves. Transesophageal echocardiography is particularly useful for
detecting an aortic root abscess or involvement of the sinus of Valsalva in adults and should be considered in children with
aortic valve endocarditis and changing aortic root dimensions on a standard transthoracic echocardiogram. However, even
though transesophageal echocardiography significantly increases the detection of vegetations in bigger (> 60 kg) children, it
does not significantly improve on the results of transthoracic echocardiography in smaller children. A negative
echocardiogram does not rule out endocarditis.

Blood culture­negative endocarditis (BCNE) accounts for 2.5% to 31% of all cases of endocarditis. This variation may be
explained by differences in the diagnostic criteria and sampling strategies used, specific epidemiologic factors affecting some
fastidious zoonotic agents, variations in the early use of antibiotics prior to blood sampling, and involvement of unknown
pathogens. Blood culture­negative endocarditis refers to definite or probable endocarditis in which 3 or more aerobic and
anaerobic blood cultures collected over 48 hours remain negative despite prolonged (> 1 week) incubation. The BCNE are
classified into 3 main categories:

1. bacterial endocarditis with blood cultures sterilized by previous antibacterial treatment ­ blood cultures are obtained 3
days or longer after the start of antibacterial therapy. This is caused by the usual endocarditis causing bacteria, ie,
streptococci, staphylococci or enterococci in 45% to 60% of the cases as evidence by positive polymerase chain
reaction (PCR) identification of DNA from these organisms;
2. endocarditis related to fastidious microorganisms eg, HACEK bacteria; defective streptococci – Gemella,
Granulicatella, and Abiotrophia species; Propionibacterium acnes, and Candida species. For these organisms,
prolonged incubation will allow identifying the causative pathogen in many cases;
3. the “true” blood culture­negative endocarditis, due to intracellular bacteria that cannot be routinely cultured in blood
with currently available techniques, eg, Bartonella species, Coxiella burnetii, and Tropheryma whipplei.

Noninfective endocarditis is rare and is mostly limited to marantic endocarditis or endocarditis related to lupus or Bechet.
Causative organisms in BCNE are shown in Table 3.

Risk factors for culture­negative IE include the risk of exposure to slow­growing bacteria, especially Coxiella burnetii and
Bartonella species, exposure to fastidious nonbacterial organisms, underlying valvular heart disease, right­sided endocarditis,
and endocarditis in patients with an intracardiac or vascular device or lines or other foreign bodies in contact with the blood. 

PREP Pearls
Blood culture is the most important diagnostic procedure in a patient suspected of having endocarditis. The bacteremia is
usually low grade and continuous. The first 2 cultures yield the organism 90% of the time; in two­thirds of cases, all the
blood cultures are positive.

Blood culture negative endocarditis accounts for 2.5% to 31% of all cases of endocarditis.

Echocardiography is a valuable adjunct to the diagnosis and treatment of infective endocarditis in children.

American Board of Pediatrics Content Specification(s)
Plan the diagnostic evaluation of a patient with suspected endocarditis

Formulate a microbiologic differential diagnosis of "culture­negative" endocarditis

Suggested Readings
Ansari A, Rigolin VH. Infective endocarditis: An update on the role of echocardiography. Curr Cardiol Rep. 2010;12(3):265­
271. doi: http://dx.doi.org/10.1007/s11886­010­0107­8

Dunne B, Marr T, Kim D, et al. Infective endocarditis. Heart Lung Circ. 2014;23(7):628­635. doi:
http://dx.doi.org/10.1016/j.hlc.2014.02.010

Dunne B, Marr T, Kim D, et al. Infective endocarditis. Heart Lung Circ. 2014;23(7):628­635. doi:
http://dx.doi.org/10.1016/j.hlc.2014.02.010

http://2016.prepid.courses.aap.org/script/december?req=201612122213076892&status=submit# 2/3
12/13/2016 December
Katsouli A, Massad MG. Current issues in the diagnosis and management of blood culture­negative infective and non­
infective endocarditis. Ann Thorac Surg. 2013;95(4):1467­1474. doi: http://dx.doi.org/10.1016/j.athoracsur.2012.10.044

Madico GE, Rice PA. 16S­ribosomal DNA to diagnosis culture negative endocarditis. Curr Infect Dis Rep. 2008;10(4):280­
286.

Starke JR. Infective endocarditis. In: Cherry JD, Demmler­Harrison GJ, Kaplan SL, Steinbach WJ, Hotez PJ, eds. Feigin and
Cherry’s Textbook of Pediatric Infectious Diseases. 7th ed. Philadelphia, PA: Saunders Elsevier; 2013:350­370.

Tattevin R, Watt G, Revest M, et al. Update on blood culture negative endocarditis. Med Mal Infect. 2015;45(1­2):1­8. doi:
http://dx.doi.org/10.1016/j.medmal.2014.11.003

Thanavaro KL, Nixon JV. Endocarditis 2014: an update. Heart Lung. 2014;43(4):334­337. doi:
http://dx.doi.org/10.1016/j.hrtlng.2014.03.009

Comment On This Question

Page 112 of 115

Contact Us
Links may open in separate window
Copyright © 2016   American Academy of Pediatrics. All rights reserved. Pop­up Blocker may need to be disabled

http://2016.prepid.courses.aap.org/script/december?req=201612122213076892&status=submit# 3/3
12/13/2016 December

Welcome  mohammed alsaiary [ Logout ]

Home PREP Pearls FAQs My Bookmarks CME Information

Overview Home  > December

Editorial Board
December   Enter Keyword Search
January
Question View:   All (8) Jump to Question
February
Print this Page Add to my Bookmarks Page 113 of 115
March

April Assessment History

May ASSESSMENT PROGRESS:  Total Questions:  8  Questions Answered:  6  Correct Answers:  2

June

July You currently have 2 questions unanswered in this assessment

August

September Question: 7
October The hematology­oncology service calls the pediatric infectious diseases team to inquire about appropriate pets for their patients, with
November an emphasis on diminishing zoonotic infections. The specific question is whether a hamster is an appropriate pet for a 3­year­old
child with acute myeloblastic leukemia who is facing a bone marrow transplant in the coming weeks.
December
Of the following, the organism MOST likely to be transmitted from the droppings of the hamster to the child is

A. Francisella tularensis
B. lymphocytic choriomeningitis virus
C. Streptobacillus moniliformis
Claim Credit
D. Yersinia pestis
Evaluation

My Learning Plan
Submit   Reset  

Page 113 of 115

Contact Us
Links may open in separate window
Copyright © 2016   American Academy of Pediatrics. All rights reserved. Pop­up Blocker may need to be disabled

http://2016.prepid.courses.aap.org/script/december?q­page=7&req=201612122213081894 1/1
12/13/2016 December

Welcome  mohammed alsaiary [ Logout ]

Home PREP Pearls FAQs My Bookmarks CME Information

Overview Home  > December

Editorial Board
December   Enter Keyword Search
January
Question View:   All (8) Jump to Question
February
Print this Page Add to my Bookmarks Page 113 of 115
March

April Assessment History

May ASSESSMENT PROGRESS:  Total Questions:  8  Questions Answered:  7  Correct Answers:  2

June

July You currently have 1 questions unanswered in this assessment

August

September Question: 7
October The hematology­oncology service calls the pediatric infectious diseases team to inquire about appropriate pets for their patients, with
November an emphasis on diminishing zoonotic infections. The specific question is whether a hamster is an appropriate pet for a 3­year­old
child with acute myeloblastic leukemia who is facing a bone marrow transplant in the coming weeks.
December
Of the following, the organism MOST likely to be transmitted from the droppings of the hamster to the child is

A. Francisella tularensis
B. lymphocytic choriomeningitis virus
C. Streptobacillus moniliformis
Claim Credit
D. Yersinia pestis
Evaluation

My Learning Plan
Incorrect View Peer Results

Correct Answer: B Average Percent Correct:  70.00%

Lymphocytic choriomeningitis (LCM) is a rodent­borne viral infectious disease caused by lymphocytic
choriomeningitis virus (LCMV), a member of the family arenaviridae. The primary host of LCMV is the
common house mouse, Mus musculus. Infection in house mice populations may vary by geographic
location; it is estimated that 5% of house mice in the United States carry LCMV and are able to transmit
virus for the duration of their lives without showing any sign of illness. Hamsters and gerbils are not the
natural reservoir, but can become infected with LCMV from wild mice at the breeder, in the pet store, or
home environment. Humans are more likely to contract LCMV from house mice, but infections from
hamsters and gerbils have also been reported. Data are insufficient to determine the potential for infection of other rodent
species (eg, chinchillas or dwarf hamsters). Husbandry practices in breeding facilities, distribution centers, and pet stores
make cross­contamination with LCMV a possibility.

Tularemia is caused by the bacterium Francisella tularensis. Rabbits, hares, and rats are especially susceptible and often die
in large numbers during outbreaks. Humans can become infected through several routes including tick and deer fly bites, skin
contact with infected animals, ingestion of contaminated water, inhalation of contaminated aerosols or agricultural dusts, and
laboratory exposure. Rat­bite fever, caused by Streptobacillus moniliformis in North America and Europe and Spirillum minus
in Asia and Africa, is transmitted by the bite or scratch from an infected rodent (rat or rarely a mouse), contact with a dead
rodent, or eating/drinking food or water contaminated by rat feces. Plague is caused by the bacterium Yersinia pestis.
Humans usually get plague after being bitten by a rodent flea that is carrying the plague bacterium or by handling an animal
infected with plague. Many types of animals, mainly rats, but also rock squirrels, wood rats, ground squirrels, prairie dogs,
chipmunks, mice, voles, and rabbits, can be affected by plague. Wild carnivores can become infected by eating other infected
animals.

The virions, or virus­like particles of arenavirus, display characteristic "sandy" or granular capsid (ie, outer skin), an
appearance from which the Latin name "arena" was derived (Figure 1 ).

Other members of arenavirus include the West African Lassa fever virus and South American arenaviruses: Argentine
hemorrhagic fever, Bolivian hemorrhagic fever, also known as Machupo virus, Sabiá­associated hemorrhagic fever, and
Venezuelan hemorrhagic fever, all of which are spread to humans through inhalation of airborne particulates or exposure to
fresh urine, droppings, saliva, or nesting materials from infected rodents. Transmission may also occur when these materials
are directly introduced into broken skin, the nose, the eyes, or the mouth, or presumably, via the bite of an infected rodent.
Spread to humans occurs through inhalation of airborne particulates originating from rodent excrement during the simple act
of sweeping a cage or a floor.

Infection with LCMV in humans with normal immune systems usually causes either asymptomatic or mild, self­limited illness.
For infected individuals who do become ill, onset of symptoms usually occurs 8 to 13 days after exposure to the virus as part

http://2016.prepid.courses.aap.org/script/december?req=201612122213353455&status=submit# 1/3
12/13/2016 December
of a biphasic febrile illness. This initial phase, which may last as long as a week, typically begins with fever, malaise, lack of
appetite, muscle aches, headache, nausea, or vomiting. Other symptoms appearing less frequently include sore throat,
cough, joint pain, chest pain, testicular pain, or parotid pain.

Following a few days of recovery, a second phase of illness includes aseptic meningitis and encephalitis. In rare instances,
myelitis with muscle weakness, paralysis, or paresthesias result. An association between LCMV infection and myocarditis has
been suggested. Mortality from LCMV infection is less than 1%. No chronic infection has been described in humans. Central
nervous system sequelae, particularly following encephalitis, include temporary or permanent neurological deficits,
hydrocephalus, and sensorineural deafness. Arthritis has also been reported.

Person­to­person transmission has not been reported, with the exception of vertical transmission from infected mother to
fetus, and rarely, through organ transplantation. The frequency with which developmental defects occur after in utero LCMV
infection is not known. LCMV infection during the first or second trimester of pregnancy can cause severe illness or
developmental defects in the fetus, including hydrocephalus, psychomotor retardation, chorioretinitis, blindness, and fetal
death (Figure 2 ). Pregnant women may recall a flu­like illness during pregnancy or may not recall any illness.

Infections with LCMV have historically been underreported, often making it difficult to determine incidence rates or estimates
of prevalence by geographic region. Several serologic studies conducted in urban areas have shown that the prevalence of
LCMV antibodies in human populations range from 2% to 5%. LCMV is most commonly recognized as causing neurological
disease, although infections with mild to no symptoms are more common clinical manifestations.

Laboratory workers who work with the virus or handle infected animals are also at risk. However, this risk can be minimized
by utilizing animals from sources that regularly test for the virus, wearing proper protective laboratory gear, and following
appropriate safety precautions.

During the first phase of the disease, the most common laboratory abnormalities are leukopenia and thrombocytopenia;
hepatic transaminases may be mildly elevated. With meningoencephalitis during the second phase, elevated protein, an
increase in the number of white blood cells and hypoglycorrhachia in the cerebrospinal fluid (CSF) is usually found.

Laboratory diagnosis is usually made by detecting immunoglobulin M and immunoglobulin G antibodies in the CSF and
serum. Virus can be detected by PCR or virus isolation in the CSF during the acute stage of illness. Ribavirin is effective
against LCMV in vitro, however, there is no established evidence to support its routine use for treatment of LCM in humans.

There are numerous reports of LCMV organ transplant­associated infections in the United States in the past decade.
Physicians and public health practitioners should be aware that organ donors with suspected central nervous system infection
and some with intracranial hemorrhage without evidence of infection could be infected with LCMV, especially when rodent
exposure has occurred. Testing for LCMV should be considered in organ recipients who develop febrile illness, neurologic
changes, or multiorgan dysfunction in the early post­transplant period, especially if multiple recipients from the same donor
become ill. The recommended treatment for LCMV infections obtained through organ transplantation includes reduced
immunosuppression and ribavirin. The efficacy of ribavirin in these cases has not been determined; however, early detection
of LCMV and prompt treatment initiation might improve outcome.

* Required *
Take Survey  

PREP Pearls
Humans are more likely to contract lymphocytic choriomeningitis virus (LCMV) from house mice, but infections from pet
rodents (hamsters and gerbils) have also been reported.

There are numerous reports of LCMV organ transplant­associated infections in the United States in the past decade.

The frequency with which developmental defects occur after in utero LCMV infection is not known; LCMV infection during
the first or second trimester of pregnancy can cause severe illness or developmental defects in the fetus, including
hydrocephalus, psychomotor retardation, chorioretinitis, blindness, and fetal death.

Laboratory diagnosis is usually made by detecting immunoglobulin M and immunoglobulin G antibodies in the
cerebrospinal fluid and serum. Virus can be detected by polymerase chain reaction or virus isolation in the cerebrospinal
fluid during the acute stage of illness.

American Board of Pediatrics Content Specification(s)
Know the epidemiology (eg, role of rodent exposure) and modes of transmission of lymphocytic choriomeningitis virus
infection

Recognize the clinical manifestations and cerebrospinal fluid findings suggestive of lymphocytic choriomeningitis virus
infection

Plan appropriate laboratory evaluation for the diagnosis of lymphocytic choriomeningitis virus infection (eg, serology)

Suggested Readings
American Academy of Pediatrics. Lymphocytic choriomeningitis. In: Kimberlin DW, Brady MT, Jackson MA, Long SS, eds.
Red Book: 2015 Report of the Committee on Infectious Diseases. 30th ed. Elk Grove Village, IL: American Academy of
Pediatrics; 2015:527­528.

Division of Viral and Rickettsial Diseased, National Center for Infectious Diseases. Update: interim guidance for minimizing
risk for human lymphocytic choriomeningitis virus infection associated with pet rodents. MMWR Dispatch.
2005;54(Dispatch):1­3.

Schafer IJ, Miller R, Ströher U, Knust B, Nichol ST, Rollin PE. Notes from the field: a cluster of lymphocytic choriomeningitis
virus infections transmitted through organ transplantation — Iowa, 2013. MMWR Morb Mortal Wkly Rep. 2014;63(11):249.
http://www.cdc.gov/mmwr/preview/mmwrhtml/mm6311a6.htm

US Centers for Disease Control and Prevention. Lymphocytic choriomeningitis (LCM). US Centers for Disease Control and
Prevention website. http://www.cdc.gov/vhf/lcm/index.html. Updated May 6, 2014

http://2016.prepid.courses.aap.org/script/december?req=201612122213353455&status=submit# 2/3
12/13/2016 December

Comment On This Question

Page 113 of 115

Contact Us
Links may open in separate window
Copyright © 2016   American Academy of Pediatrics. All rights reserved. Pop­up Blocker may need to be disabled

http://2016.prepid.courses.aap.org/script/december?req=201612122213353455&status=submit# 3/3
12/13/2016 December

Welcome  mohammed alsaiary [ Logout ]

Home PREP Pearls FAQs My Bookmarks CME Information

Overview Home  > December

Editorial Board
December   Enter Keyword Search
January
Question View:   All (8) Jump to Question
February
Print this Page Add to my Bookmarks Page 114 of 115
March

April Assessment History

May ASSESSMENT PROGRESS:  Total Questions:  8  Questions Answered:  7  Correct Answers:  2

June

July You currently have 1 questions unanswered in this assessment

August

September Question: 8
October A 21­day­old neonate born at 24 weeks gestational age has catheter­associated bloodstream infection and 7 days of positive blood
November cultures with Candida albicans. A peripherally inserted central catheter was removed on illness day 5, and physical examination has
not revealed the cause of ongoing fungemia. No murmur is appreciated on cardiac examination and there is no evidence of
December thrombophlebitis at catheter sites. Cerebrospinal fluid studies and cultures performed at the onset of illness and repeated on illness
day 5 are negative. Ophthalmologic examination on day 6 of positive cultures did not reveal fungal involvement. There is no
evidence of urinary tract involvement by ultrasonography or urine culture. The patient has been tolerating intravenous amphotericin
B deoxycholate therapy. The neonatal intensive care unit team is maintaining access with peripheral arterial and venous catheters.

Of the following, the BEST next step in the management of this patient is to

Claim Credit A. change antifungal therapy to micafungin
Evaluation B. perform an echocardiogram
My Learning Plan C. remove peripheral arterial and venous catheters
D. repeat cerebrospinal fluid studies and culture

Submit   Reset  

Page 114 of 115

Contact Us
Links may open in separate window
Copyright © 2016   American Academy of Pediatrics. All rights reserved. Pop­up Blocker may need to be disabled

http://2016.prepid.courses.aap.org/script/december?q­page=8&req=201612122213358611 1/1
12/13/2016 December

Welcome  mohammed alsaiary [ Logout ]

Home PREP Pearls FAQs My Bookmarks CME Information

Overview Home  > December

Editorial Board
December   Enter Keyword Search
January
Question View:   All (8) Jump to Question
February
Print this Page Add to my Bookmarks Page 114 of 115
March

April Assessment History

May ASSESSMENT PROGRESS:  Total Questions:  8  Questions Answered:  8  Correct Answers:  2

June

July You currently have 1 required survey(s) that are incomplete. Please complete the survey(s) for the following
question(s) on this assessment:  Question 7
August

September

October Question: 8
November A 21­day­old neonate born at 24 weeks gestational age has catheter­associated bloodstream infection and 7 days of positive blood
cultures with Candida albicans. A peripherally inserted central catheter was removed on illness day 5, and physical examination has
December not revealed the cause of ongoing fungemia. No murmur is appreciated on cardiac examination and there is no evidence of
thrombophlebitis at catheter sites. Cerebrospinal fluid studies and cultures performed at the onset of illness and repeated on illness
day 5 are negative. Ophthalmologic examination on day 6 of positive cultures did not reveal fungal involvement. There is no
evidence of urinary tract involvement by ultrasonography or urine culture. The patient has been tolerating intravenous amphotericin
B deoxycholate therapy. The neonatal intensive care unit team is maintaining access with peripheral arterial and venous catheters.

Of the following, the BEST next step in the management of this patient is to
Claim Credit

Evaluation A. change antifungal therapy to micafungin
B. perform an echocardiogram
My Learning Plan
C. remove peripheral arterial and venous catheters
D. repeat cerebrospinal fluid studies and culture

Incorrect View Peer Results
Correct Answer: B Average Percent Correct:  72.50%

The patient described in the vignette has invasive candidiasis with Candida albicans and no obvious
source of ongoing infection on physical examination and follow­up studies. Although endocarditis is an
uncommon complication of invasive candidiasis in neonates, an echocardiogram is the best next step
in management because of persistently positive blood cultures and delayed removal (ie, after 5 days of
positive blood cultures) of the central catheter. Neonates with infective endocarditis can present without
a murmur and typically lack the embolic or immunologic phenomena associated with endocarditis in
older children and adults.

The incidence of neonatal candidiasis has decreased over the last decade, but very low birthweight infants remain at highest
risk of developing infection, death, and neurodevelopmental impairment. Neonates with invasive candidiasis are more likely
than older patients to present with nonspecific signs or symptoms of infection and to develop central nervous system disease
(eg, meningoencephalitis, meningitis), especially in the setting of ongoing fungemia. Therefore, repeating cerebrospinal fluid
studies (CSF) and cultures in neonates with ongoing fungemia is recommended. However, because CSF findings in neonates
with central nervous system disease are unreliable, central nervous system involvement should be assumed to be present in
neonates with candidemia and signs and symptoms suggesting meningoencephalitis because the incidence of this
complication is high. In the patient described in the vignette, a repeat lumbar puncture had already had been performed, so it
would not be the best next step in management.

For neonates and nonneutropenic patients with candidemia, it is highly recommended that central catheters be removed as
early as possible in in the course of infection whenever feasible. For neutropenic patients, catheter removal is considered on
an individual basis, but generally is recommended unless the gastrointestinal tract or another source of infection is suspected.
Removal of peripheral arterial and venous catheters is unnecessary, except in cases of associated thrombophlebitis. In all
patients, daily blood cultures should be performed to establish that the blood stream infection has cleared.

The evaluation and management of neonates with invasive candidiasis includes cultures of blood, cerebrospinal fluid, and
urine, in addition to prompt administration of amphotericin B deoxycholate at a dose of 1 mg/kg daily. Fluconazole (12 mg/kg
daily) is a reasonable alternative in patients with susceptible isolates and in those who have not been on fluconazole
prophylaxis. Lipid formulations of amphotericin B should be used with caution in patients with urinary tract involvement. An
echinocandin (eg, caspofungin, micafungin, anidulafungin) is recommended as initial therapy in older children and adult
patients with candidemia. However, there is uncertainty regarding the optimal dose and efficacy of echinocandins in neonates,

http://2016.prepid.courses.aap.org/script/december?req=201612122214017049&status=submit# 1/2
12/13/2016 December
so their use generally is not recommended unless resistance or toxicity preclude the use of the preferred antifungal agents.
All neonates with positive cultures for Candida from urine or blood should undergo a dilated retinal examination and those
with persistently positive cultures should have imaging (eg, ultrasonography or computed tomography) of the genitourinary
tract, liver, and spleen. The duration of antifungal therapy depends on the host and infectious complications. In general,
candidemia without metastatic complications is treated for a minimum of 2 weeks after blood stream infection has cleared.
Treatment of metastatic disease, including central nervous system infection, generally is longer (ie, at least 3­4 weeks).

PREP Pearls
For neonates and nonneutropenic patients with candidemia, it is highly recommended that central catheters be removed as
early as possible in the course of infection whenever feasible.

For neutropenic patients, catheter removal is considered on an individual basis, but generally is recommended unless the
gastrointestinal tract or another source of infection is suspected.

Neonates with invasive candidiasis are more likely than older patients to develop central nervous system disease.

All neonates with positive cultures for Candida from urine or blood should undergo a dilated retinal examination and those
with persistently positive cultures should have imaging (eg, ultrasonography or computed tomography) of the genitourinary
tract, liver, and spleen.

The evaluation and management of neonates with invasive candidiasis includes cultures of blood, cerebrospinal fluid, and
urine, in addition to prompt administration of amphotericin B deoxycholate at a dose of 1 mg/kg daily.

American Board of Pediatrics Content Specification(s)
Plan management for a patient with catheter related candidemia remove catheter, antifungal therapy, follow up blood
cultures)

Evaluate a patient with persistent candidemia after removal of venous catheter (suppurative phlebitis, imaging for
thrombosis or endocarditis)

Suggested Readings
American Academy of Pediatrics. Candidiasis. In: Kimberlin DW, Brady MT, Jackson MA, Long SS, eds. Red Book: 2015
Report of the Committee on Infectious Diseases. 30th ed. Elk Grove Village, IL: American Academy of Pediatrics; 2015:275­
280.

Pappas PG, Kauffman CA, Andes DR, et al. Clinical practice guideline for the management of candidiasis: 2016 update by
the Infectious Diseases Society of America. Clin Infect Dis. 2015. doi: http://dx.doi.org/10.1093/cid/civ933

Comment On This Question

Page 114 of 115

Contact Us
Links may open in separate window
Copyright © 2016   American Academy of Pediatrics. All rights reserved. Pop­up Blocker may need to be disabled

http://2016.prepid.courses.aap.org/script/december?req=201612122214017049&status=submit# 2/2
‫ﺷﻜﺮ ﺧﺎص ﻟﻜﻞ ﻣﻦ ﺳﺎﻫﻢ ﰲ ﻫﺬا اﻟﻌﻤﻞ‬
‫وﻓﻘﻨﺎ اﻟﻠﻪ واﻳﺎﻛﻢ‬

You might also like